Sie sind auf Seite 1von 187

RESOLUTION

Republic of the Philippines


SUPREME COURT
Manila PER CURIAM:

EN BANC Petitioners Isagani Cruz and Cesar Europa brought this suit for prohibition and
mandamus as citizens and taxpayers, assailing the constitutionality of certain provisions
of Republic Act No. 8371 (R.A. 8371), otherwise known as the Indigenous Peoples
G.R. No. 135385 December 6, 2000
Rights Act of 1997 (IPRA), and its Implementing Rules and Regulations (Implementing
Rules).
ISAGANI CRUZ and CESAR EUROPA, petitioners,
vs.
In its resolution of September 29, 1998, the Court required respondents to comment. 1 In
SECRETARY OF ENVIRONMENT AND NATURAL RESOURCES, SECRETARY OF
compliance, respondents Chairperson and Commissioners of the National Commission
BUDGET AND MANAGEMENT and CHAIRMAN and COMMISSIONERS OF THE
on Indigenous Peoples (NCIP), the government agency created under the IPRA to
NATIONAL COMMISSION ON INDIGENOUS PEOPLES, respondents.
implement its provisions, filed on October 13, 1998 their Comment to the Petition, in
HON. JUAN M .FLAVIER, HON. PONCIANO BENNAGEN, BAYANI ASCARRAGA,
which they defend the constitutionality of the IPRA and pray that the petition be
EDTAMI MANSAYANGAN, BASILIO WANDAG, EVELYN DUNUAN, YAOM TUGAS,
dismissed for lack of merit.
ALFREMO CARPIANO, LIBERATO A. GABIN, MATERNIDAD M. COLAS, NARCISA
M. DALUPINES, BAI KIRAM-CONNIE SATURNO, BAE MLOMO-BEATRIZ T.
ABASALA, DATU BALITUNGTUNG-ANTONIO D. LUMANDONG, DATU On October 19, 1998, respondents Secretary of the Department of Environment and
MANTUMUKAW TEOFISTO SABASALES, DATU EDUAARDO BANDA, DATU JOEL Natural Resources (DENR) and Secretary of the Department of Budget and
UNAD, DATU RAMON BAYAAN, TIMUAY JOSE ANOY, TIMUAY MACARIO D. Management (DBM) filed through the Solicitor General a consolidated Comment. The
SALACAO, TIMUAY EDWIN B. ENDING, DATU SAHAMPONG MALANAW VI, DATU Solicitor General is of the view that the IPRA is partly unconstitutional on the ground
BEN PENDAO CABIGON, BAI NANAPNAY-LIZA SAWAY, BAY INAY DAYA- that it grants ownership over natural resources to indigenous peoples and prays that the
MELINDA S. REYMUNDO, BAI TINANGHAGA HELINITA T. PANGAN, DATU petition be granted in part.
MAKAPUKAW ADOLINO L. SAWAY, DATU MAUDAYAW-CRISPEN SAWAY, VICKY
MAKAY, LOURDES D. AMOS, GILBERT P. HOGGANG, TERESA GASPAR, On November 10, 1998, a group of intervenors, composed of Sen. Juan Flavier, one of
MANUEL S. ONALAN, MIA GRACE L. GIRON, ROSEMARIE G. PE, BENITO the authors of the IPRA, Mr. Ponciano Bennagen, a member of the 1986 Constitutional
CARINO, JOSEPH JUDE CARANTES, LYNETTE CARANTES-VIVAL, LANGLEY Commission, and the leaders and members of 112 groups of indigenous peoples
SEGUNDO, SATUR S. BUGNAY, CARLING DOMULOT, ANDRES MENDIOGRIN, (Flavier, et. al), filed their Motion for Leave to Intervene. They join the NCIP in defending
LEOPOLDO ABUGAN, VIRGILIO CAYETANO, CONCHITA G. DESCAGA, LEVY the constitutionality of IPRA and praying for the dismissal of the petition.
ESTEVES, ODETTE G. ESTEVEZ, RODOLFO C. AGUILAR, MAURO VALONES,
PEPE H. ATONG, OFELIA T. DAVI, PERFECTO B. GUINOSAO, WALTER N. TIMOL,
MANUEL T. SELEN, OSCAR DALUNHAY, RICO O. SULATAN, RAFFY MALINDA, On March 22, 1999, the Commission on Human Rights (CHR) likewise filed a Motion to
ALFREDO ABILLANOS, JESSIE ANDILAB, MIRLANDO H. MANGKULINTAS, Intervene and/or to Appear as Amicus Curiae. The CHR asserts that IPRA is an
SAMIE SATURNO, ROMEO A. LINDAHAY, ROEL S. MANSANG-CAGAN, PAQUITO expression of the principle of parens patriae and that the State has the responsibility to
S. LIESES, FILIPE G. SAWAY, HERMINIA S. SAWAY, JULIUS S. SAWAY, protect and guarantee the rights of those who are at a serious disadvantage like
LEONARDA SAWAY, JIMMY UGYUB, SALVADOR TIONGSON, VENANCIO indigenous peoples. For this reason it prays that the petition be dismissed.
APANG, MADION MALID, SUKIM MALID, NENENG MALID, MANGKATADONG
AUGUSTO DIANO, JOSEPHINE M. ALBESO, MORENO MALID, MARIO MANGCAL, On March 23, 1999, another group, composed of the Ikalahan Indigenous People and
FELAY DIAMILING, SALOME P. SARZA, FELIPE P. BAGON, SAMMY the Haribon Foundation for the Conservation of Natural Resources, Inc. (Haribon, et
SALNUNGAN, ANTONIO D. EMBA, NORMA MAPANSAGONOS, ROMEO SALIGA, al.), filed a motion to Intervene with attached Comment-in-Intervention. They agree with
SR., JERSON P. GERADA, RENATO T. BAGON, JR., SARING MASALONG, the NCIP and Flavier, et al. that IPRA is consistent with the Constitution and pray that
SOLEDAD M. GERARDA, ELIZABETH L. MENDI, MORANTE S. TIWAN, DANILO M. the petition for prohibition and mandamus be dismissed.
MALUDAO, MINORS MARICEL MALID, represented by her father CORNELIO
MALID, MARCELINO M. LADRA, represented by her father MONICO D. LADRA, The motions for intervention of the aforesaid groups and organizations were granted.
JENNYLYN MALID, represented by her father TONY MALID, ARIEL M.
EVANGELISTA, represented by her mother LINAY BALBUENA, EDWARD M.
EMUY, SR., SUSAN BOLANIO, OND, PULA BATO B'LAAN TRIBAL FARMER'S Oral arguments were heard on April 13, 1999. Thereafter, the parties and intervenors
ASSOCIATION, INTER-PEOPLE'S EXCHANGE, INC. and GREEN FORUM- filed their respective memoranda in which they reiterate the arguments adduced in their
WESTERN VISAYAS, intervenors. earlier pleadings and during the hearing.
COMMISSION ON HUMAN RIGHTS, intervenor.
IKALAHAN INDIGENOUS PEOPLE and HARIBON FOUNDATION FOR THE Petitioners assail the constitutionality of the following provisions of the IPRA and its
CONSERVATION OF NATURAL RESOURCES, INC., intervenor. Implementing Rules on the ground that they amount to an unlawful deprivation of the
States ownership over lands of the public domain as well as minerals and other natural
resources therein, in violation of the regalian doctrine embodied in Section 2, Article XII Commissioner of the National Development Corporation, the jurisdiction of said
of the Constitution: officials over said area terminates;

"(1) Section 3(a) which defines the extent and coverage of ancestral domains, and "(3) Section 63 which provides the customary law, traditions and practices of
Section 3(b) which, in turn, defines ancestral lands; indigenous peoples shall be applied first with respect to property rights, claims
of ownership, hereditary succession and settlement of land disputes, and that
"(2) Section 5, in relation to section 3(a), which provides that ancestral domains any doubt or ambiguity in the interpretation thereof shall be resolved in favor of
including inalienable public lands, bodies of water, mineral and other resources found the indigenous peoples;
within ancestral domains are private but community property of the indigenous peoples;
"(4) Section 65 which states that customary laws and practices shall be used
"(3) Section 6 in relation to section 3(a) and 3(b) which defines the composition of to resolve disputes involving indigenous peoples; and
ancestral domains and ancestral lands;
"(5) Section 66 which vests on the NCIP the jurisdiction over all claims and
"(4) Section 7 which recognizes and enumerates the rights of the indigenous peoples disputes involving rights of the indigenous peoples." 5
over the ancestral domains;
Finally, petitioners assail the validity of Rule VII, Part II, Section 1 of the NCIP
(5) Section 8 which recognizes and enumerates the rights of the indigenous peoples Administrative Order No. 1, series of 1998, which provides that "the administrative
over the ancestral lands; relationship of the NCIP to the Office of the President is characterized as a lateral but
autonomous relationship for purposes of policy and program coordination." They
contend that said Rule infringes upon the Presidents power of control over executive
"(6) Section 57 which provides for priority rights of the indigenous peoples in the departments under Section 17, Article VII of the Constitution.6
harvesting, extraction, development or exploration of minerals and other natural
resources within the areas claimed to be their ancestral domains, and the right to enter
into agreements with nonindigenous peoples for the development and utilization of Petitioners pray for the following:
natural resources therein for a period not exceeding 25 years, renewable for not more
than 25 years; and "(1) A declaration that Sections 3, 5, 6, 7, 8, 52[I], 57, 58, 59, 63, 65 and 66
and other related provisions of R.A. 8371 are unconstitutional and invalid;
"(7) Section 58 which gives the indigenous peoples the responsibility to maintain,
develop, protect and conserve the ancestral domains and portions thereof which are "(2) The issuance of a writ of prohibition directing the Chairperson and
found to be necessary for critical watersheds, mangroves, wildlife sanctuaries, Commissioners of the NCIP to cease and desist from implementing the
wilderness, protected areas, forest cover or reforestation." 2 assailed provisions of R.A. 8371 and its Implementing Rules;

Petitioners also content that, by providing for an all-encompassing definition of "(3) The issuance of a writ of prohibition directing the Secretary of the
"ancestral domains" and "ancestral lands" which might even include private lands found Department of Environment and Natural Resources to cease and desist from
within said areas, Sections 3(a) and 3(b) violate the rights of private landowners. 3 implementing Department of Environment and Natural Resources Circular No.
2, series of 1998;
In addition, petitioners question the provisions of the IPRA defining the powers and
jurisdiction of the NCIP and making customary law applicable to the settlement of "(4) The issuance of a writ of prohibition directing the Secretary of Budget and
disputes involving ancestral domains and ancestral lands on the ground that these Management to cease and desist from disbursing public funds for the
provisions violate the due process clause of the Constitution.4 implementation of the assailed provisions of R.A. 8371; and

These provisions are: "(5) The issuance of a writ of mandamus commanding the Secretary of
Environment and Natural Resources to comply with his duty of carrying out the
"(1) sections 51 to 53 and 59 which detail the process of delineation and States constitutional mandate to control and supervise the exploration,
recognition of ancestral domains and which vest on the NCIP the sole authority development, utilization and conservation of Philippine natural resources." 7
to delineate ancestral domains and ancestral lands;
After due deliberation on the petition, the members of the Court voted as follows:
"(2) Section 52[i] which provides that upon certification by the NCIP that a
particular area is an ancestral domain and upon notification to the following Seven (7) voted to dismiss the petition. Justice Kapunan filed an opinion, which the
officials, namely, the Secretary of Environment and Natural Resources, Chief Justice and Justices Bellosillo, Quisumbing, and Santiago join, sustaining the
Secretary of Interior and Local Governments, Secretary of Justice and validity of the challenged provisions of R.A. 8371. Justice Puno also filed a separate
opinion sustaining all challenged provisions of the law with the exception of Section 1, THE SECRETARY OF THE DEPARTMENT OF ENVIRONMENT AND NATURAL
Part II, Rule III of NCIP Administrative Order No. 1, series of 1998, the Rules and RESOURCES, THE REGIONAL EXECUTIVE DIRECTOR, DENR-REGION VI,
Regulations Implementing the IPRA, and Section 57 of the IPRA which he contends REGIONAL TECHNICAL DIRECTOR FOR LANDS, LANDS MANAGEMENT
should be interpreted as dealing with the large-scale exploitation of natural resources BUREAU, REGION VI PROVINCIAL ENVIRONMENT AND NATURAL RESOURCES
and should be read in conjunction with Section 2, Article XII of the 1987 Constitution. OFFICER OF KALIBO, AKLAN, REGISTER OF DEEDS, DIRECTOR OF LAND
On the other hand, Justice Mendoza voted to dismiss the petition solely on the ground REGISTRATION AUTHORITY, DEPARTMENT OF TOURISM SECRETARY,
that it does not raise a justiciable controversy and petitioners do not have standing to DIRECTOR OF PHILIPPINE TOURISM AUTHORITY, Petitioners, v. MAYOR JOSE S.
question the constitutionality of R.A. 8371. YAP, LIBERTAD TALAPIAN, MILA Y. SUMNDAD, and ANICETO YAP, in their
behalf and in behalf of all those similarly situated, Respondents.
Seven (7) other members of the Court voted to grant the petition. Justice Panganiban
filed a separate opinion expressing the view that Sections 3 (a)(b), 5, 6, 7 (a)(b), 8, and [G.R. NO. 173775 : October 8, 2008]
related provisions of R.A. 8371 are unconstitutional. He reserves judgment on the
constitutionality of Sections 58, 59, 65, and 66 of the law, which he believes must await DR. ORLANDO SACAY and WILFREDO GELITO, joined by THE LANDOWNERS OF
the filing of specific cases by those whose rights may have been violated by the IPRA. BORACAY SIMILARLY SITUATED NAMED IN A LIST, ANNEX "A" OF THIS
Justice Vitug also filed a separate opinion expressing the view that Sections 3(a), 7, and PETITION, Petitioners, v. THE SECRETARY OF THE DEPARTMENT OF
57 of R.A. 8371 are unconstitutional. Justices Melo, Pardo, Buena, Gonzaga-Reyes, ENVIRONMENT AND NATURAL RESOURCES, THE REGIONAL TECHNICAL
and De Leon join in the separate opinions of Justices Panganiban and Vitug. DIRECTOR FOR LANDS, LANDS MANAGEMENT BUREAU, REGION VI,
PROVINCIAL ENVIRONMENT AND NATURAL RESOURCES OFFICER, KALIBO,
As the votes were equally divided (7 to 7) and the necessary majority was not obtained, AKLAN, Respondents.
the case was redeliberated upon. However, after redeliberation, the voting remained the
same. Accordingly, pursuant to Rule 56, Section 7 of the Rules of Civil Procedure, the DECISION
petition is DISMISSED.
REYES, R.T., J.:
Attached hereto and made integral parts thereof are the separate opinions of Justices
Puno, Vitug, Kapunan, Mendoza, and Panganiban. AT stake in these consolidated cases is the right of the present occupants of Boracay
Island to secure titles over their occupied lands.
SO ORDERED.
There are two consolidated petitions. The first is G.R. No. 167707, a Petition for Review
on Certiorari of the Decision1 of the Court of Appeals (CA) affirming that2 of the
Regional Trial Court (RTC) in Kalibo, Aklan, which granted the petition for declaratory
relief filed by respondents-claimants Mayor Jose Yap, et al. and ordered the survey of
Boracay for titling purposes. The second is G.R. No. 173775, a petition for
prohibition, mandamus, and nullification of Proclamation No. 1064 5">[3] issued by
President Gloria Macapagal-Arroyo classifying Boracay into reserved forest and
agricultural land.

The Antecedents

G.R. No. 167707

Boracay Island in the Municipality of Malay, Aklan, with its powdery white sand beaches
and warm crystalline waters, is reputedly a premier Philippine tourist destination. The
island is also home to 12,003 inhabitants4 who live in the bone-shaped island's
three barangays.5

On April 14, 1976, the Department of Environment and Natural Resources (DENR)
approved the National Reservation Survey of Boracay Island,6 which identified several
lots as being occupied or claimed by named persons.7

On November 10, 1978, then President Ferdinand Marcos issued Proclamation


EN BANC No. 18018 declaring Boracay Island, among other islands, caves and peninsulas in the
Philippines, as tourist zones and marine reserves under the administration of the
[G.R. NO. 167707 : October 8, 2008]
Philippine Tourism Authority (PTA). President Marcos later approved the issuance RTC and CA Dispositions
of PTA Circular 3-829 dated September 3, 1982, to implement Proclamation No. 1801.
On July 14, 1999, the RTC rendered a decision in favor of respondents-claimants, with
Claiming that Proclamation No. 1801 and PTA Circular No 3-82 precluded them from a fallo reading:
filing an application for judicial confirmation of imperfect title or survey of land for titling
purposes, respondents-claimants WHEREFORE, in view of the foregoing, the Court declares that Proclamation No. 1801
Mayor Jose S. Yap, Jr., Libertad Talapian, Mila Y. Sumndad, and Aniceto Yap filed a and PTA Circular No. 3-82 pose no legal obstacle to the petitioners and those similarly
petition for declaratory relief with the RTC in Kalibo, Aklan.
situated to acquire title to their lands in Boracay, in accordance with the applicable laws
and in the manner prescribed therein; and to have their lands surveyed and approved
In their petition, respondents-claimants alleged that Proclamation No. 1801 and PTA
Circular No. 3-82 raised doubts on their right to secure titles over their occupied lands. by respondent Regional Technical Director of Lands as the approved survey does not in
They declared that they themselves, or through their predecessors-in-interest, had been itself constitute a title to the land.
in open, continuous, exclusive, and notorious possession and occupation in Boracay
since June 12, 1945, or earlier since time immemorial. They declared their lands for tax SO ORDERED.17
purposes and paid realty taxes on them.10
The RTC upheld respondents-claimants' right to have their occupied lands titled in their
Respondents-claimants posited that Proclamation No. 1801 and its implementing name. It ruled that neither Proclamation No. 1801 nor PTA Circular No. 3-82 mentioned
Circular did not place Boracay beyond the commerce of man. Since the Island was that lands in Boracay were inalienable or could not be the subject of disposition.18 The
classified as a tourist zone, it was susceptible of private ownership. Under Section 48(b) Circular itself recognized private ownership of lands. 19 The trial court cited Sections
of Commonwealth Act (CA) No. 141, otherwise known as the Public Land Act, they had 8720 and 5321 of the Public Land Act as basis for acknowledging private ownership of
the right to have the lots registered in their names through judicial confirmation of lands in Boracay and that only those forested areas in public lands were declared as
imperfect titles. part of the forest reserve.22

The Republic, through the Office of the Solicitor General (OSG), opposed the petition The OSG moved for reconsideration but its motion was denied. 23 The Republic then
for declaratory relief. The OSG countered that Boracay Island was an unclassified appealed to the CA.
land of the public domain. It formed part of the mass of lands classified as "public
forest," which was not available for disposition pursuant to Section 3(a) of Presidential On December 9, 2004, the appellate court affirmed in toto the RTC decision, disposing
Decree (PD) No. 705 or the Revised Forestry Code,11 as amended. as follows:

The OSG maintained that respondents-claimants' reliance on PD No. 1801 and PTA WHEREFORE, in view of the foregoing premises, judgment is hereby rendered by us
Circular No. 3-82 was misplaced. Their right to judicial confirmation of title was
DENYING the appeal filed in this case and AFFIRMING the decision of the lower
governed by CA No. 141 and PD No. 705. Since Boracay Island had not been classified
as alienable and disposable, whatever possession they had cannot ripen into court.24
ownership.
The CA held that respondents-claimants could not be prejudiced by a declaration that
During pre-trial, respondents-claimants and the OSG stipulated on the following facts: the lands they occupied since time immemorial were part of a forest reserve.
(1) respondents-claimants were presently in possession of parcels of land in Boracay
Island; (2) these parcels of land were planted with coconut trees and other natural Again, the OSG sought reconsideration but it was similarly denied. 25 Hence, the present
growing trees; (3) the coconut trees had heights of more or less twenty (20) meters and petition under Rule 45.
were planted more or less fifty (50) years ago; and (4) respondents-claimants declared
the land they were occupying for tax purposes.12

The parties also agreed that the principal issue for resolution was purely legal: whether [G.R. NO. 173775]
Proclamation No. 1801 posed any legal hindrance or impediment to the titling of the
lands in Boracay. They decided to forego with the trial and to submit the case for On May 22, 2006, during the pendency of G.R. No. 167707, President Gloria
resolution upon submission of their respective memoranda.13 Macapagal-Arroyo issued Proclamation No. 106426 classifying Boracay Island into four
hundred (400) hectares of reserved forest land (protection purposes) and six hundred
The RTC took judicial notice14 that certain parcels of land in Boracay Island, more twenty-eight and 96/100 (628.96) hectares of agricultural land (alienable and
particularly Lots 1 and 30, Plan PSU-5344, were covered by Original Certificate of Title disposable). The Proclamation likewise provided for a fifteen-meter buffer zone on each
No. 19502 (RO 2222) in the name of the Heirs of Ciriaco S. Tirol. These lots were side of the centerline of roads and trails, reserved for right-of-way and which shall form
involved in Civil Case Nos. 5222 and 5262 filed before the RTC of Kalibo, Aklan. 15 The part of the area reserved for forest land protection purposes.
titles were issued on August 7, 1933.16
On August 10, 2006, petitioners-claimants Dr. Orlando Sacay,27 Wilfredo Gelito,28 and HAVE PETITIONERS OCCUPANTS ACQUIRED PRIOR VESTED RIGHT OF
other landowners29 in Boracay filed with this Court an original petition for PRIVATE OWNERSHIP OVER THEIR OCCUPIED PORTIONS OF BORACAY LAND,
prohibition, mandamus, and nullification of Proclamation No. 1064.30 They allege that DESPITE THE FACT THAT THEY HAVE NOT APPLIED YET FOR JUDICIAL
the Proclamation infringed on their "prior vested rights" over portions of Boracay. They
CONFIRMATION OF IMPERFECT TITLE?
have been in continued possession of their respective lots in Boracay since time
immemorial. They have also invested billions of pesos in developing their lands and
building internationally renowned first class resorts on their lots.31 III.

Petitioners-claimants contended that there is no need for a proclamation reclassifying IS THE EXECUTIVE DECLARATION OF THEIR AREAS AS ALIENABLE AND
Boracay into agricultural land. Being classified as neither mineral nor timber land, the DISPOSABLE UNDER SEC 6, CA 141 [AN] INDISPENSABLE PRE-REQUISITE FOR
island is deemed agricultural pursuant to the Philippine Bill of 1902 and Act No. 926, PETITIONERS TO OBTAIN TITLE UNDER THE TORRENS SYSTEM?
known as the first Public Land Act.32 Thus, their possession in the concept of owner for
the required period entitled them to judicial confirmation of imperfect title.
IV.
Opposing the petition, the OSG argued that petitioners-claimants do not have a vested
right over their occupied portions in the island. Boracay is an unclassified public forest IS THE ISSUANCE OF PROCLAMATION 1064 ON MAY 22, 2006, VIOLATIVE OF
land pursuant to Section 3(a) of PD No. 705. Being public forest, the claimed portions of THE PRIOR VESTED RIGHTS TO PRIVATE OWNERSHIP OF PETITIONERS OVER
the island are inalienable and cannot be the subject of judicial confirmation of imperfect THEIR LANDS IN BORACAY, PROTECTED BY THE DUE PROCESS CLAUSE OF
title. It is only the executive department, not the courts, which has authority to reclassify THE CONSTITUTION OR IS PROCLAMATION 1064 CONTRARY TO SEC. 8, CA 141,
lands of the public domain into alienable and disposable lands. There is a need for a OR SEC. 4(a) OF RA 6657.
positive government act in order to release the lots for disposition.
V.
On November 21, 2006, this Court ordered the consolidation of the two petitions as they
principally involve the same issues on the land classification of Boracay Island. 33
CAN RESPONDENTS BE COMPELLED BY MANDAMUS TO ALLOW THE SURVEY
Issues AND TO APPROVE THE SURVEY PLANS FOR PURPOSES OF THE APPLICATION
FOR TITLING OF THE LANDS OF PETITIONERS IN BORACAY?35 (Underscoring
[G.R. No. 167707] supplied)cralawlibrary

The OSG raises the lone issue of whether Proclamation No. 1801 and PTA Circular No. In capsule, the main issue is whether private claimants (respondents-claimants in G.R.
3-82 pose any legal obstacle for respondents, and all those similarly situated, to acquire No. 167707 and petitioners-claimants in G.R. No. 173775) have a right to secure titles
title to their occupied lands in Boracay Island.34 over their occupied portions in Boracay. The twin petitions pertain to their right, if any, to
judicial confirmation of imperfect title under CA No. 141, as amended. They do not
involve their right to secure title under other pertinent laws.

[G.R. NO. 173775] Our Ruling

Petitioners-claimants hoist five (5) issues, namely: Regalian Doctrine and power of the executive to reclassify lands of the public
domain
I.
Private claimants rely on three (3) laws and executive acts in their bid for judicial
confirmation of imperfect title, namely: (a) Philippine Bill of 1902 36in relation to Act No.
AT THE TIME OF THE ESTABLISHED POSSESSION OF PETITIONERS IN
926, later amended and/or superseded by Act No. 2874 and CA No. 141;37 (b)
CONCEPT OF OWNER OVER THEIR RESPECTIVE AREAS IN BORACAY, SINCE Proclamation No. 180138 issued by then President Marcos; and (c) Proclamation No.
TIME IMMEMORIAL OR AT THE LATEST SINCE 30 YRS. PRIOR TO THE FILING OF 106439 issued by President Gloria Macapagal-Arroyo. We shall proceed to determine
THE PETITION FOR DECLARATORY RELIEF ON NOV. 19, 1997, WERE THE their rights to apply for judicial confirmation of imperfect title under these laws and
AREAS OCCUPIED BY THEM PUBLIC AGRICULTURAL LANDS AS DEFINED BY executive acts.
LAWS THEN ON JUDICIAL CONFIRMATION OF IMPERFECT TITLES OR PUBLIC
FOREST AS DEFINED BY SEC. 3a, PD 705? But first, a peek at the Regalian principle and the power of the executive to reclassify
lands of the public domain.
II.
The 1935 Constitution classified lands of the public domain into agricultural, forest or
timber.40 Meanwhile, the 1973 Constitution provided the following divisions: agricultural,
industrial or commercial, residential, resettlement, mineral, timber or forest and grazing domain in the Philippine Islands were classified into three (3) grand divisions, to wit:
lands, and such other classes as may be provided by law,41 giving the government agricultural, mineral, and timber or forest lands.61 The act provided for, among others,
great leeway for classification.42 Then the 1987 Constitution reverted to the 1935 the disposal of mineral lands by means of absolute grant (freehold system) and by lease
Constitution classification with one addition: national parks. 43 Of these, only agricultural (leasehold system).62 It also provided the definition by exclusion of "agricultural public
lands may be alienated.44 Prior to Proclamation No. 1064 of May 22, 2006, Boracay lands."63 Interpreting the meaning of "agricultural lands" under the Philippine Bill of
Island had never been expressly and administratively classified under any of these 1902, the Court declared in Mapa v. Insular Government:64
grand divisions. Boracay was an unclassified land of the public domain.
x x x In other words, that the phrase "agricultural land" as used in Act No. 926 means
The Regalian Doctrine dictates that all lands of the public domain belong to the State, those public lands acquired from Spain which are not timber or mineral lands. x x
that the State is the source of any asserted right to ownership of land and charged with x65 (Emphasis Ours)
the conservation of such patrimony.45 The doctrine has been consistently adopted
under the 1935, 1973, and 1987 Constitutions.46
On February 1, 1903, the Philippine Legislature passed Act No. 496, otherwise known
as the Land Registration Act. The act established a system of registration by which
All lands not otherwise appearing to be clearly within private ownership are presumed to
recorded title becomes absolute, indefeasible, and imprescriptible. This is known as the
belong to the State.47 Thus, all lands that have not been acquired from the government,
Torrens system.66
either by purchase or by grant, belong to the State as part of the inalienable public
domain.48 Necessarily, it is up to the State to determine if lands of the public domain will
be disposed of for private ownership. The government, as the agent of the state, is Concurrently, on October 7, 1903, the Philippine Commission passed Act
possessed of the plenary power as the persona in law to determine who shall be the No. 926, which was the first Public Land Act. The Act introduced the homestead system
favored recipients of public lands, as well as under what terms they may be granted and made provisions for judicial and administrative confirmation of imperfect titles and
such privilege, not excluding the placing of obstacles in the way of their exercise of what for the sale or lease of public lands. It permitted corporations regardless of the
otherwise would be ordinary acts of ownership.49 nationality of persons owning the controlling stock to lease or purchase lands of the
public domain.67 Under the Act, open, continuous, exclusive, and notorious possession
and occupation of agricultural lands for the next ten (10) years preceding July 26, 1904
Our present land law traces its roots to the Regalian Doctrine. Upon the Spanish
was sufficient for judicial confirmation of imperfect title. 68
conquest of the Philippines, ownership of all lands, territories and possessions in the
Philippines passed to the Spanish Crown.50 The Regalian doctrine was first introduced
in the Philippines through the Laws of the Indies and the Royal Cedulas, which laid the On November 29, 1919, Act No. 926 was superseded by Act No. 2874, otherwise
foundation that "all lands that were not acquired from the Government, either by known as the second Public Land Act. This new, more comprehensive law limited the
purchase or by grant, belong to the public domain." 51 exploitation of agricultural lands to Filipinos and Americans and citizens of other
countries which gave Filipinos the same privileges. For judicial confirmation of title,
possession and occupation en concepto dueo since time immemorial, or since July
The Laws of the Indies was followed by the Ley Hipotecaria or the Mortgage Law of
26, 1894, was required.69
1893. The Spanish Mortgage Law provided for the systematic registration of titles and
deeds as well as possessory claims.52
After the passage of the 1935 Constitution, CA No. 141 amended Act No. 2874
on December 1, 1936. To this day, CA No. 141, as amended, remains as the existing
The Royal Decree of 1894 or the Maura Law53 partly amended the Spanish Mortgage
general law governing the classification and disposition of lands of the public domain
Law and the Laws of the Indies. It established possessory information as the method of
other than timber and mineral lands,70and privately owned lands which reverted to the
legalizing possession of vacant Crown land, under certain conditions which were set
State.71
forth in said decree.54 Under Section 393 of the Maura Law, an informacion
posesoria or possessory information title,55 when duly inscribed in the Registry of
Property, is converted into a title of ownership only after the lapse of twenty (20) years Section 48(b) of CA No. 141 retained the requirement under Act No. 2874 of possession
of uninterrupted possession which must be actual, public, and adverse,56 from the date and occupation of lands of the public domain since time immemorial or since July 26,
of its inscription.57 However, possessory information title had to be perfected one year 1894. However, this provision was superseded by Republic Act (RA) No. 1942, 72 which
after the promulgation of the Maura Law, or until April 17, 1895. Otherwise, the lands provided for a simple thirty-year prescriptive period for judicial confirmation of imperfect
would revert to the State.58 title. The provision was last amended by PD No. 1073,73 which now provides for
possession and occupation of the land applied for since June 12, 1945, or earlier.74
In sum, private ownership of land under the Spanish regime could only be founded on
royal concessions which took various forms, namely: (1) titulo real or royal grant; The issuance of PD No. 89275 on February 16, 1976 discontinued the use of Spanish
(2) concesion especial or special grant; (3) composicion con el estado or adjustment titles as evidence in land registration proceedings.76 Under the decree, all holders of
title; (4) titulo de compra or title by purchase; and (5) informacion posesoria or Spanish titles or grants should apply for registration of their lands under Act No. 496
possessory information title.59 within six (6) months from the effectivity of the decree on February 16, 1976. Thereafter,
the recording of all unregistered lands77 shall be governed by Section 194 of the
Revised Administrative Code, as amended by Act No. 3344.
The first law governing the disposition of public lands in the Philippines under American
rule was embodied in the Philippine Bill of 1902.60 By this law, lands of the public
On June 11, 1978, Act No. 496 was amended and updated by PD No. 1529, known as Spouses Pedro S. Palanca and Soterranea Rafols Vda. De Palanca v. Republic,92 in
the Property Registration Decree. It was enacted to codify the various laws relative to which it stated, through Justice Adolfo Azcuna, viz.:
registration of property.78 It governs registration of lands under the Torrens system as
well as unregistered lands, including chattel mortgages. 79 x x x Petitioners furthermore insist that a particular land need not be formally released
by an act of the Executive before it can be deemed open to private ownership, citing the
A positive act declaring land as alienable and disposable is required. In keeping cases of Ramos v. Director of Lands and Ankron v. Government of the Philippine
with the presumption of State ownership, the Court has time and again emphasized that
Islands.
there must be a positive act of the government, such as an official
proclamation,80 declassifying inalienable public land into disposable land for agricultural
or other purposes.81 In fact, Section 8 of CA No. 141 limits alienable or disposable lands x x x
only to those lands which have been "officially delimited and classified." 82
Petitioner's reliance upon Ramos v. Director of Lands and Ankron v. Government is
The burden of proof in overcoming the presumption of State ownership of the lands of misplaced. These cases were decided under the Philippine Bill of 1902 and the first
the public domain is on the person applying for registration (or claiming ownership), who Public Land Act No. 926 enacted by the Philippine Commission on October 7, 1926,
must prove that the land subject of the application is alienable or disposable.83 To under which there was no legal provision vesting in the Chief Executive or President of
overcome this presumption, incontrovertible evidence must be established that the land
the Philippines the power to classify lands of the public domain into mineral, timber and
subject of the application (or claim) is alienable or disposable. 84 There must still be a
positive act declaring land of the public domain as alienable and disposable. To prove agricultural so that the courts then were free to make corresponding classifications in
that the land subject of an application for registration is alienable, the applicant must justiciable cases, or were vested with implicit power to do so, depending upon the
establish the existence of a positive act of the government such as a presidential preponderance of the evidence.93
proclamation or an executive order; an administrative action; investigation reports of
Bureau of Lands investigators; and a legislative act or a statute. 85 The applicant may To aid the courts in resolving land registration cases under Act No. 926, it was then
also secure a certification from the government that the land claimed to have been necessary to devise a presumption on land classification. Thus evolved the dictum
possessed for the required number of years is alienable and disposable. 86 in Ankron that "the courts have a right to presume, in the absence of evidence to the
contrary, that in each case the lands are agricultural lands until the contrary is shown." 94
In the case at bar, no such proclamation, executive order, administrative action, report,
statute, or certification was presented to the Court. The records are bereft of evidence But We cannot unduly expand the presumption in Ankron and De Aldecoa to an
showing that, prior to 2006, the portions of Boracay occupied by private claimants were argument that all lands of the public domain had been automatically reclassified as
subject of a government proclamation that the land is alienable and disposable. Absent disposable and alienable agricultural lands. By no stretch of imagination did the
such well-nigh incontrovertible evidence, the Court cannot accept the submission that presumption convert all lands of the public domain into agricultural lands.
lands occupied by private claimants were already open to disposition before 2006.
Matters of land classification or reclassification cannot be assumed. They call for If We accept the position of private claimants, the Philippine Bill of 1902 and Act No.
proof.87 926 would have automatically made all lands in the Philippines, except those already
classified as timber or mineral land, alienable and disposable lands. That would take
Ankron and De Aldecoa did not make the whole of Boracay Island, or portions of these lands out of State ownership and worse, would be utterly inconsistent with and
it, agricultural lands. Private claimants posit that Boracay was already an agricultural totally repugnant to the long-entrenched Regalian doctrine.
land pursuant to the old cases Ankron v. Government of the Philippine Islands
(1919)88 and De Aldecoa v. The Insular Government (1909).89 These cases were The presumption in Ankron and De Aldecoa attaches only to land registration cases
decided under the provisions of the Philippine Bill of 1902 and Act No. 926. There is a brought under the provisions of Act No. 926, or more specifically those cases dealing
statement in these old cases that "in the absence of evidence to the contrary, that in with judicial and administrative confirmation of imperfect titles. The presumption applies
each case the lands are agricultural lands until the contrary is shown." 90 to an applicant for judicial or administrative conformation of imperfect title under Act No.
926. It certainly cannot apply to landowners, such as private claimants or their
Private claimants' reliance on Ankron and De Aldecoa is misplaced. These cases did predecessors-in-interest, who failed to avail themselves of the benefits of Act No. 926.
not have the effect of converting the whole of Boracay Island or portions of it into As to them, their land remained unclassified and, by virtue of the Regalian doctrine,
agricultural lands. It should be stressed that the Philippine Bill of 1902 and Act No. 926 continued to be owned by the State.
merely provided the manner through which land registration courts would classify lands
of the public domain. Whether the land would be classified as timber, mineral, or In any case, the assumption in Ankron and De Aldecoa was not absolute. Land
agricultural depended on proof presented in each case. classification was, in the end, dependent on proof. If there was proof that the land was
better suited for non-agricultural uses, the courts could adjudge it as a mineral or timber
Ankron and De Aldecoa were decided at a time when the President of the Philippines land despite the presumption. In Ankron, this Court stated:
had no power to classify lands of the public domain into mineral, timber, and
agricultural. At that time, the courts were free to make corresponding classifications in In the case of Jocson v. Director of Forestry (supra), the Attorney-General admitted in
justiciable cases, or were vested with implicit power to do so, depending upon the
preponderance of the evidence.91 This was the Court's ruling in Heirs of the Late effect that whether the particular land in question belongs to one class or another is a
question of fact. The mere fact that a tract of land has trees upon it or has mineral within We note that the RTC decision99 in G.R. No. 167707 mentioned Krivenko v. Register of
it is not of itself sufficient to declare that one is forestry land and the other, mineral land. Deeds of Manila,100 which was decided in 1947 when CA No. 141, vesting the Executive
There must be some proof of the extent and present or future value of the forestry and with the sole power to classify lands of the public domain was already in
effect. Krivenko cited the old cases Mapa v. Insular Government,101 De Aldecoa v. The
of the minerals. While, as we have just said, many definitions have been given for
Insular Government,102 and Ankron v. Government of the Philippine Islands.103
"agriculture," "forestry," and "mineral" lands, and that in each case it is a question of
fact, we think it is safe to say that in order to be forestry or mineral land the proof must Krivenko, however, is not controlling here because it involved a totally different issue.
show that it is more valuable for the forestry or the mineral which it contains than it is for The pertinent issue in Krivenko was whether residential lots were included in the
agricultural purposes. (Sec. 7, Act No. 1148.) It is not sufficient to show that there exists general classification of agricultural lands; and if so, whether an alien could acquire a
some trees upon the land or that it bears some mineral. Land may be classified as residential lot. This Court ruled that as an alien, Krivenko was prohibited by the 1935
forestry or mineral today, and, by reason of the exhaustion of the timber or mineral, be Constitution104 from acquiring agricultural land, which included residential lots. Here, the
classified as agricultural land tomorrow. And vice-versa, by reason of the rapid growth issue is whether unclassified lands of the public domain are automatically deemed
agricultural.
of timber or the discovery of valuable minerals, lands classified as agricultural today
may be differently classified tomorrow. Each case must be decided upon the proof in
Notably, the definition of "agricultural public lands" mentioned in Krivenko relied on the
that particular case, having regard for its present or future value for one or the old cases decided prior to the enactment of Act No. 2874, including Ankron and De
other purposes. We believe, however, considering the fact that it is a matter of public Aldecoa.105 As We have already stated, those cases cannot apply here, since they were
knowledge that a majority of the lands in the Philippine Islands are agricultural lands decided when the Executive did not have the authority to classify lands as agricultural,
that the courts have a right to presume, in the absence of evidence to the contrary, that timber, or mineral.
in each case the lands are agricultural lands until the contrary is shown. Whatever the
land involved in a particular land registration case is forestry or mineral land Private claimants' continued possession under Act No. 926 does not create a
must, therefore, be a matter of proof. Its superior value for one purpose or the presumption that the land is alienable. Private claimants also contend that their
continued possession of portions of Boracay Island for the requisite period of ten (10)
other is a question of fact to be settled by the proof in each particular case. The
years under Act No. 926106 ipso facto converted the island into private ownership.
fact that the land is a manglar [mangrove swamp] is not sufficient for the courts to Hence, they may apply for a title in their name.
decide whether it is agricultural, forestry, or mineral land. It may perchance belong to
one or the other of said classes of land. The Government, in the first instance, under the A similar argument was squarely rejected by the Court in Collado v. Court of
provisions of Act No. 1148, may, by reservation, decide for itself what portions of public Appeals.107 Collado, citing the separate opinion of now Chief Justice Reynato S. Puno
land shall be considered forestry land, unless private interests have intervened before in Cruz v. Secretary of Environment and Natural Resources,107-a ruled:
such reservation is made. In the latter case, whether the land is agricultural, forestry, or
mineral, is a question of proof. Until private interests have intervened, the Government, "Act No. 926, the first Public Land Act, was passed in pursuance of the provisions of the
by virtue of the terms of said Act (No. 1148), may decide for itself what portions of the Philippine Bill of 1902. The law governed the disposition of lands of the public domain. It
"public domain" shall be set aside and reserved as forestry or mineral land. (Ramos v. prescribed rules and regulations for the homesteading, selling and leasing of portions of
Director of Lands, 39 Phil. 175; Jocson v. Director of Forestry, supra)95 (Emphasis ours) the public domain of the Philippine Islands, and prescribed the terms and conditions to
enable persons to perfect their titles to public lands in the Islands. It also provided for
Since 1919, courts were no longer free to determine the classification of lands from the the "issuance of patents to certain native settlers upon public lands," for the
facts of each case, except those that have already became private lands.96 Act establishment of town sites and sale of lots therein, for the completion of imperfect titles,
No. 2874, promulgated in 1919 and reproduced in Section 6 of CA No. 141, gave the and for the cancellation or confirmation of Spanish concessions and grants in the
Executive Department, through the President, the exclusive prerogative to classify or Islands." In short, the Public Land Act operated on the assumption that title to public
reclassify public lands into alienable or disposable, mineral or forest. 96-a Since then, lands in the Philippine Islands remained in the government; and that the government's
courts no longer had the authority, whether express or implied, to determine the
title to public land sprung from the Treaty of Paris and other subsequent treaties
classification of lands of the public domain.97
between Spain and the United States. The term "public land" referred to all lands of the
Here, private claimants, unlike the Heirs of Ciriaco Tirol who were issued their title in public domain whose title still remained in the government and are thrown open to
1933,98 did not present a justiciable case for determination by the land registration court private appropriation and settlement, and excluded the patrimonial property of the
of the property's land classification. Simply put, there was no opportunity for the courts government and the friar lands."
then to resolve if the land the Boracay occupants are now claiming were agricultural
lands. When Act No. 926 was supplanted by Act No. 2874 in 1919, without an Thus, it is plain error for petitioners to argue that under the Philippine Bill of 1902
application for judicial confirmation having been filed by private claimants or their and Public Land Act No. 926, mere possession by private individuals of lands
predecessors-in-interest, the courts were no longer authorized to determine the
creates the legal presumption that the lands are alienable and
property's land classification. Hence, private claimants cannot bank on Act No. 926.
disposable.108 (Emphasis Ours)
Except for lands already covered by existing titles, Boracay was an unclassified Private claimants cannot rely on Proclamation No. 1801 as basis for judicial
land of the public domain prior to Proclamation No. 1064. Such unclassified lands confirmation of imperfect title. The proclamation did not convert Boracay into an
are considered public forest under PD No. 705. The DENR109 and the National agricultural land. However, private claimants argue that Proclamation No. 1801 issued
Mapping and Resource Information Authority110 certify that Boracay Island is an by then President Marcos in 1978 entitles them to judicial confirmation of imperfect title.
unclassified land of the public domain. The Proclamation classified Boracay, among other islands, as a tourist zone. Private
claimants assert that, as a tourist spot, the island is susceptible of private ownership.
PD No. 705 issued by President Marcos categorized all unclassified lands of the public
domain as public forest. Section 3(a) of PD No. 705 defines a public forest as "a mass Proclamation No. 1801 or PTA Circular No. 3-82 did not convert the whole of Boracay
of lands of the public domain which has not been the subject of the present system of into an agricultural land. There is nothing in the law or the Circular which made Boracay
classification for the determination of which lands are needed for forest purpose and Island an agricultural land. The reference in Circular No. 3-82 to "private lands"117 and
which are not." Applying PD No. 705, all unclassified lands, including those in Boracay "areas declared as alienable and disposable" 118 does not by itself classify the entire
Island, are ipso facto considered public forests. PD No. 705, however, respects titles island as agricultural. Notably, Circular No. 3-82 makes reference not only to private
already existing prior to its effectivity. lands and areas but also to public forested lands. Rule VIII, Section 3 provides:

The Court notes that the classification of Boracay as a forest land under PD No. 705 No trees in forested private lands may be cut without prior authority from the PTA. All
may seem to be out of touch with the present realities in the island. Boracay, no doubt, forested areas in public lands are declared forest reserves. (Emphasis
has been partly stripped of its forest cover to pave the way for commercial supplied)cralawlibrary
developments. As a premier tourist destination for local and foreign tourists, Boracay
appears more of a commercial island resort, rather than a forest land.
Clearly, the reference in the Circular to both private and public lands merely recognizes
that the island can be classified by the Executive department pursuant to its powers
Nevertheless, that the occupants of Boracay have built multi-million peso beach resorts
under CA No. 141. In fact, Section 5 of the Circular recognizes the then Bureau of
on the island;111 that the island has already been stripped of its forest cover; or that the
Forest Development's authority to declare areas in the island as alienable and
implementation of Proclamation No. 1064 will destroy the island's tourism industry,
disposable when it provides:
do not negate its character as public forest.
Subsistence farming, in areas declared as alienable and disposable by the Bureau of
Forests, in the context of both the Public Land Act and the Constitution 112 classifying
lands of the public domain into "agricultural, forest or timber, mineral lands, and national Forest Development.
parks," do not necessarily refer to large tracts of wooded land or expanses covered by
dense growths of trees and underbrushes.113 The discussion in Heirs of Amunategui v. Therefore, Proclamation No. 1801 cannot be deemed the positive act needed to classify
Director of Forestry114 is particularly instructive: Boracay Island as alienable and disposable land. If President Marcos intended to
classify the island as alienable and disposable or forest, or both, he would have
A forested area classified as forest land of the public domain does not lose such identified the specific limits of each, as President Arroyo did in Proclamation No. 1064.
classification simply because loggers or settlers may have stripped it of its forest cover. This was not done in Proclamation No. 1801.
Parcels of land classified as forest land may actually be covered with grass or planted to
The Whereas clauses of Proclamation No. 1801 also explain the rationale behind the
crops by kaingin cultivators or other farmers. "Forest lands" do not have to be on
declaration of Boracay Island, together with other islands, caves and peninsulas in the
mountains or in out of the way places. Swampy areas covered by mangrove trees, nipa Philippines, as a tourist zone and marine reserve to be administered by the PTA - to
palms, and other trees growing in brackish or sea water may also be classified as forest ensure the concentrated efforts of the public and private sectors in the development of
land. The classification is descriptive of its legal nature or status and does not the areas' tourism potential with due regard for ecological balance in the marine
have to be descriptive of what the land actually looks like. Unless and until the land environment. Simply put, the proclamation is aimed at administering the islands
classified as "forest" is released in an official proclamation to that effect so that it may for tourism and ecological purposes. It does not address the areas' alienability.119
form part of the disposable agricultural lands of the public domain, the rules on
confirmation of imperfect title do not apply.115 (Emphasis supplied)cralawlibrary More importantly, Proclamation No. 1801 covers not only Boracay Island, but sixty-four
(64) other islands, coves, and peninsulas in the Philippines, such as Fortune and Verde
Islands in Batangas, Port Galera in Oriental Mindoro, Panglao and Balicasag Islands in
There is a big difference between "forest" as defined in a dictionary and "forest or timber Bohol, Coron Island, Puerto Princesa and surrounding areas in Palawan, Camiguin
land" as a classification of lands of the public domain as appearing in our statutes. One Island in Cagayan de Oro, and Misamis Oriental, to name a few. If the designation of
is descriptive of what appears on the land while the other is a legal status, a Boracay Island as tourist zone makes it alienable and disposable by virtue of
classification for legal purposes.116 At any rate, the Court is tasked to determine Proclamation No. 1801, all the other areas mentioned would likewise be declared wide
the legal status of Boracay Island, and not look into its physical layout. Hence, even if open for private disposition. That could not have been, and is clearly beyond, the intent
its forest cover has been replaced by beach resorts, restaurants and other commercial of the proclamation.
establishments, it has not been automatically converted from public forest to alienable
agricultural land.
It was Proclamation No. 1064 of 2006 which positively declared part of Boracay as While it is true that the land classification map does not categorically state that
alienable and opened the same to private ownership. Sections 6 and 7 of CA No. the islands are public forests, the fact that they were unclassified lands leads to
141120 provide that it is only the President, upon the recommendation of the proper the same result. In the absence of the classification as mineral or timber land, the land
department head, who has the authority to classify the lands of the public domain into
remains unclassified land until released and rendered open to disposition. 125 (Emphasis
alienable or disposable, timber and mineral lands. 121
supplied)cralawlibrary
In issuing Proclamation No. 1064, President Gloria Macapagal-Arroyo merely exercised
the authority granted to her to classify lands of the public domain, presumably subject to Moreover, the prohibition under the CARL applies only to a "reclassification" of land. If
existing vested rights. Classification of public lands is the exclusive prerogative of the the land had never been previously classified, as in the case of Boracay, there can be
Executive Department, through the Office of the President. Courts have no authority to no prohibited reclassification under the agrarian law. We agree with the opinion of the
do so.122 Absent such classification, the land remains unclassified until released and Department of Justice126 on this point:
rendered open to disposition.123
Indeed, the key word to the correct application of the prohibition in Section 4(a) is the
Proclamation No. 1064 classifies Boracay into 400 hectares of reserved forest land and word "reclassification." Where there has been no previous classification of public forest
628.96 hectares of agricultural land. The Proclamation likewise provides for a 15-meter [referring, we repeat, to the mass of the public domain which has not been the subject
buffer zone on each side of the center line of roads and trails, which are reserved for of the present system of classification for purposes of determining which are needed for
right of way and which shall form part of the area reserved for forest land protection forest purposes and which are not] into permanent forest or forest reserves or some
purposes.
other forest uses under the Revised Forestry Code, there can be no "reclassification of
forest lands" to speak of within the meaning of Section 4(a).
Contrary to private claimants' argument, there was nothing invalid or irregular, much
less unconstitutional, about the classification of Boracay Island made by the President
through Proclamation No. 1064. It was within her authority to make such classification, Thus, obviously, the prohibition in Section 4(a) of the CARL against the reclassification
subject to existing vested rights. of forest lands to agricultural lands without a prior law delimiting the limits of the public
domain, does not, and cannot, apply to those lands of the public domain, denominated
Proclamation No. 1064 does not violate the Comprehensive Agrarian Reform as "public forest" under the Revised Forestry Code, which have not been previously
Law. Private claimants further assert that Proclamation No. 1064 violates the provision determined, or classified, as needed for forest purposes in accordance with the
of the Comprehensive Agrarian Reform Law (CARL) or RA No. 6657 barring conversion provisions of the Revised Forestry Code.127
of public forests into agricultural lands. They claim that since Boracay is a public forest
under PD No. 705, President Arroyo can no longer convert it into an agricultural land
without running afoul of Section 4(a) of RA No. 6657, thus: Private claimants are not entitled to apply for judicial confirmation of imperfect
title under CA No. 141. Neither do they have vested rights over the occupied
lands under the said law. There are two requisites for judicial confirmation of imperfect
SEC. 4. Scope. - The Comprehensive Agrarian Reform Law of 1988 shall cover,
or incomplete title under CA No. 141, namely: (1) open, continuous, exclusive, and
regardless of tenurial arrangement and commodity produced, all public and private notorious possession and occupation of the subject land by himself or through his
agricultural lands as provided in Proclamation No. 131 and Executive Order No. 229, predecessors-in-interest under a bona fide claim of ownership since time immemorial or
including other lands of the public domain suitable for agriculture. from June 12, 1945; and (2) the classification of the land as alienable and disposable
land of the public domain.128
More specifically, the following lands are covered by the Comprehensive Agrarian
Reform Program: As discussed, the Philippine Bill of 1902, Act No. 926, and Proclamation No. 1801 did
(a) All alienable and disposable lands of the public domain devoted to or suitable for not convert portions of Boracay Island into an agricultural land. The island remained an
unclassified land of the public domain and, applying the Regalian doctrine, is
agriculture. No reclassification of forest or mineral lands to agricultural lands shall be
considered State property.
undertaken after the approval of this Act until Congress, taking into account ecological,
developmental and equity considerations, shall have determined by law, the specific Private claimants' bid for judicial confirmation of imperfect title, relying on the Philippine
limits of the public domain. Bill of 1902, Act No. 926, and Proclamation No. 1801, must fail because of the absence
of the second element of alienable and disposable land. Their entitlement to a
That Boracay Island was classified as a public forest under PD No. 705 did not bar the government grant under our present Public Land Act presupposes that the land
Executive from later converting it into agricultural land. Boracay Island still remained an possessed and applied for is already alienable and disposable. This is clear from the
unclassified land of the public domain despite PD No. 705. wording of the law itself.129 Where the land is not alienable and disposable, possession
of the land, no matter how long, cannot confer ownership or possessory rights. 130
In Heirs of the Late Spouses Pedro S. Palanca and Soterranea Rafols v.
Republic,124 the Court stated that unclassified lands are public forests. Neither may private claimants apply for judicial confirmation of imperfect title under
Proclamation No. 1064, with respect to those lands which were classified as agricultural
lands. Private claimants failed to prove the first element of open, continuous, exclusive, In issuing Proclamation No. 1064, the government has taken the step necessary to
and notorious possession of their lands in Boracay since June 12, 1945. open up the island to private ownership. This gesture may not be sufficient to appease
some sectors which view the classification of the island partially into a forest reserve as
We cannot sustain the CA and RTC conclusion in the petition for declaratory relief that absurd. That the island is no longer overrun by trees, however, does not becloud the
private claimants complied with the requisite period of possession. vision to protect its remaining forest cover and to strike a healthy balance between
progress and ecology. Ecological conservation is as important as economic progress.
The tax declarations in the name of private claimants are insufficient to prove the first
element of possession. We note that the earliest of the tax declarations in the name of To be sure, forest lands are fundamental to our nation's survival. Their promotion and
private claimants were issued in 1993. Being of recent dates, the tax declarations are protection are not just fancy rhetoric for politicians and activists. These are needs that
not sufficient to convince this Court that the period of possession and occupation become more urgent as destruction of our environment gets prevalent and difficult to
commenced on June 12, 1945. control. As aptly observed by Justice Conrado Sanchez in 1968 in Director of Forestry
v. Munoz:134
Private claimants insist that they have a vested right in Boracay, having been in
possession of the island for a long time. They have invested millions of pesos in The view this Court takes of the cases at bar is but in adherence to public policy that
developing the island into a tourist spot. They say their continued possession and should be followed with respect to forest lands. Many have written much, and many
investments give them a vested right which cannot be unilaterally rescinded by more have spoken, and quite often, about the pressing need for forest preservation,
Proclamation No. 1064. conservation, protection, development and reforestation. Not without justification. For,
forests constitute a vital segment of any country's natural resources. It is of common
The continued possession and considerable investment of private claimants do not
knowledge by now that absence of the necessary green cover on our lands produces a
automatically give them a vested right in Boracay. Nor do these give them a right to
apply for a title to the land they are presently occupying. This Court is constitutionally number of adverse or ill effects of serious proportions. Without the trees, watersheds
bound to decide cases based on the evidence presented and the laws applicable. As dry up; rivers and lakes which they supply are emptied of their contents. The fish
the law and jurisprudence stand, private claimants are ineligible to apply for a judicial disappear. Denuded areas become dust bowls. As waterfalls cease to function, so will
confirmation of title over their occupied portions in Boracay even with their continued hydroelectric plants. With the rains, the fertile topsoil is washed away; geological
possession and considerable investment in the island. erosion results. With erosion come the dreaded floods that wreak havoc and destruction
to property - crops, livestock, houses, and highways - not to mention precious human
One Last Note lives. Indeed, the foregoing observations should be written down in a lumberman's
decalogue.135
The Court is aware that millions of pesos have been invested for the development of
Boracay Island, making it a by-word in the local and international tourism industry. The
Court also notes that for a number of years, thousands of people have called the island WHEREFORE, judgment is rendered as follows:
their home. While the Court commiserates with private claimants' plight, We are bound
to apply the law strictly and judiciously. This is the law and it should prevail. Ito ang 1. The Petition for Certiorariin G.R. No. 167707 is GRANTED and the Court of Appeals
batas at ito ang dapat umiral. Decision in CA-G.R. CV No. 71118 REVERSED AND SET ASIDE.

All is not lost, however, for private claimants. While they may not be eligible to apply for 2. The Petition for Certiorari in G.R. No. 173775 is DISMISSED for lack of merit.
judicial confirmation of imperfect title under Section 48(b) of CA No. 141, as amended,
this does not denote their automatic ouster from the residential, commercial, and other SO ORDERED.
areas they possess now classified as agricultural. Neither will this mean the loss of their
substantial investments on their occupied alienable lands. Lack of title does not
necessarily mean lack of right to possess.

For one thing, those with lawful possession may claim good faith as builders of
improvements. They can take steps to preserve or protect their possession. For
another, they may look into other modes of applying for original registration of title, such
as by homestead131 or sales patent,132subject to the conditions imposed by law.

More realistically, Congress may enact a law to entitle private claimants to acquire title
to their occupied lots or to exempt them from certain requirements under the present
land laws. There is one such bill133 now pending in the House of Representatives.
Whether that bill or a similar bill will become a law is for Congress to decide.
Cagayan, covering Lot 2472-B consisting of 100,000 square meters and Transfer
Certificate of Title No. T-1278,7 issued in the name of the private respondents, covering
Lot 2472-A consisting of 6,997,921 square meters.
On 19 May 1994, Bienvenida Taguiam Vda. De Dayag and others filed with the
Regional Office No. 2 of the Department of Environment and Natural Resources
(DENR), Tuguegarao, Cagayan, a letter-petition requesting the DENR to initiate the
filing of an action for the annulment of Decree No. 381928 on the ground that the trial
court did not have jurisdiction to adjudicate a portion of the subject property which was
allegedly still classified as timber land at the time of the issuance of Decree No. 381928.
The Regional Executive Director of the DENR created an investigating team to conduct
ground verification and ocular inspection of the subject property.
The investigating team reported that:
A) The portion of Lot 2472 Cad-151 as shown in the Plan prepared for spouses
Carag, and covered under LC Project 3-L of Tuguegarao, Cagayan, was found
to be still within the timberland area at the time of the issuance of the Decree
and O.C.T. of the spouses Antonio Carag and Victoria Turingan, and the same
Republic of the Philippines
SUPREME COURT was only released as alienable and disposable on February 22, 1982, as
certified by USEC Jose G. Solis of the NAMRIA on 27 May 1994.
Manila
FIRST DIVISION B) Petitioner Bienvenida Taguiam Vda. De Dayag and others have possessed
and occupied by themselves and thru their predecessors-in-interest the portion
G.R. No. 155450 August 6, 2008 of Lot 2472 Cad-151, covered by LC Project 3-L of LC Map 2999, since time
REPUBLIC OF THE PHILIPPINES represented by the Regional Executive Director, immemorial.8
Department of Environment and Natural Resources, Regional Office No. Thus, the investigating team claimed that "a portion of Lot 2472 Cad-151" was "only
2, petitioners, released as alienable and disposable on 22 February 1982."
vs.
COURT OF APPEALS, HEIRS OF ANTONIO CARAG AND VICTORIA TURINGAN, In a Memorandum dated 9 September 1996, the Legal Division of the Land
THE REGISTER OF DEEDS OF CAGAYAN, and the COURT OF FIRST INSTANCE Management Bureau recommended to the Director of Lands that an action for the
OF CAGAYAN, respondents. cancellation of OCT No. 11585, as well as its derivative titles, be filed with the proper
court. The Director of Lands approved the recommendation.
DECISION
On 10 June 1998, or 68 years after the issuance of Decree No. 381928, petitioner
CARPIO, J.: filed with the Court of Appeals a complaint for annulment of judgment, cancellation and
The Case declaration of nullity of titles9 on the ground that in 1930 the trial court had no
jurisdiction to adjudicate a portion of the subject property, which portion consists of
review1
This is a petition for of the 21 May 20012 and 25 September 20023 Resolutions 2,640,000 square meters (disputed portion). The disputed portion was allegedly still
of the Court of Appeals in CA-G.R. SP No. 47965. The 21 May 2001 Resolution classified as timber land at the time of issuance of Decree No. 381928 and, therefore,
dismissed petitioner Republic of the Philippines (petitioner) amended complaint for was not alienable and disposable until 22 February 1982 when the disputed portion was
reversion, annulment of decree, cancellation and declaration of nullity of titles. The 25 classified as alienable and disposable.
September 2002 Resolution denied petitioners motion for reconsideration.
On 19 October 1998, private respondents filed a motion to dismiss. 10 Private
The Facts respondents alleged that petitioner failed to comply with Rule 47 of the Rules of Court
On 2 June 1930, the then Court of First Instance of Cagayan (trial court) issued Decree because the real ground for the complaint was mistake, not lack of jurisdiction, and that
No. 3819284in favor of spouses Antonio Carag and Victoria Turingan (spouses Carag), petitioner, as a party in the original proceedings, could have availed of the ordinary
predecessors-in-interest of private respondents Heirs of Antonio Carag and Victoria remedies of new trial, appeal, petition for relief or other appropriate remedies but failed
Turingan (private respondents), covering a parcel of land identified as Lot No. 2472, to do so. Private respondents added that petitioner did not attach to the complaint a
Cad. 151, containing an area of 7,047,673 square meters (subject property), situated in certified true copy of the decision sought to be annulled. Private respondents also
Tuguegarao, Cagayan. On 19 July 1938, pursuant to said Decree, the Register of maintained that the complaint was barred by the doctrines of res judicata and law of the
Deeds of Cagayan issued Original Certificate of Title No. 11585 5 (OCT No. 11585) in case and by Section 38 of Act No. 496.11 Private respondents also stated that not all the
the name of spouses Carag. heirs of spouses Carag were brought before the Court of Appeals for an effective
resolution of the case. Finally, private respondents claimed that the real party in interest
On 2 July 1952, OCT No. 11585 was cancelled to discharge the encumbrance was not petitioner but a certain Alfonso Bassig, who had an ax to grind against private
expressly stated in Decree No. 381928. Two transfer certificates of title were issued: respondents.12
Transfer Certificate of Title No. T-1277,6 issued in the name of the Province of
On 3 March 1999, petitioner filed an amended complaint for reversion, annulment of Petitioner Complied with Rule 47 of the Rules of Court
decree, cancellation and declaration of nullity of titles.13
First, the Court of Appeals ruled that petitioner failed to allege either of the grounds of
The Ruling of the Court of Appeals extrinsic fraud or lack of jurisdiction in the complaint for annulment of decree. 15
On 21 May 2001, the Court of Appeals dismissed the complaint because of lack of We find otherwise. In its complaint and amended complaint, petitioner stated:
jurisdiction over the subject matter of the case. The Court of Appeals declared:
11. In view of the fact that in 1930 or in 1938, only the Executive Branch of the
The rule is clear that such judgments, final orders and resolutions in civil Government had the authority and power to declassify or reclassify land of the
actions which this court may annul are those which the "ordinary remedies of public domain, the Court did not, therefore, have the power and authority
new trial, appeal, petition for relief or other appropriate remedies are no longer to adjudicate in favor of the spouses Antonio Carag and Victoria
available." The Amended Complaint contains no such allegations which are Turingan the said tract of timberland, portion of the Lot 2472 Cad-151, at
jurisdictional neither can such circumstances be divined from its allegations. the time of the issuance of the Decree and the Original Certificate of Title
Furthermore, such actions for Annulment may be based only on two (2) of the said spouses; and such adjudication and/or Decree and Title issued
grounds: extrinsic fraud and lack of jurisdiction. Neither ground is alleged in the covering the timberland area is null and void ab initio considering the
Amended Complaint which is for Reversion/Annulment of Decree, Cancellation provisions of the 1935, 1973 and 1987 Philippine constitution.
and Declaration of Nullity of Titles. It merely alleges that around 2,640,000
xxxx
square meters of timberland area within Lot 2472 Cad. 151, had been
erroneously included in the title of the Spouses Antonio Carag and Victoria 15. The issuance of Decree No. 381928 and O.C.T. No. 11585 in the name of
Turingan under Decree No. 381928 and O.C.T. No. 11585 issued on June 2, spouses Antonio Carag and Victoria Turingan, and all the derivative titles
1930 and July 19, 1938, respectively; that hence, such adjudication and/or thereto in the name of the Heirs and said spouses, specifically with respect to
Decree and Title covering a timberland area is null and void ab initio under the the inclusion thereto of timberland area, by the then Court of First Instance
provisions of the 1935, 1973 and 1987 Constitutions. (now the Regional Trial Court), and the Register of Deeds of Cagayan is
patently illegal and erroneous for the reason that said Court and/or the
Finally, it is clear that the issues raised in the Amended Complaint as well as those in
Register of Deeds of Cagayan did not have any authority or jurisdiction
the Motion to dismiss are factual in nature and should be threshed out in the proper trial
to decree or adjudicate the said timberland area of Lot 2472 Cad-151,
court in accordance with Section 101 of the Public Land Act.14 (Citations omitted)
consequently, the same are null and void ab initio, and of no force and effect
Petitioner filed a motion for reconsideration. In its 25 September 2002 Resolution, the whatsoever.16 (Emphasis supplied; citations omitted)
Court of Appeals denied the motion for reconsideration.
Petitioner clearly alleged in the complaint and amended complaint that it was seeking to
Hence, this petition. annul Decree No. 381928 on the ground of the trial courts lack of jurisdiction over the
subject land, specifically over the disputed portion, which petitioner maintained was
The Issues
classified as timber land and was not alienable and disposable.
Petitioner raises the following issues:
Second, the Court of Appeals also dismissed the complaint on the ground of petitioners
1. Whether the allegations of the complaint clearly stated that the ordinary failure to allege that the "ordinary remedies of new trial, appeal, petition for relief or
remedies of new trial, appeal, petition for relief and other appropriate remedies other appropriate remedies are no longer available."
are no longer available;
In Ancheta v. Ancheta,17 we ruled:
2. Whether the amended complaint clearly alleged the ground of lack of
In a case where a petition for annulment of judgment or final order of the RTC
jurisdiction;
filed under Rule 47 of the Rules of Court is grounded on lack of jurisdiction
3. Whether the Court of Appeals may try the factual issues raised in the over the person of the defendant/respondent or over the nature or subject of
amended complaint and in the motion to dismiss; the action, the petitioner need not allege in the petition that the ordinary
remedy of new trial or reconsideration of the final order or judgment or appeal
4. Whether the then Court of First Instance of Cagayan had jurisdiction to
therefrom are no longer available through no fault of her own. This is so
adjudicate a tract of timberland in favor of respondent spouses Antonio Carag
because a judgment rendered or final order issued by the RTC without
and Victoria Turingan;
jurisdiction is null and void and may be assailed any time either collaterally or
5. Whether the fact that the Director of Lands was a party to the original in a direct action or by resisting such judgment or final order in any action or
proceedings changed the nature of the land and granted jurisdiction to the then proceeding whenever it is invoked, unless barred by laches.18
Court of First Instance over the land;
Since petitioners complaint is grounded on lack of jurisdiction over the subject of the
6. Whether the doctrine of res judicata applies in this case; and action, petitioner need not allege that the ordinary remedies of new trial, appeal, petition
7. Whether Section 38 of Act No. 496 is applicable in this case. for relief or other appropriate remedies are no longer available through no fault of
petitioner.
The Ruling of the Court
Third, the Court of Appeals ruled that the issues raised in petitioners complaint were
While the Court of Appeals erred in dismissing the complaint on procedural grounds, we factual in nature and should be threshed out in the proper trial court in accordance with
will still deny the petition because the complaint for annulment of decree has no merit. Section 101 of the Public Land Act.19
Section 6, Rule 47 of the Rules of Court provides: and may at any time and in a like manner transfer such lands from one class to
another, for the purposes of their government and disposition.
SEC. 6. Procedure. - The procedure in ordinary civil cases shall be observed.
Should a trial be necessary, the reception of evidence may be referred to a Petitioner has not alleged that the Governor-General had declared the disputed portion
member of the court or a judge of a Regional Trial Court. of the subject property timber or mineral land pursuant to Section 6 of Act No. 2874.
Therefore, the Court of Appeals may try the factual issues raised in the complaint for the It is true that Section 8 of Act No. 2874 opens to disposition only those lands which
complete and proper determination of the case. have been declared alienable or disposable. Section 8 provides:
However, instead of remanding the complaint to the Court of Appeals for further SECTION 8. Only those lands shall be declared open to disposition or
proceedings, we shall decide the case on the merits. concession which have been officially delimited and classified and, when
practicable, surveyed, and which have not been reserved for public or quasi-
Complaint for Annulment of Decree Has No Merit
public uses, not appropriated by the Government, nor in any manner become
Petitioner contends that the trial court had no jurisdiction to adjudicate to spouses Carag private property, nor those on which a private right authorized and
the disputed portion of the subject property. Petitioner claims that the disputed portion recognized by this Act or any other valid law may be claimed, or which,
was still classified as timber land, and thus not alienable and disposable, when Decree having been reserved or appropriated, have ceased to be so. However, the
No. 381928 was issued in 1930. In effect, petitioner admits that the adjacent 4,407,673 Governor-General may, for reasons of public interest, declare lands of the
square meters of the subject property, outside of the disputed portion, were alienable public domain open to disposition before the same have had their boundaries
and disposable in 1930. Petitioner argues that in 1930 or in 1938, only the Executive established or been surveyed, or may, for the same reasons, suspend their
Branch of the Government, not the trial courts, had the power to declassify or reclassify concession or disposition by proclamation duly published or by Act of the
lands of the public domain. Legislature. (Emphasis supplied)
Lack of jurisdiction, as a ground for annulment of judgment, refers to either lack of However, Section 8 provides that lands which are already private lands, as well as
jurisdiction over the person of the defending party or over the subject matter of the lands on which a private claim may be made under any law, are not covered by the
claim.20 Jurisdiction over the subject matter is conferred by law and is determined by the classification requirement in Section 8 for purposes of disposition. This exclusion in
statute in force at the time of the filing of the action.21 Section 8 recognizes that during the Spanish regime, Crown lands were per se
alienable unless falling under timber or mineral zones, or otherwise reserved for some
Under the Spanish regime, all Crown lands were per se alienable. In Aldecoa v. Insular
public purpose in accordance with law.
Government,22 we ruled:
Clearly, with respect to lands excluded from the classification requirement in Section 8,
From the language of the foregoing provisions of law, it is deduced that, with
the exception of those comprised within the mineral and timber zone, all lands trial courts had jurisdiction to adjudicate these lands to private parties. Petitioner has not
owned by the State or by the sovereign nation are public in character, alleged that the disputed portion had not become private property prior to the enactment
and per se alienable and, provided they are not destined to the use of the of Act No. 2874. Neither has petitioner alleged that the disputed portion was not land on
which a private right may be claimed under any existing law at that time.
public in general or reserved by the Government in accordance with law, they
may be acquired by any private or juridical person x x x23 (Emphasis supplied) In Republic of the Philippines v. Court of Appeals,27 the Republic sought to annul the
judgment of the Court of First Instance (CFI) of Rizal, sitting as a land registration court,
Thus, unless specifically declared as mineral or forest zone, or reserved by the State for
some public purpose in accordance with law, all Crown lands were deemed alienable. because when the application for land registration was filed in 1927 the land was
alleged to be unclassified forest land. The Republic also alleged that the CFI of Rizal
In this case, petitioner has not alleged that the disputed portion had been declared as had no jurisdiction to determine whether the land applied for was forest or agricultural
mineral or forest zone, or reserved for some public purpose in accordance with law, land since the authority to classify lands was then vested in the Director of Lands as
during the Spanish regime or thereafter. The land classification maps24 petitioner provided in Act Nos. 92628 and 2874. The Court ruled:
attached to the complaint also do not show that in 1930 the disputed portion was part of
We are inclined to agree with the respondent that it is legally doubtful if the
the forest zone or reserved for some public purpose. The certification of the National
authority of the Governor General to declare lands as alienable and disposable
Mapping and Resources Information Authority, dated 27 May 1994, contained no
would apply to lands that have become private property or lands that have
statement that the disputed portion was declared and classified as timber land. 25
been impressed with a private right authorized and recognized by Act 2874 or
The law prevailing when Decree No. 381928 was issued in 1930 was Act No. any valid law. By express declaration of Section 45 (b) of Act 2874 which is
2874,26 which provides: quoted above, those who have been in open, continuous, exclusive and
SECTION 6. The Governor-General, upon the recommendation of the notorious possession and occupation of agricultural lands of the public domain
Secretary of Agriculture and Natural Resources, shall from time to time classify under a bona fide claim of acquisition of ownership since July 26, 1894 may
the lands of the public domain into - file an application with the Court of First Instance of the province where the
land is located for confirmation of their claims and these applicants shall be
(a) Alienable or disposable conclusively presumed to have performed all the conditions essential to a
(b) Timber and government grant and shall be entitled to a certificate of title. When the land
registration court issued a decision for the issuance of a decree which
(c) Mineral lands was the basis of an original certificate of title to the land, the court had
already made a determination that the land was agricultural and that the
applicant had proven that he was in open and exclusive possession of
the subject land for the prescribed number of years. It was the land
registration court which had the jurisdiction to determine whether the
land applied for was agricultural, forest or timber taking into account the
proof or evidence in each particular case. (Emphasis supplied)
As with this case, when the trial court issued the decision for the issuance of Decree
No. 381928 in 1930, the trial court had jurisdiction to determine whether the subject
property, including the disputed portion, applied for was agricultural, timber or mineral
land. The trial court determined that the land was agricultural and that spouses Carag
proved that they were entitled to the decree and a certificate of title. The government,
which was a party in the original proceedings in the trial court as required by law, did not
appeal the decision of the trial court declaring the subject land as agricultural. Since the Republic of the Philippines
SUPREME COURT
trial court had jurisdiction over the subject matter of the action, its decision rendered in
1930, or 78 years ago, is now final and beyond review. Manila
EN BANC
The finality of the trial courts decision is further recognized in Section 1, Article XII of
the 1935 Constitution which provides: G.R. No. L-8936 October 2, 1915
SECTION 1. All agricultural, timber, and mineral lands of the public domain, CONSUELO LEGARDA, with her husband MAURO PRIETO, plaintiffs-appellants,
waters, minerals, coal, petroleum, and other mineral oils, all forces of potential vs.
energy, and other natural resources of the Philippines belong to the State, and N.M. SALEEBY, defendant-appellee.
their disposition, exploitation, development, or utilization shall be limited to
Singson, Ledesma and Lim for appellants.
citizens of the Philippines, or to corporations or associations at least sixty per
D.R. Williams for appellee.
centum of the capital of which is owned by such citizens, subject to any
existing right, grant, lease, or concession at the time of the inauguration
of the Government established under this Constitution. (Emphasis
JOHNSON, J.:
supplied)
From the record the following facts appear:
Thus, even as the 1935 Constitution declared that all agricultural, timber and mineral
lands of the public domain belong to the State, it recognized that these lands First. That the plaintiffs and the defendant occupy, as owners, adjoining lots in the
were "subject to any existing right, grant, lease or concession at the time of the district of Ermita in the city of Manila.
inauguration of the Government established under this Constitution." 29 When the
Second. That there exists and has existed a number of years a stone wall between the
Commonwealth Government was established under the 1935 Constitution, spouses
said lots. Said wall is located on the lot of the plaintiffs.
Carag had already an existing right to the subject land, including the disputed portion,
pursuant to Decree No. 381928 issued in 1930 by the trial court. Third. That the plaintiffs, on the 2d day of March, 1906, presented a petition in the Court
of Land Registration for the registration of their lot. After a consideration of said petition
WHEREFORE, we DENY the petition. We DISMISS petitioner Republic of the
the court, on the 25th day of October, 1906, decreed that the title of the plaintiffs should
Philippines complaint for reversion, annulment of decree, cancellation and declaration
be registered and issued to them the original certificate provided for under the torrens
of nullity of titles for lack of merit.
system. Said registration and certificate included the wall.
SO ORDERED.
Fourth. Later the predecessor of the defendant presented a petition in the Court of Land
Registration for the registration of the lot now occupied by him. On the 25th day of
March, 1912, the court decreed the registration of said title and issued the original
certificate provided for under the torrens system. The description of the lot given in the
petition of the defendant also included said wall.
Fifth. Several months later (the 13th day of December, 1912) the plaintiffs discovered
that the wall which had been included in the certificate granted to them had also been
included in the certificate granted to the defendant .They immediately presented a
petition in the Court of Land Registration for an adjustment and correction of the error
committed by including said wall in the registered title of each of said parties. The lower
court however, without notice to the defendant, denied said petition upon the theory
that, during the pendency of the petition for the registration of the defendant's land, they
failed to make any objection to the registration of said lot, including the wall, in the name
of the defendant.
Sixth. That the land occupied by t he wall is registered in the name of each of the nothing in the Act which indicates who should be the owner of land which has been
owners of the adjoining lots. The wall is not a joint wall. registered in the name of two different persons.
Under these facts, who is the owner of the wall and the land occupied by it? The rule, we think, is well settled that the decree ordering the registration of a particular
parcel of land is a bar to future litigation over the same between the same parties .In
The decision of the lower court is based upon the theory that the action for the
view of the fact that all the world are parties, it must follow that future litigation over the
registration of the lot of the defendant was a judicial proceeding and that the judgment
title is forever barred; there can be no persons who are not parties to the action. This,
or decree was binding upon all parties who did not appear and oppose it. In other
we think, is the rule, except as to rights which are noted in the certificate or which arise
words, by reason of the fact that the plaintiffs had not opposed the registration of that
subsequently, and with certain other exceptions which need not be dismissed at
part of the lot on which the wall was situate they had lost it, even though it had been
present. A title once registered can not be defeated, even by an adverse, open, and
theretofore registered in their name. Granting that theory to be correct one, and granting
notorious possession. Registered title under the torrens system can not be defeated by
even that the wall and the land occupied by it, in fact, belonged to the defendant and his
prescription (section 46, Act No. 496). The title, once registered, is notice to the world.
predecessors, then the same theory should be applied to the defendant himself.
All persons must take notice. No one can plead ignorance of the registration.
Applying that theory to him, he had already lost whatever right he had therein, by
permitting the plaintiffs to have the same registered in their name, more than six years The question, who is the owner of land registered in the name of two different persons,
before. Having thus lost hid right, may he be permitted to regain it by simply including it has been presented to the courts in other jurisdictions. In some jurisdictions, where the
in a petition for registration? The plaintiffs having secured the registration of their lot, "torrens" system has been adopted, the difficulty has been settled by express statutory
including the wall, were they obliged to constantly be on the alert and to watch all the provision. In others it has been settled by the courts. Hogg, in his excellent discussion of
proceedings in the land court to see that some one else was not having all, or a portion the "Australian Torrens System," at page 823, says: "The general rule is that in the case
of the same, registered? If that question is to be answered in the affirmative, then the of two certificates of title, purporting to include the same land, the earlier in date
whole scheme and purpose of the torrens system of land registration must fail. The real prevails, whether the land comprised in the latter certificate be wholly, or only in part,
purpose of that system is to quiet title to land; to put a stop forever to any question of comprised in the earlier certificate. (Oelkers vs. Merry, 2 Q.S.C.R., 193; Miller vs. Davy,
the legality of the title, except claims which were noted at the time of registration, in the 7 N.Z.R., 155; Lloyd vs. Myfield, 7 A.L.T. (V.) 48; Stevens vs. Williams, 12 V.L. R., 152;
certificate, or which may arise subsequent thereto. That being the purpose of the law, it Register of Titles, vs. Esperance Land Co., 1 W.A.R., 118.)" Hogg adds however that,
would seem that once a title is registered the owner may rest secure, without the "if it can be very clearly ascertained by the ordinary rules of construction relating to
necessity of waiting in the portals of the court, or sitting in the "mirador de su casa," to written documents, that the inclusion of the land in the certificate of title of prior date is a
avoid the possibility of losing his land. Of course, it can not be denied that the mistake, the mistake may be rectified by holding the latter of the two certificates of title
proceeding for the registration of land under the torrens system is judicial (Escueta vs. to be conclusive." (See Hogg on the "Australian torrens System," supra, and cases
.Director of Lands, 16 Phil. Rep., 482). It is clothed with all the forms of an action and cited. See also the excellent work of Niblack in his "Analysis of the Torrens System,"
the result is final and binding upon all the world. It is an action in rem. (Escueta vs. page 99.) Niblack, in discussing the general question, said: "Where two certificates
Director of Lands (supra); Grey Alba vs. De la Cruz, 17 Phil. rep., 49 Roxas vs. purport to include the same land the earlier in date prevails. ... In successive
Enriquez, 29 Phil. Rep., 31; Tyler vs. Judges, 175 Mass., 51 American Land Co. vs. registrations, where more than one certificate is issued in respect of a particular estate
Zeiss, 219 U.S., 47.) or interest in land, the person claiming under the prior certificates is entitled to the
estate or interest; and that person is deemed to hold under the prior certificate who is
While the proceeding is judicial, it involves more in its consequences than does an
the holder of, or whose claim is derived directly or indirectly from the person who was
ordinary action. All the world are parties, including the government. After the registration
the holder of the earliest certificate issued in respect thereof. While the acts in this
is complete and final and there exists no fraud, there are no innocent third parties who
country do not expressly cover the case of the issue of two certificates for the same
may claim an interest. The rights of all the world are foreclosed by the decree of
land, they provide that a registered owner shall hold the title, and the effect of this
registration. The government itself assumes the burden of giving notice to all parties. To
undoubtedly is that where two certificates purport to include the same registered land,
permit persons who are parties in the registration proceeding (and they are all the
the holder of the earlier one continues to hold the title" (p. 237).
world) to again litigate the same questions, and to again cast doubt upon the validity of
the registered title, would destroy the very purpose and intent of the law. The Section 38 of Act No. 496, provides that; "It (the decree of registration) shall be
registration, under the torrens system, does not give the owner any better title than he conclusive upon and against all persons, including the Insular Government and all the
had. If he does not already have a perfect title, he can not have it registered. Fee simple branches thereof, whether mentioned by name in the application, notice, or citation, or
titles only may be registered. The certificate of registration accumulates in open included in the general description "To all whom it may concern." Such decree shall not
document a precise and correct statement of the exact status of the fee held by its be opened by reason of the absence, infancy, or other disability of any person affected
owner. The certificate, in the absence of fraud, is the evidence of title and shows exactly thereby, nor by any proceeding in any court for reversing judgments or decrees; subject,
the real interest of its owner. The title once registered, with very few exceptions, should however, to the right of any person deprived of land or of any estate or interest therein
not thereafter be impugned, altered, changed, modified, enlarged, or diminished, except by decree of registration obtained by fraud to file in the Court of Land Registration a
in some direct proceeding permitted by law. Otherwise all security in registered titles petition for review within one year after entry of the decree (of registration), provided no
would be lost. A registered title can not be altered, modified, enlarged, or diminished in innocent purchaser for value has acquired an interest.
a collateral proceeding and not even by a direct proceeding, after the lapse of the period
It will be noted, from said section, that the "decree of registration" shall not be opened,
prescribed by law.
for any reason, in any court, except for fraud, and not even for fraud, after the lapse of
For the difficulty involved in the present case the Act (No. 496) providing for the one year. If then the decree of registration can not be opened for any reason, except for
registration of titles under the torrens system affords us no remedy. There is no fraud, in a direct proceeding for that purpose, may such decree be opened or set aside
provision in said Act giving the parties relief under conditions like the present. There is in a collateral proceeding by including a portion of the land in a subsequent certificate or
decree of registration? We do not believe the law contemplated that a person could be We have decided, in case of double registration under the Land Registration Act, that
deprived of his registered title in that way. the owner of the earliest certificate is the owner of the land. That is the rule between
original parties. May this rule be applied to successive vendees of the owners of such
We have in this jurisdiction a general statutory provision which governs the right of the
certificates? Suppose that one or the other of the parties, before the error is discovered,
ownership of land when the same is registered in the ordinary registry in the name of
transfers his original certificate to an "innocent purchaser." The general rule is that the
two persons. Article 1473 of the Civil Code provides, among other things, that when one
vendee of land has no greater right, title, or interest than his vendor; that he acquires
piece of real property had been sold to two different persons it shall belong to the
the right which his vendor had, only. Under that rule the vendee of the earlier certificate
person acquiring it, who first inscribes it in the registry. This rule, of course,
would be the owner as against the vendee of the owner of the later certificate.
presupposes that each of the vendees or purchasers has acquired title to the land. The
real ownership in such a case depends upon priority of registration. While we do not We find statutory provisions which, upon first reading, seem to cast some doubt upon
now decide that the general provisions of the Civil Code are applicable to the Land the rule that the vendee acquires the interest of the vendor only. Sections 38, 55, and
Registration Act, even though we see no objection thereto, yet we think, in the absence 112 of Act No. 496 indicate that the vendee may acquire rights and be protected against
of other express provisions, they should have a persuasive influence in adopting a rule defenses which the vendor would not. Said sections speak of available rights in favor of
for governing the effect of a double registration under said Act. Adopting the rule which third parties which are cut off by virtue of the sale of the land to an "innocent purchaser."
we believe to be more in consonance with the purposes and the real intent of the That is to say, persons who had had a right or interest in land wrongfully included in an
torrens system, we are of the opinion and so decree that in case land has been original certificate would be unable to enforce such rights against an "innocent
registered under the Land Registration Act in the name of two different persons, the purchaser," by virtue of the provisions of said sections. In the present case Teus had his
earlier in date shall prevail. land, including the wall, registered in his name. He subsequently sold the same to the
appellee. Is the appellee an "innocent purchaser," as that phrase is used in said
In reaching the above conclusion, we have not overlooked the forceful argument of the
sections? May those who have been deprived of their land by reason of a mistake in the
appellee. He says, among other things; "When Prieto et al. were served with notice of
original certificate in favor of Teus be deprived of their right to the same, by virtue of the
the application of Teus (the predecessor of the defendant) they became defendants in a
sale by him to the appellee? Suppose the appellants had sold their lot, including the
proceeding wherein he, Teus, was seeking to foreclose their right, and that of orders, to
wall, to an "innocent purchaser," would such purchaser be included in the phrase
the parcel of land described in his application. Through their failure to appear and
"innocent purchaser," as the same is used in said sections? Under these examples
contest his right thereto, and the subsequent entry of a default judgment against them,
there would be two innocent purchasers of the same land, is said sections are to be
they became irrevocably bound by the decree adjudicating such land to Teus. They had
applied .Which of the two innocent purchasers, if they are both to be regarded as
their day in court and can not set up their own omission as ground for impugning the
innocent purchasers, should be protected under the provisions of said sections? These
validity of a judgment duly entered by a court of competent jurisdiction. To decide
questions indicate the difficulty with which we are met in giving meaning and effect to
otherwise would be to hold that lands with torrens titles are above the law and beyond
the phrase "innocent purchaser," in said sections.
the jurisdiction of the courts".
May the purchaser of land which has been included in a "second original certificate"
As was said above, the primary and fundamental purpose of the torrens system is to
ever be regarded as an "innocent purchaser," as against the rights or interest of the
quiet title. If the holder of a certificate cannot rest secure in this registered title then the
owner of the first original certificate, his heirs, assigns, or vendee? The first original
purpose of the law is defeated. If those dealing with registered land cannot rely upon the
certificate is recorded in the public registry. It is never issued until it is recorded. The
certificate, then nothing has been gained by the registration and the expense incurred
record notice to all the world. All persons are charged with the knowledge of what it
thereby has been in vain. If the holder may lose a strip of his registered land by the
contains. All persons dealing with the land so recorded, or any portion of it, must be
method adopted in the present case, he may lose it all. Suppose within the six years
charged with notice of whatever it contains. The purchaser is charged with notice of
which elapsed after the plaintiff had secured their title, they had mortgaged or sold their
every fact shown by the record and is presumed to know every fact which the record
right, what would be the position or right of the mortgagee or vendee? That mistakes
discloses .This rule is so well established that it is scarcely necessary to cite authorities
are bound to occur cannot be denied, and sometimes the damage done thereby is
in its support (Northwestern National Bank vs. Freeman, 171 U.S., 620, 629; Delvin on
irreparable. It is the duty of the courts to adjust the rights of the parties under such
Real Estate, sections 710, 710 [a]).
circumstances so as to minimize such damages, taking into consideration al of the
conditions and the diligence of the respective parties to avoid them. In the present case, When a conveyance has been properly recorded such record is constructive notice of
the appellee was the first negligent (granting that he was the real owner, and if he was its contents and all interests, legal and equitable, included therein. (Grandin vs.
not the real owner he can not complain) in not opposing the registration in the name of Anderson, 15 Ohio State, 286, 289; Orvis vs. Newell, 17 Conn., 97; Buchanan vs.
the appellants. He was a party-defendant in an action for the registration of the lot in Intentional Bank, 78 Ill., 500; Youngs vs. Wilson, 27 N.Y., 351; McCabe vs. Grey, 20
question, in the name of the appellants, in 1906. "Through his failure to appear and to Cal., 509; Montefiore vs. Browne, 7 House of Lords Cases, 341.)
oppose such registration, and the subsequent entry of a default judgment against him,
Under the rule of notice, it is presumed that the purchaser has examined every
he became irrevocably bound by the decree adjudicating such land to the appellants.
instrument of record affecting the title. Such presumption is irrebutable. He is charged
He had his day in court and should not be permitted to set up his own omissions as the
with notice of every fact shown by the record and is presumed to know every fact which
ground for impugning the validity of a judgment duly entered by a court of competent
an examination of the record would have disclosed. This presumption cannot be
jurisdiction." Granting that he was the owner of the land upon which the wall is located,
overcome by proof of innocence or good faith. Otherwise the very purpose and object of
his failure to oppose the registration of the same in the name of the appellants, in the
the law requiring a record would be destroyed. Such presumption cannot be defeated
absence of fraud, forever closes his mouth against impugning the validity of that
by proof of want of knowledge of what the record contains any more than one may be
judgment. There is no more reason why the doctrine invoked by the appellee should be
permitted to show that he was ignorant of the provisions of the law. The rule that all
applied to the appellants than to him.
persons must take notice of the facts which the public record contains is a rule of law.
The rule must be absolute. Any variation would lead to endless confusion and useless certificate and his successors should be permitted to rest secure in their title, against
litigation. one who had acquired rights in conflict therewith and who had full and complete
knowledge of their rights. The purchaser of land included in the second original
While there is no statutory provision in force here requiring that original deeds of
certificate, by reason of the facts contained in the public record and the knowledge with
conveyance of real property be recorded, yet there is a rule requiring mortgages to be
which he is charged and by reason of his negligence, should suffer the loss, if any,
recorded. (Arts. 1875 and 606 of the Civil Code.) The record of a mortgage is
resulting from such purchase, rather than he who has obtained the first certificate and
indispensable to its validity. (Art .1875.) In the face of that statute would the courts allow
who was innocent of any act of negligence.
a mortgage to be valid which had not been recorded, upon the plea of ignorance of the
statutory provision, when third parties were interested? May a purchaser of land, The foregoing decision does not solve, nor pretend to solve, all the difficulties resulting
subsequent to the recorded mortgage, plead ignorance of its existence, and by reason from double registration under the torrens system and the subsequent transfer of the
of such ignorance have the land released from such lien? Could a purchaser of land, land. Neither do we now attempt to decide the effect of the former registration in the
after the recorded mortgage, be relieved from the mortgage lien by the plea that he was ordinary registry upon the registration under the torrens system. We are inclined to the
a bona fide purchaser? May there be a bona fide purchaser of said land, bona fide in view, without deciding it, that the record under the torrens system, supersede all other
the sense that he had no knowledge of the existence of the mortgage? We believe the registries. If that view is correct then it will be sufficient, in dealing with land registered
rule that all persons must take notice of what the public record contains in just as and recorded alone. Once land is registered and recorded under the torrens system,
obligatory upon all persons as the rule that all men must know the law; that no one can that record alone can be examined for the purpose of ascertaining the real status of the
plead ignorance of the law. The fact that all men know the law is contrary to the title to the land.
presumption. The conduct of men, at times, shows clearly that they do not know the
It would be seen to a just and equitable rule, when two persons have acquired equal
law. The rule, however, is mandatory and obligatory, notwithstanding. It would be just as
rights in the same thing, to hold that the one who acquired it first and who has complied
logical to allow the defense of ignorance of the existence and contents of a public
with all the requirements of the law should be protected.
record.
In view of our conclusions, above stated, the judgment of the lower court should be and
In view, therefore, of the foregoing rules of law, may the purchaser of land from the
is hereby revoked. The record is hereby returned to the court now having and exercising
owner of the second original certificate be an "innocent purchaser," when a part or all of
the jurisdiction heretofore exercised by the land court, with direction to make such
such land had theretofore been registered in the name of another, not the vendor? We
orders and decrees in the premises as may correct the error heretofore made in
are of the opinion that said sections 38, 55, and 112 should not be applied to such
including the land in the second original certificate issued in favor of the predecessor of
purchasers. We do not believe that the phrase "innocent purchaser should be applied to
the appellee, as well as in all other duplicate certificates issued.
such a purchaser. He cannot be regarded as an "innocent purchaser" because of the
facts contained in the record of the first original certificate. The rule should not be Without any findings as to costs, it is so ordered.
applied to the purchaser of a parcel of land the vendor of which is not the owner of the Arellano, C.J., Torrens, and Araullo, JJ., concur.
original certificate, or his successors. He, in nonsense, can be an "innocent purchaser"
of the portion of the land included in another earlier original certificate. The rule of notice
of what the record contains precludes the idea of innocence. By reason of the prior Separate Opinions
registry there cannot be an innocent purchaser of land included in a prior original
certificate and in a name other than that of the vendor, or his successors. In order to TRENT, J., dissenting:
minimize the difficulties we think this is the safe rule to establish. We believe the phrase I dissent.
"innocent purchaser," used in said sections, should be limited only to cases where
unregistered land has been wrongfully included in a certificate under the torrens system. In cases of double or overlapping registration, I am inclined to agree with the reasoning
When land is once brought under the torrens system, the record of the original and authority on which it is held in the majority opinion (first) that the original holder of
certificate and all subsequent transfers thereof is notice to all the world. That being the the prior certificate is entitled to the land as against the original holder of the later
rule, could Teus even regarded as the holder in good fifth of that part of the land certificate, where there has been no transfer of title by either party to an innocent
included in his certificate of the appellants? We think not. Suppose, for example, that purchaser; both, as is shown in the majority opinion, being at fault in permitting the
Teus had never had his lot registered under the torrens system. Suppose he had sold double registration to take place; (second) that an innocent purchaser claiming under
his lot to the appellee and had included in his deed of transfer the very strip of land now the prior certificate is entitled to the land as against the original holder of the later
in question. Could his vendee be regarded as an "innocent purchaser" of said strip? certificate, and also as against innocent purchasers from the holder of the later
Would his vendee be an "innocent purchaser" of said strip? Certainly not. The record of certificate; the innocent purchaser being in no wise at fault in connection with the
the original certificate of the appellants precludes the possibility. Has the appellee issuance of the later certificate.
gained any right by reason of the registration of the strip of land in the name of his But I am of opinion that neither the authorities cited, nor the reasoning of the majority
vendor? Applying the rule of notice resulting from the record of the title of the opinion sustains the proposition that the original holder of the prior certificate is entitled
appellants, the question must be answered in the negative. We are of the opinion that to the land as against an innocent purchaser from the holder of the later certificate.
these rules are more in harmony with the purpose of Act No. 496 than the rule
contended for by the appellee. We believe that the purchaser from the owner of the later As to the text-book authorities cited in the majority opinion, it is sufficient to say that the
certificate, and his successors, should be required to resort to his vendor for damages, rules laid down by both Hogg and Niblack are mere general rules, admittedly subject to
in case of a mistake like the present, rather than to molest the holder of the first exception, and of course of no binding force or authority where the reasoning upon
certificate who has been guilty of no negligence. The holder of the first original which these rules are based is applicable to the facts developed in a particular case.
In its last analysis the general rule laid down in the majority opinion rests upon the always be imputed to such a purchaser, so that in no event can he claim to be without
proposition set forth in the last page of the opinion wherein it is said that "it would seem fault when it appears that the lands purchased by him from the holder of a duly
to be a just and equitable rule, when two persons have acquired equal rights in the registered certificate of title are included within the bounds of the lands described in a
same thing, to hold that the one who acquired it first and who has complied with all the certificate of title of an earlier date.
requirements of the law should be protected." The rule, as applied to the matter in hand,
At considerable length the majority opinion (in reliance upon the general rule laid down
may be stated as follows: It would seem to be a just and equitable rule when two
under the various systems of land registration, other than those based on the torrens
persons have acquired separate and independent registered titles to the same land,
system) insists that a purchaser of land land duly registered in the Land Registration
under the Land Registration Act, to hold that the one who first acquired registered title
Court, is charged with notice of the contents of each and every one of the thousands
and who has complied with all the requirements of the law in that regard should be
and tens of thousands of certificates of registry on file in the land registry office, so that
protected, in the absence of any express statutory provision to the contrary.
negligence may be imputed to him if he does not ascertain that all or any part of the
Thus stated I have no quarrel with the doctrine as a statement of the general rule to be land purchased by him is included within the boundary lines of any one of the thousands
applied in cases of double or overlapping registration under the Land Registration Act; or tens of thousands of tracts of land whose original registry bears an earlier date than
for it is true as stated in the majority opinion that in the adjudication and registration of the date of the original registry of the land purchased by him. It is contended that he
titles by the Courts of Land Registration "mistakes are bound to occur, and sometimes cannot claim to be without fault should he buy such land because, as it is said, it was
the damage done thereby is irreparable;" and that in the absence of statutory provisions possible for him to discover that the land purchased by him had been made the subject
covering such cases, "it is the duty of the courts to adjust the rights of the parties, under of double or overlapping registration by a comparison of the description and boundary
such circumstances, so as to minimize such damages, taking into consideration all of lines of the thousands of tracts and parcels of land to be found in the land registry office.
the conditions, and the diligence of the respective parties to avoid them."
But such ruling goes far to defeat one of the principal objects sought to be attained by
But like most such general rules, it has its exceptions and should not be applied in a the introduction and adoption of the so-called torrens system for the registration of land.
case wherein the reasons on which it is based do not exist, or in cases wherein still The avowed intent of that system of land registration is to relieve the purchase of
more forceful reasons demand the application of a contrary rule. registered lands from the necessity of looking farther than the certificate of title of the
vendor in order that he may rest secure as to the validity of the title to the lands
The general rule relied upon in the majority opinion is a mere application of a well
conveyed to him. And yet it is said in the majority opinion that he is charged with notice
settled equity rule that: "Where conflicting equities are otherwise equal in merit, that
of the contents of every other certificate of title in the office of the registrar so that his
which first occurred will be given the preference." But it is universally laid down by all
failure to acquaint himself with its contents may be imputed to him as negligence.
the courts which have had occasion to apply this equity rule that "it should be the last
test resorted to," and that "it never prevails when any other equitable ground for If the rule announced in the majority opinion is to prevail, the new system of land
preference exists." (See 19 Cent. Dig., tit. Equity, par. 181; and may cases cited in 16 registration, instead of making transfers of real estate simple, expenditious and secure,
Cyc., 139, note 57.) It follows that the general rules, that in cases of double or and instead of avoiding the necessity for expensive and oftimes uncertain searches of
overlapping registration the earlier certificate should be protected, ought not to prevail the land record and registries, in order to ascertain the true condition of the title before
so as to deprive an innocent purchaser under the later certificate of his title of the earlier purchase, will, in many instances, add to the labor, expense and uncertainty of any
certificate contributed to the issuance of the later certificate. Hence the holder of the attempt by a purchaser to satisfy himself as to the validity of the title to lands purchased
earlier certificate of title should not be heard to invoke the "just and equitable rule" as by him.
laid down in the majority opinion, in order to have his own title protected and the title of
As I have said before, one of the principal objects, if not the principal object, of the
an innocent purchaser of a later certificate cancelled or annulled, in any case wherein it
torrens system of land registration upon which our Land Registration Act is avowedly
appears that the holder of the later certificate was wholly without fault, while the holder
modelled is to facilitate the transfer of real estate. To that end the Legislature
of the issuance of the later certificate, in that he might have prevented its issuance by
undertakes to relieve prospective purchasers and all others dealing in registered lands
merely entering his appearance in court in response to lawful summons personally
from the necessity of looking farther than the certificate of title to such lands furnished
served upon him in the course of the proceedings for the issuance of the second
by the Court of Land Registration, and I cannot, therefore, give my consent to a ruling
certificate, and pleading his superior rights under the earlier certificate, instead of
which charges a purchaser or mortgage of registered lands with notice of the contents
keeping silent and by his silence permitting a default judgment to be entered against
of every other certificate of title in the land registry, so that negligence and fault may be
him adjudicating title in favor of the second applicant.
imputed to him should he be exposed to loss or damages as a result of the lack of such
The majority opinion clearly recognizes the soundness of the principles I am contending knowledge.
for by reasoning (with which I am inclined to agree) whereby it undertakes to
Suppose a prospective purchaser of lands registered under the Land Registration Act
demonstrate that as between the original holders of the double or overlapping
desires to avoid the imputation of negligence in the event that, unknown to him, such
registration the general rule should prevail, because both such original parties must held
lands have been made the subject of double or overlapping registration, what course
to have been fault and, their equities being equal, preference should be given to the
should he pursue? What measures should he adopt in order to search out the
earlier title.
information with notice of which he is charged? There are no indexes to guide him nor is
The majority opinion further recognizes the soundness of my contention by the there anything in the record or the certificate of title of the land he proposes to buy
reasoning whereby it undertakes to sustain the application of the general rule in favor of which necessarily or even with reasonable probability will furnish him a clue as to the
the original holder of the earlier certificate against purchasers from the original holder of fact of the existence of such double or overlapping registration. Indeed the only course
the later certificate, by an attempt to demonstrate that such purchasers can in no event open to him, if he desires to assure himself against the possibility of double or
be held to be innocent purchasers; because, as it is said, negligence may and should overlapping registration, would even seem to be a careful, laborious and extensive
comparison of the registered boundary lines contained in the certificate of title of the and defend his title so as to defeat his right to the benefit of the equitable rule. It is true
tract of land he proposes to buy with those contained in all the earlier certificates of title that the order of publication in such cases having been duly complied with, all the world
to be found in the land registry. Assuredly it was never the intention of the author of the is charged with notice thereof, but it does not necessarily follow that, in the absence of
new Land Registration Act to impose such a burden on a purchaser of duly registered actual notice, culpable negligence in permitting a default judgment to be entered against
real estate, under penalty that a lack of the knowledge which might thus be acquired him may be imputed to the holder of the earlier certificate so as to defeat his right to the
may be imputed to him by this court as negligence in ruling upon the respective equities land under the equitable rule favoring the earlier certificate. Such a holding would have
of the holders of lands which have been the subject of double or overlapping the effect (to quote the language of the majority opinion) of requiring the holder of a
registration. certificate of title to wait indefinitely "in the portals of the court" and to sit in the "mirador
de su casa" in order to avoid the possibility of losing his lands; and I agree with the
On the other hand, I think that negligence and fault may fairly be imputed to a holder of
writer of the majority opinion that to do so would place an unreasonable burden on the
a registered certificate of title who stood supinely by and let a default judgment be
holders of such certificate, which was not contemplated by the authors of the Land
entered against him, adjudicating all or any part of his registered lands to another
Registration Act. But no unreasonable burden is placed upon the holder of a registered
applicant, if it appears that he was served with notice or had actual notice of the
title by a rule which imputes culpable negligence to him when he sits supinely by and
pendency of the proceedings in the Court of Land Registration wherein such default
lets a judgment in default be entered against him adjudicating title to his lands in favor
judgment was entered.
of another applicant, despite the fact that he has actual knowledge of the pendency of
The owner of land who enjoys the benefits secured to him by its registry in the Court of the proceedings in which such judgment is entered and despite the fact that he has
Land Registration may reasonably be required to appear and defend his title when he been personally served with summons to appear and default his title.
has actual notice that proceedings are pending in that court wherein another applicant,
"Taking into consideration all of the conditions and the diligence of the respective
claiming the land as his own, is seeking to secure its registry in his name. All that is
parties," it seems to me that there is no "equality in merit" between the conflicting
necessary for him to do is to enter his appearance in those proceedings, invite the
equities set up by an innocent purchaser who acquires title to the land under a
court's attention to the certificate of title registered in his name, and thus, at the cost of
registered certificate, and the holder of an earlier certificate who permitted a default
the applicant, avoid all the damage and inconvenience flowing from the double or
judgment to be entered against him, despite actual notice of the pendency of the
overlapping registration of the land in question. There is nothing in the new system of
proceedings in the course of which the later certificate was issued.
land registration which seems to render it either expedient or necessary to relieve a
holder of a registered title of the duty of appearing and defending that title, when he has I am convinced, furthermore, that aside from the superior equities of the innocent
actual notice that it is being attacked in a court of competent jurisdiction, and if, as a purchaser in cases such as that now under discussion, there are strong reasons of
result of his neglect or failure so to do, his lands become subject to double or convenience and public policy which militate in favor of the recognition of his title rather
overlapping registration, he should not be permitted to subject an innocent purchaser, than that of the holder of the earlier title.
holding under the later certificate, to all the loss and damage resulting from the double
One ruling exposes all persons purchasing or dealing in registered lands to unknown,
or overlapping registration, while he goes scot free and holds the land under a manifest
unspecified and uncertain dangers, to guard against which all such persons will be put
misapplication of the equitable rule that "where conflicting equities are otherwise equal
to additional cost, annoyance and labor on every occasion when any transaction is had
in merit, that which first accrued will be given the preference." It is only where both or
with regard to such lands; while the other ruling tends to eliminate consequences so
neither of the parties are at fault that the rule is properly applicable as between
directly adverse to the purpose and object for which the land registration law was
opposing claimants under an earlier and a later certificate of registry to the same land.
enacted, and imposes no burden upon any holder of a certificate of registered lands
Of course all that is said in the briefs of counsel and the majority opinion as to the right other than that of defending his title on those rare, definite and specific occasions
of the holder of a certificate to rest secure in his registered title so that those dealing wherein he has actual notice that his title is being challenged in a Court of Land
with registered lands can confidently rely upon registry certificates thereto is equally Registration, a proceeding in which the cost and expense is reduced to the minimum by
forceful by way of argument in favor of the holder of one or the other certificate in case the conclusive character of his certificate of title in support of his claim of ownership.
of double or overlapping registration. The problem is to determine which of the Furthermore, judgment against the innocent purchaser and in favor of the holder of the
certificate holders is entitled to the land. The decision of that question in favor of either earlier certificate in a case such as that under consideration must inevitably tend to
one must necessarily have the effect of destroying the value of the registered title of the increase the danger of double or overlapping registrations by encouraging holders of
other and to that extent shaking the public confidence in the value of the whole system registered titles, negligently or fraudulently and conclusively, to permit default judgments
for the registration of lands. But, in the language of the majority opinion, "that mistakes to be entered against them adjudicating title to all or a part of their registered lands in
are bound to occur cannot be denied and sometimes the damage done thereby is favor of other applicants, despite actual notice of the pendency of judicial proceedings
irreparable. It is the duty of the courts to adjust the rights of the parties under such had for that purpose, and this, without adding in any appreciable degree to the security
circumstances so as to minimize the damages, taking into consideration all the of thir titles, and merely to save them the very slight trouble or inconvenience incident to
conditions and the diligence of the respective parties to avoid them."lawphil.net an entry of appearance in the court in which their own titles were secured, and inviting
attention to the fact that their right, title and ownership in the lands in questions has
It will be observed that I limit the exception to the general equitable rule, as laid down in
already been conclusively adjudicated.
the majority opinion, to case wherein the holder of the earlier certificate of title has
actual notice of the pendency of the proceedings in the course of which the latter The cases wherein there is a practical possibility of double or overlapping registration
certificate of title was issued, or to cases in which he has received personal notice of the without actual notice to the holder of the earlier certificate must in the very nature of
pendency of those proceedings. Unless he has actual notice of the pendency of such things to be so rare as to be practically negligible. Double or overlapping registration
proceedings I readily agree with the reasoning of the majority opinion so far as it holds almost invariably occurs in relation to lands held by adjoining occupants or claimants. It
that negligence, culpable negligence, should not be imputed to him for failure to appear is difficult to conceive of a case wherein double registration can take place, in the
absence of fraud, without personal service of notice of the pendency of the proceedings Republic of the Philippines
upon the holder of the earlier certificate, the statute requiring such notice to be served SUPREME COURT
upon the owner or occupant of all lands adjoining those for which application for Manila
registration is made; and the cases wherein an adjoining land owner can, even by the
FIRST DIVISION
use of fraud, conduct proceedings for the registration of his land to a successful
conclusion without actual notice to the adjoining property owners must be rare indeed.
In the case at bar the defendant purchased the land in question from the original holder G.R. No. 86787 May 8, 1992
of a certificate of title issued by the Court of Land Registration, relying upon the records MILAGROS TUMULAK BISHOP, JUANA PANGILINAN, EMILIO MAXIMO, ANITA
of the Court of Land Registration with reference thereto and with no knowledge that any PANGILINAN, MAGDALENA ROSETE, MANUEL DACUT, RECTO DIESTA,
part of the land thus purchased was included in an earlier certificate of title issued to the VIRGINIA NOVICIO, and LINDA BONILLA, petitioners,
plaintiff. The plaintiff, the holder of the earlier certificate of title, negligently permitted a
vs.
default judgment to be entered against him in the Court of Land Registration, HONORABLE COURT OF APPEALS and SPOUSES MANUEL AND JESUSA
adjudicating part of the lands included in his own certificate of title in favor of another SALANG, respondents.
applicant, from whom the defendant in this action acquired title, and this despite the fact
that he was an adjoining land owner, had actual notice of the pendency of the Saturnino Bactad for petitioners.
proceedings and was personally served with summons to appear and defends his rights
in the premises. It seems to me that there can be no reason for doubt as to the
respective merits of the equities of the parties, and further that the judgment of the CRUZ, J.:
majority in favor of the plaintiff will inevitably tend to increase the number of cases The question presented in this case is not novel. As in previous cases resolving the
wherein registered land owners in the future will fail to appear and defend their titles same issue, the answer will not change.
when challenged in other proceedings in the Courts of Land Registration, thereby
enormously increasing the possibility and probability of loss and damage to innocent In dispute are certain portions of a parcel of land situated in Calapacuan, Subic,
third parties and dealers in registered lands generally, arising out of erroneous, double Zambales, with a total area of 1,652 square meters. These portions are in the
or overlapping registration of lands by the Courts of Land Registration. possession of the petitioners. The entire parcel is registered in the name of the private
respondents under Transfer Certificate of Title No. T-29018.
Carson, J., concurs.
On January 22, 1985, the private respondents sued the petitioners for recovery of
possession of the lots in question. The plaintiffs invoked their rights as registered
owners of the land. In their answer, the defendants claimed that the lots were part of the
public domain and could not have been registered under the Torrens system. All alleged
long and continuous possession of the lots and produced tax declarations in their
names. Two of them maintained that they had acquired their respective lots by virtue of
valid contracts of sale. Another based her claim on inheritance.
After trial, Judge Nicias O. Mendoza of the Regional Trial Court of Olongapo City
rendered judgment in favor of the plaintiffs. 1 He held in part as follows:
The plaintiffs, being the registered owners in fee simple of the land in
question, necessarily have the lawful right to the physical possession
of the land. The owner of a land has a right to enjoy and possess it,
and he has also the right to recover and repossess the same from
any person occupying it unlawfully.
Art. 428 New Civil Code
The owner has the right to enjoy and dispose of a
thing, without other limitations than those
established by law.
The owner has also a right of action against the
holder and possessor of the thing in order to
recover it.
There is, therefore, no doubt in law, that the plaintiffs being the
registered owners of the land in question have also the corresponding
right to the recovery and possession of the same. The defendants
who are in physical occupancy of the land belonging to the plaintiffs
have no right whatsoever to unjustly withhold the possession of the
said land from the plaintiffs. The defendants' occupancy of the land in legal basis. To be sure, a certification from an administrative body cannot prevail
question is unlawful and in violation of plaintiffs right to the recovery against court decision declaring the land to be registrable.
and possession of the land they owned. The evidence presented by
Significantly, it does not appear in the record that the Director of Forestry, or any other
the defendants claiming as per certifications of the Bureau of Forestry
representative of the Government for that matter, entered any opposition to the land
that the land occupied by them is within the alienable and disposable
registration proceedings that led to the issuance of the Original Certificate of Title. No
public land, deserves scant consideration as the said certification are
less importantly, an action to invalidate a certificate of title on the ground of fraud
without basis in law. The moment the land in question was titled in the
prescribes after the expiration of one (1) year from the entry of the decree of registration
name of the plaintiffs, it ceased to become a part of the public domain 4 and cannot now be resorted to by the petitioners at this late hour. And collaterally at
as the same became the private property of the registered owner, the
that.
herein plaintiffs. Tax declarations of the land made in the names of
the defendants are not evidence of title, it appearing that the land is The strange theory submitted by the petitioners that the owner of registered land must
already titled to the plaintiffs. The registration of the land in the names also possess it does not merit serious attention. The non-presentation by the private
of the defendants with the Assessor's Office for taxation purposes respondents of their tax declarations on the land is no indication that they have never
and the payments of real property taxes by the defendants can not acquired ownership thereof or have lost it by such omission.
and does not defeat the title of the plaintiffs to the land. The fact that
The second ground must also be rejected.
the defendants have been in occupancy of the land in question for
quite a period of time is of no moment as prescription will not ripen As registered owners of the lots in question, the private respondents have a right to
into ownership because the land is covered by a torrens title. eject any person illegally occupying their property. This right is imprescriptible. Even if it
Acquisitive prescription will not be available to land titled under Art. be supposed that they were aware of the petitioners' occupation of the property, and
496. regardless of the length of that possession, the lawful owners have a right to demand
the return of their property at any time as long as the possession was unauthorized or
PREMISED THEREFORE on the foregoing consideration, the Court merely tolerated, if at all. This right is never barred by laches.
finds and so holds that the plaintiffs being the registered owners of
the land in question are entitled to the possession of the same, and In urging laches against the private respondents for not protesting their long and
that the defendants who are occupying the land belonging to the continuous occupancy of the lots in question, the petitioners are in effect contending
plaintiffs in violation of the right of the latter, are duty-bound to restore that they have acquired the said lots by acquisitive prescription. It is an elementary
possession of the same to the titled owners, the herein plaintiffs. principle that the owner of a land registered under the Torrens system cannot lose it by
prescription. 5
On appeal, this decision was affirmed by the respondent court on August 22, 1988. 2
Their motion for reconsideration having been denied, the petitioners then came to this As the Court observed in the early case Legarda v. Saleeby: 6
Court, urging reversal of the courts below. The real purpose of the Torrens system of land registration is to quite
They allege that: title to land; to put a stop forever to any question of the legality of the
title, except claims which were noted at the time of registration in the
1. The land in question is part of the public domain and could not
certificate, or which may arise subsequent thereto. That being the
have been validly registered under the Torrens system.
purpose of the law, it would seem that once the title was registered,
2. The petitioners have acquired title to their respective lots by laches. the owner may rest secure, without the necessity of waiting in the
portals of the court, or sitting in the "mirador de su casa," to avoid the
3. In the alternative, they should be considered builders in good faith
possibility of losing his land.
entitled to the rights granted by Articles 448, 546, 547 and 548 of the
Civil Code. Applied consistently these many years, this doctrine has been burnished bright with use
and has long become a settled rule of law.
The petition has no merit.
In light of the observations already made, it is obvious that the petitioners cannot invoke
On the first ground, the Court notes that the private respondents' title is traceable to an
the status of builders in good faith to preserve their claimed rights to the constructions
Original Certificate of Title issued way back in 1910 or eighty-two years ago. That
they have made on the lots in dispute.
certificate is now incontrovertible and conclusive against the whole world. The
resumption of regularity applies to the issuance of that certificate. This presumption A builder in good faith is one who is unaware of any flaw in his title to the land at the
covers the finding that the land subject of the certificate was private in nature and time he builds on it. 7 This definition cannot apply to the petitioners because they knew
therefore registrable under the Torrens system. at the very outset that they had no right at all to occupy the subject lots.
To sustain an action for annulment of a Torrens certificate for being void ab initio, it The petitioners have consistently insisted that the lots were part of the public domain
must be shown that the registration court had not acquired jurisdiction over the case and even submitted a certification to that effect from the Bureau of Forestry. The land
and that there was actual fraud in securing the title. 3 Neither of these requirements has was in fact registered under the Torrens system and such registration was constructive
been established by the petitioners. All they submitted was the certification of the notice to the whole world, including the petitioners. Apparently, the petitioners did not
Bureau of Forestry that the land in question was alienable and disposable public land. take the trouble of checking such registration. At any rate, the point is that, whether the
The trial court was correct in ruling that this deserved scant consideration for lack of land be public or private, the petitioners knew they had no right to occupy it and build on
it. The Court of Appeals was correct in calling them squatters for having entered, Republic of the Philippines
without permission or authority, land that did not belong to them. SUPREME COURT
Manila
In urging reversal of the trial court and the respondent court, the petitioners are asking
us to overturn long established doctrines guaranteeing the integrity of the Torrens FIRST DIVISION
system and the indefeasibility of titles issued thereunder for the protection and peace of
G.R. No. L-68741 January 28, 1988
mind of the registered owner against illegal encroachments upon his property. We are
not disposed to take this drastic step on the basis alone of their feeble arguments. NATIONAL GRAINS AUTHORITY, plaintiff-appellee,
vs.
WHEREFORE, the petition is DENIED, with costs against the petitioners. It is so
INTERMEDIATE APPELLATE COURT, MELECIO MAGCAMIT, NENA COSICO and
ordered.
EMELITA MAGCAMIT, defendants-appellants.
Narvasa, C.J., Grio-Aquino, Medialdea and Bellosillo, JJ., concur.

PARAS, J.:
Footnotes
This is a petition for review of the decision of the then Intermediate Appellate Court *
1 Original Records, pp. 95-103. (now Court of Appeals) dated January 31, 1984, reversing the decision of the Court of
First Instance of Laguna and San Pablo City, 8th Judicial District, Branch III, and of the
2 Rollo, pp. 11-15; Melo, J., ponente, with Herrera, M. and Imperial,
resolution dated August 28, 1984 denying the motion for reconsideration filed thereof.
JJ., concurring.
The undisputed facts of this case as found by the Trial Court and the Intermediate
3 J.M. Tuason & Co., Inc. v. Macalindog, 6 SCRA 938; Bernardo v.
Appellate Court are as follows:
Siojo, 58 Phil. 89.
On December 2,1971, the spouses Paulino Vivas and Engracia Lizards, as owners of a
4 Sec. 38 of Act No. 496 (now Section 32 of PD 1529); Hernandez v.
parcel of land situated in Bo. San Francisco, Victoria, Laguna, comprising more or less
CA, 160 SCRA 821; Natalia Realty Corp. v. Vallez, 144 SCRA 292;
105,710 square meters, sold for P30,000.00 said property in favor of spouses Melencio
Municipality of Hagonoy v. Sec. of Agriculture and Natural Resources,
Magcamit and Nena Cosico, and Amelita Magcamit (herein private respondents) as
73 SCRA 507.
evidenced by "Kasulatan Ng Bilihang Mabiling Muli." This sale with right to repurchase
5 Sec. 47 of PD 1529; Natalia Realty Corp. v. Vallez, supra; Umbay v. was recorded in the Office of the Register of Deeds of Laguna on December 6,1971
Alecha, 135 SCRA 427; Barcelona, et al. v. Barcelona, et al., 100 under Act No. 3344. On January 31,1972 the sale was made absolute by the spouses
Phil. 251. Vivas and Lizardo in favor of the private respondents for the sum of P90,000.00;
P50,000.00 of which was paid upon the execution of the instrument, entitled "Kasulatan
6 31 Phil. 590.
Ng Bilihan Tuluyan," after being credited with the P30,000.00 consideration of the
7 Mercado v. Court of Appeals, 162 SCRA 75; Granados v. Monton, "Kasulatan Ng Mabibiling Muli," and the balance of P40,000.00 was to be paid the
86 Phil. 42; Caram v. Laureta, 103 SCRA 7; Arriola v. Gomez de la moment that the certificate of title is issued. From the execution of said Kasulatan,
Serna, 14 Phil. 627. private respondent have remained in peaceful, adverse and open possession of subject
property.
On February 26, 1975, an Original Certificate of Title No. T-1728 covering the property
in question was issued to and in the name of the spouses Vivas and Lizardo without the
knowledge of the private respondents and on April 30, 1975, said Spouses executed a
Special Power of Attorney in favor of Irenea Ramirez authorizing the latter to mortgage
the property with the petitioner, National Grains Authority.
On May 2, 1974, the counsel for the petitioner wrote the Provincial Sheriff in Sta. Cruz,
Laguna, requesting for the extrajudicial foreclosure of the mortgage executed by Irenea
Ramirez on May 18, 1975, covering, among others, the property involved in this case
covered by OCT No. T-1728, for unpaid indebtedness in the amount of P63,948.80 in
favor of the petitioner.
On May 31, 1974, the Provincial Sheriff caused the issuance of the notice of sale of the
property in question, scheduling the public auction sale on June 28, 1974. The petitioner
was the highest and successful bidder so that a Certificate of Sale was issued in its
favor on the same date by the Provincial Sheriff.
On July 10, 1974, the petitioner in its capacity as attorney-in-fact of the mortgagor sold
the subject real property in favor of itself. By virtue of the deed of absolute sale, TCT
No. T-75171 of the Register of Deeds for the Province of Laguna was issued in the
name of the petitioner on July 16, 1974. It was only in July 1974, that private
respondents learned that a title in the name of the Vivas spouses had been issued Authority to execute a deed of reconveyance sufficient in law for
covering the property in question and that the same property had been mortgaged in purposes of registration and cancellation of transfer Certificate of Title
favor of the petitioner. Private respondent Nena Magcamit offered to pay the petitioner No. T-75171 and the issuance of another title in the names of plaintiff-
NGA the amount of P40,000.00 which is the balance of the amount due the Vivas appellants, and ordering defendants-appellees Paulino Vivas and
spouses under the terms of the absolute deed of sale but the petitioner refused to Engracia Lizardo to pay the National Grains Authority the sum of
accept the payment. On July 31, 1974, counsel for private respondents made a formal P78,375.00 (Exh. 3) within thirty (30) days from the receipts of the writ
demand on the spouses Vivas and Lizardo to comply with their obligation under the of execution. No damages and costs. (Rollo, p. 19).
terms of the absolute deed of sale; and soon after reiterated to the NGA, the offer to pay
The petitioner filed a motion for reconsideration of the said decision but the same was
the balance of P40,000.00 due under the absolute deed of sale. On August 13, 1974
denied. (Rollo, p. 26).
petitioner in its reply informed counsel of private respondents that petitioner is now the
owner of the property in question and has no intention of disposing of the same. Hence, this petition.
The private respondents, who as previously stated, are in possession of subject In the resolution of May 20, 1985, the petition was given due course and the parties
property were asked by petitioner to vacate it but the former refused. Petitioner filed a were required to submit simultaneous memoranda (Rollo, p. 128). The memorandum for
suit for ejectment against private respondents in the Municipal Court of Victoria, the petitioner was filed on July 3, 1985 (Rollo, p. 129) while the memorandum for the
Laguna, but the case was dismissed. private respondents was filed on August 26, 1985 1 Rollo p. 192).
On June 4, 1975, private respondents filed a complaint before the then Court of First The main issue in this case is whether or not violation of the terms of the agreement
Instance of Laguna and San Pablo City, Branch III, San Pablo City, against the between the spouses Vivas and Lizardo, the sellers, and private respondents, the
petitioner and the spouses Vivas and Lizardo, praying, among others, that they be buyers, to deliver the certificate of title to the latter, upon its issuance, constitutes a
declared the owners of the property in question and entitled to continue in possession of breach of trust sufficient to defeat the title and right acquired by petitioner NGA, an
the same, and if the petitioner is declared the owner of the said property, then, to order innocent purchaser for value.
it to reconvey or transfer the ownership to them under such terms and conditions as the
It is undisputed that: (1) there are two deeds of sale of the same land in favor of private
court may find just, fair and equitable under the premises. (Record on Appeal, pp. 2-11).
respondents, namely: (a) the conditional sale with right to repurchase or the 'Kasulatan
In its answer to the complaint, the petitioner (defendant therein) maintained that it was Ng Bilihang Mabibiling Muli" which was registered under Act 3344 and (b) the deed of
never a privy to any transaction between the private respondents (plaintiffs therein) and absolute sale or "Kasulatan ng Bilihang Tuluyan" which was not registered; (2) the
the spouses Paulino Vivas and Engracia Lizardo that it is a purchaser in good faith and condition that the Certificate of Title will be delivered to the buyers upon its issuance
for value of the property formerly covered by OCT No. 1728; and that the title is now and upon payment of the balance of P40,000.00 is contained in the deed of absolute
indefeasible, hence, private respondents' cause of action has' already prescribed. sale; and (3) the land in question at the time of the execution of both sales was not yet
(Record on Appeal, pp. 16-22). covered by the Torrens System of registration.
After due hearing, the trial court ** rendered its decision on March 17, 1981, in favor of It is axiomatic, that while the registration of the conditional sale with right of repurchase
the petitioner, the dispositive portion of said judgment reading as follows: may be binding on third persons, it is by provision of law "understood to be without
prejudice to third party who has better right" (Section 194 of the Administrative Code, as
WHEREFORE, judgment is hereby rendered as follows:
amended by Act No. 3344). In this case, it will be noted that the third party NGA, is a
(1) declaring defendant National Grains Authority the lawful owner of registered owner under the Torrens System and has obviously a better right than private
the property in question by virtue of its indefeasible title to the same; respondents and that the deed of absolute sale with the suspensive condition is not
registered and is necessarily binding only on the spouses Vivas and Lizardo and private
(2) ordering plaintiffs to turn over possession of the land to defendant
respondents.
National Grains Authority;
In their complaint at the Regional Trial Court, private respondents prayed among others,
(3) ordering defendants-spouses Paulino Vivas and Engracia Lizardo
for two alternative reliefs, such as: (a) to be declared the owners of the property in
to pay plaintiffs the sum of P56,000.00 representing the amount paid
question or (b) to order the declared owner to reconvey or transfer the ownership of the
pursuant to the Kasulatan Ng Bilihang Tuluyan marked Exhibit "3",
property in their favor.
with legal interest thereon from January 31, 1972 until the amount is
paid, to pay an additional amount of P5,000.00 for and as attorney's Private respondents claim a better right to the property in question by virtue of the
fees, an additional amount of Pl0,000.00 as moral damages, another Conditional Sale, later changed to a deed of Absolute Sale which although unregistered
amount of P5,000.00 by way of exemplary damages and to pay the under the Torrens System allegedly transferred to them the ownership and the
costs of this suit. (Rollo, P. 35). possession of the property in question. In fact, they argue that they have been and are
still in possession of the same openly, continuously, publicly under a claim of ownership
The private respondents interposed an appeal from the decision of the trial court to the
adverse to all other claims since the purchase on December 2, 1971 (Rollo, p. 165). It is
Intermediate Appellate Court.
stressed that not until the month of July, 1974 did the plaintiff learn that a title had been
After proper proceedings, the appellate court rendered its decision on January 31, 1984, issued covering the property in question (Rollo, p. 15).
reversing and setting aside the decision of the trial court as follows:
Time and time again, this Court has ruled that the proceedings for the registration of title
WHEREFORE, the decision of the lower court is hereby reversed and to land under the Torrens System is an action in rem not in personam, hence, personal
set aside and another one is rendered ordering the National Grains notice to all claimants of the res is not necessary in order that the court may have
jurisdiction to deal with and dispose of the res. Neither may lack of such personal notice mortgagor and the NGA was the highest bidder in the public auction. Unquestionably,
vitiate or invalidate the decree or title issued in a registration proceeding, for the State, therefore, the NGA is an innocent purchaser for value, first as an innocent mortgagee
as sovereign over the land situated within it, may provide for the adjudication of title in a under Section 32 of P.D. 1529 and later as innocent purchaser for value in the public
proceeding in rem or one in the nature of or akin a to proceeding in rem which shall be auction sale.
binding upon all persons, known or unknown (Moscoso vs. Court of appeals, 128 SCRA
Private respondents claim that NGA did not even field any representative to the land
719 [1984], citing: City of Manila vs. Lack, et al., 19 Phil. 324, 337; Roxas vs. Enriquez,
which was not even in the possession of the supposed mortgagors, nor present any
29 Phil. 31; Director of Lands vs. Roman Catholic Archbishop of Manila, 41 Phil. 120;
witness to prove its allegations in the ANSWER nor submit its DEED OF MORTGAGE
Aguilar vs. Caogdan, 105 Phil. 661). It is thus evident that respondents' right over the
to show its being a mortgages in good faith and for value (Rollo, p. 110).
property was barred by res judicata when the decree of registration was issued to
spouses Vivas and Lizards. It does not matter that they may have had some right even Such contention is, however, untenable. Well settled is the rule that all persons dealing
the right of ownership, BEFORE the grant of the Torrens Title. with property covered by a torrens certificate of title are not required to go beyond what
appears on the face of the title. When there is nothing on the certificate of title to
Thus, under Section 44 of P.D. 1529, every registered owner receiving a certificate of
indicate any cloud or vice in the ownership of the property, or any encumbrance
title in pursuance of a decree of registration, and every subsequent purchaser of
thereon, the purchaser is not required to explore further than what the torrens title upon
registered land taking a certificate of title for value and in good faith, shall hold the same
its face indicates in quest for any hidden defect or inchoate right that may subsequently
free from all encumbrances except those noted on the certificate and any of the
defeat his right thereto (Centeno vs. Court of Appeals, 139 SCRA 545 [1985]).
encumbrances which may be subsisting, and enumerated in the law. Under said
provision, claims and liens of whatever character, except those mentioned by law as More specifically, the Court has ruled that a bank is not required before accepting a
existing, against the land prior to the issuance of certificate of title, are cut off by such mortgage to make an investigation of the title of the property being given as security
certificate if not noted thereon, and the certificate so issued binds the whole world, (Phil. National Cooperative Bank vs. Carandang Villalon, 139 SCRA 570 [1985]), and
including the government (Aldecoa and Co. vs. Warner Barns & Co., 30 Phil. 209 where innocent third persons like mortgagee relying on the certificate of title acquire
[1915]; Snyder vs. Fiscal of Cebu and Avila, 42 Phil. 766 [1922]). Under said ruling, if rights over the property, their rights cannot be disregarded (Duran vs. IAC, 138 SCRA
the purchaser is the only party who appears in the deeds and the registration of titles in 489 [1985]).
the property registry, no one except such purchaser may be deemed by law to be the
Under the circumstances, the Regional Trial Court could not have erred in ruling that
owner of the properties in question (Ibid). Moreover, no title to registered land in
plaintiffs (private respondents herein) complaint insofar as it prays that they be declared
derogation to that of the registered owner shall be acquired by prescription or adverse
owners of the land in question can not prosper in view of the doctrine of indefeasibility of
possession (Umbay vs. Alecha, 135 SCRA 427 [1985]).
title under the Torrens System, because it is an established principle that a petition for
It does not appear that private respondents' claim falls under any of the exceptions review of the decree of registration will not prosper even if filed within one year from the
provided for under Section 44 of P.D. 1529 which can be enforced against petitioner entry of the decree if the title has passed into the hands of an innocent purchaser for
herein. value (Pres. Decree No. 1529, Sec. 32). The setting aside of the decree of registration
issued in land registration proceedings is operative only between the parties to the fraud
Thus, it has been invariably restated by this Court, that "The real purpose of the Torrens
and the parties defrauded and their privies, but not against acquirers in good faith and
System is to quiet title to land and to stop forever any question as to its legality. "Once a
for value and the successors in interest of the latter; as to them the decree shall remain
title is registered, the owner may rest secure, without the necessity of waiting in the
in full force and effect forever (Domingo vs. The Mayon Realty Corp. et al., 102 Phil. 32
portals of the court, or sitting on the "mirador su casato," avoid the possibility of losing
[19571). Assuming, therefore, that there was fraud committed by the sellers against the
his land." "An indirect or collateral attack on a Torrens Title is not allowed (Dominga vs.
buyers in the instant case, petitioner NGA who was not privy therein cannot be made to
Santos, 55 Phil. 361; Singian vs. Manila Railroad, 62 Phil. 467)."
suffer the consequences thereof As correctly declared by the trial court, the National
The only exception to this rule is where a person obtains a certificate of title to a land Grains Authority is the lawful owner of the property in question by virtue of its
belonging to another and he has full knowledge of the rights of the true owner. He is indefeasible title.
then considered as guilty of fraud and he may be compelled to transfer the land to the
As to private respondents' alternative prayer that the declared owner be ordered to
defrauded owner so long as the property has not passed to the hands of an innocent
reconvey or transfer the ownership of the property in their favor, it is clear that there is
purchaser for value (Angeles vs. Sania, 66 Phil. 444 [1938], emphasis supplied).
absolutely no reason why petitioner, an innocent purchaser for value, should reconvey
It will be noted that the spouses Vivas and Lizardo never committed any fraud in the land to the private respondents.
procuring the registration of the property in question. On the contrary, their application
PREMISES CONSIDERED, the decision of the Court of Appeals is REVERSED and
for registration which resulted in the issuance of OCT No. 1728 was with complete
SET ASIDE, and the decision of the Court of First Instance of Laguna and San Pablo
knowledge and implied authority of private respondents who retained a portion of the
City, now Regional Trial Court, is REINSTATED.
consideration until the issuance to said spouses of a certificate of title applied for under
the Torrens Act and the corresponding delivery of said title to them. The question SO ORDERED.
therefore, is not about the validity of OCT No. 1728 but in the breach of contract Teehankee, C.J., Narvasa, Cruz and Gancayco, JJ., concur.
between private respondents and the Vivas spouses. Petitioner NGA was never a privy
to this transaction. Neither was it shown that it had any knowledge at the time of the
execution of the mortgage, of the existence of the suspensive condition in the deed of Footnotes
absolute sale much less of its violation. Nothing appeared to excite suspicion. The
Special Power of Attorney was regular on its face; the OCT was in the name of the
* IAC, 4th Civil Cases Division, penned by Justice Porfirio V.
D.B.T. MAR-BAY CONSTRUCTION, G.R. No. 167232
Sison,with the concurrence of Justices Abdulwahid A. Bidin,
INCORPORATED,
Mareelino R. Veloso and Desiderio P. Jurado.
** Presided by Judge Conrado T. Limcaoco. Petitioner,
Present:

YNARES-SANTIAGO, J.,
- versus -
Chairperson,

CHICO-NAZARIO,

VELASCO, JR.,

NACHURA, and
RICAREDO PANES, ANGELITO PANES,
SALVADOR CEA, ABOGADO MAUTIN, DONARDO PERALTA, JJ.
PACLIBAR, ZOSIMO PERALTA and HILARION
MANONGDO,

Respondents.

Promulgated:

July 31, 2009

x------------------------------------------------------------------------------------x

DECISION

NACHURA, J.:

THIRD DIVISION
Before this Court is a Petition1[1] for Review on Certiorari under Rule 45 of the respondents filed an Amended Complaint8[8] and a Second Amended Complaint9[9]
Rules of Civil Procedure, assailing the Court of Appeals (CA) Decision2[2] dated particularly impleading DBT as one of the defendants.
3
October 25, 2004 which reversed and set aside the Order [3] of the Regional Trial Court
(RTC) of Quezon City, Branch 216, dated November 8, 2001. In the Complaints, Ricaredo alleged that he is the lawful owner and claimant of
the subject property which he had declared for taxation purposes in his name, and
The Facts assessed in the amount of P2,602,190.00 by the City Assessor of Quezon City as of the
year 1985. Respondents alleged that per Certification10[10] of the Department of
Subject of this controversy is a parcel of land identified as Lot Plan Psu- Environment and Natural Resources (DENR) National Capital Region (NCR) dated May
123169,4[4] containing an area of Two Hundred Forty Thousand, One Hundred Forty- 7, 1992, Lot Plan Psu-123169 was verified to be correct and on file in said office, and
Six (240,146) square meters, and situated at Barangay (Brgy.) Pasong Putik, approved on July 23, 1948.
Novaliches, Quezon City (subject property). The property is included in Transfer
Certificate of Title (TCT) No. 200519,5[5] entered on July 19, 1974 and issued in favor of Respondents also claimed that Ricaredo, his immediate family members, and
B.C. Regalado & Co. (B.C. Regalado). It was conveyed by B.C. Regalado to petitioner the other respondents had been, and still are, in actual possession of the portions of the
D.B.T. Mar-Bay Construction, Inc. (DBT) through a dacion en pago6[6] for services subject property, and their possession preceded the Second World War. To perfect his
rendered by the latter to the former. title in accordance with Act No. 496 (The Land Registration Act) as amended by
Presidential Decree (P.D.) No. 1529 (The Property Registration Decree), Ricaredo filed
On June 24, 1992, respondents Ricaredo P. Panes (Ricaredo), his son with the RTC of Quezon City, Branch 82 a case docketed as LRC Case No. Q-91-011,
Angelito P. Panes (Angelito), Salvador Cea, Abogado Mautin, Donardo Paclibar, with LRC Rec. No. N-62563.11[11]
Zosimo P. Peralta, and Hilarion Manongdo (herein collectively referred to as
respondents) filed a Complaint7[7] for Quieting of Title with Cancellation of TCT No. Respondents averred that in the process of complying with the publication
200519 and all Titles derived thereat (sic), Damages, with Petition for the Issuance of requirements for the Notice of Initial Hearing with the Land Registration Authority (LRA),
Injunction with Prayer for the Issuance of Restraining Order Ex-Parte, Etc. against B.C. it was discovered by the Mapping Services of the LRA that there existed an overlapping
Regalado, Mar-Bay Realty, Inc., Spouses Gereno Brioso and Criselda M. Brioso, of portions of the land subject of Ricaredos application, with the subdivision plan of B.C.
Spouses Ciriaco and Nellie Mariano, Avelino C. Perdido and Florentina Allado, Regalado. The said portion had, by then, already been conveyed by B.C. Regalado to
Eufrocina A. Maborang and Fe Maborang, Spouses Jaime and Rosario Tabangcura, DBT.
Spouses Oscar Ikalina and the Register of Deeds (RD) of Quezon City. Subsequently,
Ricaredo asseverated that upon verification with the LRA, he found that the
subdivision plan of B.C. Regalado was deliberately drawn to cover portions of the

[1]
1
Rollo, pp. 3-19. subject property. Respondents claimed that the title used by B.C. Regalado in the
2
[2] Particularly docketed as CA-G.R. CV No. 75550, penned by Associate preparation of the subdivision plan did not actually cover the subject property. They
Justice Eloy R. Bello, Jr., with Associate Justices Regalado E. Maambong and asserted that from the records of B.C. Regalado, they gathered that TCT Nos.
Lucenito N. Tagle, concurring; rollo, pp. 22-36.
[3]
3
Rollo, pp. 82-85. [8]
8
Id. at 197-209.
[4]
4
Records, Vol. 1, p. 15. [9]
9
Id. at 266-278.
[5]
5
Records, Vol. 3, pp. 723-739.
[6]
6
Id. at 740-755. 10
[10] Id. at 16.
[7]
7
Records, Vol. 1, pp. 1-13. 11
[11] Id. at 17-20.
211081,12[12] 21109513[13] and 211132,14[14] which allegedly included portions of the Estate. Other lots included therein are Lot 890-B of Psd 36854, Lot 2 of (LRC) Pcs
subject property, were derived from TCT No. 200519. However, TCT No. 200519 only 12892 and Lot 3 of (LRC) Pcs 12892. Thus, respondents' allegation that Lots 661, 664,
covered Lot 503 of the Tala Estate with an area of Twenty-Two Thousand Six Hundred 665, 693 and 694 of the Tala Estate were not included in TCT No. 200519 was not true.
Fifteen (22,615) square meters, and was different from those mentioned in TCT Nos.
211081, 211095 and 211132. According to respondents, an examination of TCT No. On December 28, 1993, then defendants Spouses Jaime and Rosario
200519 would show that it was derived from TCT Nos. 14814,15[15] 14827,16[16] Tabangcura (Spouses Tabangcura) filed their Answer20[20] with Counterclaim, claiming
1481517[17] and T-28. that they were buyers in good faith and for value when they bought a house and lot
covered by TCT No. 211095 from B.C. Regalado, the latter being a subdivision
In essence, respondents alleged that B.C. Regalado and DBT used the developer and registered owner thereof, on June 30, 1986. When respondent Abogado
derivative titles which covered properties located far from Pasong Putik, Novaliches, Mautin entered and occupied the property, Spouses Tabangcura filed a case for
Quezon City where the subject property is located, and B.C. Regalado and DBT then Recovery of Property before the RTC, Quezon City, Branch 97 which rendered a
offered the same for sale to the public. Respondents thus submitted that B.C Regalado decision21[21] in their favor.
and DBT through their deliberate scheme, in collusion with others, used (LRC) Pcs-
18345 as shown in the consolidation-subdivision plan to include the subject property
covered by Lot Plan Psu-123169.
On its part, DBT, traversing the complaint, alleged that it is the legitimate
In his Answer18[18] dated July 24, 1992, the RD of Quezon City interposed the owner and occupant of the subject property pursuant to a dacion en pago executed by
defense that at the time of registration, he found all documents to be in order. B.C. Regalado in the formers favor; that respondents were not real parties-in-interests
Subsequently, on December 5, 1994, in his Motion19[19] for Leave to Admit Amended because Ricaredo was a mere claimant whose rights over the property had yet to be
Answer, with the Amended Answer attached, he admitted that he committed a grave determined by the RTC where he filed his application for registration; that the other
mistake when he earlier said that TCT No. 200519 covered only one lot, i.e. Lot 503. He respondents did not allege matters or invoke rights which would entitle them to the relief
averred that upon careful examination, he discovered that TCT No. 200519 is prayed for in their complaint; that the complaint was premature; and that the action
composed of 17 pages, and actually covered 54 lots, namely: Lots 503, 506, 507, 508, inflicted a chilling effect on the lot buyers of DBT.22[22]
509, 582, 586, 655, 659, 686, 434, 495, 497, 299, 498, 499, 500, 501, 502, 493, 692,
776, 496, 785, 777, 786, 780, 783, 505, 654, 660, 661, 663, 664, 665, 668, 693, 694, The RTC's Rulings

713, 716, 781, 779, 784, 782, 787, 893, 1115, 1114, 778, 669 and 788, all of the Tala
On June 15, 2000, the RTC through Judge Marciano I. Bacalla (Judge

[12]
12
Id. at 21. Bacalla), rendered a Decision23[23] in favor of the respondents. The RTC held that the

[13]
13
Id. at 22. testimony of Ricaredo that he occupied the subject property since 1936 when he was
only 16 years old had not been rebutted; that Ricaredo's occupation and cultivation of
14
[14] In the pleadings filed by respondents, they alleged that the
aforementioned TCT bore the number 211152. However, a perusal of the said the subject property for more than thirty (30) years in the concept of an owner vested in
title reveals that the TCT bears the number 211132; Records, Vol. 1, p. 288. him equitable ownership over the same by virtue of an approved plan, Psu 123169; that
[15]
15
Records, Vol. 1, p. 290.
[16]
16
Id. at 291. [20]
20
Id. at 350-354.
[17]
17
Id. at 292. [21]
21
Penned by former RTC Judge Oscar Leviste.
[18]
18
Id. at 49-50. [22]
22
Records, Vol. 1, pp. 355-358.
[19]
19
Id. at 395-397. [23]
23
Rollo, pp. 56-61.
the subject property was declared under the name of Ricaredo for taxation superiority of OCT 779 be upheld; and that the subject property be declared as
purposes;24[24] and that the subject property per survey should not have been included belonging to the Estate of Don Pedro/Don Jose de Ocampo.
in TCT No. 200519, registered in the name of B.C. Regalado and ceded to DBT. The
RTC further held that Spouses Tabangcura failed to present satisfactory evidence to In its Order28[28] dated March 13, 2001, the RTC, through Acting Judge
prove their claim. Thus, the RTC disposed of the case in this wise: Modesto C. Juanson (Judge Juanson), denied Atty. Pulumbarits Motion for Intervention
because a judgment had already been rendered pursuant to Section 2, 29[29] Rule 19 of
WHEREFORE, in view of the foregoing considerations, the 1997 Rules of Civil Procedure.
judgment is hereby rendered declaring Certificate of Title No. 200519
and all titles derived thereat as null and void insofar as the same
embrace the land covered by Plan PSU-123169 with an area of On April 10, 2001, the RTC issued an Order30[30] stating that there appeared
240,146 square meters in the name of Ricaredo Panes; ordering to be a need for a clarificatory hearing before it could act on DBT's Motion for
defendant DBT Marbay Realty, Inc. to pay plaintiff Ricaredo Panes
the sum of TWENTY THOUSAND (P20,000) pesos as attorneys fees Reconsideration. Thus, a hearing was held on May 17, 2001. Thereafter, supplemental
plus costs of suit. memoranda were required of the parties.31[31] Both parties complied.32[32] However,
SO ORDERED. having found that the original copy of TCT No. 200519 was not submitted to it for
comparison with the photocopy thereof on file, the RTC directed DBT to present the
original or certified true copy of the TCT on August 21, 2001.33[33] Respondents moved
On September 12, 2000, DBT filed a Motion25[25] for Reconsideration, based
to reconsider the said directive34[34] but the same was denied.35[35] DBT, on the other
on the grounds of prescription and laches. DBT also disputed Ricaredos claim of open,
hand, manifested that a copy of TCT No. 200519, consisting of 17 pages, had already
adverse, and continuous possession of the subject property for more than thirty (30)
been admitted in evidence; and that because of the fire in the Office of the RD in
years, and asserted that the subject property could not be acquired by prescription or
Quezon City sometime in 1988, DBT, despite diligent effort, could not secure an
adverse possession because it is covered by TCT No. 200519.
original or certified true copy of said TCT. Instead, DBT submitted a certified true copy

While the said Motion for Reconsideration was pending, Judge Bacalla passed of Consolidated Subdivision Plan Pcs 18345.36[36]

away.

Meanwhile, on January 2, 2001, a Motion26[26] for Intervention and a


Complaint in Intervention were filed by Atty. Andres B. Pulumbarit (Atty. Pulumbarit),
representing the Don Pedro/Don Jose de Ocampo Estate. The intervenor alleged that [28]
28
Id. at 866.
the subject property formed part of the vast tract of land with an area of 117,000 29
[29] SEC. 2. Time to intervene.- The motion to intervene may be filed at
hectares, covered by Original Certificate of Title (OCT) No. 779 issued by the any time before rendition of judgment by the trial court. A copy of the pleading-
in-intervention shall be attached to the motion and served on the original
Honorable Norberto Romualdez on March 14, 1913 under Decree No. 10139, which parties.
belongs to the Estate of Don Pedro/Don Jose de Ocampo. Thus, the Complaint27[27] in
[30]
30
Records, Vol. 3, p. 867.
Intervention prayed that the RTCs Decision be reconsidered; that the legitimacy and
[31]
31
Id. at 884.
[32]
32
Id. at 885-888 and 890-893.
[24]
24
Records, Vol. 2, pp. 709-710. [33]
33
Id. at 894.
[25]
25
Records, Vol. 3, pp. 799-808. [34]
34
Id. at 896-900.
[26]
26
Id. at 837-838. [35]
35
Id. at 902.
[27]
27
Id. at 839-843. [36]
36
Id. at 903-906.
On November 8, 2001, the RTC, through Judge Juanson, issued an Petitioner filed a Motion for Reconsideration,42[42] which was, however, denied
Order37[37] reversing the earlier RTC Decision and dismissing the Complaint for lack of by the CA in its Resolution43[43] dated February 22, 2005.
merit. The RTC held that prescription does not run against registered land; hence, a
title once registered cannot be defeated even by adverse, open or notorious Hence, this Petition.
possession. Moreover, the RTC opined that even if the subject property could be
acquired by prescription, respondents' action was already barred by prescription and/or
laches because they never asserted their rights when B.C. Regalado registered the
The Issues
subject property in 1974; and later developed, subdivided and sold the same to
individual lot buyers.
Petitioner raises the following as grounds for this Petition:

On December 18, 2001, respondents filed a Motion for Reconsideration 38[38]


I.
which the RTC denied in its Order39[39] dated June 17, 2002. Aggrieved, respondents
appealed to the CA.40[40] PETITIONER'S FAILURE TO ALLEGE PRESCRIPTION IN ITS
ANSWER IS NOT A WAIVER OF SUCH DEFENSE.
The CA's Ruling
II.

On October 25, 2004, the CA reversed and set aside the RTC Orders dated IT IS NOT ERRONEOUS TO REQUIRE THE PRODUCTION OF A
November 8, 2001 and June 17, 2002 and reinstated the RTC Decision dated June 15, CERTIFIED TRUE COPY OF TCT NO. 200519 AFTER THE
2000. The CA held that the properties described and included in TCT No. 200519 are DECISION ON THE MERITS HAS BEEN RENDERED BUT
BEFORE IT BECAME FINAL.
located in San Francisco del Monte, San Juan del Monte, Rizal and Cubao, Quezon
City while the subject property is located in Brgy. Pasong Putik, Novaliches, Quezon III.
City. Furthermore, the CA held that Engr. Vertudazo's testimony that there is a gap of
A REGISTERED LAND CAN NOT BE ACQUIRED BY ACQUISITIVE
around 1,250 meters between Lot 503 and Psu 123169 was not disproved or refuted.
PRESCRIPTION.
The CA found that Judge Juanson committed a procedural infraction when he
entertained issues and admitted evidence presented by DBT in its Motion for IV.
Reconsideration which were never raised in the pleadings and proceedings prior to the
THE TESTIMONY OF ENGR. VERTUDAZO ON THE BASIS OF
rendition of the RTC Decision. The CA opined that DBT's claims of laches and THE TECHNICAL DESCRIPTION OF LOT 503 IN AN INCOMPLETE
prescription clearly appeared to be an afterthought. Lastly, the CA held that DBT's DOCUMENT IS UNRELIABLE.
Motion for Reconsideration was not based on grounds enumerated in the Rules of
V.
Procedure.41[41]
MR. PANES HAS NEVER BEEN IN OPEN, ADVERSE AND
CONTINUOUS POSSESSION OF THE SUBJECT PROPERTY FOR
[37]
37
Rollo, pp. 82-85. MORE THAN THIRTY (30) YEARS.44[44]

[38]
38
Id. at 86-92. [42]
42
Rollo, pp. 150-163.
[39]
39
Id. at 101-103. [43]
43
Id. at 37-38.
[40]
40
Records, Vol. 3, pp. 939-940. 44

[41]
41
Supra note 2. [44] Supra note 1 at 7-8.
Distilled from the petition and the responsive pleadings, and culled from the judgment, and adherence to its decision would cause injustice. 46[46] Thus, the RTC
arguments of the parties, the issues may be reduced to two questions, namely: in its Order dated November 8, 2001 could validly entertain the defenses of prescription
and laches in DBT's motion for reconsideration.
1) Did the RTC err in upholding DBT's defenses of prescription and laches as
raised in the latter's Motion for Reconsideration? However, the conclusion reached by the RTC in its assailed Order was
erroneous. The RTC failed to consider that the action filed before it was not simply for
2) Which between DBT and the respondents have a better right over the reconveyance but an action for quieting of title which is imprescriptible.
subject property?
Verily, an action for reconveyance can be barred by prescription. When an
Our Ruling action for reconveyance is based on fraud, it must be filed within four (4) years from
discovery of the fraud, and such discovery is deemed to have taken place from the
We answer the first question in the affirmative. issuance of the original certificate of title. On the other hand, an action for reconveyance
based on an implied or constructive trust prescribes in ten (10) years from the date of
It is true that in Dino v. Court of Appeals45[45] we ruled:
the issuance of the original certificate of title or transfer certificate of title. The rule is that
the registration of an instrument in the Office of the RD constitutes constructive notice to
(T)rial courts have authority and discretion to dismiss an
action on the ground of prescription when the parties' pleadings or the whole world and therefore the discovery of the fraud is deemed to have taken place
other facts on record show it to be indeed time-barred; (Francisco v. at the time of registration.47[47]
Robles, Feb. 15, 1954; Sison v. McQuaid, 50 O.G. 97; Bambao v.
Lednicky, Jan. 28, 1961; Cordova v. Cordova, Jan. 14, 1958;
Convets, Inc. v. NDC, Feb. 28, 1958; 32 SCRA 529; Sinaon v. However, the prescriptive period applies only if there is an actual need to
Sorongan, 136 SCRA 408); and it may do so on the basis of a
reconvey the property as when the plaintiff is not in possession of the property. If the
motion to dismiss (Sec. 1, [f] Rule 16, Rules of Court), or an answer
which sets up such ground as an affirmative defense (Sec. 5, Rule plaintiff, as the real owner of the property also remains in possession of the property,
16), or even if the ground is alleged after judgment on the
the prescriptive period to recover title and possession of the property does not run
merits, as in a motion for reconsideration (Ferrer v. Ericta, 84
SCRA 705); or even if the defense has not been asserted at all, against him. In such a case, an action for reconveyance, if nonetheless filed, would be
as where no statement thereof is found in the pleadings (Garcia
in the nature of a suit for quieting of title, an action that is imprescriptible. 48[48] Thus, in
v. Mathis, 100 SCRA 250; PNB v. Pacific Commission House, 27
SCRA 766; Chua Lamco v. Dioso, et al., 97 Phil. 821); or where a Vda. de Gualberto v. Go,49[49] this Court held:
defendant has been declared in default (PNB v. Perez; 16 SCRA
270). What is essential only, to repeat, is that the facts
demonstrating the lapse of the prescriptive period be otherwise
sufficiently and satisfactorily apparent on the record; either in
the averments of the plaintiff's complaint, or otherwise [46]
46
Mauricio v. National Labor Relations Commission, G.R. No. 164635,
established by the evidence. (Emphasis supplied)
November 17, 2005, 475 SCRA 323, 331, citing Tocao v. Court of Appeals,
G.R. No. 127405, September 20, 2001, 365 SCRA 463, 464; and Astraquillo v.
Indeed, one of the inherent powers of courts is to amend and control its Javier, G.R. No. L-20034, January 30, 1965, 13 SCRA 125.
processes so as to make them conformable to law and justice. This includes the right to
[47]
47
Millena v. Court of Appeals, 381 Phil. 132, 138 (2000).
reverse itself, especially when in its opinion it has committed an error or mistake in
[48]
48
Aguirre v. Heirs of Lucas Villanueva, G.R. No. 169898, June 8, 2007,
524 SCRA 492, 494.
[49]
49
G.R. No. 139843, July 21, 2005, 463 SCRA 671, 681, citing
[45] 411 Phil. 594, 603-604 (2001), citing Gicano v. Gegato, No. L-63575,
45 Development Bank of the Phils. v. CA, G.R. No. 129471, April 28, 2000, 331
January 20, 1988, 157 SCRA 140. SCRA 267, 270.
[A]n action for reconveyance of a parcel of land based on implied or to avoid recognizing a right when to do so would result in a clearly inequitable
constructive trust prescribes in ten years, the point of reference
situation.52[52]
being the date of registration of the deed or the date of the issuance
of the certificate of title over the property, but this rule applies only
when the plaintiff or the person enforcing the trust is not in Albeit the conclusion of the RTC in its Order dated November 8, 2001, which
possession of the property, since if a person claiming to be the
owner thereof is in actual possession of the property, as the dismissed respondents' complaint on grounds of prescription and laches, may have
defendants are in the instant case, the right to seek reconveyance, been erroneous, we, nevertheless, resolve the second question in favor of DBT.
which in effect seeks to quiet title to the property, does not
prescribe. The reason for this is that one who is in actual
possession of a piece of land claiming to be the owner thereof may It is a well-entrenched rule in this jurisdiction that no title to registered land in
wait until his possession is disturbed or his title is attacked before
taking steps to vindicate his right, the reason for the rule being, that derogation of the rights of the registered owner shall be acquired by prescription or
his undisturbed possession gives him a continuing right to seek the adverse possession.53[53]
aid of a court of equity to ascertain and determine the nature of the
adverse claim of a third party and its effect on his own title, which
right can be claimed only by one who is in possession. Article 112654[54] of the Civil Code in connection with Section 4655[55] of Act
No. 496 (The Land Registration Act), as amended by Section 4756[56] of P.D. No. 1529
(The Property Registration Decree), clearly supports this rule. Prescription is unavailing
not only against the registered owner but also against his hereditary successors.
Insofar as Ricaredo and his son, Angelito, are concerned, they established in Possession is a mere consequence of ownership where land has been registered under
their testimonies that, for some time, they possessed the subject property and that the Torrens system, the efficacy and integrity of which must be protected. Prescription is
Angelito bought a house within the subject property in 1987. 50[50] Thus, the rightly regarded as a statute of repose whose objective is to suppress fraudulent and
respondents are proper parties to bring an action for quieting of title because persons stale claims from springing up at great distances of time and surprising the parties or
having legal, as well as equitable, title to or interest in a real property may bring such
action, and title here does not necessarily denote a certificate of title issued in favor of
the person filing the suit.51[51] [52]
52
Maestrado v. Court of Appeals, 384 Phil. 418, 430 (2000).
[53]
53
Abadiano v. Martir, G.R. No. 156310, July 31, 2008, 560 SCRA 676,
Although prescription and laches are distinct concepts, we have held, 693; Ragudo v. Fabella Estate Tenants Association, Inc., G.R. No. 146823,
nonetheless, that in some instances, the doctrine of laches is inapplicable where the August 9, 2005, 466 SCRA 136, 148; Alcantara-Daus v. Sps. De Leon, 452
Phil. 92, 102 (2003); Velez, Sr. v. Rev. Demetrio, 436 Phil. 1, 9 (2002); Villegas
action was filed within the prescriptive period provided by law. Therefore, laches will not
v. Court of Appeals, 403 Phil. 791, 801 (2001); Bishop v. Court of Appeals,
apply to this case, because respondents' possession of the subject property has G.R. No. 86787, May 8, 1992, 208 SCRA 636, 641; and Barcelona, et. al. v.
rendered their right to bring an action for quieting of title imprescriptible and, hence, not Barcelona and Ct. of Appeals, 100 Phil. 251, 256-257 (1956).
barred by laches. Moreover, since laches is a creation of equity, acts or conduct alleged [54] ARTICLE 1126. Against a title recorded in the Registry of Property,
54

ordinary prescription of ownership or real rights shall not take place to the
to constitute the same must be intentional and unequivocal so as to avoid injustice.
prejudice of a third person, except in virtue of another title also recorded; and
Laches will operate not really to penalize neglect or sleeping on one's rights, but rather the time shall begin to run from the recording of the latter.
As to the lands registered under the Land Registration Act, the
provisions of that special law shall govern.
[55] SECTION 46. No title to registered land in derogation to that of the
55

registered owner shall be acquired by prescription or adverse possession.


[50]
50
TSN, February 2, 1996, pp. 53-55.
[56] SECTION 47. Registered land not subject to prescription. No title to
56

[51]
51
Art. 477, New Civil Code; Mamadsual v. Moson, G.R. No. 92557, registered land in derogation of the title of the registered owner shall be
September 27, 1990, 190 SCRA 82, 89. acquired by prescription or adverse possession.
their representatives when the facts have become obscure from the lapse of time or the Dacion en pago is the delivery and transmission of ownership of a thing by the
defective memory or death or removal of witnesses.57[57] debtor to the creditor as an accepted equivalent of the performance of the obligation. It
is a special mode of payment where the debtor offers another thing to the creditor, who
Thus, respondents' claim of acquisitive prescription over the subject property is accepts it as an equivalent of the payment of an outstanding debt. In its modern
baseless. Under Article 1126 of the Civil Code, acquisitive prescription of ownership of concept, what actually takes place in dacion en pago is an objective novation of the
lands registered under the Land Registration Act shall be governed by special laws. obligation where the thing offered as an accepted equivalent of the performance of an
Correlatively, Act No. 496, as amended by PD No. 1529, provides that no title to obligation is considered as the object of the contract of sale, while the debt is
registered land in derogation of that of the registered owner shall be acquired by considered as the purchase price.62[62]
adverse possession. Consequently, in the instant case, proof of possession by the
respondents is immaterial and inconsequential.58[58]

Moreover, it may be stressed that there was no ample proof that DBT It must also be noted that portions of the subject property had already been
participated in the alleged fraud. While factual issues are admittedly not within the sold to third persons who, like DBT, are innocent purchasers in good faith and for value,
province of this Court, as it is not a trier of facts and is not required to re-examine or relying on the certificates of title shown to them, and who had no knowledge of any
contrast the oral and documentary evidence anew, we have the authority to review and, defect in the title of the vendor, or of facts sufficient to induce a reasonably prudent man
in proper cases, reverse the factual findings of lower courts when the findings of fact of to inquire into the status of the subject property.63[63] To disregard these circumstances
the trial court are in conflict with those of the appellate court. 59[59] In this regard, we simply on the basis of alleged continuous and adverse possession of respondents
reviewed the records of this case and found no clear evidence that DBT participated in
nor by any proceeding in any court for reversing judgment, subject, however,
the fraudulent scheme. In Republic v. Court of Appeals,60[60] this Court gave due
to the right of any person, including the government and the branches thereof,
importance to the fact that the private respondent therein did not participate in the fraud deprived of land or of any estate or interest therein by such adjudication or
averred. We accord the same benefit to DBT in this case. To add, DBT is an innocent confirmation of title obtained by actual fraud, to file in the proper Court of First
purchaser for value and good faith which, through a dacion en pago duly entered into Instance a petition for reopening and review of the decree of registration not
with B.C. Regalado, acquired later than one year from and after the date of the entry of such decree of
registration, but in no case shall such petition be entertained by the court
where an innocent purchaser for value has acquired the land or an
ownership over the subject property, and whose rights must be protected under Section
interest therein whose rights may be prejudiced. Whenever the phrase
3261[61] of P.D. No. 1529.
"innocent purchaser for value" or an equivalent phrase occurs in this
Decree, it shall be deemed to include an innocent lessee, mortgagee, or
other encumbrancer for value.
[57]
57
Gallardo v. Intermediate Appellate Court, G.R. No. L-67742, October
29, 1987, 155 SCRA 248, 260. (Citations omitted) Upon the expiration of said period of one year, the decree of
[58]
58
Feliciano v. Zaldivar, G.R. No. 162593, September 26, 2006, 503 registration and the certificate of title issued shall become incontrovertible. Any
SCRA 182, 197, citing Natalia Realty Corporation v. Vallez, et al., G.R. Nos. person aggrieved by such decree of registration in any case may pursue his
78290-94, May 23, 1989, 173 SCRA 534. remedy by action for damages against the applicant or any other person
[59]
59
Tan v. Court of Appeals, 421 Phil. 134, 141-142 (2001). responsible for the fraud (Emphasis supplied).
[60]
60
G.R. No. 116111, January 21, 1999, 301 SCRA 366, 370. [62]
62
Uy v. Sandiganbayan, G.R. No. 111544, July 6, 2004, 433 SCRA 424,
[61]
61
SECTION 32. Review of decree of registration; Innocent purchaser for 438. (Citations omitted)
value. The decree of registration shall not be reopened or revised by reason of [63]
63
Agag v. Alpha Financing Corporation, G.R. No. 154826, July 31, 2003,
absence, minority, or other disability of any person adversely affected thereby, 407 SCRA 602, 610.
would not only be inimical to the rights of the aforementioned titleholders, but would new judgment is hereby entered DISMISSING the Complaint filed by the respondents
ultimately wreak havoc on the stability of the Torrens system of registration. for lack of merit.

A final note.

While the Torrens system is not a mode of acquiring title, but merely a system SO ORDERED.
of registration of titles to lands, justice and equity demand that the titleholder should not
be made to bear the unfavorable effect of the mistake or negligence of the State's
agents, in the absence of proof of his complicity in a fraud or of manifest damage to
third persons. The real purpose of the Torrens system is to quiet title to land and put a
stop forever to any question as to the legality of the title, except claims that were noted
in the certificate at the time of the registration or that may arise subsequent thereto.
Otherwise, the integrity of the Torrens system would forever be sullied by the ineptitude
and inefficiency of land registration officials, who are ordinarily presumed to have
regularly performed their duties.64[64] Thus, where innocent third persons, relying on
the correctness of the certificate of title thus issued, acquire rights over the property, the
court cannot disregard those rights and order the cancellation of the certificate. The
effect of such outright cancellation will be to impair public confidence in the certificate of
title. The sanctity of the Torrens system must be preserved; otherwise, everyone
dealing with the property registered under the system will have to inquire in every
instance on whether the title had been regularly or irregularly issued, contrary to the
evident purpose of the law. Every person dealing with the registered land may safely
rely on the correctness of the certificate of title issued therefor, and the law will in no
way oblige him to go behind the certificate to determine the condition of the
property.65[65]

WHEREFORE, the instant Petition is GRANTED and the assailed Court of


Appeals Decision dated October 25, 2004 is hereby REVERSED and SET ASIDE. A

[64]
64
Republic v. Guerrero, G.R. No. 133168, March 28, 2006, 485 SCRA
424, 445.
[65]
65
Republic v. Orfinada, Sr., G.R. No. 141145, November 12, 2004, 442
SCRA 342, 359, citing Heirs of Spouses Benito Gavino and Juana Euste v.
Court of Appeals, G.R. No. 120154, June 29, 1998, 291 SCRA 495, 509.
GUTIERREZ, JR., J.:
Eduardo S. Baranda and Alfonso Hitalia were the petitioners in G.R. No. 64432 and the
private respondents in G.R. No. 62042. The subject matter of these two (2) cases and
the instant case is the same a parcel of land designated as Lot No. 4517 of the
Cadastral Survey of Sta. Barbara, Iloilo covered by Original Certificate of Title No. 6406.
The present petition arose from the same facts and events which triggered the filing of
the earlier petitions. These facts and events are cited in our resolution dated December
29, 1983 in G.R. No. 64432, as follows:
. . . This case has its origins in a petition for reconstitution of title filed
with the Court of First Instance of Iloilo involving a parcel of land
known as Lot No. 4517 of the Sta. Barbara Cadastre covered by
Original Certificate of Title No. 6406 in the name of Romana Hitalia.
Eventually, Original Certificate of Title No. 6406 was cancelled and
Transfer Certificate of Title No. 106098 was issued in the names of
Alfonso Hitalia and Eduardo S. Baranda The Court issued a writ of
possession which Gregorio Perez, Maria P. Gotera and Susana Silao
refused to honor on the ground that they also have TCT No. 25772
over the same Lot No. 4517. The Court, after considering the private
respondents' opposition and finding TCT No. 25772 fraudulently
acquired, ordered that the writ of possession be carried out. A motion
for reconsideration having been denied, a writ of demolition was
issued on March 29, 1982. Perez and Gotera filed a petition for
certiorari and prohibition with the Court of Appeals. On August 6,
1982, the Court of Appeals denied the petition. Perez and Gotera filed
the petition for review on certiorari denominated as G.R. No. 62042
before the Supreme Court. As earlier stated the petition was denied in
a resolution dated January 7,1983. The motion for reconsideration
was denied in another resolution dated March 25, 1983, which also
stated that the denial is final. This decision in G.R. No. 62042, in
accordance with the entry of judgment, became final on March 25,
1983. The petitioners in the instant case G.R. No. 64432--contend
that the writs of possession and demolition issued in the respondent
court should now be implemented; that Civil Case No. 00827 before
the Intermediate Appellate Court was filed only to delay the
implementation of the writ; that counsel for the respondent should be
held in contempt of court for engaging in a concerted but futile effort to
delay the execution of the writs of possession and demolition and that
petitioners are entitled to damages because of prejudice caused by
the filing of this petition before the Intermediate Appellate Court. On
September 26, 1983, this Court issued a Temporary Restraining
Order ' to maintain the status quo, both in the Intermediate Appellate
G.R. No. 81163 September 26, 1988 Court and in the Regional Trial Court of Iloilo. Considering that (l)there
is merit in the instant petition for indeed the issues discussed in G.R.
EDUARDO S. BARANDA and ALFONSO HITALIA, petitioners,
No. 64432 as raised in Civil Case No. 00827 before the respondent
vs.
court have already been passed upon in G.R. No. 62042; and (2) the
HONORABLE JUDGE TITO GUSTILO, ACTING REGISTER OF DEEDS AVITO
Temporary Restraining Order issued by the Intermediate Appellate
SACLAUSO, HONORABLE COURT OF APPEALS, and ATTY. HECTOR P.
Court was only intended not to render the petition moot and academic
TEODOSIO, respondents.
pending the Court's consideration of the issues, the Court
Eduardo S. Baranda for petitioners. RESOLVED to DIRECT the respondent Intermediate Appellate Court
not to take cognizance of issues already resolved by this Court and
Rico & Associates for private respondents. accordingly DISMISS the petition in Civil Case No. 00827. Immediate
implementation of the writs of possession and demolition is likewise 2. To cancel No.T-25772. Likewise to cancel No.T-106098 and once
ordered. (pp. 107-108, Rollo G.R. No. 64432) cancelled to issue new certificates of title to each of Eduardo S.
Baranda and Alfonso Hitalia;
On May 9, 1984, the Court issued a resolution denying with finality a motion for
reconsideration of the December 29, 1983 resolution in G.R. No. 64432. On this same Plus other relief and remedies equitable under the premises. (p. 473,
date, another resolution was issued, this time in G.R. No. 62042, referring to the 64432 Rollo)
Regional Trial Court of Iloilo the ex-parte motion of the private respondents (Baranda
and Hitalia) for execution of the judgment in the resolutions dated January 7, 1983 and Acting on these motions, we issued on September 17,1986 a Resolution in G.R. No.
March 9, 1983. In the meantime, the then Intermediate Appellate Court issued a 62042 and G.R. No. 64432 granting the motions as prayed for. Acting on another
resolution dated February 10, 1984, dismissing Civil Case No. 00827 which covered the motion of the same nature filed by the petitioners, we issued another Resolution dated
same subject matter as the Resolutions above cited pursuant to our Resolution dated October 8, 1986 referring the same to the Court Administrator for implementation by the
December 29, 1983. The resolution dated December 29, 1983 in G.R. No. 64432 judge below.
became final on May 20, 1984. In compliance with our resolutions, the Regional Trial Court of Iloilo, Branch 23 presided
Upon motions of the petitioners, the Regional Trial Court of Iloilo, Branch 23 presided by by Judge Tito G. Gustilo issued two (2) orders dated November 6,1986 and January
Judge Tito G. Gustilo issued the following order: 6,1987 respectively, to wit:

Submitted are the following motions filed by movants Eduardo S. ORDER


Baranda and Alfonso Hitalia through counsel dated August 28, 1984: This is an Ex-parte Motion and Manifestation submitted by the
(a) Reiterating Motion for Execution of Judgment of Resolutions dated movants through counsel on October 20, 1986; the Manifestation of
January 7, 1983 and March 9, 1983 Promulgated by Honorable Atty. Helen Sornito, Register of Deeds of the City of Iloilo, and
Supreme Court (First Division) in G.R. No. 62042; formerly acting register of deeds for the Province of Iloilo dated
October 23, 1986 and the Manifestation of Atty. Avito S. Saclauso,
(b) Motion for Execution of Judgment of Resolution dated December Acting Register of Deeds, Province of Iloilo dated November 5, 1986.
29, 1983 Promulgated by Honorable Supreme Court (First Division) in
G.R. No. 64432; Considering that the motion of movants Atty. Eduardo S. Baranda and
Alfonso Hitalia dated August 12, 1986 seeking the full implementation
(c) The Duties of the Register of Deeds are purely ministerial under of the writ of possession was granted by the Honorable Supreme
Act 496, therefore she must register all orders, judgment, resolutions Court, Second Division per its Resolution dated September 17,1986,
of this Court and that of Honorable Supreme Court. the present motion is hereby GRANTED.
Finding the said motions meritorious and there being no opposition WHEREFORE, the Acting Register of Deeds, Province of Iloilo, is
thereto, the same is hereby GRANTED. hereby ordered to register the Order of this Court dated September 5,
1984 as prayed for.
WHEREFORE, Transfer Certificate of Title No. T-25772 is hereby
declared null and void and Transfer Certificate of Title No. T-106098 xxx xxx xxx
is hereby declared valid and subsisting title concerning the ownership
of Eduardo S. Baranda and Alfonso Hitalia, all of Sta. Barbara ORDER
Cadastre. This is a Manifestation and Urgent Petition for the Surrender of
The Acting Register of Deeds of Iloilo is further ordered to register the Transfer Certificate of Title No. T-25772 submitted by the petitioners
Subdivision Agreement of Eduardo S. Baranda and Alfonso Hitalia as Atty. Eduardo S. Baranda and Alfonso Hitalia on December 2, 1986,
prayed for." (p. 466, Rollo--G.R. No. 64432) in compliance with the order of this Court dated November 25, 1 986,
a Motion for Extension of Time to File Opposition filed by Maria
The above order was set aside on October 8, 1984 upon a motion for reconsideration Provido Gotera through counsel on December 4, 1986 which was
and manifestation filed by the Acting Registrar of Deeds of Iloilo, Atty. Helen P. Sornito, granted by the Court pursuant to its order dated December 15, 1986.
on the ground that there was a pending case before this Court, an Action for Considering that no Opposition was filed within the thirty (30) days
Mandamus, Prohibition, Injunction under G.R. No. 67661 filed by Atty. Eduardo period granted by the Court finding the petition tenable, the same is
Baranda, against the former which remained unresolved. hereby GRANTED.
In view of this development, the petitioners filed in G.R. No. 62042 and G.R. No. 64432 WHEREFORE, Maria Provido Gotera is hereby ordered to surrender
ex-parte motions for issuance of an order directing the Regional Trial Court and Acting Transfer Certificate of Title No. T-25772 to this Court within ten (10)
Register of Deeds to execute and implement the judgments of this Court. They prayed days from the date of this order, after which period, Transfer
that an order be issued: Certificate of Title No. T-25772 is hereby declared annulled and the
Register of Deeds of Iloilo is ordered to issue a new Certificate of Title
1. Ordering both the Regional Trial Court of Iloilo Branch XXIII, under in lieu thereof in the name of petitioners Atty. Eduardo S. Baranda and
Hon. Judge Tito G. Gustilo and the acting Register of Deeds Helen P. Alfonso Hitalia, which certificate shall contain a memorandum of the
Sornito to register the Order dated September 5, 1984 of the lower annulment of the outstanding duplicate. (pp. 286-287, Rollo 64432)
court;
On February 9, 1987, Atty. Hector Teodosio, the counsel of Gregorio Perez, private The records show that after the Acting Register of Deeds annotated a notice of is
respondent in G.R. No. 64432 and petitioner in G.R. No. 62042, filed a motion for pendens on the new certificates of titles issued in the name of the petitioners, the
explanation in relation to the resolution dated September 17, 1986 and manifestation petitioners filed in the reconstitution case an urgent ex-parte motion to immediately
asking for clarification on the following points: cancel notice of lis pendens annotated thereon.
a. As to the prayer of Atty. Eduardo Baranda for the cancellation of In his order dated February 12, 1987, respondent Judge Gustilo granted the motion and
TCT T-25772, should the same be referred to the Court of Appeals directed the Acting Register of Deeds of Iloilo to cancel the lis pendens found on
(as mentioned in the Resolution of November 27, 1985) or is it already Transfer Certificate of Title Nos. T-106098; T-111560; T-111561 and T-111562.
deemed granted by implication (by virtue of the Resolution dated
September 17, 1986)? Respondent Acting Register of Deeds Avito Saclauso filed a motion for reconsideration
of the February 12, 1987 order stating therein:
b. Does the Resolution dated September 17, 1986 include not only
the implementation of the writ of possession but also the cancellation That the undersigned hereby asks for a reconsideration of the said
of TCT T-25772 and the subdivision of Lot 4517? (p. 536, Rollo order based on the second paragraph of Section 77 of P.D. 1529, to
4432) wit:

Acting on this motion and the other motions filed by the parties, we issued a resolution "At any time after final judgment in favor of the
dated May 25, 1987 noting all these motions and stating therein: defendant or other disposition of the action such as
to terminate finally all rights of the plaintiff in and to
xxx xxx xxx the land and/or buildings involved, in any case in
which a memorandum or notice of Lis Pendens has
Since entry of judgment in G.R. No. 62042 was made on January 7, been registered as provided in the preceding
1983 and in G.R. No. 64432 on May 30, 1984, and all that remains is section, the notice of Lis Pendens shall be deemed
the implementation of our resolutions, this COURT RESOLVED to cancelled upon the registration of a certificate of the
refer the matters concerning the execution of the decisions to the clerk of court in which the action or proceeding was
Regional Trial Court of Iloilo City for appropriate action and to apply pending stating the manner of disposal thereof."
disciplinary sanctions upon whoever attempts to trifle with the
implementation of the resolutions of this Court. No further motions in That the lis pendens under Entry No. 427183 was annotated on T-
these cases will be entertained by this Court. (p. 615, Rollo-64432) 106098, T-111560, T-111561 and T-111562 by virtue of a case
docketed as Civil Case No. 15871, now pending with the Intermediate
In the meantime, in compliance with the Regional Trial Court's orders dated November Court of Appeals, entitled, "Calixta Provido, Ricardo Provido, Sr.,
6, 1986 and January 6, 1987, Acting Register of Deeds AvitoSaclauso annotated the Maria Provido and Perfecto Provido, Plaintiffs, versus Eduardo
order declaring Transfer Certificate of Title No. T-25772 as null and void, cancelled the Baranda and Alfonso Hitalia, Respondents."
same and issued new certificates of titles numbers T-111560, T-111561 and T-111562
in the name of petitioners Eduardo S. Baranda and Alfonso Hitalia in lieu of Transfer That under the above-quoted provisions of P.D. 152, the cancellation
Certificate of TItle No. T-106098. of subject Notice of Lis Pendens can only be made or deemed
cancelled upon the registration of the certificate of the Clerk of Court
However, a notice of lis pendens "on account of or by reason of a separate case (Civil
in which the action or proceeding was pending, stating the manner of
Case No. 15871) still pending in the Court of Appeals" was carried out and annotated in disposal thereof.
the new certificates of titles issued to the petitioners. This was upheld by the trial court
after setting aside its earlier order dated February 12, 1987 ordering the cancellation of Considering that Civil Case No. 1587, upon which the Notice of Lis
lis pendens. Pendens was based is still pending with the Intermediate Court of
Appeals, only the Intermediate Court of Appeals and not this
This prompted the petitioners to file another motion in G.R, No. 62042 and G.R. No. Honorable Court in a mere cadastral proceedings can order the
64432 to order the trial court to reinstate its order dated February 12, 1987 directing the cancellation of the Notice of Lis Pendens. (pp. 68-69, Rollo)
Acting Register of Deeds to cancel the notice of lis pendens in the new certificates of
titles. Adopting these arguments and on the ground that some if not all of the plaintiffs in Civil
Case No. 15871 were not privies to the case affected by the Supreme Court resolutions,
In a resolution dated August 17, 1987, we resolved to refer the said motion to the respondent Judge Tito Gustilo set aside his February 12, 1987 order and granted the
Regional Trial Court of Iloilo City, Branch 23 for appropriate action. Acting Register of Deeds' motion for reconsideration.
Since respondent Judge Tito Gustilo of the Regional Trial Court of Iloilo, Branch 23 The issue hinges on whether or not the pendency of the appeal in Civil Case No. 15871
denied the petitioners' motion to reinstate the February 12, 1987 order in another order with the Court of Appeals prevents the court from cancelling the notice of lis pendens in
dated September 17, 1987, the petitioners filed this petition for certiorari, prohibition and the certificates of titles of the petitioners which were earlier declared valid and
mandamus with preliminary injunction to compel the respondent judge to reinstate his subsisting by this Court in G.R. No. 62042 and G.R. No. 64432. A corollary issue is on
order dated February l2, 1987 directing the Acting Register of Deeds to cancel the the nature of the duty of a Register of Deeds to annotate or annul a notice of lis
notice of lis pendens annotated in the new certificates of titles issued in the name of the pendens in a torrens certificate of title.
petitioners.
Civil Case No. 15871 was a complaint to seek recovery of Lot No. 4517 of Sta. Barbara
Cadastre Iloilo, (the same subject matter of G.R. No 62042 and G.R. No. 64432) from RuIes of Court; Jamora v. Duran, et al., 69 Phil. 3, 11; I Martin, Rules
petitioners Baranda and Hitalia filed by Calixta Provido, Ricardo Provido, Maxima of Court, p. 415, footnote 3, citing cases.) (Natanov. Esteban, 18
Provido and Perfecta Provido before the Regional Trial Court of Iloilo, Branch 23. At the SCRA 481, 485-486)
instance of Atty. Hector P. Teodosio, the Provides' counsel, a notice of is pendens was
annotated on petitioners' Certificate of Title No. T-106098 covering Lot No. 4517, Sta. The private respondents are not entitled to this protection. The facts obtaining in this
Barbara Cadastre. case necessitate the application of the rule enunciated in the cases of Victoriano v.
Rovila (55 Phil. 1000), Municipal Council of Paranaque v. Court of First Instance of
Acting on a motion to dismiss filed by the petitioners, the court issued an order dated Rizal (70 Phil., 363) and Sarmiento v. Ortiz (10 SCRA 158), to the effect that:
October 24, 1984 dismissing Civil Case No. 15871.
We have once held that while ordinarily a notice of pendency which
The order was then appealed to the Court of Appeals. This appeal is the reason why has been filed in a proper case, cannot be cancelled while the action
respondent Judge Gustilo recalled the February 12, 1987 order directing the Acting is pending and undetermined, the proper court has the discretionary
Register of Deeds to cancel the notice of lis pendens annotated on the certificates of power to cancel it under peculiar circumstances, as for instance,
titles of the petitioners. where the evidence so far presented by the plaintiff does not bear out
the main allegations of his complaint, and where the continuances of
This petition is impressed with merit. the trial, for which the plaintiff is responsible, are unnecessarily
Maria Provido Gotera was one of the petitioners in G.R. No. 62042. Although Calixta delaying the determination of the case to the prejudice of the
Provido, Ricardo Provido, Maxima Provido and Perfecta Provido, the plaintiffs in Civil defendant. (Victoriano v. Rovira, supra; The Municipal Council of
Case No. 15871 were not impleaded as parties, it is very clear in the petition that Maria Paranaque v. Court of First Instance of Rizal, supra)
Provido was acting on behalf of the Providos who allegedly are her co-owners in Lot No. The facts of this case in relation to the earlier cases brought all the way to the Supreme
4517, Sta. Barbara Cadastre as shown by Transfer Certificate of Title No. T-25772 Court illustrate how the private respondents tried to block but unsuccessfuly the already
issued in her name and the names of the plaintiffs in Civil Case No. 15871, among final decisions in G.R. No. 62042 and G.R. No. 64432.
others. (Annex "E" G.R. No. 62042, p. 51, Rollo) In fact, one of the issues raised by
petitioners Maria Provido Gotera and Gregoria Perez in G.R. No. 62042 was as follows: Parenthetically, respondent Judge Tito Gustilo abused his discretion in sustaining the
respondent Acting Register of Deeds' stand that, the notice of lis pendens in the
xxx xxx xxx certificates of titles of the petitioners over Lot No. 4571, Barbara Cadastre cannot be
2. Whether or not, in the same reconstitution proceedings, respondent cancelled on the ground of pendency of Civil Case No. 15871 with the Court of Appeals.
Judge Midpantao L. Adil had the authority to declare as null and void In upholding the position of the Acting Register of Deeds based on Section 77 of
the transfer certificate of title in the name of petitioner Maria Provido Presidential Decree No. 1529, he conveniently forgot the first paragraph thereof which
Gotera and her other co-owners. (p. 3, Rollo; Emphasis supplied) provides:

It thus appears that the plaintiffs in Civil Case No. 15871 were privies to G.R. No. 62042 Cancellation of lis pendens. Before final judgment, a notice of lis
contrary to the trial court's findings that they were not. pendens may be cancelled upon Order of the Court after proper
showing that the notice is for the purpose of molesting the adverse
G.R. No. 62042 affirmed the order of the then Court of First Instance of Iloilo in the party, or that it is not necessary to protect the rights of the party who
reconstitution proceedings declaring TCT No. 25772 in the name of Providos over Lot caused it to be registered. It may also be cancelled by the Register of
No. 4517, Sta. Barbara Cadastre null and void for being fraudulently obtained and Deeds upon verified petition of the party who caused the registration
declaring TCT No. 106098 over the same parcel Lot No. 4517, Sta. Barbara Cadastre in thereof.
the name of petitioners Eduardo Baranda and Alfonso Hitalia valid and subsisting.
This Court cannot understand how respondent Judge Gustilo could have been misled
The decision in G.R. No. 62042 became final and executory on March 25,1983 long by the respondent Acting Register of Deeds on this matter when in fact he was the
before Civil Case No. 15871 was filed. same Judge who issued the order dismissing Civil Case No. 15871 prompting the
Under these circumstances, it is crystal clear that the Providos, private respondents private respondents to appeal said order dated October 10, 1984 to the Court of
herein, in filing Civil Case No. 15871 were trying to delay the full implementation of the Appeals. The records of the main case are still with the court below but based on the
final decisions in G.R. No. 62042 as well as G.R. No. 64432 wherein this Court ordered order, it can be safely assumed that the various pleadings filed by the parties
immediate implementation of the writs of possession and demolition in the reconstitution subsequent to the motion to dismiss filed by the petitioners (the defendants therein)
proceedings involving Lot No. 4517, Sta. Barbara Cadastre. touched on the issue of the validity of TCT No. 25772 in the name of the Providos over
Lot Number 4571, Sta. Barbara Cadastre in the light of the final decisions in G.R. No.
The purpose of a notice of lis pendens is defined in the following manner: 62042 and G.R. No. 64432.
Lis pendens has been conceived to protect the real rights of the party The next question to be determined is on the nature of the duty of the Register of Deeds
causing the registration thereof With the lis pendens duly recorded, he to annotate and/or cancel the notice of lis pendens in a torrens certificate of title.
could rest secure that he would not lose the property or any part of it.
For, notice of lis pendens serves as a warning to a prospective Section 10, Presidential Decree No. 1529 states that "It shall be the duty of the Register
purchaser or incumbrancer that the particular property is in litigation; of Deeds to immediately register an instrument presented for registration dealing with
and that he should keep his hands off the same, unless of course he real or personal property which complies with all the requisites for registration. ... . If the
intends to gamble on the results of the litigation. (Section 24, Rule 14, instrument is not registrable, he shall forthwith deny registration thereof and inform the
presentor of such denial in writing, stating the ground or reasons therefore, and advising
him of his right to appeal by consulta in accordance with Section 117 of this Decree."
Section 117 provides that "When the Register of Deeds is in doubt with regard to the
proper step to be taken or memoranda to be made in pursuance of any deed, mortgage
or other instrument presented to him for registration or where any party in interest does
not agree with the action taken by the Register of Deeds with reference to any such
instrument, the question shall be submitted to the Commission of Land Registration by
the Register of Deeds, or by the party in interest thru the Register of Deeds. ... ."
The elementary rule in statutory construction is that when the words and phrases of the
statute are clear and unequivocal, their meaning must be determined from the language
employed and the statute must be taken to mean exactly what it says. (Aparri v. Court
of Appeals, 127 SCRA 231; Insular Bank of Asia and America Employees' Union
[IBAAEU] v. Inciong, 132 SCRA 663) The statute concerning the function of the
Register of Deeds to register instruments in a torrens certificate of title is clear and
leaves no room for construction. According to Webster's Third International Dictionary of
the English Language the word shall means "ought to, must, ...obligation used to
express a command or exhortation, used in laws, regulations or directives to express
what is mandatory." Hence, the function of a Register of Deeds with reference to the
registration of deeds encumbrances, instruments and the like is ministerial in nature.
The respondent Acting Register of Deeds did not have any legal standing to file a
motion for reconsideration of the respondent Judge's Order directing him to cancel the
notice of lis pendens annotated in the certificates of titles of the petitioners over the
subject parcel of land. In case of doubt as to the proper step to be taken in pursuance of
any deed ... or other instrument presented to him, he should have asked the opinion of
the Commissioner of Land Registration now, the Administrator of the National Land Title
and Deeds Registration Administration in accordance with Section 117 of Presidential
Decree No. 1529.
In the ultimate analysis, however, the responsibility for the delays in the full
implementation of this Court's already final resolutions in G.R. No. 62042 and G.R. No.
64432 which includes the cancellation of the notice of lis pendens annotated in the
certificates of titles of the petitioners over Lot No. 4517 of the Sta. Barbara Cadastre
falls on the respondent Judge. He should never have allowed himself to become part of
dilatory tactics, giving as excuse the wrong impression that Civil Case No. 15871 filed
by the private respondents involves another set of parties claiming Lot No. 4517 under
their own Torrens Certificate of Title. G.R. No. L-20611 May 8, 1969
WHEREFORE, the instant petition is GRANTED. The February 12, 1987 order of the AURELIO BALBIN and FRANCISCO BALBIN, petitioners,
Regional Trial Court of Iloilo, Branch 23 is REINSTATED. All subsequent orders issued vs.
by the trial court which annulled the February 12, 1987 order are SET ASIDE. Costs REGISTER OF DEEDS OF ILOCOS SUR, respondent.
against the private respondents.
Vicente Llanes for petitioners.
SO ORDERED. Office of the Solicitor General for respondent.
Manuel A. Argel for respondents third parties affected.
MAKALINTAL, J.:
Appeal from the resolution of the Commissioner of Land Registration in LRC Consulta
No. 366.
On November 15, 1961 petitioners presented to the register of deeds of Ilocos Sur a
duplicate copy of the registered owner's certificate of title (OCT No. 548) and an
instrument entitled "Deed of Donation inter-vivos," with the request that the same be
annotated on the title. Under the terms of the instrument sought to be annotated one
Cornelio Balbin, registered owner of the parcel of land described in OCT No. 548,
appears to have donated inter-vivos an undivided two-thirds (/) portion thereof in favor
of petitioners. The entire area of the land is 11.2225 hectares. Unsatisfied, petitioners referred the matter to the Commissioner of Land Registration,
who subsequently upheld the action of the Register of Deeds in a resolution dated April
The register of deeds denied the requested annotation for being "legally defective or 10, 1962. With respect to the principal point in controversy, the Commissioner
otherwise not sufficient in law." It appears that previously annotated in the observed:
memorandum of encumbrances on the certificate are three separate sales of undivided
portions of the land earlier executed by Cornelio Balbin in favor of three different buyers. (1) It appears that the donor is now merely a co-owner of the property
The pertinent entries read: described in the Original Certificate of Title No. 548, having previously sold
undivided portions thereof on three different occasions in favor of three
Entry No. 5658. Sales. different buyers. Consequently, aside from the owner's duplicate issued to
Cornelio Balbin, there are now three co-owner's duplicates which are
Sale for the sum of P400.00 executed by the registered owner, conveying presumably in the possession of the three buyers. Accordingly, in addition to
an undivided portion of an area of 3,710 square meters only in favor of the owner's duplicate of Original Certificate of Title No. 548, the three co-
Florentino Gabayan, this Original Certificate of Title No. 548 is hereby owner's duplicates must likewise be surrendered. The claim of counsel for the
cancelled with respect to said area of 3,710 square meters and in lieu donees that the issuance of the three co-owner's duplicates was unauthorized
thereof, the name of the vendee ... is hereby substituted to succeed to all is beside the point. Unless and until a court of competent jurisdiction rules to
rights, participation in interest of the vendor. ... the contrary, these titles are presumed to have been lawfully
issued.lawphi1.et
Date of Instrument: January 25, 1955, ... Without presenting those three (3) other duplicates of the title, petitioners would want to
compel annotation of the deed of donation upon the copy in their possession, citing
xxx xxx xxx section 55 of Act 496, which provides that "the production of the owner's duplicate
certificate of title whenever any voluntary instrument is presented for registration shall
be conclusive authority from the registered owner to the register of deeds to make a
Entry No. 5659. Sale of portion.
memorandum of registration in accordance with such instrument." Under this provision,
according to petitioners, the presentation of the other copies of the title is not required,
Sale for the sum of P100.00 executed by the registered owner, conveying first, because it speaks of "registered owner" and not one whose claim to or interest in
an undivided portion of an area of 16,713 square meters in favor of the property is merely annotated on the title, such as the three vendees-co-owners in
Roberto Bravo, this Original Certificate of Title No. 548 is hereby this case; and secondly, because the issuance of the duplicate copies in their favor was
cancelled with respect to said undivided portion ... and in lieu thereof the illegal or unauthorized.
name of the vendee ... is hereby substituted to succeed to all rights,
participation and interest of the vendor ... We find no merit in petitioners' contention. Section 55, supra, obviously assumes that
there is only one duplicate copy of the title in question, namely, that of the registered
owner himself, such that its production whenever a voluntary instrument is presented
Date of Instrument: June 9, 1953. ... constitutes sufficient authority from him for the register of deeds to make the
corresponding memorandum of registration. In the case at bar, the three other copies of
Entry No. 5660. Sale of portion. the title were in existence, presumably issued under section 43 * of Act 496. As correctly
observed by the Land Registration Commissioner, petitioners' claim that the issuance of
Sale for the sum of P400.00 executed by the registered owner, those copies was unauthorized or illegal is beside the point, its legality being presumed
conveying an undivided portion of an area of 15,000 square until otherwise declared by a court of competent jurisdiction. There being several copies
meters in favor of Juana Gabayan, this Certificate of Title No. of the same title in existence, it is easy to see how their integrity may be adversely
548 is hereby cancelled with respect to said undivided portion ... affected if an encumbrance, or an outright conveyance, is annotated on one copy and
and in lieu thereof the name of the vendee ... is hereby not on the others. The law itself refers to every copy authorized to be issued as a
substituted to succeed to all rights, participation and interest of duplicate of the original, which means that both must contain identical entries of the
the vendor ... transactions, particularly voluntary ones, affecting the land covered by the title. If this
were not so, if different copies were permitted to carry differing annotations, the whole
system of Torrens registration would cease to be reliable.
Date of Instrument: February 12, 1952. ...
One other ground relied upon by the Land Registration Commissioner in upholding the
action taken by the Register of Deeds of Ilocos Sur is that since the property subject of
The final part of the annotations referring to the abovementioned sales contains an
the donation is presumed conjugal, that is, property of the marriage of the donor,
additional memorandum stating that "three co-owner's duplicate certificates of title No.
Cornelio Balbin, and his deceased wife, Nemesia Mina, "there should first be a
548 have been issued (by the register of deeds of Ilocos Sur) in the name of Florentino
liquidation of the partnership before the surviving spouse may make such a
Gabayan, Roberto Bravo and Juana Gabayan upon verbal request of Mr. Andres
conveyance." This legal conclusion may appear too general and sweeping in its
Cabeldo, Notary Public of Caoayan, I. Sur, for and in the name of the vendees, this 5th
implications, for without a previous settlement of the partnership a surviving spouse may
day of January, 1956 at Vigan, I. Sur." Mainly because these three other co-owner's
dispose of his aliquot share or interest therein subject of course to the result of future
copies of the certificate of title No. 548 had not been presented by petitioners, the
liquidation. Nevertheless, it is not to be denied that, if the conjugal character of the
Register of Deeds refused to make the requested annotation.
property is assumed, the deed of donation executed by the husband, Cornelio Balbin, situated in the municipality of Esperanza, province of Agusan, and covered by original
bears on its face an infirmity which justified the denial of its registration, namely, the fact certificate of title P-1237 in the name of "Arcenio Abalo, married to Nicolasa M. Abalo."
that the two-thirds portion of said property which he donated was more than his one-half Sometime in May, 1962 Almirol went to the office of the Register of Deeds of Agusan in
share, not to say more than what remained of such share after he had sold portions of Butuan City to register the deed of sale and to secure in his name a transfer certificate
the same land to three other parties. of title. Registration was refused by the Register of Deeds upon the following grounds,
inter alia, stated in his letter of May 21, 1962:
It appears that there is a case pending in the Court of First Instance of Ilocos Sur (CC
No. 2221), wherein the civil status of the donor Cornelio Balbin and the character of the 1. That Original Certificate of Title No. P-1237 is registered in the name of
land in question are in issue, as well as the validity of the different conveyances Arcenio Abalo, married to Nicolasa M. Abalo, and by legal presumption, is
executed by him. The matter of registration of the deed of donation may well await the considered conjugal property;
outcome of that case, and in the meantime the rights of the interested parties could be
protected by filing the proper notices of lis pendens. 2. That in the sale of a conjugal property acquired after the effectivity of the
New Civil Code it is necessary that both spouses sign the document; but
IN VIEW OF THE FOREGOING, the decisions of the Register of Deeds of Ilocos Sur
and that of the Commissioner of Land Registration are affirmed. No pronouncement as 3. Since, as in this case, the wife has already died when the sale was made,
to costs. the surviving husband can not dispose of the whole property without violating
the existing law (LRC Consulta No. 46 dated June 10, 1958).
Reyes, J.B.L., Dizon, Zaldivar, Sanchez, Fernando, Teehankee and Barredo, JJ.,
concur. To effect the registration of the aforesaid deed of absolute Sale, it is
Capistrano, J., took no part. necessary that the property be first liquidated and transferred in the name of
Concepcion, C.J., and Castro, J., are on leave. the surviving spouse and the heirs of the deceased wife by means of
extrajudicial settlement or partition and that the consent of such other heir or
Footnotes heirs must be procured by means of another document ratifying this sale
*Section executed by their father.
43. Certificates where land registered in names of two or more
persons. Where two or more persons are registered owners as tenants in In view of such refusal, Almirol went to the Court of First Instance of Agusan on a
common, or otherwise, one owner's duplicate certificate may be issued for the petition for mandamus (sp. civ. case 151), to compel the Register of Deeds to register
whole land, or a separate duplicate may be issued to each for his undivided the deed of sale and to issue to him the corresponding transfer certificate of title, and to
share. recover P5,000 in moral damages and P1,000 attorney's fees and expenses of
litigation. It is Almirol's assertion that it is but a ministerial duty of the respondent to
perform the acts required of him, and that he (Almirol) has no other plain, speedy and
adequate remedy in the ordinary course of law.
In his answer with counterclaim for P10,000 damages, the respondent reiterated
the grounds stated in his letter of May 21, 1962, averred that the petitioner has "other
legal, plain, speedy and adequate remedy at law by appealing the decision of the
respondent to the Honorable Commissioner of Land Registration," and prayed for
dismissal of the petition.
In its resolution of October 16, 1963 the lower court, declaring that "mandamus
does not lie . . . because the adequate remedy is that provided by Section 4 of Rep. Act
1151", dismissed the petition, with costs against the petitioner.
Hence the present appeal by Almirol.
The only question of law tendered for resolution is whether mandamus will lie to
compel the respondent to register the deed of sale in question.
Although the reasons relied upon by the respondent evince a sincere desire on
G.R. No. L-22486 March 20, 1968 his part to maintain inviolate the law on succession and transmission of rights over real
TEODORO ALMIROL, petitioner-appellant, properties, these do not constitute legal grounds for his refusal to register the deed.
vs. Whether a document is valid or not, is not for the register of deeds to determine; this
THE REGISTER OF DEEDS OF AGUSAN, respondent-appellee. function belongs properly to a court of competent jurisdiction.1

Tranquilino O. Calo, Jr. for petitioner-appellant. Whether the document is invalid, frivolous or intended to harass, is not
Office of the Solicitor General for respondent-appellee. the duty of a Register of Deeds to decide, but a court of competent jurisdiction.
(Gabriel vs. Register of Deeds of Rizal, et al., L-17956, Sept. 30, 1953).
CASTRO, J.:
. . . the supposed invalidity of the contracts of lease is no valid objection
On June 28, 1961 Teodoro Almirol purchased from Arcenio Abalo a parcel of land to their registration, because invalidity is no proof of their non-existence or a
valid excuse for denying their registration. The law on registration does not of Deeds, Pasig, Rizal vs. heirs of Hi Caiji, et al., 99 Phil. 25, 29-31; Mendoza
require that only valid instruments shall be registered. How can parties affected vs. Abrera, et al., L-10519, April 30, 1959; Agricultural Credit Cooperative
thereby be supposed to know their invalidity before they become aware, Association of Hinibiran vs. Yulo Yusay, et al., L-13313, April 28, 1960; Dulay,
actually or constructively, of their existence or of their provisions? If the et al., vs. Herrera, L-17084, August 30, 1962.
purpose of registration is merely to give notice, then questions regarding the
effect or invalidity of instruments are expected to be decided after, not before,
registration. It must follow as a necessary consequence that registration must
first be allowed, and validity or effect litigated afterwards. (Gurbax Singh Pablo
& Co. vs. Reyes and Tantoco, 92 Phil. 182-183).
Indeed, a register of deeds is entirely precluded by section 4 of Republic Act 1151
from exercising his personal judgment and discretion when confronted with the problem
of whether to register a deed or instrument on the ground that it is invalid. For under the
said section, when he is in doubt as to the proper step to be taken with respect to any
deed or other instrument presented to him for registration, all that he is supposed to do
is to submit and certify the question to the Commissioner of Land Registration who
shall, after notice and hearing, enter an order prescribing the step to be taken on the
doubtful question. Section 4 of R.A. 1151 reads as follows:
Reference of doubtful matters to Commissioner of Land Registration.
When the Register of Deeds is in doubt with regard to the proper step to be
taken or memorandum to be made in pursuance of any deed, mortgage, or
other instrument presented to him for registration, or where any party in
interest does not agree with the Register of Deeds with reference to any such
matter, the question shall be submitted to the Commissioner of Land
Registration either upon the certification of the Register of Deeds, stating the
question upon which he is in doubt, or upon the suggestion in writing by the
party in interest; and thereupon the Commissioner, after consideration of the
matter shown by the records certified to him, and in case of registered lands,
after notice to the parties and hearing, shall enter an order prescribing the step
to be taken or memorandum to be made. His decision in such cases shall be Republic of the Philippines
conclusive and binding upon all Registers of Deeds: Provided, further, That SUPREME COURT
when a party in interest disagrees with the ruling or resolution of the Manila
Commissioner and the issue involves a question of law, said decision may be
appealed to the Supreme Court within thirty days from and after receipt of the THIRD DIVISION
notice thereof.
G.R. No. 175746 March 12, 2008
The foregoing notwithstanding, the court a quo correctly dismissed the petition for
mandamus. Section 4 abovequoted provides that "where any party in interest does not CHARLES L. ONG, Petitioner,
agree with the Register of Deeds . . . the question shall be submitted to the
vs.
Commissioner of Land Registration," who thereafter shall "enter an order prescribing
the step to be taken or memorandum to be made," which shall be "conclusive and REPUBLIC OF THE PHILIPPINES, Respondent.
binding upon all Registers of Deeds." This administrative remedy must be resorted to by
DECISION
the petitioner before he can have recourse to the courts.
ACCORDINGLY, the Resolution of the lower court of October 16, 1969, is YNARES-SANTIAGO, J.:
affirmed, at petitioner's cost.1wph1.t
This petition for review on certiorari assails the April 25, 2006 Decision1 of the Court of
Reyes, J.B.L., Dizon, Makalintal, Bengzon, J.P., Zaldivar, Angeles and Fernando, JJ., Appeals in CA-G.R. CV No. 76085, which reversed and set aside the January 16, 2002
concur.
Decision2 of the Municipal Trial Court of Mangaldan, Pangasinan in Land Registration
Concepcion, C.J., is on leave.
Sanchez, J., concurs in the result. Case No. 99-023, and the November 20, 2006 Resolution3 which denied petitioners
motion for reconsideration.
Footnotes
1Inre Consulta filed by Francisco on behalf of Cabantog, 67 Phil. 222, 228; The antecedent facts are as follows.
Smith Bell & Co., Ltd. vs. Register of Deeds of Davao, 96 Phil. 53, 61; Register
On July 1, 1999, petitioner Charles L. Ong (petitioner) in his behalf and as duly Furnish copies of this Decision to the Office of the Solicitor General, Makati City, Metro
authorized representative of his brothers, namely, Roberto, Alberto and Cesar, filed an Manila, the Office of the Provincial Prosecutor, Dagupan City, Atty. Celestino Domingo
Application for Registration of Title4 over Lot 15911 (subject lot) situated in Barangay Jr., the Office of the Land Registration Authority, Quezon City, as well as the applicant.
Anolid, Mangaldan, Pangasinan with an area of five hundred seventy four (574) square
SO ORDERED.5
meters, more or less. They alleged that they are the co-owners of the subject lot; that
the subject lot is their exclusive property having acquired the same by purchase from Aggrieved, respondent appealed to the Court of Appeals which rendered the assailed
spouses Tony Bautista and Alicia Villamil on August 24, 1998; that the subject lot is Decision, the dispositive portion of which reads:
presently unoccupied; and that they and their predecessors-in-interest have been in
open, continuous and peaceful possession of the subject lot in the concept of owners WHEREFORE, the instant appeal is GRANTED. Accordingly, the decision of the court a
for more than thirty (30) years. quo granting the application for registration of title of applicants-appellees is
REVERSED and SET ASIDE. No pronouncement as to costs.
After due notice and publication, only respondent Republic of the Philippines
(respondent), represented by the Office of the Solicitor General, opposed the SO ORDERED.6
application for registration of title. Respondent asserted that neither applicants nor their In reversing the decision of the trial court, the Court of Appeals found that the subject lot
predecessors-in-interest have been in open, continuous, exclusive and notorious is part of the alienable and disposable lands of the public domain. Thus, it was
possession and occupation of the subject lot since June 12, 1945 or earlier as required incumbent upon petitioner to prove that they possessed the subject lot in the nature and
by Section 48(b) of Commonwealth Act No. 141, as amended by Presidential Decree for the duration required by law. However, petitioner failed to prove that he or his
(P.D.) No. 1073; that applicants failed to adduce any muniment of title to prove their predecessors-in-interest have been in adverse possession of the subject lot in the
claims; that the tax declaration appended to the application does not appear genuine concept of owner since June 12, 1945 or earlier as mandated by Section 14(1) of P.D.
and merely shows pretended possession of recent vintage; that the application was filed 1529. It noted that the earliest tax declaration which petitioner presented is dated 1971.
beyond the period allowed under P.D. No. 892; and that the subject lot is part of the Consequently, petitioner could not fairly claim possession of the land prior to 1971.
public domain which cannot be the subject of private appropriation. Neither was petitioner able to prove that he or his predecessors-in-interest actually
On January 16, 2002, the trial court rendered a Decision in favor of petitioner and his occupied the subject lot prior to the filing of the application. Thus, the trial court erred in
brothers, viz: granting the application for registration of title over the subject lot.

The foregoing evidences presented by the applicant indubitably established sufficient Hence, this petition raising the following issues:
basis to grant the applicant (sic) for registration. Originally, the whole parcel of land was 1. WHETHER OR NOT PETITIONER, TOGETHER WITH HIS BROTHERS,
owned by spouses Teofilo Abellara and Abella Charmine who acquired the same by NAMELY, ROBERTO L. ONG, ALBERTO L. ONG AND CEZAR L. ONG,
virtue of a Deed of Sale from Cynthia Cacho, Agustin Cacho, Jr., Jasmin Cacho, Jover HAVE REGISTRABLE OWNERSHIP OVER THE REAL PROPERTY
Cacho and Lauro Cacho. Later, they sold the same parcel of land to spouses Tony C. SUBJECT MATTER OF LAND REGISTRATION CASE NO. 99-023, AND
Villamil and Alicia Bautista, who in turn sold the same land to herein applicants.
2. WHETHER OR NOT THE FINDINGS AND CONCLUSION OF THE
The same parcel of land has been declared in the name of the applicant and her FORMER SPECIAL FOURTH DIVISION OF THE COURT OF APPEALS
predecessors-in-interest and its taxes has (sic) been religiously paid. THAT THE SUBJECT REAL PROPERTY IS A PUBLIC LAND IS CORRECT.7
The said circumstances further show that the possession and ownership of the The petition lacks merit.
applicant and her (sic) predecessors-in-interest over the same parcel of land has (sic)
been continuous and peaceful under bona fide claim of ownership before the filing of Section 14(1) of P.D. 1529 ("Property Registration Decree"), as amended, provides
the instant application for registration on [July 1, 1999].
SEC. 14. Who may apply. The following persons may file in the proper Court of First
WHEREFORE, after confirming the Order of General Default, the Court hereby orders Instance an application for registration of title to land, whether personally or through
and decrees the registration of a parcel of land as shown on plan ap-01-004897 their duly authorized representatives:
approved by the Bureau of Land(s) situated in Barangay Anolid, Mangaldan,
(1) Those who by themselves or through their predecessors-in-interest have been in
Pangasinan, containing an area of Five Hundred Seventy Four (574) square meters,
open, continuous, exclusive and notorious possession and occupation of alienable and
subject of the application for registration of title, in accordance with Presidential Decree
disposable lands of the public domain under a bona fide claim of ownership since June
No. 1529, in favor of CHARLIE L. ONG in his behalf and as representative of his
12, 1945, or earlier.
brothers namely, ROBERTO L. ONG, ALBERTO L. ONG and CESAR L. ONG.
Thus, pursuant to the aforequoted provision of law, applicants for registration of title possession of a land consists in the manifestation of acts of dominion over it of such a
must prove: (1) that the subject land forms part of the disposable and alienable lands of nature as a party would naturally exercise over his own property.20
the public domain, and (2) that they have been in open, continuous, exclusive and
Petitioner admitted that after he and his brothers bought the subject lot from spouses
notorious possession and occupation of the same under a bona fide claim of ownership
Tony Bautista and Alicia Villamil in 1998, neither he nor his brothers actually occupied
since June 12, 1945, or earlier.8 These requisites involve questions of fact which are
the subject lot.21 No improvements were made thereon and the most that they did was
not proper in a petition for review on certiorari. Factual findings of the court a quo are
to visit the lot on several occasions.22 Petitioners predecessor-in-interest, Tony
generally binding on this Court except for certain recognized exceptions, as is the case
Bautista testified that he and his wife never actually occupied the subject lot from the
here, where the trial court and the Court of Appeals arrived at conflicting findings.9 After
time they bought the same from spouses Teofilo Abellera and Abella Sarmen in
a careful review of the records, we sustain the findings and conclusions of the Court of
1997.23 Aside from these two testimonies, no other evidence was presented to
Appeals.
establish the character of the possession of the subject lot by petitioners other alleged
There is no dispute that the subject lot is classified as alienable and disposable land of predecessors-in-interest. Clearly, petitioners evidence failed to establish specific acts of
the public domain. The Report10 dated January 17, 2000 of the Bureau of Lands stated ownership to substantiate the claim that he and his predecessors-in-interest possessed
that the subject lot is "within the alienable and disposable zone as classified under and occupied the subject lot in the nature and duration required by law.
Project 50 L.C. Map No. 698 and released and classified as such on November 21,
The burden of proof in land registration cases rests on the applicant who must show by
1927."11 This finding is, likewise, embodied in the Report12 dated January 7, 1999 of
clear, positive and convincing evidence that his alleged possession and occupation of
the Department of Environment and Natural Resources Community Environment and
the land is of the nature and duration required by law.24 Unfortunately, petitioners
Natural Resources Office (DENR-CENRO) and the blue print Copy13 of the plan
evidence do not constitute the "well-nigh incontrovertible" evidence necessary in cases
covering the subject lot. However, petitioner failed to prove that he or his predecessors-
of this nature.25 Accordingly, the Court of Appeals did not err in reversing the Decision
in-interest have been in open, continuous, exclusive and notorious possession and
of the trial court and in denying his application for registration of title over the subject lot.
occupation of the subject lot since June 12, 1945 or earlier.
WHEREFORE, in view of the foregoing, the petition is DENIED. The April 25, 2006
The records show that petitioner and his brothers bought the subject lot from spouses
Decision of the Court of Appeals in CA-G.R. CV No. 76085 which reversed and set
Tony Bautista and Alicia Villamil on August 24, 1998,14 who in turn purchased the
aside the January 16, 2002 Decision of the Municipal Trial Court of Mangaldan,
same from spouses Teofilo Abellera and Abella Sarmen on January 16, 1997.15 The
Pangasinan in Land Registration Case No. 99-023, and the November 20, 2006
latter bought the subject lot from Cynthia, Agustin Jr., Jasmin, Omir and Lauro, all
Resolution denying the motion for reconsideration, are AFFIRMED.
surnamed Cacho, on July 10, 1979.16 The earliest tax declaration which was submitted
in evidence was Tax Declaration No. 2560617 issued in 1971 in the names of spouses Costs against petitioner.
Agustin Cacho and Eufrosinia Baustista. While tax declarations are not conclusive proof
of ownership, they constitute good indicia of possession in the concept of owner and a SO ORDERED.
claim of title over the subject property.18 Even if we were to tack petitioners claim of CONSUELO YNARES-SANTIAGO
ownership over the subject lot to that of their alleged predecessors-in-interest, spouses
Agustin Cacho and Eufrosinia Baustista in 1971, still this would fall short of the required Associate Justice
possession from June 12, 1945 or earlier.1avvphi1 WE CONCUR:
Further, as correctly pointed by the Court of Appeals, possession alone is not sufficient
to acquire title to alienable lands of the public domain because the law requires
possession and occupation. As held in Republic v. Alconaba:19

The law speaks of possession and occupation. Since these words are separated by the
conjunction and, the clear intention of the law is not to make one synonymous with the
other. Possession is broader than occupation because it includes constructive
possession. When, therefore, the law adds the word occupation, it seeks to delimit the
all encompassing effect of constructive possession. Taken together with the words
open, continuous, exclusive and notorious, the word occupation serves to highlight the
fact that for an applicant to qualify, his possession must not be a mere fiction. Actual
property, rendering them ineligible to acquire title to the said property under the Friar
Lands Act.[8] Petitioners also sought to nullify Original Certificate of Title (OCT) No. 614
from which the foregoing titles sought to be cancelled originated or were derived.

Respondent Genuino Ice Co., Inc. filed a motion to dismiss[9] on the ground that the
complaint states no cause of action because petitioners are not real parties-in-interest;
that no relief may be granted as a matter of law; and that petitioners failed to exhaust
administrative remedies, but it was denied by the trial court. Respondent moved for
reconsideration but the same was denied.

On November 4, 1999, petitioners filed a Second Amended Complaint[10] which sought


to annul, in addition to the titles already alleged in the original complaint, TCT Nos.
274095 and 274096;[11] 274097 and 274098;[12] and 274099.[13]

The Second Amended Complaint alleged the following causes of action, as well as the
remedy sought to be obtained, thus:
THIRD DIVISION
NELSIE B. CAETE, RONA ANAS, G.R. No. 154080 4. That plaintiffs (petitioners) and their predecessors-in-interest are
Petitioners, Present: among those who have been in actual, adverse, peaceful and
Ynares-Santiago, J. (Chairperson), continuous possession in concept of owners of unregistered parcels
- versus - Austria-Martinez, of land situated at Sitio Mabilog, Barangay Culiat, Quezon City, Metro
Corona,* Manila, which parcels of land are more particularly described as
Nachura, and follows:
Reyes, JJ.
GENUINO ICE COMPANY, INC., (1) A parcel of unregistered land known as Lot 668,
Respondent. Promulgated: situated at Barangay Culiat, Quezon City x x x.

January 22, 2008 (2) A parcel of unregistered land known as Lot 669,
x --------------------------------------------------------------------------x situated at Barangay Culiat, Quezon City x x x.
DECISION
5. That the above-described real property is a portion of a friar land
YNARES-SANTIAGO, J.: known as Piedad Estate, which property is intended for distribution
among the bona fide occupants thereof pursuant to the Friar Lands
This petition for review on certiorari seeks to set aside the Decision[1] of the Court of Act.
Appeals dated January 9, 2002 in CA-G.R. SP No. 64337 entitled Genuino Ice
Company, Inc. vs. Hon. Victorino P. Evangelista, Nelsie B. Caete, et al., and its 6. That transfer certificates of title allegedly having originated or
Resolution[2] dated June 26, 2002, dismissing petitioners Second Amended Complaint derived from Original Certificate of Title No. 614 were issued by the
in Civil Case No. Q-99-36483 filed in Branch 223 of the Regional Trial Court of Quezon Register of Deeds of Quezon City, which transfer certificates of title
City. are in truth and in fact fictitious, spurious and null and void, for the
following reasons: (a) that no record of any agency of the government
Records show that on January 11, 1999, petitioners filed a complaint for cancellation of shows as to how and in what manner was OCT 614 issued; (b) that
title to property covered by Transfer Certificate of Title (TCT) Nos. N- no record of any proceedings whatsoever, whether judicial or
140441;[3] 14399;[4] RT-94384 (292245);[5] RT-94794 (292246);[6] and administrative, can support defendants claim that the above-
292247.[7] Petitioners alleged that said titles are spurious, fictitious and were issued described property originated from OCT 614; and (c) that the transfer
under mysterious circumstances, considering that the holders thereof including their certificates of title over the above-described property were issued
predecessors-in-interest were never in actual, adverse and physical possession of the
under mysterious circumstances for the above-named defendants (2) based on the allegations and prayer of the complaint, no relief, as
and their so-called predecessors-in-interest never had any actual, a matter of law, may be granted;
adverse, physical possession of the said property, thus, not allowed
to acquire title over the property in litigation pursuant to the Friar b) Prescription has set in;
Lands Act.
c) There are earlier similar complaints (Civil Case Nos. Q-95-22834
7. That defendants are holders of transfer certificates of title of the and Q-95-23111) filed by a different set of plaintiffs against a different
above-described property, which transfer certificates of title are null set of defendants but which involve the same subject matter, cause of
and void, for reasons specifically mentioned in Paragraph 6 hereof x x action and allegations of the plaintiffs, with respect to the cancellation
x; of OCT 614 and succeeding titles derived from it. Said complaints
have since been dismissed by Branch 93 of the Regional Trial Court
8. That the acts in acquiring and keeping the said transfer certificates of Quezon City, the dismissal of which is the subject of a pending
of title in violation of the Friar Lands Act and other existing laws are certiorari proceeding in the appellate court.[15]
prejudicial to plaintiffs rights over the above-described property.
On January 3, 2001,[16] the trial court denied respondents motion to dismiss the
9. That equity demands that defendants transfer certificates of title as Second Amended Complaint. Its motion for reconsideration was likewise denied hence
specified in Paragraph 7 hereof be declared fictitious, spurious and respondent filed a petition for certiorari with the Court of Appeals.
null and void ab initio.
The appellate court granted respondents petition for certiorari and dismissed petitioners
PRAYER Second Amended Complaint for failure to state a cause of action. Hence, the instant
petition raising the following issues:
WHEREFORE, premises considered, it is most respectfully prayed of
this Honorable Court that judgment be rendered in favor of plaintiffs A. THAT THE COURT OF APPEALS ERRED IN
and against defendants: DECLARING THAT THE COMPLAINT FILED BY THE
PETITIONERS WITH THE REGIONAL TRIAL COURT
(1) Declaring as null and void ab initio OCT 614 and all transfer OF QUEZON CITY IN CIVIL CASE NO. Q-99-36483 DOES
certificates of title derived therefrom; NOT STATE A VALID CAUSE OF ACTION;

(2) Declaring as null and void defendants transfer certificates of title B. THAT THE COURT OF APPEALS ERRED IN
over the property in litigation; DECLARING THAT THE PETITIONERS ARE NOT REAL
PARTIES IN INTEREST;
(3) Ordering defendant Register of Deeds of Quezon City to cancel
defendants transfer certificates of title and all transfer certificates of C. THAT THE COURT OF APPEALS ERRED IN APPLYING
title derived therefrom; THE DOCTRINE OF EXHAUSTION OF ADMINISTRATIVE
REMEDIES; and,
(4) Declaring the plaintiffs as bona fide occupants of the property in
litigation pursuant to the provisions of the Friar Lands Act and other D. THAT THE COURT OF APPEALS ACTED WITH GRAVE
existing laws.[14] ABUSE OF DISCRETION AND DENIED PETITIONERS
RIGHT TO DUE PROCESS WHEN IT DISMISSED THEIR
Respondent moved to dismiss the Second Amended Complaint on the following COMPLAINT.[17]
grounds:
We deny the petition.
a) The complaint states no cause of action because: (1) on the
allegations alone, plaintiffs (petitioners) are not real parties in interest The subject lots are part of the Piedad Estate, Quezon City, a Friar Land acquired on
who may bring suit to cancel defendants (including respondent) titles; December 23, 1903 by the Philippine Government from the Philippine Sugar Estates
Development Company, Ltd., La Sociedad Agricola de Ultramar, the British-Manila jurisdiction. The said allegations are not statements of ultimate facts
Estate Company, Ltd., and the Recoleto Order of the Philippine Islands, as indicated in but are mere conclusions of law.
Public Act No. 1120 (Friar Lands Act) enacted on April 26, 1904.[18]
A pleading should state the ultimate facts essential to the rights of
After the Piedad Estate was registered in OCT No. 614 in the name of the Philippine action or defense asserted, as distinguished from mere conclusions of
Government in 1910 under the provisions of Act 496, the area was subdivided originally fact, or conclusions of law. General allegations that a contract is valid
into 874 lots. As a result of subsequent surveys executed in the course of disposition, or legal, or is just, fair and reasonable, are mere conclusions of
the number of lots increased to 1,305. Disposition of these lots was made by the Bureau law. Likewise, allegations that a contract is void, voidable, invalid,
of Lands thru sales, under the Friar Lands Act, as early as 1910 and records show that illegal, ultra vires, or against public policy, without stating facts
even before the Second World War, all lots in the Piedad Estate have been disposed showing its invalidity, are mere conclusions of law.[24]
of.[19] The Piedad Estate has long been segregated from the mass of the public domain
and has become private land duly registered under the Torrens system following the Ultimate facts means the essential facts constituting the plaintiff's cause of action, or
procedure for the confirmation of private lands prescribed in Act 496. Thus the lands such facts as are so essential that they cannot be stricken out without leaving the
inside the Piedad Estate are no longer lands of the public domain.[20] statement of the cause of action inadequate.[25]Cause of action has been defined as an
act or omission of one party in violation of the legal right or rights of the other;[26] and
One who acquires land under the Friar Lands Act, as well as his successors-in-interest, its essential elements are: 1) a right in favor of the plaintiff by whatever means and
may not claim successional rights to purchase by reason of occupation from time under whatever law it arises or is created; 2) an obligation on the part of the named
immemorial, as this contravenes the historical fact that friar lands were bought by the defendant to respect or not to violate such right; and 3) an act or omission on the part of
Government of the Philippine Islands, pursuant to an Act of Congress of the United the named defendant violative of the right of the plaintiff or constituting a breach of the
States, approved on July 1, 1902, not from individual persons but from certain obligation of defendant to the plaintiff for which the latter may maintain an action for
companies, a society and a religious order. Under the Friar Lands Act, only actual recovery of damages. If these elements are not extant, the complaint becomes
settlers and occupants at the time said lands are acquired by the Government were vulnerable to a motion to dismiss on the ground of failure to state a cause of
given preference to lease, purchase, or acquire their holdings, in disregard of the action.[27] In the resolution of a motion to dismiss based on failure to state a cause of
settlement and occupation of persons before the government acquired the lands.[21] action, only the facts alleged in the complaint as well as its annexes must be
considered.[28] The test in such case is whether a court can render a valid judgment on
The basic rules of proper pleading and procedure require that every pleading shall the complaint based upon the facts alleged and pursuant to the prayer therein.[29]
contain in a methodical and logical form, a plain, concise and direct statement of the
ultimate facts on which the party pleading relies for his claim or defense, as the case Corollarily, the question of whether or not a complaint states a cause of action against a
may be, omitting the statement of mere evidentiary facts.[22] And in all averments of defendant or the action is premature is one of law. The trial court can consider all the
fraud or mistake, the circumstances constituting fraud or mistake must be stated with pleadings filed, including annexes, motions and the evidence on record. However in so
particularity.[23] doing, the trial court does not rule on the truth or falsity of such documents. It merely
includes such documents in the hypothetical admission. Any review of a finding of lack
It is axiomatic that the averments of the complaint determine the of cause of action based on these documents would not involve a calibration of the
nature of the action, and consequently, the jurisdiction of the probative value of such pieces of evidence but would only limit itself to the inquiry of
courts. This is because the complaint must contain a concise whether the law was properly applied given the facts and these supporting
statement of the ultimate facts constituting the plaintiff's cause of documents. Therefore, what would inevitably arise from such a review are pure
action and must specify the relief sought. No rule is better established questions of law, and not questions of fact.
than that which requires the complaint to contain a statement of all
the facts constituting the plaintiff's cause of action. Additionally, The trial court must likewise apply relevant statutes and jurisprudence in determining
Section 5, Rule 8 of the Rules of Court provides that in all averments whether the allegations in a complaint establish a cause of action. While it focuses on
of fraud or mistake, the circumstances constituting fraud or mistake the complaint, a court clearly cannot disregard decisions material to the proper
must be stated with particularity. In the case at bar, while there are appreciation of the questions before it. In resolving a motion to dismiss, every court
allegations of fraud in the above quoted complaints, the same are not must take cognizance of decisions this Court has rendered because they are proper
particular enough to bring the controversy within the SEC's subjects of mandatory judicial notice. The said decisions, more importantly, form part of
the legal system, and failure of any court to apply them shall constitute an abdication of
its duty to resolve a dispute in accordance with law, and shall be a ground for As to the second issue raised, petitioners claim that they are bona fide occupants of the
administrative action against an inferior court magistrate.[30] subject property within the contemplation of the Friar Lands Act, having allegedly been
in actual, adverse, peaceful and continuous possession of the property, although it is
Considering the foregoing, it is not difficult to see the need for particularity and incipient not stated for how long and since when. In their second amended complaint, they seek
substantiation in the petitioners Second Amended Complaint. judgment

First, their initial claim that OCT 614 of which all the other subject titles are derivatives (4) Declaring the plaintiffs as bona fide occupants of the property in
is null and void, has been proven wrong. As has been held in Pinlac and other cases, litigation pursuant to the provisions of the Friar Lands Act and other
OCT 614 did legally exist and was previously issued in the name of the Philippine existing laws. (Emphasis supplied)
Government in 1910 under the provisions of Act 496.
They do not pray to be declared owners of the subject property despite their alleged
Second, the Ad Hoc Committee of the then Ministry of Natural Resources, which was adverse possession but only to be adjudged as the bona fide occupants thereof. In
specifically tasked to investigate the historical background of the Piedad Estate, found other words, petitioners concede the States ownership of the property.
that as early as the period prior to the Second World War, all lots in the Piedad Estate
had already been disposed of. Being so, petitioners may not be considered the real parties in interest for the purpose
of maintaining the suit for cancellation of the subject titles. The Court of Appeals is
Third, the Piedad Estate has been placed under the Torrens system of land correct in declaring that only the State, through the Solicitor General, may institute such
registration, which means that all lots therein are titled. suit. Jurisprudence on the matter has been settled and the issue need not be
belabored. Thus
Fourth, as held in the Balicudiong case, one who acquires land under the Friar Lands
Act, as well as his successors-in-interest, may not claim successional rights to purchase The Court also holds that private respondents are not the proper
by reason of occupation from time immemorial, which means that petitioners claimed parties to initiate the present suit. The complaint, praying as it did for
actual, adverse, peaceful and continuous possession of the subject property is really of the cancellation of the transfer certificates of title of petitioners on the
no moment unless it is shown that their predecessors-in-interest were actual settlers ground that they were derived from a spurious OCT No. 4216,
and occupants at the time said lands were acquired by the Government, and whose assailed in effect the validity of said title. While private respondents
rights were not disregarded even though they were in occupation of the same before the did not pray for the reversion of the land to the government, we agree
government acquired the land; yet, no period of time in relation to adverse possession is with the petitioners that the prayer in the complaint will have the same
alleged. result of reverting the land to the government under the Regalian
doctrine. Gabila vs. Barriga ruled that only the government is entitled
Petitioners Second Amended Complaint betrays no more than an incomplete narration to this relief. The Court in that case held:
of facts unsupported by documentary or other exhibits; the allegations therein partake of
conclusions of law unsupported by a particular averment of circumstances that will show The present motion to dismiss is actually predicated
why or how such inferences or conclusions were arrived at. It is replete with sweeping on Section 1(g), Rule 16 of the Revised Rules of
generalizations and inferences derived from facts that are not found therein. While there Court, i.e., failure of the complaint to state a cause
are allegations of fraud upon the claim that the subject titles were fictitious, spurious and of action, for it alleges in paragraph 12 thereof that
obtained under mysterious circumstances, the same are not specific to bring the the plaintiff admits that he has no right to demand
controversy within the trial courts jurisdiction. There is no explanation or narration of the cancellation or amendment of the defendants
facts as would show why said titles are claimed to be fictitious or spurious, contrary to title, because, even if the said title were canceled or
the requirement of the Rules that the circumstances constituting fraud must be stated amended, the ownership of the land embraced
with particularity; otherwise, the allegation of fraud would simply be an unfounded therein, or of the portion thereof affected by the
conclusion of law. In the absence of specific averments, the complaint is defective, for it amendment, would revert to the public domain. In
presents no basis upon which the court should act, or for the defendant to meet it with his amended complaint the plaintiff makes no
an intelligent answer. pretense at all that any part of the land covered by
the defendants title was privately owned by him or
by his predecessors-in-interest. Indeed, it is
admitted therein that the said land was at all times On the issue of exhaustion of administrative remedies, suffice it to state that since
a part of the public domain until December 18, petitioners do not possess the necessary interest to prosecute the case for cancellation
1964, when the government issued a title thereon in of title in the courts, neither do they have the right to pursue administrative remedies
favor of defendant. Thus, if there is any person or outside thereof. They are not the owners; nor are they qualified applicants therefor. It
entity to relief, it can only be the government. has not been shown by their complaint that they have previously taken steps to avail of
the benefits under the Friar Lands Act, since all they seek, should the questioned titles
In the case at bar, the plaintiffs own averments be nullified, is to be declared bona fide occupants of the property covered by the
negate the existence of such right, for it would questioned titles. Neither is there any indication that they possess the qualifications
appear therefrom that whatever right might have necessary to enable them to avail of the preference granted under the Act.
been violated by the defendant belonged to the
government, not to the plaintiff. Plaintiff-appellant Finally, there is no merit in petitioners contention that respondent belatedly filed the
argues that although his complaint is captioned as petition for certiorari with the Court of Appeals, and that the appellate court gravely
one for cancellation of title, he has nevertheless abused its discretion when it entertained and resolved the same.
stated therein several causes of action based on
his alleged rights of possession and ownership over The Order of the trial court dated January 3, 2001 denying respondents motion to
the improvements, on defendant-appellees alleged dismiss the Second Amended Complaint was received by the respondent on January
fraudulent acquisition of the land, and on the 16, 2001. Respondent filed a motion for reconsideration on January 18, 2001 which was
damages allegedly incurred by him (plaintiff- denied on February 28, 2001. Respondent received the order denying its motion for
appellant) in relation to the improvements. These reconsideration on March 27, 2001. On the same day, it filed a Notice to File Petition for
matters are merely ancillary to the central issue of Certiorari. On April 2, 2001, the petition for certiorari was filed with the Court of
whether or not defendant-appellees title should be Appeals. Clearly, the same was timely filed hence, the appellate court correctly
canceled or amended, and they may not be leaned entertained the same.
upon in an effort to make out a cause of action in
relation to the said focal issue. Indeed, the principal WHEREFORE, the petition is DENIED. The Decision of the Court of Appeals
relief prayed for in the amended complaint is the dated January 9, 2002 in CA-G.R. SP No. 64337 dismissing petitioners Second
cancellation or amendment of defendant-appellees Amended Complaint in Civil Case No. Q-99-36483 and the Resolution dated June 26,
title.[31] 2002 denying the motion for reconsideration, are AFFIRMED.
Under Rule 3, Section 2 of the Rules of Court, a real party in interest is the party who SO ORDERED.
stands to be benefited or injured by the judgment in the suit, or the party entitled to the
avails of the suit. Interest within the meaning of the rule means material interest, an
interest in issue and to be affected by the decree, as distinguished from mere interest in
the question involved, or a mere incidental interest. The interest of the party must also
be personal and not one based on a desire to vindicate the constitutional right of some
third and unrelated party. Real interest, on the other hand, means a present substantial
interest, as distinguished from a mere expectancy or a future, contingent, subordinate,
or consequential interest.[32]

If petitioners are to be believed, they would possess a mere inchoate interest in the
properties covered by the subject titles, a mere expectancy conditioned upon the fact
that if the questioned titles are cancelled and the property is reverted to the State, they
would probably or possibly be given preferential treatment as qualified buyers or
lessees of the property under the Friar Lands Act. But this certainly is not the interest
required by law that grants them license or the personality to prosecute their case. Only
to the State does the privilege belong.
- Hernando De Soto[1]

This decision inevitably affects all untitled lands currently in possession of persons and
entities other than the Philippine government. The petition, while unremarkable as to the
EN BANC
facts, was accepted by the Court en bancin order to provide definitive clarity to the
HEIRS OF MARIO MALABANAN, G.R. No.179987 applicability and scope of original registration proceedings under Sections 14(1) and
Petitioner, 14(2) of the Property Registration Decree. In doing so, the Court confronts not only the
Present: relevant provisions of the Public Land Act and the Civil Code, but also the reality on the
PUNO,C.J., ground. The countrywide phenomenon of untitled lands, as well as the problem of
QUISUMBING, informal settlement it has spawned, has unfortunately been treated with benign neglect.
YNARES-SANTIAGO, Yet our current laws are hemmed in by their own circumscriptions in addressing the
CARPIO, phenomenon. Still, the duty on our part is primarily to decide cases before us in accord
-versus- with the Constitution and the legal principles that have developed our public land law,
AUSTRIA-MARTINEZ, though our social obligations dissuade us from casting a blind eye on the endemic
CORONA, CARPIO MORALES, TINGA, problems.
CHICO-NAZARIO, VELASCO, JR., NACHURA, LEONARDO DE CASTRO, BRION, I.
REPUBLIC OF THE PHILIPPINES, PERALTA, and
Respondent. BERSAMIN, JJ. On 20 February 1998, Mario Malabanan filed an application for land registration
covering a parcel of land identified as Lot 9864-A, Cad-452-D, Silang Cadastre,[2]
Promulgated: situated in Barangay Tibig, Silang Cavite, and consisting of 71,324 square meters.
April 29, 2009 Malabanan claimed that he had purchased the property from Eduardo Velazco,[3] and
x---------------------------------------------------x that he and his predecessors-in-interest had been in open, notorious, and continuous
DECISION adverse and peaceful possession of the land for more than thirty (30) years.
TINGA, J.:
One main reason why the informal sector has not become formal is The application was raffled to the Regional Trial Court of (RTC) Cavite-Tagaytay City,
that from Indonesia to Brazil, 90 percent of the informal lands are not Branch 18. The Office of the Solicitor General (OSG) duly designated the Assistant
titled and registered. This is a generalized phenomenon in the so- Provincial Prosecutor of Cavite, Jose Velazco, Jr., to appear on behalf of the
called Third World. And it has many consequences. State.[4] Apart from presenting documentary evidence, Malabanan himself and his
xxx witness, Aristedes Velazco, testified at the hearing. Velazco testified that the property
The question is: How is it that so many governments, from Suharto's in was originally belonged to a twenty-two hectare property owned by his great-
Indonesia to Fujimori's in Peru, have wanted to title these people and grandfather, Lino Velazco. Lino had four sons Benedicto, Gregorio, Eduardo and
have not been able to do so effectively? One reason is that none of the Estebanthe fourth being Aristedess grandfather. Upon Linos death, his four sons
state systems in Asia or Latin America can gather proof of informal inherited the property and divided it among themselves. But by 1966, Estebans
titles. In Peru, the informals have means of proving property ownership wife, Magdalena, had become the administrator of all the properties inherited by the
to each other which are not the same means developed by the Spanish Velazco sons from their father, Lino. After the death of Esteban and Magdalena, their
legal system. The informals have their own papers, their own forms of son Virgilio succeeded them in administering the properties, including Lot 9864-A, which
agreements, and their own systems of registration, all of which are very originally belonged to his uncle, Eduardo Velazco. It was this property that was sold by
clearly stated in the maps which they use for their own informal Eduardo Velazco to Malabanan.[5]
business transactions.
Assistant Provincial Prosecutor Jose Velazco, Jr. did not cross-examine Aristedes
If you take a walk through the countryside, from Indonesia to Peru, and Velazco. He further manifested that he also [knew] the property and I affirm the truth of
you walk by field after field--in each field a different dog is going to bark the testimony given by Mr. Velazco.[6] The Republic of the Philippines likewise did not
at you. Even dogs know what private property is all about. The only present any evidence to controvert the application.
one who does not know it is the government. The issue is that there
exists a "common law" and an "informal law" which the Latin American Among the evidence presented by Malabanan during trial was a Certification dated 11
formal legal system does not know how to recognize. June 2001, issued by the Community Environment & Natural Resources Office,
Department of Environment and Natural Resources (CENRO-DENR), which stated that since the requisite notice of hearing was published only after the hearing had already
the subject property was verified to be within the Alienable or Disposable land per Land begun. Naguit, petitioners argue, remains the controlling doctrine, especially when the
Classification Map No. 3013 established under Project No. 20-A and approved as such property in question is agricultural land. Therefore, with respect to agricultural lands,
under FAO 4-1656 on March 15, 1982.[7] any possession prior to the declaration of the alienable property as disposable may be
On 3 December 2002, the RTC rendered judgment in favor of Malabanan, the counted in reckoning the period of possession to perfect title under the Public Land Act
dispositive portion of which reads: and the Property Registration Decree.

WHEREFORE, this Court hereby approves this application for The petition was referred to the Court en banc,[12] and on 11 November 2008, the case
registration and thus places under the operation of Act 141, Act 496 was heard on oral arguments. The Court formulated the principal issues for the oral
and/or P.D. 1529, otherwise known as Property Registration Law, the arguments, to wit:
lands described in Plan Csd-04-0173123-D, Lot 9864-A and containing
an area of Seventy One Thousand Three Hundred Twenty Four 1.In order that an alienable and disposable land of the public domain
(71,324) Square Meters, as supported by its technical description now may be registered under Section 14(1) of Presidential Decree No.
forming part of the record of this case, in addition to other proofs 1529, otherwise known as the Property Registration Decree, should
adduced in the name of MARIO MALABANAN, who is of legal age, the land be classified as alienable and disposable as of June 12, 1945
Filipino, widower, and with residence at Munting Ilog, Silang, Cavite. or is it sufficient that such classification occur at any time prior to the
filing of the applicant for registration provided that it is established that
Once this Decision becomes final and executory, the corresponding the applicant has been in open, continuous, exclusive and notorious
decree of registration shall forthwith issue. possession of the land under a bona fide claim of ownership since
June 12, 1945 or earlier?
SO ORDERED.
2. For purposes of Section 14(2) of the Property Registration Decree
The Republic interposed an appeal to the Court of Appeals, arguing that Malabanan may a parcel of land classified as alienable and disposable be deemed
had failed to prove that the property belonged to the alienable and disposable land of private land and therefore susceptible to acquisition by prescription in
the public domain, and that the RTC had erred in finding that he had been in possession accordance with the Civil Code?
of the property in the manner and for the length of time required by law for confirmation
of imperfect title. 3.May a parcel of land established as agricultural in character either
because of its use or because its slope is below that of forest lands be
On 23 February 2007, the Court of Appeals rendered a Decision[8] reversing the RTC registrable under Section 14(2) of the Property Registration Decree in
and dismissing the application of Malabanan. The appellate court held that under relation to the provisions of the Civil Code on acquisitive prescription?
Section 14(1) of the Property Registration Decree any period of possession prior to the
classification of the lots as alienable and disposable was inconsequential and should be 4.Are petitioners entitled to the registration of the subject land in their
excluded from the computation of the period of possession. Thus, the appellate court names under Section 14(1) or Section 14(2) of the Property
noted that since the CENRO-DENR certification had verified that the property was Registration Decree or both?[13]
declared alienable and disposable only on 15 March 1982, the Velazcos possession
prior to that date could not be factored in the computation of the period of possession. Based on these issues, the parties formulated their respective positions.
This interpretation of the Court of Appeals of Section 14(1) of the Property Registration
Decree was based on the Courts ruling in Republic v. Herbieto.[9] With respect to Section 14(1), petitioners reiterate that the analysis of the Court in
Naguit is the correct interpretation of the provision. The seemingly contradictory
Malabanan died while the case was pending with the Court of Appeals;[10] hence, it pronouncement in Herbieto, it is submitted, should be considered obiter dictum, since
was his heirs who appealed the decision of the appellate court. Petitioners, before this the land registration proceedings therein was void ab initio due to lack of publication of
Court, rely on our ruling in Republic v. Naguit,[11] which was handed down just four the notice of initial hearing. Petitioners further point out that in Republic v.
months prior to Herbieto. Petitioners suggest that the discussion in Herbieto cited by the Bibonia,[14]promulgated in June of 2007, the Court applied Naguit and adopted the
Court of Appeals is actually obiter dictum since the Metropolitan Trial Court therein same observation that the preferred interpretation by the OSG of Section 14(1) was
which had directed the registration of the property had no jurisdiction in the first place patently absurd. For its part, the OSG remains insistent that for Section 14(1) to apply,
the land should have been classified as alienable and disposable as of 12 June 1945. that public lands suitable for agricultural purposes may be disposed of by confirmation
Apart from Herbieto, the OSG also cites the subsequent rulings in Buenaventura v. of imperfect or incomplete titles through judicial legalization.[22]Section 48(b) of the
Republic,[15] Fieldman Agricultural Trading v. Republic[16] and Republic v. Imperial Public Land Act, as amended by P.D. No. 1073, supplies the details and unmistakably
Credit Corporation,[17] as well as the earlier case of Director of Lands v. Court of grants that right, subject to the requisites stated therein:
Appeals.[18]
Sec. 48. The following described citizens of the Philippines,
With respect to Section 14(2), petitioners submit that open, continuous, exclusive and occupying lands of the public domain or claiming to own any such
notorious possession of an alienable land of the public domain for more than 30 years land or an interest therein, but whose titles have not been perfected
ipso jure converts the land into private property, thus placing it under the coverage of or completed, may apply to the Court of First Instance of the
Section 14(2). According to them, it would not matter whether the land sought to be province where the land is located for confirmation of their claims
registered was previously classified as agricultural land of the public domain so long as, and the issuance of a certificate of title therefor, under the Land
at the time of the application, the property had already been converted into private Registration Act, to wit:
property through prescription. To bolster their argument, petitioners cite extensively
from our 2008 ruling in Republic v. T.A.N. Properties.[19] xxx

The arguments submitted by the OSG with respect to Section 14(2) are more extensive. (b) Those who by themselves or through their predecessors in
The OSG notes that under Article 1113 of the Civil Code, the acquisitive prescription of interest have been in open, continuous, exclusive, and notorious
properties of the State refers to patrimonial property, while Section 14(2) speaks of possession and occupation of alienable and disposable lands of the
private lands. It observes that the Court has yet to decide a case that presented Section public domain, under a bona fide claim of acquisition of ownership,
14(2) as a ground for application for registration, and that the 30-year possession period since June 12, 1945, or earlier, immediately preceding the filing of
refers to the period of possession under Section 48(b) of the Public Land Act, and not the application for confirmation of title except when prevented by war
the concept of prescription under the Civil Code. The OSG further submits that, or force majeure. These shall be conclusively presumed to have
assuming that the 30-year prescriptive period can run against public lands, said period performed all the conditions essential to a Government grant and
should be reckoned from the time the public land was declared alienable and shall be entitled to a certificate of title under the provisions of this
disposable. chapter.

Both sides likewise offer special arguments with respect to the particular factual Section 48(b) of Com. Act No. 141 received its present wording in 1977 when the law
circumstances surrounding the subject property and the ownership thereof. was amended by P.D. No. 1073. Two significant amendments were introduced by P.D.
No. 1073. First, the term agricultural lands was changed to alienable and disposable
II. lands of the public domain. The OSG submits that this amendment restricted the scope
First, we discuss Section 14(1) of the Property Registration Decree. For a full of the lands that may be registered. This is not actually the case. Under Section 9 of the
understanding of the provision, reference has to be made to the Public Land Act. Public Land Act, agricultural lands are a mere subset of lands of the public domain
alienable or open to disposition. Evidently, alienable and disposable lands of the public
A. domain are a larger class than only agricultural lands.
Commonwealth Act No. 141, also known as the Public Land Act, has, since its
enactment, governed the classification and disposition of lands of the public domain. Second, the length of the requisite possession was changed from possession for thirty
The President is authorized, from time to time, to classify the lands of the public domain (30) years immediately preceding the filing of the application to possession since June
into alienable and disposable, timber, or mineral lands.[20] Alienable and disposable 12, 1945 or earlier. The Court in Naguitexplained:
lands of the public domain are further classified according to their uses into (a)
agricultural; (b) residential, commercial, industrial, or for similar productive purposes; (c) When the Public Land Act was first promulgated in 1936, the period of
educational, charitable, or other similar purposes; or (d) reservations for town sites and possession deemed necessary to vest the right to register their title to
for public and quasi-public uses.[21] agricultural lands of the public domain commenced from July 26, 1894.
However, this period was amended by R.A. No. 1942, which provided
May a private person validly seek the registration in his/her name of alienable and that the bona fide claim of ownership must have been for at least thirty
disposable lands of the public domain? Section 11 of the Public Land Act acknowledges (30) years. Then in 1977, Section 48(b) of the Public Land Act was
again amended, this time by P.D. No. 1073, which pegged the Sec. 14 [of the Property Registration Decree]. Who may
reckoning date at June 12, 1945. xxx apply. The following persons may file in the proper Court of First
Instance an application for registration of title to land, whether
It bears further observation that Section 48(b) of Com. Act No, 141 is virtually the same personally or through their duly authorized representatives:
as Section 14(1) of the Property Registration Decree. Said Decree codified the various xxx
laws relative to the registration of property, including lands of the public domain. It is It is clear that Section 48 of the Public Land Act is more descriptive of the nature of the
Section 14(1) that operationalizes the registration of such lands of the public domain. right enjoyed by the possessor than Section 14 of the Property Registration Decree,
The provision reads: which seems to presume the pre-existence of the right, rather than establishing the right
itself for the first time. It is proper to assert that it is the Public Land Act, as amended by
SECTION 14. Who may apply. The following persons may P.D. No. 1073 effective 25 January 1977, that has primarily established the right of a
file in the proper Court of First Instance an application for registration Filipino citizen who has been in open, continuous, exclusive, and notorious possession
of title to land, whether personally or through their duly authorized and occupation of alienable and disposable lands of the public domain, under a bona
representatives: fide claim of acquisition of ownership, since June 12, 1945 to perfect or complete his
title by applying with the proper court for the confirmation of his ownership claim and the
(1)those who by themselves or through their issuance of the corresponding certificate of title.
predecessors-in-interest have been in open,
continuous, exclusive and notorious Section 48 can be viewed in conjunction with the afore-quoted Section 11 of the Public
possession and occupation of alienable and Land Act, which provides that public lands suitable for agricultural purposes may be
disposable lands of the public domain under disposed of by confirmation of imperfect or incomplete titles, and given the notion that
a bona fide claim of ownership since June 12, both provisions declare that it is indeed the Public Land Act that primarily establishes
1945, or earlier. the substantive ownership of the possessor who has been in possession of the property
since 12 June 1945. In turn, Section 14(a) of the Property Registration Decree
Notwithstanding the passage of the Property Registration Decree and the inclusion of recognizes the substantive right granted under Section 48(b) of the Public Land Act, as
Section 14(1) therein, the Public Land Act has remained in effect. Both laws commonly well provides the corresponding original registration procedure for the judicial
refer to persons or their predecessors-in-interest who have been in open, continuous, confirmation of an imperfect or incomplete title.
exclusive and notorious possession and occupation of alienable and disposable lands of
the public domain under a bona fide claim of ownership since June 12, 1945, or earlier. There is another limitation to the right granted under Section 48(b). Section 47 of the
That circumstance may have led to the impression that one or the other is a Public Land Act limits the period within which one may exercise the right to seek
redundancy, or that Section 48(b) of the Public Land Act has somehow been repealed registration under Section 48. The provision has been amended several times, most
or mooted. That is not the case. recently by Rep. Act No. 9176 in 2002. It currently reads thus:

The opening clauses of Section 48 of the Public Land Act and Section 14 of the Section 47. The persons specified in the next following section are
Property Registration Decree warrant comparison: hereby granted time, not to extend beyond December 31, 2020 within
which to avail of the benefits of this Chapter: Provided, That this period
Sec. 48 [of the Public Land Act]. The following described citizens of shall apply only where the area applied for does not exceed twelve
the Philippines, occupying lands of the public domain or claiming to (12) hectares: Provided, further, That the several periods of time
own any such land or an interest therein, but whose titles have not designated by the President in accordance with Section Forty-Five of
been perfected or completed, may apply to the Court of First this Act shall apply also to the lands comprised in the provisions of this
Instance of the province where the land is located for confirmation of Chapter, but this Section shall not be construed as prohibiting any said
their claims and the issuance of a certificate of title therefor, under persons from acting under this Chapter at any time prior to the period
the Land Registration Act, to wit: fixed by the President.[24]

xxx Accordingly under the current state of the law, the substantive right granted under
Section 48(b) may be availed of only until 31 December 2020.
B. irrespective of the length of adverse possession even if in good faith.
However, if the property has already been classified as alienable and
Despite the clear text of Section 48(b) of the Public Land Act, as amended and Section disposable, as it is in this case, then there is already an intention on
14(a) of the Property Registration Decree, the OSG has adopted the position that for the part of the State to abdicate its exclusive prerogative over the
one to acquire the right to seek registration of an alienable and disposable land of the property.
public domain, it is not enough that the applicant and his/her predecessors-in-interest
be in possession under a bona fide claim of ownership since 12 June 1945; the The Court declares that the correct interpretation of Section 14(1) is that which was
alienable and disposable character of the property must have been declared also as of adopted in Naguit. The contrary pronouncement in Herbieto, as pointed out in Naguit,
12 June 1945. Following the OSGs approach, all lands certified as alienable and absurdly limits the application of the provision to the point of virtual inutility since it
disposable after 12 June 1945 cannot be registered either under Section 14(1) of the would only cover lands actually declared alienable and disposable prior to 12 June
Property Registration Decree or Section 48(b) of the Public Land Act as amended. The 1945, even if the current possessor is able to establish open, continuous, exclusive and
absurdity of such an implication was discussed in Naguit. notorious possession under a bona fide claim of ownership long before that date.

Petitioner suggests an interpretation that the alienable and disposable Moreover, the Naguit interpretation allows more possessors under a bona fide claim of
character of the land should have already been established since June ownership to avail of judicial confirmation of their imperfect titles than what would be
12, 1945 or earlier. This is not borne out by the plain meaning of feasible under Herbieto. This balancing fact is significant, especially considering our
Section 14(1). Since June 12, 1945, as used in the provision, qualifies forthcoming discussion on the scope and reach of Section 14(2) of the Property
its antecedent phrase under a bonafide claim of ownership. Generally Registration Decree.
speaking, qualifying words restrict or modify only the words or
phrases to which they are immediately associated, and not those Petitioners make the salient observation that the contradictory passages
distantly or remotely located.[25] Ad proximum antecedents fiat from Herbieto are obiter dictasince the land registration proceedings therein is void ab
relation nisi impediatur sentencia. initio in the first place due to lack of the requisite publication of the notice of initial
hearing. There is no need to explicitly overturn Herbieto, as it suffices that the Courts
Besides, we are mindful of the absurdity that would result if we adopt acknowledgment that the particular line of argument used therein concerning Section
petitioners position. Absent a legislative amendment, the rule would 14(1) is indeed obiter.
be, adopting the OSGs view, that all lands of the public domain which It may be noted that in the subsequent case of Buenaventura,[26] the Court,
were not declared alienable or disposable before June 12, 1945 would citing Herbieto, again stated that [a]ny period of possession prior to the date when the
not be susceptible to original registration, no matter the length of [s]ubject [property was] classified as alienable and disposable is inconsequential and
unchallenged possession by the occupant. Such interpretation renders should be excluded from the computation of the period of possession That statement, in
paragraph (1) of Section 14 virtually inoperative and even precludes the context of Section 14(1), is certainly erroneous. Nonetheless, the passage as cited
the government from giving it effect even as it decides to reclassify in Buenaventura should again be considered as obiter. The application therein was
public agricultural lands as alienable and disposable. The ultimately granted, citing Section 14(2). The evidence submitted by petitioners therein
unreasonableness of the situation would even be aggravated did not establish any mode of possession on their part prior to 1948, thereby precluding
considering that before June 12, 1945, the Philippines was not yet the application of Section 14(1). It is not even apparent from the decision whether
even considered an independent state. petitioners therein had claimed entitlement to original registration following Section
14(1), their position being that they had been in exclusive possession under a bona fide
Accordingly, the Court in Naguit explained: claim of ownership for over fifty (50) years, but not before 12 June 1945.

Thus, neither Herbieto nor its principal discipular ruling Buenaventura has any
[T]he more reasonable interpretation of Section 14(1) is that it merely
precedental value with respect to Section 14(1). On the other hand, the ratio of Naguit is
requires the property sought to be registered as already alienable and
embedded in Section 14(1), since it precisely involved situation wherein the applicant
disposable at the time the application for registration of title is filed. If
had been in exclusive possession under a bona fide claim of ownership prior to 12 June
the State, at the time the application is made, has not yet deemed it
1945. The Courts interpretation of Section 14(1) therein was decisive to the resolution of
proper to release the property for alienation or disposition, the
the case. Any doubt as to which between Naguit or Herbieto provides the final word of
presumption is that the government is still reserving the right to utilize
the Court on Section 14(1) is now settled in favor of Naguit.
the property; hence, the need to preserve its ownership in the State
We noted in Naguit that it should be distinguished from Bracewell v. Court of possession of the subject land even before the year 1927. As a rule,
Appeals[27] since in the latter, the application for registration had been filed before the we are bound by the factual findings of the Court of Appeals.
land was declared alienable or disposable. The dissent though Although there are exceptions, petitioner did not show that this is one
pronounces Bracewell as the better rule between the two. Yet two years after Bracewell, of them.[29]
its ponente, the esteemed Justice Consuelo Ynares-Santiago, penned the ruling
in Republic v. Ceniza,[28] which involved a claim of possession that extended back to Why did the Court in Ceniza, through the same eminent member who
1927 over a public domain land that was declared alienable and disposable only in authored Bracewell, sanction the registration under Section 48(b) of public domain
1980. Ceniza cited Bracewell, quoted extensively from it, and following the mindset of lands declared alienable or disposable thirty-five (35) years and 180 days after 12 June
the dissent, the attempt at registration in Ceniza should have failed. Not so. 1945? The telling difference is that in Ceniza, the application for registration was filed
nearly six (6) years after the land had been declared alienable or disposable, while
To prove that the land subject of an application for registration is in Bracewell, the application was filed nine (9) years before the land was declared
alienable, an applicant must establish the existence of a positive act alienable or disposable. That crucial difference was also stressed in Naguit to
of the government such as a presidential proclamation or an contradistinguish it from Bracewell, a difference which the dissent seeks to belittle.
executive order; an administrative action; investigation reports of III.
Bureau of Lands investigators; and a legislative act or a statute. We next ascertain the correct framework of analysis with respect to Section 14(2). The
provision reads:
In this case, private respondents presented a certification dated
November 25, 1994, issued by Eduardo M. Inting, the Community SECTION 14. Who may apply. The following persons may
Environment and Natural Resources Officer in the Department of file in the proper Court of First Instance an application for registration
Environment and Natural Resources Office in Cebu City, stating that of title to land, whether personally or through their duly authorized
the lots involved were "found to be within the alienable and representatives:
disposable (sic) Block-I, Land Classification Project No. 32-A, per
map 2962 4-I555 dated December 9, 1980." This is sufficient xxx
evidence to show the real character of the land subject of private
respondents application. Further, the certification enjoys a (2)Those who have acquired ownership over
presumption of regularity in the absence of contradictory private lands by prescription under the
evidence, which is true in this case. Worth noting also was the provisions of existing laws.
observation of the Court of Appeals stating that:

[n]o opposition was filed by the Bureaus of Lands The Court in Naguit offered the following discussion concerning Section 14(2), which we
and Forestry to contest the application of appellees did even then recognize, and still do, to be an obiter dictum, but we nonetheless refer to
on the ground that the property still forms part of it as material for further discussion, thus:
the public domain. Nor is there any showing that
the lots in question are forestal land.... Did the enactment of the Property Registration Decree and the
amendatory P.D. No. 1073 preclude the application for registration of
Thus, while the Court of Appeals erred in ruling that mere possession alienable lands of the public domain, possession over which
of public land for the period required by law would entitle its occupant commenced only after June 12, 1945? It did not, considering Section
to a confirmation of imperfect title, it did not err in ruling in favor of 14(2) of the Property Registration Decree, which governs and
private respondents as far as the first requirement in Section 48(b) of authorizes the application of those who have acquired ownership of
the Public Land Act is concerned, for they were able to overcome the private lands by prescription under the provisions of existing laws.
burden of proving the alienability of the land subject of their
application. Prescription is one of the modes of acquiring ownership under the Civil
Code.[[30]] There is a consistent jurisprudential rule that properties
As correctly found by the Court of Appeals, private respondents were classified as alienable public land may be converted into private
able to prove their open, continuous, exclusive and notorious property by reason of open, continuous and exclusive possession of at
least thirty (30) years.[[31]] With such conversion, such property may The Naguit obiter had adverted to a frequently reiterated jurisprudence holding that
now fall within the contemplation of private lands under Section 14(2), properties classified as alienable public land may be converted into private property by
and thus susceptible to registration by those who have acquired reason of open, continuous and exclusive possession of at least thirty (30)
ownership through prescription. Thus, even if possession of the years.[36] Yet if we ascertain the source of the thirty-year period, additional complexities
alienable public land commenced on a date later than June 12, 1945, relating to Section 14(2) and to how exactly it operates would emerge. For there are in
and such possession being been open, continuous and exclusive, then fact two distinct origins of the thirty (30)-year rule.
the possessor may have the right to register the land by virtue of
Section 14(2) of the Property Registration Decree. The first source is Rep. Act No. 1942, enacted in 1957, which amended Section 48(b) of
the Public Land Act by granting the right to seek original registration of alienable public
Naguit did not involve the application of Section 14(2), unlike in this case where lands through possession in the concept of an owner for at least thirty years.
petitioners have based their registration bid primarily on that provision, and where the
evidence definitively establishes their claim of possession only as far back as 1948. It is The following-described citizens of the Philippines, occupying lands
in this case that we can properly appreciate the nuances of the provision. of the public domain or claiming to own any such lands or an interest
therein, but whose titles have not been perfected or completed, may
A. apply to the Court of First Instance of the province where the land is
The obiter in Naguit cited the Civil Code provisions on prescription as the possible basis located for confirmation of their claims and the issuance of a
for application for original registration under Section 14(2). Specifically, it is Article 1113 certificate of title therefor, under the Land Registration Act, to wit:
which provides legal foundation for the application. It reads:
xxx xxx xxx
All things which are within the commerce of men are susceptible of
prescription, unless otherwise provided. Property of the State or any of (b) Those who by themselves or through their predecessors in
its subdivisions not patrimonial in character shall not be the object of interest have been in open, continuous, exclusive and notorious
prescription. possession and occupation of agricultural lands of the public domain,
under a bona fide claim of acquisition of ownership, for at least
It is clear under the Civil Code that where lands of the public domain are patrimonial in thirty years immediately preceding the filing of the application
character, they are susceptible to acquisitive prescription. On the other hand, among for confirmation of title, except when prevented by war or force
the public domain lands that are not susceptible to acquisitive prescription are timber majeure. These shall be conclusively presumed to have performed
lands and mineral lands. The Constitution itself proscribes private ownership of timber all the conditions essential to a Government grant and shall be
or mineral lands. entitled to a certificate of title under the provisions of this Chapter.
(emphasis supplied)[37]
There are in fact several provisions in the Civil Code concerning the acquisition of real
property through prescription. Ownership of real property may be acquired by ordinary
prescription of ten (10) years,[32] or through extraordinary prescription of thirty (30) This provision was repealed in 1977 with the enactment of P.D. 1073, which made the
years.[33] Ordinary acquisitive prescription requires possession in good faith,[34] as date 12 June 1945the reckoning point for the first time. Nonetheless, applications for
well as just title.[35] registration filed prior to 1977 could have invoked the 30-year rule introduced by Rep.
Act No. 1942.
When Section 14(2) of the Property Registration Decree explicitly provides that persons
who have acquired ownership over private lands by prescription under the provisions of The second source is Section 14(2) of P.D. 1529 itself, at least by implication, as it
existing laws, it unmistakably refers to the Civil Code as a valid basis for the registration applies the rules on prescription under the Civil Code, particularly Article 1113 in
of lands. The Civil Code is the only existing law that specifically allows the acquisition by relation to Article 1137. Note that there are two kinds of prescription under the Civil
prescription of private lands, including patrimonial property belonging to the State. Thus, Codeordinary acquisitive prescription and extraordinary acquisitive prescription, which,
the critical question that needs affirmation is whether Section 14(2) does encompass under Article 1137, is completed through uninterrupted adverse possession for thirty
original registration proceedings over patrimonial property of the State, which a private years, without need of title or of good faith.
person has acquired through prescription.
Obviously, the first source of the thirty (30)-year period rule, Rep. Act No. 1942, became (2) Those which belong to the State, without being for public
unavailable after 1977. At present, the only legal basis for the thirty (30)-year period is use, and are intended for some public service or for the
the law on prescription under the Civil Code, as mandated under Section 14(2). development of the national wealth.
However, there is a material difference between how the thirty (30)-year rule operated
under Rep. Act No. 1942 and how it did under the Civil Code. Art. 421. All other property of the State, which is not of the character
stated in the preceding article, is patrimonial property
Section 48(b) of the Public Land Act, as amended by Rep. Act No. 1942, did not refer to
or call into application the Civil Code provisions on prescription. It merely set forth a It is clear that property of public dominion, which generally includes property belonging
requisite thirty-year possession period immediately preceding the application for to the State, cannot be the object of prescription or, indeed, be subject of the commerce
confirmation of title, without any qualification as to whether the property should be of man.[39] Lands of the public domain, whether declared alienable and disposable or
declared alienable at the beginning of, and continue as such, throughout the entire not, are property of public dominion and thus insusceptible to acquisition by
thirty-(30) years. There is neither statutory nor jurisprudential basis to assert Rep. Act prescription.
No. 1942 had mandated such a requirement,[38] similar to our earlier finding with
respect to the present language of Section 48(b), which now sets 12 June 1945 as the Let us now explore the effects under the Civil Code of a declaration by the President or
point of reference. any duly authorized government officer of alienability and disposability of lands of the
public domain. Would such lands so declared alienable and disposable be converted,
Then, with the repeal of Rep. Act No. 1942, the thirty-year possession period as basis under the Civil Code, from property of the public dominion into patrimonial property?
for original registration became Section 14(2) of the Property Registration Decree, After all, by connotative definition, alienable and disposable lands may be the object of
which entitled those who have acquired ownership over private lands by prescription the commerce of man; Article 1113 provides that all things within the commerce of man
under the provisions of existing laws to apply for original registration. Again, the thirty- are susceptible to prescription; and the same provision further provides that patrimonial
year period is derived from the rule on extraordinary prescription under Article 1137 of property of the State may be acquired by prescription.
the Civil Code. At the same time, Section 14(2) puts into operation the entire regime of
prescription under the Civil Code, a fact which does not hold true with respect to Section Nonetheless, Article 422 of the Civil Code states that [p]roperty of public dominion,
14(1). when no longer intended for public use or for public service, shall form part of the
patrimonial property of the State. It is this provision that controls how public dominion
B. property may be converted into patrimonial property susceptible to acquisition by
prescription. After all, Article 420 (2) makes clear that those property which belong to
Unlike Section 14(1), Section 14(2) explicitly refers to the principles on prescription the State, without being for public use, and are intended for some public service or for
under existing laws. Accordingly, we are impelled to apply the civil law concept of the development of the national wealth are public dominion property. For as long as the
prescription, as set forth in the Civil Code, in our interpretation of Section 14(2). There is property belongs to the State, although already classified as alienable or disposable, it
no similar demand on our part in the case of Section 14(1). remains property of the public dominion if when it is intended for some public service or
for the development of the national wealth.
The critical qualification under Article 1113 of the Civil Code is thus: [p]roperty of the
State or any of its subdivisions not patrimonial in character shall not be the object of Accordingly, there must be an express declaration by the State that the public
prescription. The identification what consists of patrimonial property is provided by dominion property is no longer intended for public service or the development of
Articles 420 and 421, which we quote in full: the national wealth or that the property has been converted into patrimonial.
Without such express declaration, the property, even if classified as alienable or
Art. 420. The following things are property of public dominion: disposable, remains property of the public dominion, pursuant to Article 420(2),
and thus incapable of acquisition by prescription. It is only when such alienable
(1) Those intended for public use, such as roads, canals, rivers, and disposable lands are expressly declared by the State to be no longer
torrents, ports and bridges constructed by the State, banks, intended for public service or for the development of the national wealth that the
shores, roadsteads, and others of similar character; period of acquisitive prescription can begin to run. Such declaration shall be in
the form of a law duly enacted by Congress or a Presidential Proclamation in
cases where the President is duly authorized by law.
It is comprehensible with ease that this reading of Section 14(2) of the Property
Registration Decree limits its scope and reach and thus affects the registrability even of The limitation imposed by Article 1113 dissuades us from ruling that the period of
lands already declared alienable and disposable to the detriment of the bona fide possession before the public domain land becomes patrimonial may be counted for the
possessors or occupants claiming title to the lands. Yet this interpretation is in accord purpose of completing the prescriptive period. Possession of public dominion property
with the Regalian doctrine and its concomitant assumption that all lands owned by the before it becomes patrimonial cannot be the object of prescription according to the Civil
State, although declared alienable or disposable, remain as such and ought to be used Code. As the application for registration under Section 14(2) falls wholly within the
only by the Government. framework of prescription under the Civil Code, there is no way that possession during
the time that the land was still classified as public dominion property can be counted to
Recourse does not lie with this Court in the matter. The duty of the Court is to apply the meet the requisites of acquisitive prescription and justify registration.
Constitution and the laws in accordance with their language and intent. The remedy is
to change the law, which is the province of the legislative branch. Congress can very Are we being inconsistent in applying divergent rules for Section 14(1) and Section
well be entreated to amend Section 14(2) of the Property Registration Decree and 14(2)? There is no inconsistency. Section 14(1) mandates registration on the basis
pertinent provisions of the Civil Code to liberalize the requirements for judicial of possession, while Section 14(2) entitles registration on the basis
confirmation of imperfect or incomplete titles. of prescription. Registration under Section 14(1) is extended under the aegis of
the Property Registration Decree and the Public Land Act while registration under
The operation of the foregoing interpretation can be illustrated by an actual example. Section 14(2) is made available both by the Property Registration Decree and the
Republic Act No. 7227, entitled An Act Accelerating The Conversion Of Military Civil Code.
Reservations Into Other Productive Uses, etc., is more commonly known as the BCDA
law. Section 2 of the law authorizes the sale of certain military reservations and portions In the same manner, we can distinguish between the thirty-year period under Section
of military camps in Metro Manila, including Fort Bonifacio and Villamor Air Base.For 48(b) of the Public Land Act, as amended by Rep. Act No. 1472, and the thirty-year
purposes of effecting the sale of the military camps, the law mandates the President to period available through Section 14(2) of the Property Registration Decree in relation to
transfer such military lands to the Bases Conversion Development Authority Article 1137 of the Civil Code. The period under the former speaks of a thirty-year
(BCDA)[40]which in turn is authorized to own, hold and/or administer them.[41]The period of possession, while the period under the latter concerns a thirty-year
President is authorized to sell portions of the military camps, in whole or in period of extraordinary prescription. Registration under Section 48(b) of the
part.[42].Accordingly, the BCDA law itself declares that the military lands subject thereof Public Land Act as amended by Rep. Act No. 1472 is based on thirty years of
are alienable and disposable pursuant to the provisions of existing laws and regulations possession alone without regard to the Civil Code, while the registration under
governing sales of government properties.[43] Section 14(2) of the Property Registration Decree is founded on extraordinary
prescription under the Civil Code.
From the moment the BCDA law was enacted the subject military lands have become
alienable and disposable. However, said lands did not become patrimonial, as the It may be asked why the principles of prescription under the Civil Code should not apply
BCDA law itself expressly makes the reservation that these lands are to be sold in order as well to Section 14(1). Notwithstanding the vaunted status of the Civil Code, it
to raise funds for the conversion of the former American bases ultimately is just one of numerous statutes, neither superior nor inferior to other statutes
at Clark and Subic.[44]Such purpose can be tied to either public service or the such as the Property Registration Decree. The legislative branch is not bound to adhere
development of national wealth under Article 420(2). Thus, at that time, the lands to the framework set forth by the Civil Code when it enacts subsequent legislation.
remained property of the public dominion under Article 420(2), notwithstanding their Section 14(2) manifests a clear intent to interrelate the registration allowed under that
status as alienable and disposable. It is upon their sale as authorized under the BCDA provision with the Civil Code, but no such intent exists with respect to Section 14(1).
law to a private person or entity that such lands become private property and cease to
be property of the public dominion. IV.
One of the keys to understanding the framework we set forth today is seeing how our
C. land registration procedures correlate with our law on prescription, which, under the
Should public domain lands become patrimonial because they are declared as such in a Civil Code, is one of the modes for acquiring ownership over property.
duly enacted law or duly promulgated proclamation that they are no longer intended for
public service or for the development of the national wealth, would the period of The Civil Code makes it clear that patrimonial property of the State may be acquired by
possession prior to the conversion of such public dominion into patrimonial be reckoned private persons through prescription. This is brought about by Article 1113, which states
in counting the prescriptive period in favor of the possessors? We rule in the negative. that [a]ll things which are within the commerce of man are susceptible to prescription,
and that [p]roperty of the State or any of its subdivisions not patrimonial in character the Torrens system. It should be remembered that registration of property is not a mode
shall not be the object of prescription. of acquisition of ownership, but merely a mode of confirmation of ownership.[48]

There are two modes of prescription through which immovables may be acquired under Looking back at the registration regime prior to the adoption of the Property Registration
the Civil Code. The first is ordinary acquisitive prescription, which, under Article 1117, Decree in 1977, it is apparent that the registration system then did not fully
requires possession in good faith and with just title; and, under Article 1134, is accommodate the acquisition of ownership of patrimonial property under the Civil Code.
completed through possession of ten (10) years. There is nothing in the Civil Code that What the system accommodated was the confirmation of imperfect title brought about
bars a person from acquiring patrimonial property of the State through ordinary by the completion of a period of possession ordained under the Public Land Act (either
acquisitive prescription, nor is there any apparent reason to impose such a rule. At the 30 years following Rep. Act No. 1942, or since 12 June 1945 following P.D. No. 1073).
same time, there are indispensable requisitesgood faith and just title. The ascertainment
of good faith involves the application of Articles 526, 527, and 528, as well as Article The Land Registration Act[49] was noticeably silent on the requisites for alienable public
1127 of the Civil Code,[45] provisions that more or less speak for themselves. lands acquired through ordinary prescription under the Civil Code, though it arguably did
not preclude such registration.[50]Still, the gap was lamentable, considering that the
On the other hand, the concept of just title requires some clarification. Under Article Civil Code, by itself, establishes ownership over the patrimonial property of persons who
1129, there is just title for the purposes of prescription when the adverse claimant came have completed the prescriptive periods ordained therein. The gap was finally closed
into possession of the property through one of the modes recognized by law for the with the adoption of the Property Registration Decree in 1977, with Section 14(2)
acquisition of ownership or other real rights, but the grantor was not the owner or could thereof expressly authorizing original registration in favor of persons who have acquired
not transmit any right. Dr. Tolentino explains: ownership over private lands by prescription under the provisions of existing laws, that
is, the Civil Code as of now.
Just title is an act which has for its purpose the transmission of V.
ownership, and which would have actually transferred ownership if the We synthesize the doctrines laid down in this case, as follows:
grantor had been the owner. This vice or defect is the one cured by
prescription. Examples: sale with delivery, exchange, donation, (1) In connection with Section 14(1) of the Property Registration Decree, Section 48(b)
succession, and dacion in payment.[46] of the Public Land Act recognizes and confirms that those who by themselves or
The OSG submits that the requirement of just title necessarily precludes the applicability through their predecessors in interest have been in open, continuous, exclusive, and
of ordinary acquisitive prescription to patrimonial property. The major premise for the notorious possession and occupation of alienable and disposable lands of the public
argument is that the State, as the owner and grantor, could not transmit ownership to domain, under a bona fide claim of acquisition of ownership, since June 12, 1945 have
the possessor before the completion of the required period of possession.[47] It is acquired ownership of, and registrable title to, such lands based on the length and
evident that the OSG erred when it assumed that the grantor referred to in Article 1129 quality of their possession.
is the State. The grantor is the one from whom the person invoking ordinary acquisitive
prescription derived the title, whether by sale, exchange, donation, succession or any (a)Since Section 48(b) merely requires possession since 12 June 1945 and
other mode of the acquisition of ownership or other real rights. does not require that the lands should have been alienable and disposable
during the entire period of possession, the possessor is entitled to secure
Earlier, we made it clear that, whether under ordinary prescription or extraordinary judicial confirmation of his title thereto as soon as it is declared alienable and
prescription, the period of possession preceding the classification of public dominion disposable, subject to the timeframe imposed by Section 47 of the Public Land
lands as patrimonial cannot be counted for the purpose of computing prescription. But Act.[51]
after the property has been become patrimonial, the period of prescription begins to run
in favor of the possessor. Once the requisite period has been completed, two legal (b)The right to register granted under Section 48(b) of the Public Land Act is
events ensue: (1) the patrimonial property is ipso jure converted into private land; and further confirmed by Section 14(1) of the Property Registration Decree.
(2) the person in possession for the periods prescribed under the Civil Code acquires
ownership of the property by operation of the Civil Code. (2) In complying with Section 14(2) of the Property Registration Decree, consider that
under the Civil Code, prescription is recognized as a mode of acquiring ownership of
It is evident that once the possessor automatically becomes the owner of the converted patrimonial property. However, public domain lands become only patrimonial property
patrimonial property, the ideal next step is the registration of the property under not only with a declaration that these are alienable or disposable. There must also be an
express government manifestation that the property is already patrimonial or no longer
retained for public service or the development of national wealth, under Article 422 of The informal settlement of public lands, whether declared alienable or not, is a
the Civil Code. And only when the property has become patrimonial can the prescriptive phenomenon tied to long-standing habit and cultural acquiescence, and is common
period for the acquisition of property of the public dominion begin to run. among the so-called Third World countries. This paradigm powerfully evokes the
disconnect between a legal system and the reality on the ground. The law so far has
(a)Patrimonial property is private property of the government. The person been unable to bridge that gap. Alternative means of acquisition of these
acquires ownership of patrimonial property by prescription under the Civil Code public domain lands, such as through homestead or free patent, have proven
is entitled to secure registration thereof under Section 14(2) of the Property unattractive due to limitations imposed on the grantee in the encumbrance or alienation
Registration Decree. of said properties.[52] Judicial confirmation of imperfect title has emerged as the most
viable, if not the most attractive means to regularize the informal settlement of alienable
(b)There are two kinds of prescription by which patrimonial property may be or disposable lands of the public domain, yet even that system, as revealed in this
acquired, one ordinary and other extraordinary. Under ordinary acquisitive decision, has considerable limits.
prescription, a person acquires ownership of a patrimonial property through
possession for at least ten (10) years, in good faith and with just title. Under There are millions upon millions of Filipinos who have individually or exclusively held
extraordinary acquisitive prescription, a persons uninterrupted adverse residential lands on which they have lived and raised their families. Many more have
possession of patrimonial property for at least thirty (30) years, regardless of tilled and made productive idle lands of the State with their hands. They have been
good faith or just title, ripens into ownership. regarded for generation by their families and their communities as common law
owners. There is much to be said about the virtues of according them legitimate states.
B. Yet such virtues are not for the Court to translate into positive law, as the law itself
We now apply the above-stated doctrines to the case at bar. considered such lands as property of the public dominion. It could only be up to
Congress to set forth a new phase of land reform to sensibly regularize and formalize
It is clear that the evidence of petitioners is insufficient to establish that Malabanan has the settlement of such lands which in legal theory are lands of the public domain before
acquired ownership over the subject property under Section 48(b) of the Public Land the problem becomes insoluble. This could be accomplished, to cite two examples, by
Act. There is no substantive evidence to establish that Malabanan or petitioners as his liberalizing the standards for judicial confirmation of imperfect title, or amending the Civil
predecessors-in-interest have been in possession of the property since 12 June 1945 or Code itself to ease the requisites for the conversion of public dominion property into
earlier. The earliest that petitioners can date back their possession, according to their patrimonial.
own evidencethe Tax Declarations they presented in particularis to the year 1948. Thus,
they cannot avail themselves of registration under Section 14(1) of the Property Ones sense of security over land rights infuses into every aspect of well-being not only
Registration Decree. of that individual, but also to the persons family. Once that sense of security is deprived,
life and livelihood are put on stasis. It is for the political branches to bring welcome
Neither can petitioners properly invoke Section 14(2) as basis for registration. While the closure to the long pestering problem.
subject property was declared as alienable or disposable in 1982, there is no competent
evidence that is no longer intended for public use service or for the development of the WHEREFORE, the Petition is DENIED. The Decision of the Court of Appeals dated 23
national evidence, conformably with Article 422 of the Civil Code. The classification of February 2007 and Resolution dated 2 October 2007 are AFFIRMED. No
the subject property as alienable and disposable land of the public domain does not pronouncement as to costs.
change its status as property of the public dominion under Article 420(2) of the Civil
Code. Thus, it is insusceptible to acquisition by prescription. SO ORDERED.

VI. DANTE O. TINGA


A final word. The Court is comfortable with the correctness of the legal doctrines Associate Justice
established in this decision. Nonetheless, discomfiture over the implications of todays
ruling cannot be discounted. For, every untitled property that is occupied in the country
will be affected by this ruling. The social implications cannot be dismissed lightly, and
the Court would be abdicating its social responsibility to the Filipino people if we simply
levied the law without comment.
Gregorio Bilog, Jr. for defendant Land Registration Commissioner.

ESGUERRA, J.:

This case was originally appealed to the Court of Appeals where it was docketed as CA-
G.R. No. 47438-R. The Court of Appeals certified it to this Court for final consideration
and resolution of the pure question of law involved.

The factual background of the case is as follows:

On May 9, 1969, a Complaint for Annulment of Certificate of Title was filed by the
Republic of the Philippines (represented by the Director of Lands), with the Court of
First Instance of Davao, Branch 1, alleging, among others, the following:

3. That defendant Commissioner of Land Registration and defendant


Register of Deeds of Davao City whose Offices are at Espaa
Extension, Quezon City and Davao City, respectively. "(are included
in this complaint, the first being the public Official charged under the
law with the approval )." subdivision surveys of private lands while the
second is the Official vested with the authority to issue certificates of
titles, pursuant to the provisions of Act 496, as amended, otherwise
known as the Land Registration Law;

4. That defendant Estate of Luisa Villa Abrille (now Heirs of Luisa Villa
Abrille) is the owner of a parcel of land in the City of Davao containing
an area of FIVE HUNDRED TWENTY FIVE THOUSAND SIX
HUNDRED FIFTY TWO SQUARE METERS (525.652), more or less,
under Transfer Certificate of Title No. T-1439 of the Registry of Deeds
of Davao City, issued in her name;

5. That deceased Luisa Villa Abrille during her lifetime caused the
Republic of the Philippines subdivision of the aforesaid parcel of land into two lots designated as
SUPREME COURT Lots Nos. 379-B-2-B-1 and 379-B-2-B-2 under subdivision plan (LRC)
Manila Psd-69322 which was approved by the Land Registration
FIRST DIVISION Commissioner on March 17,1967;

G.R. No. L-39248 May 7, 1976 6. That under Subdivision Plan (LRC) Psd-69322, Lot No. 379- B-2-B-
1 contains an area of 30,100 Square Meters while Lot No. 379-B-2B-2
REPUBLIC OF THE PHILIPPINES, represented by the DIRECTOR OF contains an area of 577,679 Square Meters or a total area of 607,779
LANDS, plaintiff-appellee, Square Meters, which is 82,127 Square Meters more than the original
vs. area covered in Transfer Certificate of Title No. T-1439 in the name of
HEIRS OF LUISA VILLA ABRILLE, defendant-appellant, LAND REGISTRATION said defendant Luisa Villa Abrille;
COMMISSIONER and THE REGISTER OF DEEDS OF DAVAO CITY, defendants.
7. That on March 27, 1967 or ten days after the approval by the Land
Solicitor General Estelito P. Mendoza, Assistant Solicitor General Octavio R. Ramirez Registration Commissioner, said Luisa Villa Abrille was able to secure
and Atty. Baltazar Llamas plaintiff-appellee. an order from the Court of First Instance of Davao in LRC (GLRO)
Jose R. Madrazo, Jr. for defendant-appellant. Doc. No. 9969, directing the Register of Deeds for the City of Davao
and Province of Davao, to correct the area of Certificate of Title No. T-
1439 and thereafter to cancel the same and issue in lieu thereof TCT entitled under the law to claim, as she did, the increase or excess in area of her original
Nos. T-18886 and T-18887; land as her own.

8. That on March 30, 1967, the Register of Deeds concerned On August 12, 1969, defendant Commissioner of Land Registration prays for a
registered Lot 379-B-2-B-1 and issued TCT No. 18886 therefor, in the judgment on the pleadings and avers in his answer that he has no knowledge of the
name of Luisa Villa Abrille and on the same date registered Lot No. subject matter of the complaint since the subdivision plan involved therein was
379-B-2-B-2 and issued TCT No. 18887 in the name of Luisa Villa approved by the then Commissioner of Land Registration, Antonio Noblejas; and that on
Abrille; February 19, 1968, the then Commissioner of Land Registration, Antonio Noblejas,
issued LRC Circular No. 167 directing the Register of Deeds throughout the Philippines
9. That the registration of Lot No. 379-B-2-B-2, which includes the
to, among others, deny the registration of subdivision plans with increased or expanded
aforementioned excess area of 82,127 Square Meters, was not in
areas and to withhold the issuance of the corresponding titles, or if the plans have
accordance with law for lack of the required notice and publication as
already been registered and the titles issued, to recall the titles and to take appropriate
prescribed in Act 496, as amended, otherwise known as the Land
steps for their cancellation.
Registration Law;
Some private persons, as actual possessors and occupants, tried to intervene in the
10. That the excess or enlarged area of 82,127 Square Meters as a
case as movant-intervenors but they were denied standing in court by the trial court in
result of the approval of the subdivision survey (LRC) Psd-69322 was
its order of August 16,1969.
formerly a portion of the Davao River which dried up by reason of the
change of course of the said Davao River; hence a land belonging to On January 6, 1970, the parties litigants submitted in court their "Agreed Stipulation of
the public domain; and Facts" and pray that judgment be rendered by the trial court on their case based on
their stipulation of facts. The "Agreed Stipulation of Facts" of the parties reads as
11. That as a consequence thereof, Transfer Certificate of Title No.
follows:
18887 which covers Lot No. 379-B-2-B-2 of Subdivision Survey (LRC)
Psd-69322, wherein the excess area of land belong to the public COME NOW the parties assisted by their respective attorneys, and
domain (not private land) is null and void ab initio. unto the Honorable Court, most respectfully submit the following
stipulation of facts and allege:
On June 10, 1969, defendant Register of Deeds of Davao- City filed her answer
averring that she, "in the performance of her ministerial duty, honestly and in good faith 1. That Lot 379-B-2-B was originally registered on June 28, 1916 in
effected the registration of Subdivision Lot No. 379-B-2-B-1 and Lot No. 379B-2-B-2 and the Registry Book of the Register of Deeds of Zamboanga as Vol.
the issuance of corresponding TCT No. 18886 and TCT No. 18887 therefor, A27, Page 40 under Original Certificate of Title No. 5609, Case No. 1,
respectively, in view of the approval of the Land Registration Commissioner of G.L.R.O. Rec. No. 317, in the name of Francisco Villa Abrille Lim
Subdivision Plan (LRC) Psd-69322, and in view of the Order of the Court of First Juna, father of Luisa Villa Abrille;
Instance of Davao to correct the area in Certificate of Title No. T-1439, to cancel the
2. That upon the death of the original owner, the said property was
same and to issue in lieu thereof TCT Nos. T-18886 and T-18887".
inherited by Luisa Villa Abrille and Transfer Certificate of Title No. T-
On July 2, 1969, herein defendant-appellants filed their answer admitting the allegations 1439 was issued in the name of said Luisa Villa Abrille;
contained in paragraphs 1, 3, 4, 5 and 7 of the complaint; that they admit the increase in
3. That subsequently, by virtue of an approved subdivision plan Psd-
area of the land of their predecessor but that the increase in area of the land was
69322 by the defendant, Land Registration Commissioner, Transfer
acceded to and concurred in by the defendant, Land Registration Commissioner, and
Certificate of Title Nos. T-18886 and 18887 were issued by the
the same was duly noted and approved by the Court of First Instance of Davao; that
defendant, Register of Deeds of Davao, copy of which subdivision
they admit the issuance of TCT Nos. T-18886 and T-18887 out of Certificate of Title No.
plan is hereto attached as Annex "A", and made integral part hereof;
T- 1439 in the name of their predecessor-in-interest Luisa Villa Abrille but that TCT No.
T-18886 had been cancelled and in lieu thereof, TCT No. T-19077 was issued in favor 4. That Transfer Certificate of Title No. T-18886 was subsequently
of Gaudencio Consunji, and, TCT No. T-18887 had likewise been cancelled and several cancelled by virtue of deed of sale, and Transfer Certificate of Title
Transfer Certificates of Title were issued thereunder; that the subject increase of area No. T-19077 was issued in the name of Gaudencio Consunji a
was made in accordance with law and existing jurisprudence; and that Luisa Villa purchaser in good faith and for value;
Abrille, predecessor-in-interest of herein defendant-appellant, as riparian owner was
5. That the said subdivision plan Annex "A" was also approved by the registered owner holds property adjacent to the parcel of land in
Court of First Instance of Davao, Branch IV, through an Order dated question;
March 27, 1967, copy of which order is hereto attached as Annex "B"
10. That the portion of land subject of the increase adjoins Lot 379-B-
and made part hereof;
2-B and abuts the Davao River;
6. That the said Order Annex "B" was issued by the Court of First
11. That the parcel of land subject of the increase is fully planted with
Instance of Davao, Branch IV, on the strength of the Report of the
coconuts, bananas and other seasonal crops by the defendants,
defendant, Land Registration Commissioner, copy of which report is
through their predecessor-in-interest;
hereto attached as Annex "C" and made integral part hereof;
12. That the increase in area could have taken place very long time
7. That much later on, Transfer Certificate of Title No. T-18887 was by
ago as the coconuts planted thereon had long been fruit bearing;
virtue of an Order of the Court of First Instance, Branch 1, in Special
Proceedings No. 1357, entitled: In the Matter of the Testate Estate of 13. That Transfer Certificate of Title No. 18886 does not contain any
Luisa Villa Abrille, approving a project of partition cancelled, and in portion of the increase in area;
lieu thereof, the following Transfer Certificates of Title were issued to
the following named persons, to wit: 14. That of the certificates of title issued based under subdivision plan
(LRC) Psd-71236, only Transfer Certificates of Title Nos. T- 20725; T-
(a) T-20690 - Huang Siu Sin; 20701; T-20713; and T-20690 contain the increase in area; while all
the other certificates of title issued under subdivision plan (LRC) Psd-
(b) T-20692 - Huang Siu Sin;
71236 do not contain any increase in area;
(c) T-20701 - Josefino Huang;
15. That the parties agree that the issuance of the Order Annex "B"
(d) T-20702 - Josefino Huang; was without notice to the Director of Lands.

(e) T-20703 - Josefino Huang; The trial court thereafter rendered its decision dated January 27,
1970, which reads as follows:
(f) T-20732 Huang Siu Sin, et al.;
This is an ordinary civil action for annulment of certificate of title
(g) T-20733 - Huang Siu Sin, et al.;
instituted by the Republic of the Philippines, represented by the
(h) T-20713 - Miguel Huang; Director of Lands, against the Estate of Luisa Abrille, represented by
Huang Siu Sin, Administrator, the Land Registration Commissioner
(i) T-23015 - Miguel Huang; and the Register of Deeds of the City of Davao. Because the residue
(j) T-20725 - Milagros Huang; of the intestate estate of Luisa Villa Abrille had been divided among
Huang Siu Sin, Josefino Huang, Milagros Huang, Miguel Huang and
(k) T-20726 - Milagros Huang; lap Tong Ha, heirs, they were directed to appear and to substitute for
the intestate estate and they did.
which certificates of title were issued on the basis of a subdivision
plan LRC Psd-71236 duly approved by the defendant, Land The parties submitted the following stipulation of facts:
Registration Commissioner, copy of which subdivision plan (LRC)
Psd-71236 is hereto attached as Annex "D" and made integral part xxx xxx xxx
hereof; The increase in area of the land covered by Original Certificate of Title
8. That the parties admit that there was an increase in the area of Lot No. 5609 of the Register of Deeds of Davao in the name of Francisco
379-B-2-B, but the same was with the knowledge of the defendant, Villa Abrille Lim Juna and subsequently by Transfer Certificate of Title
Land Registration Commissioner and the court of First Instance of No. T. 1439 in the name of Luisa Villa Abrille and finally, based on
Davao, Branch IV; subdivision plan (LRC) Psd-71236, by Transfer Certificates of Title
Nos. T-20725 in the name of Milagros Huang, T20701 in the name of
9. That the parties admit that no registered owner has been affected Josefino Huang, T-20713 in the name of Miguel Huang and T-20690
or prejudiced in the increase in area as only Luisa Villa Abrille as the
in the name of Huang Siu Sin, is from 525,652 square meters to directing the Register of Deeds of Davao to issue new certificates of
607,779 square meters, or 82,127 square meters. title in lieu thereof after the portions consisting of 82,127 square
meters, the land involved, shall have been segregated therefrom in
The remedy sought by defendant heirs of Luisa Villa Abrille in order to
accordance with law.
include the increase in area was a petition for approval of Subdivision
Plan (LRC) Psd-79322 recommended by the Commissioner of Land Not satisfied with the judgment of the trial court, defendant Heirs of Luisa Villa Abrille
Registration in his Report, and for issuance of new title under Section brought the case on appeal to the Court of Appeals. The Court of Appeals, however, in
44, Act 496, as amended, filed with this court, which was assigned to its Resolution dated July 22, 1974, certified the case (CA-G.R. No. 47438-R) to this
Branch IV. Court for consideration and final disposition.

Even pursuant to Section 44 of Act 496 under which the aforesaid Defendant-appellant maintains that the lower court erred in holding the approval of
remedy was sought, notice before the hearing is required. The parties Subdivision Plan (LRC) Psd-69322 of no legal effect merely on ground of lack of notice
admit that there was no notice to the persons interested, including the to interested persons, and in ordering the cancellation of Certificates of Title Nos. T-
Director of Lands, before the petition was heard. 20725, T-20701, T-20713 and T-20690. It is the contention of the defendant-appellant
that since the government agencies having to do with lands know all the time the
Worse, the increase in area could not have been included in Transfer
increase in area in subdivision plan Psd-69322, and the government agencies
Certificates of Title Nos. T-20725, T-20701, T-20713 and T-20690
concerned tolerated if not abetted the ultimate inclusion of the involved increase in area,
even assuming arguendo that the same belonged to the owner of the
defendant-appellant should not be made to suffer the effect of the allegedly wrong
land to which it is adjacent by the simple expediency of a petition for
procedure or step taken in the approval of the aforementioned subdivision plan.
approval of subdivision plan and issuance of new titles, because a
Besides, defendant-appellant claims that it is their honest belief that the legal remedy
subdivision of a registered land under Section 44 of Act 496 does not
taken by them in seeking the approval of their subdivision plan concerned was well
authorize the inclusion of land or area not embraced in the titled or in
within the law, particularly the provision of Section 44 of Act 496, as amended.
excess of what is stated in the title. And the approval by the Court of
such subdivision plan does not lend validity to it. The subdivision must Plaintiff-appellee, on the other hand, maintains that the approval of the subdivision plan,
be limited to the area stated in the title. Neither amendment of the title with the increase in area, by the defendant-appellant Land Registration Commission
under Section 112 of Act 496 would be a valid remedy 1. does not lend validity to the said subdivision plan; and that the issuance of the four
transfer certificates of title (Nos. T-20725, T-20701, T-20713 and T-20690) over the
The heirs of Luisa Villa Abrille.. owners of the adjacent estate, might
increased area in question is improper and invalid notwithstanding the conformity of the
have acquired a registrable title to the land in question but to bring it
Land Registration Commissioner and the subsequent order of the Court of First
under the operation of the Land Registration Act, a petition for
Instance of Davao, Branch IV, approving the subdivision plan concerned, as the
registration under Act 496 should have been filed. More so when the
required giving of notice to all parties interested in defendant-appellant's petition for
title acquired is by continuous possession for at least 30 years under
approval of subdivision plan was not at all followed,
a claim of ownership And even assuming that the land is an accretion,
the fact that the riparian estate is registered does not bring ipso facto Before Us, therefore, for consideration and final resolution, in order to arrive at a
effect its accretion thereto under the operation of the Land judicious disposition of the case at bar, is whether or not the lower court erred in
Registration Act. No decree of registration of the land based upon ordering the cancellation of Transfer Certificates of Title Nos. T-20725, T-20701, T-
final judgment promulgated by a court of competent jurisdiction after 20713 and T-20690 which cover the increased area in question totalling 82,127 square
due publication, notice and hearing, has been issued by the meters.
Commissioner of Land Registration and transcribed by the Register of
After a careful and thorough deliberation of the matter in controversy, We are of the
Deeds of Davao in the registry, for the reason that no initial or original
opinion and so hold that the lower court acted correctly in ordering the cancellation of
registration proceedings have been instituted by the owner. And the
Transfer Certificates of Title Nos. T-20725, T-20701, T-20713 and T-20690 which
only way by which a title to the land in question can be issued for the
admittedly covered the increased area of 82,127 square meters under Subdivision Plan
first time is for the Land Registration Commissioner to issue a decree
(LRC) Psd-71236 (and formerly under Psd-69322) for the City of Davao.
of registration based upon final judgment rendered by a court of
competent jurisdiction after trial. Certainly, the step taken by defendant-appellant in petitioning the court for the approval
of their Subdivision Plan (LRC) Psd-69322 and then Psd-71236 to include the
WHEREFORE, judgment is hereby rendered cancelling Transfer
questioned increased area of 82,127 square meters is, to say the least, unwarranted
Certificates of Title Nos. T-20725, T-20701, T-20713 and T-20690 and
and irregular. This is so, for the increased area in question, which is not a registered 12. Sending of copy of the decree of registration to the corresponding
land but formerly a river bed, is so big as to give allowance for a mere mistake in area of Register of Deeds, and
the original registration of the tracts of land of the defendant-appellant formerly
13. Transcription of the decree of registration in the registration book
belonging to and registered in the name of their grandfather, Francisco Villa Abrille Lim
and the issuance of the owner's duplicate original certificate of title to
Juna. In order to bring this increase in area, which the parties admitted to have been a
the applicant by the Register of Deeds, upon payment of the
former river bed of the Davao River, under the operation and coverage of the Land
prescribed fees.
Registration Law, Act 496, proceedings in registrations of land title should have been
filed Instead of an ordinary approval of subdivision plan. Hence, with the foregoing requisites not having been complied with, the lower court
committed no error in its appealed decision dated January 27, 1970.
It should be remembered that recourse under Section 44 of Act 496, which the
predecessor-in-interest (Luisa Villa Abrille) of the herein defendant-appellant took, is WHEREFORE, the judgment appealed from is hereby affirmed in toto.
good only insofar as it covers previously registered lands. In the instant case, part of the
tracts of land, particularly the area of 82,127 square meters, has not yet been brought No special pronouncement as to costs.
under the operation of the Torrens System. Worse still, the approval of Subdivision SO ORDERED.
Plans (LRC) Psd-69322 and Psd-71236 was without notice to all parties in interest,
more particularly the Director of Lands. For an applicant to have his imperfect or Teehankee (Chairman), Makasiar, Muoz Palma and Martin, JJ., concur.
incomplete title or claim to a land to be originally registered under Act 496, the following
requisites should all be satisfied:

1. Survey of land by the Bureau of Lands or a duly licensed private


surveyor;

2. Filing of application for registration by the applicant;

3. Setting of the date for the initial hearing of the application by the
Court;

4. Transmittal of the application and the date of initial hearing together


with all the documents or other evidences attached thereto by the
Clerk of Court to the Land Registration Commission;

5. Publication of a notice of the filing of the application and date and


place of the hearing in the Official Gazette;

6. Service of notice upon contiguous owners, occupants and those


known to have interests in the property by the sheriff;

7. Filing of answer to the application by any person whether named in


the notice or not;

8. Hearing of the case by the Court;

9. Promulgation of judgment by the Court;

10. Issuance of the decree by the Court declaring the decision final
and instructing the Land Registration Commission to issue a decree of
confirmation and registration;

11. Entry of the decree of registration in the Land Registration


Commission;
Angela Razon. From this judgment the Director of Lands took this appeal, assigning
thereto the following errors, to wit: (1) The holding that the judgment rendered in a prior
case between the plaintiff and defendant Angela Razon on the parcel of land in question
is controlling in this action; (2) the holding that plaintiff is entitled to recover the
possession of said parcel of land; the annulment of the sale made by the Director of
Lands to Angela Razon; and the ordering that the certificate of title issued by the
register of deeds of the Province of Pampanga to Angela Razon by virtue of said sale
be cancelled; and (3) the denial of the motion for new trial filed by the Director of Lands.

The evidence shows that on December 18, 1880, Nemesio Pinlac sold the land in
question, then a fish pond, tho Apolonio Garcia and Basilio Mendoza for the sum of
Republic of the Philippines P12, reserving the right to repurchase the same (Exhibit B). After having been in
SUPREME COURT possession thereof for about eight years, and the fish pond having been destroyed,
Manila Apolonio Garcia and Basilio Mendoza, on September 5, 1899, sold it to Valentin Susi for
the sum of P12, reserving the right to repurchase it (Exhibit A). Before the execution of
EN BANC the deed of sale, Valentin Susi had already paid its price and sown "bacawan" on said
land, availing himself of the firewood gathered thereon, with the proceeds of the sale of
G.R. No. L-24066 December 9, 1925
which he had paid the price of the property. The possession and occupation of the land
VALENTIN SUSI, plaintiff-appellee, in question, first, by Apolonio Garcia and Basilio Mendoza, and then by Valentin Susi
vs. has been open, continuous, adverse and public, without any interruption, except during
ANGELA RAZON and THE DIRECTOR OF LANDS, defendants. THE DIRECTOR OF the revolution, or disturbance, except when Angela Razon, on September 13, 1913,
LANDS, appellant. commenced an action in the Court of First Instance of Pampanga to recover the
possession of said land (Exhibit C), wherein after considering the evidence introduced
Acting Attorney-General Reyes for appellant.
at the trial, the court rendered judgment in favor of Valentin Susi and against Angela
Monico R. Mercado for appellee.
Razon, dismissing the complaint (Exhibit E). Having failed in her attempt to obtain
possession of the land in question through the court, Angela Razon applied to the
Director of Lands for the purchase thereof on August 15, 1914 (Exhibit C). Having
VILLA-REAL, J.:
learned of said application, Valentin Susi filed and opposition thereto on December 6,
This action was commenced in the Court of First Instance of Pampanga by a complaint 1915, asserting his possession of the land for twenty-five years (Exhibit P). After making
filed by Valentin Susi against Angela Razon and the Director of Lands, praying for the proper administrative investigation, the Director of Lands overruled the opposition of
judgment: (a) Declaring plaintiff the sole and absolute owner of the parcel of land Valentin Susi and sold the land to Angela Razon. By virtue of said grant the register of
described in the second paragraph of the complaint; (b) annulling the sale made by the deeds of Pampanga, on August 31, 1921, issued the proper certificate of title to Angela
Director of Lands in favor of Angela Razon, on the ground that the land is a private Razon. Armed with said document, Angela Razon required Valentin Susi to vacate the
property; (c) ordering the cancellation of the certificate of title issued to said Angela land in question, and as he refused to do so, she brought and action for forcible entry
Razon; and (d) sentencing the latter to pay plaintiff the sum of P500 as damages, with and detainer in the justice of the peace court of Guagua, Pampanga, which was
the costs. dismissed for lack of jurisdiction, the case being one of title to real property (Exhibit F
and M). Valentin Susi then brought this action.
For his answer to the complaint, the Director of Lands denied each and every allegation
contained therein and, as special defense, alleged that the land in question was a With these facts in view, we shall proceed to consider the questions raised by the
property of the Government of the United States under the administration and control of appellant in his assignments of error.lawphi1.net
the Philippine Islands before its sale to Angela Razon, which was made in accordance It clearly appears from the evidence that Valentin Susi has been in possession of the
with law. land in question openly, continuously, adversely, and publicly, personally and through
After trial, whereat evidence was introduced by both parties, the Court of First Instance his predecessors, since the year 1880, that is, for about forty-five years. While the
of Pampanga rendered judgment declaring the plaintiff entitled to the possession of the judgment of the Court of First Instance of Pampanga against Angela Razon in the
land, annulling the sale made by the Director of Lands in favor of Angela Razon, and forcible entry case does not affect the Director of Lands, yet it is controlling as to Angela
ordering the cancellation of the certificate of title issued to her, with the costs against Razon and rebuts her claim that she had been in possession thereof. When on August
15, 1914, Angela Razon applied for the purchase of said land, Valentin Susi had
already been in possession thereof personally and through his predecessors for thirty-
four years. And if it is taken into account that Nemesio Pinlac had already made said
land a fish pond when he sold it on December 18, 1880, it can hardly be estimated
when he began to possess and occupy it, the period of time being so long that it is
beyond the reach of memory. These being the facts, the doctrine laid down by the
Supreme Court of the United States in the case of Cario vs. Government of the
Philippine Islands (212 U. S., 449 1), is applicable here. In favor of Valentin Susi, there Republic of the Philippines
SUPREME COURT
is, moreover, the presumption juris et de jure established in paragraph (b) of section 45
Manila
of Act No. 2874, amending Act No. 926, that all the necessary requirements for a grant
EN BANC
by the Government were complied with, for he has been in actual and physical
G.R. No. 73002 December 29, 1986
possession, personally and through his predecessors, of an agricultural land of the
THE DIRECTOR OF LANDS, petitioner,
public domain openly, continuously, exclusively and publicly since July 26, 1894, with a
vs.
right to a certificate of title to said land under the provisions of Chapter VIII of said Act. INTERMEDIATE APPELLATE COURT and ACME PLYWOOD & VENEER CO. INC.,
So that when Angela Razon applied for the grant in her favor, Valentin Susi had already ETC., respondents.
acquired, by operation of law, not only a right to a grant, but a grant of the Government, D. Nacion Law Office for private respondent.
for it is not necessary that certificate of title should be issued in order that said grant
may be sanctioned by the courts, an application therefore is sufficient, under the NARVASA, J.:
provisions of section 47 of Act No. 2874. If by a legal fiction, Valentin Susi had acquired The Director of Lands has brought this appeal by certiorari from a judgment of the
the land in question by a grant of the State, it had already ceased to be the public Intermediate Appellate Court affirming a decision of the Court of First Instance of
domain and had become private property, at least by presumption, of Valentin Susi, Isabela, which ordered registration in favor of Acme Plywood & Veneer Co., Inc. of five
beyond the control of the Director of Lands. Consequently, in selling the land in question parcels of land measuring 481, 390 square meters, more or less, acquired by it from
to Angela Razon, the Director of Lands disposed of a land over which he had no longer Mariano and Acer Infiel, members of the Dumagat tribe.
any title or control, and the sale thus made was void and of no effect, and Angela Razon The registration proceedings were for confirmation of title under Section 48 of
did not thereby acquire any right. Commonwealth Act No. 141 (The Public Land Act). as amended: and the appealed
judgment sums up the findings of the trial court in said proceedings in this wise:
The Director of Lands contends that the land in question being of the public domain, the 1. That Acme Plywood & Veneer Co. Inc., represented by Mr. Rodolfo Nazario is
plaintiff-appellee cannot maintain an action to recover possession thereof.lawphi1.net a corporation duly organized in accordance with the laws of the Republic of the
Philippines and registered with the Securities and Exchange Commission on December
If, as above stated, the land, the possession of which is in dispute, had already become,
23, 1959;
by operation of law, private property of the plaintiff, there lacking only the judicial
2. That Acme Plywood & Veneer Co. Inc., represented by Mr. Rodolfo Nazario
sanction of his title, Valentin Susi has the right to bring an action to recover possession
can acquire real properties pursuant to the provisions of the Articles of Incorporation
thereof and hold it.
particularly on the provision of its secondary purposes (paragraph (9), Exhibit 'M-l');
For the foregoing, and no error having been found in the judgment appealed from, the 3. That the land subject of the Land Registration proceeding was ancestrally
same is hereby affirmed in all its parts, without special pronouncement as to costs. So acquired by Acme Plywood & Veneer Co., Inc., on October 29, 1962, from Mariano
ordered. Infiel and Acer Infiel, both members of the Dumagat tribe and as such are cultural
minorities;
Avancea, C.J., Malcolm, Street, Villamor, Ostrand, Johns, and Romualdez, JJ., 4. That the constitution of the Republic of the Philippines of 1935 is applicable as
concur. the sale took place on October 29, 1962;
Johnson, J., took no part. 5. That the possession of the Infiels over the land relinquished or sold to Acme
Plywood & Veneer Co., Inc., dates back before the Philippines was discovered by
Footnotes
Magellan as the ancestors of the Infiels have possessed and occupied the land from
1 41 Phil., 935. generation to generation until the same came into the possession of Mariano Infiel and
Acer Infiel;
6. That the possession of the applicant Acme Plywood & Veneer Co., Inc., is
continuous, adverse and public from 1962 to the present and tacking the possession of
the Infiels who were granted from whom the applicant bought said land on October 29, The Petition for Review does not dispute-indeed, in view of the quoted findings of the
1962, hence the possession is already considered from time immemorial. trial court which were cited and affirmed by the Intermediate Appellate Court, it can no
7. That the land sought to be registered is a private land pursuant to the longer controvert before this Court-the fact that Mariano and Acer Infiel, from whom
provisions of Republic Act No. 3872 granting absolute ownership to members of the Acme purchased the lands in question on October 29, 1962, are members of the
non-Christian Tribes on land occupied by them or their ancestral lands, whether with the national cultural minorities who had, by themselves and through their progenitors,
alienable or disposable public land or within the public domain; possessed and occupied those lands since time immemorial, or for more than the
8. That applicant Acme Plywood & Veneer Co. Inc., has introduced more than required 30-year period and were, by reason thereof, entitled to exercise the right
Forty-Five Million (P45,000,000.00) Pesos worth of improvements, said improvements granted in Section 48 of the Public Land Act to have their title judicially confirmed. Nor is
were seen by the Court during its ocular investigation of the land sought to be registered there any pretension that Acme, as the successor-in-interest of the Infiels, is disqualified
on September 18, 1982; to acquire and register ownership of said lands under any provisions of the 1973
9. That the ownership and possession of the land sought to be registered by the Constitution other than Section 11 of its Article XIV already referred to.
applicant was duly recognized by the government when the Municipal Officials of Given the foregoing, the question before this Court is whether or not the title that the
Maconacon, Isabela, have negotiated for the donation of the townsite from Acme Infiels had transferred to Acme in 1962 could be confirmed in favor of the latter in
Plywood & Veneer Co., Inc., and this negotiation came to reality when the Board of proceedings instituted by it in 1981 when the 1973 Constitution was already in effect,
Directors of the Acme Plywood & Veneer Co., Inc., had donated a part of the land having in mind the prohibition therein against private corporations holding lands of the
bought by the Company from the Infiels for the townsite of Maconacon Isabela (Exh. 'N') public domain except in lease not exceeding 1,000 hectares.
on November 15, 1979, and which donation was accepted by the Municipal The question turns upon a determination of the character of the lands at the time of
Government of Maconacon, Isabela (Exh. 'N-l'), during their special session on institution of the registration proceedings in 1981. If they were then still part of the public
November 22, 1979. domain, it must be answered in the negative. If, on the other hand, they were then
The Director of Lands takes no issue with any of these findings except as to the already private lands, the constitutional prohibition against their acquisition by private
applicability of the 1935 Constitution to the matter at hand. Concerning this, he asserts corporations or associations obviously does not apply.
that, the registration proceedings have been commenced only on July 17, 1981, or long In this regard, attention has been invited to Manila Electric Company vs. Castro-
after the 1973 Constitution had gone into effect, the latter is the correctly applicable law; Bartolome, et al, 1 where a similar set of facts prevailed. In that case, Manila Electric
and since section 11 of its Article XIV prohibits private corporations or associations from Company, a domestic corporation more than 60% of the capital stock of which is
holding alienable lands of the public domain, except by lease not to exceed 1,000 Filipino-owned, had purchased in 1947 two lots in Tanay, Rizal from the Piguing
hectares (a prohibition not found in the 1935 Constitution which was in force in 1962 spouses. The lots had been possessed by the vendors and, before them, by their
when Acme purchased the lands in question from the Infiels), it was reversible error to predecessor-in-interest, Olimpia Ramos, since prior to the outbreak of the Pacific War in
decree registration in favor of Acme Section 48, paragraphs (b) and (c), of 1941. On December 1, 1976, Meralco applied to the Court of First Instance of Rizal,
Commonwealth Act No. 141, as amended, reads: Makati Branch, for confirmation of title to said lots. The court, assuming that the lots
SEC. 48. The following described citizens of the Philippines, occupying lands of were public land, dismissed the application on the ground that Meralco, a juridical
the public domain or claiming to own any such lands or an interest therein, but whose person, was not qualified to apply for registration under Section 48(b) of the Public Land
titles have not been perfected or completed, may apply to the Court of First Instance of Act which allows only Filipino citizens or natural persons to apply for judicial
the province where the land is located for confirmation of their claims, and the issuance confirmation of imperfect titles to public land. Meralco appealed, and a majority of this
of a certificate of title therefor, under the Land Registration Act, to wit: Court upheld the dismissal. It was held that:
xxx xxx xxx ..., the said land is still public land. It would cease to be public land only upon the
(b) Those who by themselves or through their predecessors-in-interest have been issuance of the certificate of title to any Filipino citizen claiming it under section 48(b).
in open, continuous, exclusive and notorious possession and occupation of agricultural Because it is still public land and the Meralco, as a juridical person, is disqualified to
lands of the public domain, under a bona fide claim of acquisition or ownership, for at apply for its registration under section 48(b), Meralco's application cannot be given due
least thirty years immediately preceding the filing of the application for confirmation of course or has to be dismissed.
title except when prevented by war or force majeure. These shall be conclusively Finally, it may be observed that the constitutional prohibition makes no distinction
presumed to have performed all the conditions essential to a Government grant and between (on the one hand) alienable agricultural public lands as to which no occupant
shall be entitled to a certificate of title under the provisions of this chapter. has an imperfect title and (on the other hand) alienable lands of the public domain as to
(c) Members of the National Cultural minorities who by themselves or through which an occupant has on imperfect title subject to judicial confirmation.
their predecessors-in-interest have been in open. continuous, exclusive and notorious Since section 11 of Article XIV does not distinguish, we should not make any distinction
possession and occupation of lands of the public domain suitable to agriculture, whether or qualification. The prohibition applies to alienable public lands as to which a Torrens
disposable or not, under a bona fide claim of ownership for at least 30 years shall be title may be secured under section 48(b). The proceeding under section 48(b)
entitled to the rights granted in subsection (b) hereof.
'presupposes that the land is public' (Mindanao vs. Director of Lands, L-19535, July 30, .... Secondly, under the provisions of Republic Act No. 1942, which the respondent
1967, 20 SCRA 641, 644). Court held to be inapplicable to the petitioner's case, with the latter's proven occupation
The present Chief Justice entered a vigorous dissent, tracing the line of cases and cultivation for more than 30 years since 1914, by himself and by his predecessors-
beginning with Carino in 1909 2 thru Susi in 1925 3 down to Herico in 1980, 4 which in-interest, title over the land has vested on petitioner so as to segregate the land from
developed, affirmed and reaffirmed the doctrine that open, exclusive and undisputed the mass of public land. Thereafter, it is no longer disposable under the Public Land Act
possession of alienable public land for the period prescribed by law creates the legal as by free patent. ....
fiction whereby the land, upon completion of the requisite period ipso jure and without xxx xxx xxx
the need of judicial or other sanction, ceases to be public land and becomes private As interpreted in several cases, when the conditions as specified in the foregoing
property. That said dissent expressed what is the better and, indeed, the correct, provision are complied with, the possessor is deemed to have acquired, by operation of
view-becomes evident from a consideration of some of the principal rulings cited law, a right to a grant, a government grant, without the necessity of a certificate of title
therein, being issued. The land, therefore, ceases to be of the public domain and beyond the
The main theme was given birth, so to speak, in Carino involving the authority of the Director of Lands to dispose of. The application for confirmation is mere
Decree/Regulations of June 25, 1880 for adjustment of royal lands wrongfully occupied formality, the lack of which does not affect the legal sufficiency of the title as would be
by private individuals in the Philippine Islands. It was ruled that: evidenced by the patent and the Torrens title to be issued upon the strength of said
It is true that the language of articles 4 and 5 5 attributes title to those 'who may prove' patent. 12
possession for the necessary time and we do not overlook the argument that this means Nothing can more clearly demonstrate the logical inevitability of considering possession
may prove in registration proceedings. It may be that an English conveyancer would of public land which is of the character and duration prescribed by statute as the
have recommended an application under the foregoing decree, but certainly it was not equivalent of an express grant from the State than the dictum of the statute itself 13 that
calculated to convey to the mind of an Igorot chief the notion that ancient family the possessor(s) "... shall be conclusively presumed to have performed all the
possessions were in danger, if he had read every word of it. The words 'may prove' conditions essential to a Government grant and shall be entitled to a certificate of title
(acrediten) as well or better, in view of the other provisions, might be taken to mean .... " No proof being admissible to overcome a conclusive presumption, confirmation
when called upon to do so in any litigation. There are indications that registration was proceedings would, in truth be little more than a formality, at the most limited to
expected from all but none sufficient to show that, for want of it, ownership actually ascertaining whether the possession claimed is of the required character and length of
gained would be lost. The effect of the proof, wherever made, was not to confer title, but time; and registration thereunder would not confer title, but simply recognize a title
simply to establish it, as already conferred by the decree, if not by earlier law. ... already vested. The proceedings would not originally convert the land from public to
That ruling assumed a more doctrinal character because expressed in more categorical private land, but only confirm such a conversion already affected by operation of law
language, in Susi: from the moment the required period of possession became complete. As was so well
.... In favor of Valentin Susi, there is, moreover, the presumption juris et de jure put in Carino, "... (T)here are indications that registration was expected from all, but
established in paragraph (b) of section 45 of Act No. 2874, amending Act No. 926, that none sufficient to show that, for want of it, ownership actually gained would be lost. The
all the necessary requirements for a grant by the Government were complied with, for effect of the proof, wherever made, was not to confer title, but simply to establish it, as
he has been in actual and physical possession, personally and through his already conferred by the decree, if not by earlier law."
predecessors, of an agricultural land of the public domain openly, continuously, If it is accepted-as it must be-that the land was already private land to which the Infiels
exclusively and publicly since July 26, 1984, with a right to a certificate of title to said had a legally sufficient and transferable title on October 29, 1962 when Acme acquired it
land under the provisions of Chapter VIII of said Act. So that when Angela Razon from said owners, it must also be conceded that Acme had a perfect right to make such
applied for the grant in her favor, Valentin Susi had already acquired, by operation of acquisition, there being nothing in the 1935 Constitution then in force (or, for that matter,
law not only a right to a grant, but a grant of the Government, for it is not necessary that in the 1973 Constitution which came into effect later) prohibiting corporations from
a certificate of title should be issued in order that said grant may be sanctioned by the acquiring and owning private lands.
courts, an application therefore is sufficient, under the provisions of section 47 of Act Even on the proposition that the land remained technically "public" land, despite
No. 2874. If by a legal fiction, Valentin Susi had acquired the land in question by a grant immemorial possession of the Infiels and their ancestors, until title in their favor was
of the State, it had already ceased to be of the public domain and had become private actually confirmed in appropriate proceedings under the Public Land Act, there can be
property, at least by presumption, of Valentin Susi, beyond the control of the Director of no serious question of Acmes right to acquire the land at the time it did, there also being
Lands. Consequently, in selling the land in question of Angela Razon, the Director of nothing in the 1935 Constitution that might be construed to prohibit corporations from
Lands disposed of a land over which he had no longer any title or control, and the sale purchasing or acquiring interests in public land to which the vendor had already
thus made was void and of no effect, and Angela Razon did not thereby acquire any acquired that type of so-called "incomplete" or "imperfect" title. The only limitation then
right. 6 extant was that corporations could not acquire, hold or lease public agricultural lands in
Herico, in particular, appears to be squarely affirmative: 11 excess of 1,024 hectares. The purely accidental circumstance that confirmation
proceedings were brought under the aegis of the 1973 Constitution which forbids
corporations from owning lands of the public domain cannot defeat a right already 6. To uphold respondent judge's denial of Meralco's application on the
vested before that law came into effect, or invalidate transactions then perfectly valid technicality that the Public Land Act allows only citizens of the Philippines who are
and proper. This Court has already held, in analogous circumstances, that the natural persons to apply for confirmation of their title would be impractical and would
Constitution cannot impair vested rights. just give rise to multiplicity of court actions. Assuming that there was a technical error
We hold that the said constitutional prohibition 14 has no retroactive application to the not having filed the application for registration in the name of the Piguing spouses as
sales application of Binan Development Co., Inc. because it had already acquired a the original owners and vendors, still it is conceded that there is no prohibition against
vested right to the land applied for at the time the 1973 Constitution took effect. their sale of the land to the applicant Meralco and neither is there any prohibition
That vested right has to be respected. It could not be abrogated by the new against the application being refiled with retroactive effect in the name of the original
Constitution. Section 2, Article XIII of the 1935 Constitution allows private corporations owners and vendors (as such natural persons) with the end result of their application
to purchase public agricultural lands not exceeding one thousand and twenty-four being granted, because of their indisputable acquisition of ownership by operation of
hectares. Petitioner' prohibition action is barred by the doctrine of vested rights in law and the conclusive presumption therein provided in their favor. It should not be
constitutional law. necessary to go through all the rituals at the great cost of refiling of all such applications
xxx xxx xxx in their names and adding to the overcrowded court dockets when the Court can after
The due process clause prohibits the annihilation of vested rights. 'A state may not all these years dispose of it here and now. (See Francisco vs. City of Davao)
impair vested rights by legislative enactment, by the enactment or by the subsequent The ends of justice would best be served, therefore, by considering the applications for
repeal of a municipal ordinance, or by a change in the constitution of the State, except confirmation as amended to conform to the evidence, i.e. as filed in the names of the
in a legitimate exercise of the police power'(16 C.J.S. 1177-78). original persons who as natural persons are duly qualified to apply for formal
xxx xxx xxx confirmation of the title that they had acquired by conclusive presumption and mandate
In the instant case, it is incontestable that prior to the effectivity of the 1973 Constitution of the Public Land Act and who thereafter duly sold to the herein corporations (both
the right of the corporation to purchase the land in question had become fixed and admittedly Filipino corporations duly qualified to hold and own private lands) and
established and was no longer open to doubt or controversy. granting the applications for confirmation of title to the private lands so acquired and
Its compliance with the requirements of the Public Land Law for the issuance of a patent sold or exchanged.
had the effect of segregating the said land from the public domain. The corporation's There is also nothing to prevent Acme from reconveying the lands to the Infiels and the
right to obtain a patent for the land is protected by law. It cannot be deprived of that right latter from themselves applying for confirmation of title and, after issuance of the
without due process (Director of Lands vs. CA, 123 Phil. 919).<re||an1w> 15 certificate/s of title in their names, deeding the lands back to Acme. But this would be
The fact, therefore, that the confirmation proceedings were instituted by Acme in its own merely indulging in empty charades, whereas the same result is more efficaciously and
name must be regarded as simply another accidental circumstance, productive of a speedily obtained, with no prejudice to anyone, by a liberal application of the rule on
defect hardly more than procedural and in nowise affecting the substance and merits of amendment to conform to the evidence suggested in the dissent in Meralco.
the right of ownership sought to be confirmed in said proceedings, there being no doubt While this opinion seemingly reverses an earlier ruling of comparatively recent vintage,
of Acme's entitlement to the land. As it is unquestionable that in the light of the in a real sense, it breaks no precedent, but only reaffirms and re-established, as it were,
undisputed facts, the Infiels, under either the 1935 or the 1973 Constitution, could have doctrines the soundness of which has passed the test of searching examination and
had title in themselves confirmed and registered, only a rigid subservience to the letter inquiry in many past cases. Indeed, it is worth noting that the majority opinion, as well
of the law would deny the same benefit to their lawful successor-in-interest by valid as the concurring opinions of Chief Justice Fernando and Justice Abad Santos, in
conveyance which violates no constitutional mandate. Meralco rested chiefly on the proposition that the petitioner therein, a juridical person,
The Court, in the light of the foregoing, is of the view, and so holds, that the majority was disqualified from applying for confirmation of an imperfect title to public land under
ruling in Meralco must be reconsidered and no longer deemed to be binding precedent. Section 48(b) of the Public Land Act. Reference to the 1973 Constitution and its Article
The correct rule, as enunciated in the line of cases already referred to, is that alienable XIV, Section 11, was only tangential limited to a brief paragraph in the main opinion,
public land held by a possessor, personally or through his predecessors-in-interest, and may, in that context, be considered as essentially obiter. Meralco, in short, decided
openly, continuously and exclusively for the prescribed statutory period (30 years under no constitutional question.
The Public Land Act, as amended) is converted to private property by the mere lapse or WHEREFORE, there being no reversible error in the appealed judgment of the
completion of said period, ipso jure. Following that rule and on the basis of the Intermediate Appellate Court, the same is hereby affirmed, without costs in this
undisputed facts, the land subject of this appeal was already private property at the time instance.
it was acquired from the Infiels by Acme. Acme thereby acquired a registrable title, there SO ORDERED.
being at the time no prohibition against said corporation's holding or owning private Feria, Yap, Fernan, Alampay, Cruz, Paras and Feliciano, JJ., concur.
land. The objection that, as a juridical person, Acme is not qualified to apply for judicial
confirmation of title under section 48(b) of the Public Land Act is technical, rather than
substantial and, again, finds its answer in the dissent in Meralco: Separate Opinions
GUTIERREZ, JR., J., concurring: unlike the later 1973 Constitution which imposed an absolute prohibition. Even on the
I reiterate my concurrence in Meralco v. Castro-Bartolome, and, therefore, dissent here. erroneous assumption that the land remained public land despite the Infiels' open
possession thereof as owners from time immemorial, respondent corporation's lawful
TEEHANKEE, C.J., concurring: purchase from them of the land in 1962 and P 45million investments redounding
Under the express text and mandate of the cited Act, such possessors "shall be presumably to the welfare and progress of the community, particularly the municipality
conclusively presumed to have performed all the conditions essential to a Government of Maconacon, Isabela to which it donated part of the land for the townsite created a
grant and shall be entitled to a certificate of title under the provisions of this chapter. " vested right which could not be impaired by the prohibition adopted eleven years later.
The Court thus held in Susi that under the presumption juris et de jure established in the But as sufficiently stressed, the land of the Infiels had been ipso jure converted into
Act, the rightful possessor of the public land for the statutory period "already acquired, private land and they had a legally sufficient and transferable title conferred by the
by operation of law, not only a right to a grant, but a grant of the Government, for it is conclusive presumption of the Public Land Act (which needed only to be established in
not necessary that certificate of title should be issued an order that said grant may be confirmation of title proceedings for formalization and issuance of the certificate of title)
sanctioned by the courts, an application therefore is sufficient . . . . If by a legal fiction, which they lawfully and validly transferred to respondent corporation.
Valentin Susi had acquiredthe land in question by a grant of the State, it had already In fact, the many amendments to the Act extending the period for the filing of such
ceased to be of the public domain and had become private property, at least by applications for judicial confirmation of imperfect and incomplete titles to alienable and
presumption, of Valentin Susi, beyond the control of the Director of Lands [and beyond disposable public lands expressly reiterate that it has always been the "policy of the
his authority to sell to any other person]. " 6 State to hasten the settlement, adjudication and quieting of titles to [such] unregistered
The root of the doctrine goes back to the pronouncement of Justice Oliver Wendell lands," i.e. to recognize that such lands publicly and notoriously occupied and cultivated
Holmes for the U.S. Supreme Court in the 1909 case of Carino (the Igorot chief who under bona fide claim of acquisition or ownership have ipso jure been converted into
would have been deprived of ancestral family lands by the dismissal of his application private property and grant the possessors the opportunity to establish and record such
for registration) which reversed the dismissal of the registration court (as affirmed by the fact. Thus, the deadline for the filing of such application which would have originally
Supreme Court) and adopted the liberal view that under the decree and regulations of expired first on December 31, 1938 was successively extended to December 31, 1941,
June 25, 1880, "The words 'may prove' (acrediten), as well, or better, in view of the then extended to December 31, 1957, then to December 31, 1968, further extended to
other provisions, might be taken to mean when called upon to do so in any litigation. December 31, 1976 and lastly extended to December 31, 1987. 7
There are indications that registration was expected from all, but none sufficient to show The cited Act's provision that only natural persons may apply thereunder for
that, for want of it, ownership actually gained would be lost. The effect of the proof, confirmation of title is in effect a technicality of procedure and not of substance. My
whenever made, was not to confer title, but simply to establish it, as already conferred submittal in Meralco, mutatis mutandis, is properly applicable: "The ends of justice
by the decree, if not by earlier law." would best be served, therefore, by considering the applications for confirmation as
The Court's decision at bar now expressly overturns the Meralco and related cases amended to conform to the evidence, i.e. as filed in the names of the original persons
subsequent thereto which failed to adhere to the aforecited established doctrine dating who as natural persons are duly qualified to apply for formal confirmation of the title that
back to 1909 and was consistently applied up to June 29, 1982 (when the Meralco they had acquired by conclusive presumption and mandate of the Public Land Act and
decision was promulgated). We reaffirm the established doctrine that such acquisitive who thereafter duly sold to the herein corporations (both admittedly Filipino corporations
prescription of alienable public lands takes place ipso jure or by operation of law without duly qualified to hold and own private lands) and granting the applications for
the necessity of a prior issuance of a certificate of title. The land ipso jure ceases to be confirmation of title to the private lands so acquired and sold or exchanged." 8 Indeed,
of the public domain and becomes private property, which may be lawfully sold to and then Chief Justice Enrique M. Fernando likewise dissented along the same line from the
acquired by qualified corporations such as respondent corporation. (As stressed in majority ruling therein and held: "I dissent insofar as the opinion of the Court would
Herico supra, "the application for confirmation is a mere formality, the lack of which characterize such jurisdictional defect that the applicant was Meralco, a juridical person
does not affect the legal sufficiency of the title.") rather than the natural persons-transferors, under the particular circumstances of this
Such ipso jure conversion into private property of public lands publicly held under a case, as an insurmountable obstacle to the relief sought. I would apply by analogy,
bona fide claim of acquisition or ownership is the public policy of the Act and is so although the facts could be distinguished, the approach followed by us in Francisco v.
expressly stated therein. By virtue of such conversion into private property, qualified City of Davao, where the legal question raised, instead of being deferred and possibly
corporations may lawfully acquire them and there is no "alteration or defeating" of the taken up in another case, was resolved. By legal fiction and in the exercise of our
1973 Constitution's prohibition against corporations holding or acquiring title to lands of equitable jurisdiction, I feel that the realistic solution would be to decide the matter as if
the public domain, as claimed in the dissenting opinion, for the simple reason that no the application under Section 48(b) were filed by the Piguing spouses, who I assume
public lands are involved. suffer from no such disability." 9 Justice Vicente Abad Santos, now retired, while
It should be noted that respondent corporation purchased the land from the Infiels on concurring in the procedural result, likewise, in effect dissented from the therein majority
October 16, 1962 under the aegis of the 1935 Constitution which contained no ruling on the question of substance, and stated his opinion that "the lots which are
prohibition against corporations holding public lands (except a limit of 1,024 hectares) sought to be registered have ceased to be lands of the public domain at the time they
were acquired by the petitioner corporation. They are already private lands because of SEC. 11. .... No private corporation or association may hold alienable lands of the public
acquisitive prescription by the predecessors of the petitioner and all that is needed is domain except by lease not to exceed one thousand hectares in area; nor may any
the confirmation of the title. Accordingly, the constitutional provision that no private citizen hold such lands by lease in excess of five hundred hectares ....
corporation or association may hold alienable lands of the public domain is inapplicable. It has to be conceded that, literally, statutory law and constitutional provision prevent a
" 10 corporation from directly applying to the Courts for the issuance of Original Certificates
To my mind, the reason why the Act limits the filing of such applications to natural of Title to lands of the public domain (Manila Electric Company vs. Castro-Bartolome,
citizens who may prove their undisputed and open possession of public lands for the 114 SCRA 799; Republic vs. Villanueva, 114 SCRA 875; Republic vs. Court of Appeals,
required statutory thirty-year period, tacking on their predecessors'-in-interest 119 SCRA 449; Iglesia ni Cristo vs. Hon. Judge, CFI of Nueva Ecija, Br. 1). It is my
possession is that only natural persons, to the exclusion of juridical persons such as opinion that the literalism should be adhered to in this case.
corporations, can actually, physically and in reality possess public lands for the required The reasoning of the majority can be restated in simple terms as follows:
statutory 30-year period. That juridical persons or corporations cannot do so is obvious. (a) The INFIELS can successfully file an application for a certificate of title over
But when the natural persons have fulfilled the required statutory period of possession, the land involved in the case.
the Act confers on them a legally sufficient and transferable title. It is preferable to follow (b) After the INFIELS secure a certificate of title, they can sell the land to ACME.
the letter of the law that they file the applications for confirmation of their title, although (c) As ACME can eventually own the certificate of title, it should be allowed to
they have lawfully transferred their title to the land. But such procedural failure cannot directly apply to the Courts for the Certificate of Title, thus avoiding the circuituous
and should not defeat the substance of the law, as stressed in the above-cited opinions, "literal" requirement that the INFIELS should first apply to the courts for the titles, and
that the lands are already private lands because of acquisitive prescription by the afterwards transfer the title to ACME.
corporation's predecessors and the realistic solution would be to consider the The majority opinion, in effect, adopted the following excerpt from a dissent in Manila
application for confirmation as filed by the natural persons-transferors, and in Electric Company vs. Castro-Bartolome (114 SCRA 799, 823 [1982]).
accordance with the evidence, confirm their title to the private lands so converted by To uphold respondent judge's denial of Meralco's application on the technicality that the
operation of law and lawfully transferred by them to the corporation. The law, after all, Public Land Act allows only citizens of the Philippines who are natural persons to apply
recognizes the validity of the transfer and sale of the private land to the corporation. It for confirmation of their title would be impractical and would just give rise to multiplicity
should not be necessary to go in a round-about way and have the corporation reassign of court actions. Assuming that there was a technical error in not having filed the
its rights to the private land to natural persons-(as I understand), was done after the application for registration in the name of the Piguing spouses as the original owners
decision in the Meralco and Iglesia ni Cristo cases) just for the purpose of complying on and vendors,
paper with the technicality of having natural persons file the application for confirmation still it is conceded that there is no prohibition against their sale of the land to the
of title to the private land. applicant Meralco
and neither is there any prohibition against the application being refiled with retroactive
MELENCIO-HERRERA, J., dissenting: effect in the name of the original owners and vendors (as such natural persons) with the
Section 48 of the Public Land Act, in part, provides: end result of their application being granted, because of their indisputable acquisition of
SEC. 48. The following described citizens of the Philippines, occupying lands of ownership by operation of law and the conclusive presumption therein provided in their
the public domain or claiming to own any such lands or an interest therein, but whose favor.
titles have not been perfected or completed, may apply to the Court of First Instance of It should not be necessary to go through all the rituals at the great cost of refiling of all
the province where the land is located for confirmation of their claims and the issuance such applications in their names and adding to the overcrowded court dockets when the
of a certificate of title therefor, under the Land Registration Act, to wit: Court can after all these years dispose of it here and now." (Paragraphing supplied)
(a) ... The effect is that the majority opinion now nullifies the statutory provision that only
(b) Those who by themselves or through their predecessors in interest have been citizens (natural persons) can apply for certificates of title under Section 48(b) of the
in open, continuous, exclusive, and notorious possession and occupation of agricultural Public Land Act, as well as the constitutional provision (Article XIV, Section 11) which
lands of the public domain, under a bona fide claim of acquisition of ownership, for at prohibits corporations from acquiring title to lands of the public domain. That
least thirty years immediately preceding the filing of the application for confirmation of interpretation or construction adopted by the majority cannot be justified. "A construction
title except when prevented by war or force majeure. These shall be conclusively adopted should not be such as to nullify, destroy or defeat the intention of the
presumed to have performed are the conditions essential to a Government grant and legislature" (New York State Dept. of Social Services v. Dublino [UST 37 L. Ed 2d 688,
shall be entitled to a certificate of title under the provisions of this chapter. 93 S Ct 2507; United States v. Alpers 338 US 680, 94 L Ed 457, 70 S Ct 352; cited in
(c) ... 73 Am Jur. 2nd., p. 351).
Article XIV, Section 11, of the 1973 Constitution, in part, provides: It has also been said that:
In the construction of statutes, the courts start with the assumption that the legislature
intended to enact an effective law, and the legislature is not to be presumed to have
done a vain thing in the enactment of a statute. Hence, it is a general principle that the There are indications that registration was expected from all, but none sufficient to show
courts should, if reasonably possible to do so interpret the statute, or the provision being that, for want of it, ownership actually gained would be lost. The effect of the proof,
construed, so as to give it efficient operation and effect as a whole. An interpretation whenever made, was not to confer title, but simply to establish it, as already conferred
should, if possible, be avoided, under which the statute or provision being construed is by the decree, if not by earlier law."
defeated, or as otherwise expressed, nullified, destroyed, emasculated, repealed, The Court's decision at bar now expressly overturns the Meralco and related cases
explained away, or rendered insignificant, meaningless, inoperative, or nugatory. If a subsequent thereto which failed to adhere to the aforecited established doctrine dating
statute is fairly susceptible of two constructions, one of which will give effect to the act, back to 1909 and was consistently applied up to June 29, 1982 (when the Meralco
while the other will defeat it, the former construction is preferred. One part of a statute decision was promulgated).<re||an1w> We reaffirm the established doctrine that
may not be construed so as to render another part nugatory or of no effect. Moreover, such acquisitive prescription of alienable public lands takes place ipso jure or by
notwithstanding the general rule against the enlargement of extension of a statute by operation of law without the necessity of a prior issuance of a certificate of title. The land
construction, the meaning of a statute may be extended beyond the precise words used ipso jure ceases to be of the public domain and becomes private property, which may
in the law, and words or phrases may be altered or supplied, where this is necessary to be lawfully sold to and acquired by qualified corporations such as respondent
prevent a law from becoming a nullity. Wherever the provision of a statute is general corporation. (As stressed in Herico supra, "the application for confirmation is a mere
everything which is necessary to make such provision effectual is supplied by formality, the lack of which does not affect the legal sufficiency of the title.")
implication. (Pliakos vs. Illinois Liquor Control Com. 11 III 2d 456, 143 NE2d 47; cited in Such ipso jure conversion into private property of public lands publicly held under a
73 AM Jur. 2d pp. 422-423) bona fide claim of acquisition or ownership is the public policy of the Act and is so
The statutory provision and the constitutional prohibition express a public policy. The expressly stated therein. By virtue of such conversion into private property, qualified
proper course for the Court to take is to promote in the fullest manner the policy thus corporations may lawfully acquire them and there is no "alteration or defeating" of the
laid down and to avoid a construction which would alter or defeat that policy. 1973 Constitution's prohibition against corporations holding or acquiring title to lands of
In fine, I confirm my adherence to the ruling of this Court in Meralco vs. Hon. Castro- the public domain, as claimed in the dissenting opinion, for the simple reason that no
Bartolome, 114 SCRA 799 [1982] and related cases. public lands are involved.
It should be noted that respondent corporation purchased the land from the Infiels on
October 16, 1962 under the aegis of the 1935 Constitution which contained no
Separate Opinions prohibition against corporations holding public lands (except a limit of 1,024 hectares)
GUTIERREZ, JR., J., concurring: unlike the later 1973 Constitution which imposed an absolute prohibition. Even on the
I reiterate my concurrence in Meralco v. Castro-Bartolome, and, therefore, dissent here. erroneous assumption that the land remained public land despite the Infiels' open
possession thereof as owners from time immemorial, respondent corporation's lawful
TEEHANKEE, C.J., concurring: purchase from them of the land in 1962 and P 45million investments redounding
Under the express text and mandate of the cited Act, such possessors "shall be presumably to the welfare and progress of the community, particularly the municipality
conclusively presumed to have performed all the conditions essential to a Government of Maconacon, Isabela to which it donated part of the land for the townsite created a
grant and shall be entitled to a certificate of title under the provisions of this chapter. " vested right which could not be impaired by the prohibition adopted eleven years later.
The Court thus held in Susi that under the presumption juris et de jure established in the But as sufficiently stressed, the land of the Infiels had been ipso jure converted into
Act, the rightful possessor of the public land for the statutory period "already acquired, private land and they had a legally sufficient and transferable title conferred by the
by operation of law, not only a right to a grant, but a grant of the Government, for it is conclusive presumption of the Public Land Act (which needed only to be established in
not necessary that certificate of title should be issued an order that said grant may be confirmation of title proceedings for formalization and issuance of the certificate of title)
sanctioned by the courts, an application therefore is sufficient . . . . If by a legal fiction, which they lawfully and validly transferred to respondent corporation.
Valentin Susi had acquiredthe land in question by a grant of the State, it had already In fact, the many amendments to the Act extending the period for the filing of such
ceased to be of the public domain and had become private property, at least by applications for judicial confirmation of imperfect and incomplete titles to alienable and
presumption, of Valentin Susi, beyond the control of the Director of Lands [and beyond disposable public lands expressly reiterate that it has always been the "policy of the
his authority to sell to any other person]. " 6 State to hasten the settlement, adjudication and quieting of titles to [such] unregistered
The root of the doctrine goes back to the pronouncement of Justice Oliver Wendell lands," i.e. to recognize that such lands publicly and notoriously occupied and cultivated
Holmes for the U.S. Supreme Court in the 1909 case of Carino (the Igorot chief who under bona fide claim of acquisition or ownership have ipso jure been converted into
would have been deprived of ancestral family lands by the dismissal of his application private property and grant the possessors the opportunity to establish and record such
for registration) which reversed the dismissal of the registration court (as affirmed by the fact. Thus, the deadline for the filing of such application which would have originally
Supreme Court) and adopted the liberal view that under the decree and regulations of expired first on December 31, 1938 was successively extended to December 31, 1941,
June 25, 1880, "The words 'may prove' (acrediten), as well, or better, in view of the then extended to December 31, 1957, then to December 31, 1968, further extended to
other provisions, might be taken to mean when called upon to do so in any litigation. December 31, 1976 and lastly extended to December 31, 1987. 7
The cited Act's provision that only natural persons may apply thereunder for its rights to the private land to natural persons-(as I understand), was done after the
confirmation of title is in effect a technicality of procedure and not of substance. My decision in the Meralco and Iglesia ni Cristo cases) just for the purpose of complying on
submittal in Meralco, mutatis mutandis, is properly applicable: "The ends of justice paper with the technicality of having natural persons file the application for confirmation
would best be served, therefore, by considering the applications for confirmation as of title to the private land.
amended to conform to the evidence, i.e. as filed in the names of the original persons
who as natural persons are duly qualified to apply for formal confirmation of the title that MELENCIO-HERRERA, J., dissenting:
they had acquired by conclusive presumption and mandate of the Public Land Act and Section 48 of the Public Land Act, in part, provides:
who thereafter duly sold to the herein corporations (both admittedly Filipino corporations SEC. 48. The following described citizens of the Philippines, occupying lands of
duly qualified to hold and own private lands) and granting the applications for the public domain or claiming to own any such lands or an interest therein, but whose
confirmation of title to the private lands so acquired and sold or exchanged." 8 Indeed, titles have not been perfected or completed, may apply to the Court of First Instance of
then Chief Justice Enrique M. Fernando likewise dissented along the same line from the the province where the land is located for confirmation of their claims and the issuance
majority ruling therein and held: "I dissent insofar as the opinion of the Court would of a certificate of title therefor, under the Land Registration Act, to wit:
characterize such jurisdictional defect that the applicant was Meralco, a juridical person (a) ...
rather than the natural persons-transferors, under the particular circumstances of this (b) Those who by themselves or through their predecessors in interest have been
case, as an insurmountable obstacle to the relief sought. I would apply by analogy, in open, continuous, exclusive, and notorious possession and occupation of agricultural
although the facts could be distinguished, the approach followed by us in Francisco v. lands of the public domain, under a bona fide claim of acquisition of ownership, for at
City of Davao, where the legal question raised, instead of being deferred and possibly least thirty years immediately preceding the filing of the application for confirmation of
taken up in another case, was resolved. By legal fiction and in the exercise of our title except when prevented by war or force majeure. These shall be conclusively
equitable jurisdiction, I feel that the realistic solution would be to decide the matter as if presumed to have performed are the conditions essential to a Government grant and
the application under Section 48(b) were filed by the Piguing spouses, who I assume shall be entitled to a certificate of title under the provisions of this chapter.
suffer from no such disability." 9 Justice Vicente Abad Santos, now retired, while (c) ...
concurring in the procedural result, likewise, in effect dissented from the therein majority Article XIV, Section 11, of the 1973 Constitution, in part, provides:
ruling on the question of substance, and stated his opinion that "the lots which are SEC. 11. .... No private corporation or association may hold alienable lands of the public
sought to be registered have ceased to be lands of the public domain at the time they domain except by lease not to exceed one thousand hectares in area; nor may any
were acquired by the petitioner corporation. They are already private lands because of citizen hold such lands by lease in excess of five hundred hectares ....
acquisitive prescription by the predecessors of the petitioner and all that is needed is It has to be conceded that, literally, statutory law and constitutional provision prevent a
the confirmation of the title. Accordingly, the constitutional provision that no private corporation from directly applying to the Courts for the issuance of Original Certificates
corporation or association may hold alienable lands of the public domain is inapplicable. of Title to lands of the public domain (Manila Electric Company vs. Castro-Bartolome,
" 10 114 SCRA 799; Republic vs. Villanueva, 114 SCRA 875; Republic vs. Court of Appeals,
To my mind, the reason why the Act limits the filing of such applications to natural 119 SCRA 449; Iglesia ni Cristo vs. Hon. Judge, CFI of Nueva Ecija, Br. 1). It is my
citizens who may prove their undisputed and open possession of public lands for the opinion that the literalism should be adhered to in this case.
required statutory thirty-year period, tacking on their predecessors'-in-interest The reasoning of the majority can be restated in simple terms as follows:
possession is that only natural persons, to the exclusion of juridical persons such as (a) The INFIELS can successfully file an application for a certificate of title over
corporations, can actually, physically and in reality possess public lands for the required the land involved in the case.
statutory 30-year period. That juridical persons or corporations cannot do so is obvious. (b) After the INFIELS secure a certificate of title, they can sell the land to ACME.
But when the natural persons have fulfilled the required statutory period of possession, (c) As ACME can eventually own the certificate of title, it should be allowed to
the Act confers on them a legally sufficient and transferable title. It is preferable to follow directly apply to the Courts for the Certificate of Title, thus avoiding the circuituous
the letter of the law that they file the applications for confirmation of their title, although "literal" requirement that the INFIELS should first apply to the courts for the titles, and
they have lawfully transferred their title to the land. But such procedural failure cannot afterwards transfer the title to ACME.
and should not defeat the substance of the law, as stressed in the above-cited opinions, The majority opinion, in effect, adopted the following excerpt from a dissent in Manila
that the lands are already private lands because of acquisitive prescription by the Electric Company vs. Castro-Bartolome (114 SCRA 799, 823 [1982]).
corporation's predecessors and the realistic solution would be to consider the To uphold respondent judge's denial of Meralco's application on the technicality that the
application for confirmation as filed by the natural persons-transferors, and in Public Land Act allows only citizens of the Philippines who are natural persons to apply
accordance with the evidence, confirm their title to the private lands so converted by for confirmation of their title would be impractical and would just give rise to multiplicity
operation of law and lawfully transferred by them to the corporation. The law, after all, of court actions. Assuming that there was a technical error in not having filed the
recognizes the validity of the transfer and sale of the private land to the corporation. It application for registration in the name of the Piguing spouses as the original owners
should not be necessary to go in a round-about way and have the corporation reassign and vendors,
still it is conceded that there is no prohibition against their sale of the land to the therein to the latest 1980 case of Herico 4 that "it is established doctrine....... that an
applicant Meralco open, continuous, adverse and public possession of a land of the public domain for the
and neither is there any prohibition against the application being refiled with retroactive period provided in the Public Land Act provision in force at the time (from July 26, 1894
effect in the name of the original owners and vendors (as such natural persons) with the in Susi under the old law [this period was reduced to 'at least thirty years immediately
end result of their application being granted, because of their indisputable acquisition of preceding the filing of the application for confirmation of title' by amendment of
ownership by operation of law and the conclusive presumption therein provided in their Commonwealth Act No. 141, equivalent to the period of acquisitive prescription 5 ]) by a
favor. private individual personally and through his predecessors confers an effective title on
It should not be necessary to go through all the rituals at the great cost of refiling of all said possessor, whereby the land ceases to be land of the public domain and becomes
such applications in their names and adding to the overcrowded court dockets when the private property." I hereby reproduce the same by reference for brevity's sake. But since
Court can after all these years dispose of it here and now." (Emphasis supplied) we are reverting to the old above-cited established doctrine and precedents and
The effect is that the majority opinion now nullifies the statutory provision that only discarding the Meralco and Iglesia ni Cristo cases which departed therefrom in the
citizens (natural persons) can apply for certificates of title under Section 48(b) of the recent past, I feel constrained to write this concurrence in amplification of my views and
Public Land Act, as well as the constitutional provision (Article XIV, Section 11) which ratio decidendi.
prohibits corporations from acquiring title to lands of the public domain. That I am honored by my brethren's judgment at bar that my dissenting opinion in the June,
interpretation or construction adopted by the majority cannot be justified. "A construction 1982 Meralco and Iglesia ni Cristo cases, 1 which is herein upheld, "expressed what is
adopted should not be such as to nullify, destroy or defeat the intention of the the better. . . . and indeed the correct view." My dissent was anchored on the landmark
legislature" (New York State Dept. of Social Services v. Dublino [UST 37 L. Ed 2d 688, 1909 case of Carino 2 through the 1925 case of Susi 3 and the long line of cases cited
93 S Ct 2507; United States v. Alpers 338 US 680, 94 L Ed 457, 70 S Ct 352; cited in therein to the latest 1980 case of Herico 4 that "it is established doctrine....... that an
73 Am Jur. 2nd., p. 351). open, continuous, adverse and public possession of a land of the public domain for the
It has also been said that: period provided in the Public Land Act provision in force at the time (from July 26, 1894
In the construction of statutes, the courts start with the assumption that the legislature in Susi under the old law [this period was reduced to 'at least thirty years immediately
intended to enact an effective law, and the legislature is not to be presumed to have preceding the filing of the application for confirmation of title' by amendment of
done a vain thing in the enactment of a statute. Hence, it is a general principle that the Commonwealth Act No. 141, equivalent to the period of acquisitive prescription 5 ]) by a
courts should, if reasonably possible to do so interpret the statute, or the provision being private individual personally and through his predecessors confers an effective title on
construed, so as to give it efficient operation and effect as a whole. An interpretation said possessor, whereby the land ceases to be land of the public domain and becomes
should, if possible, be avoided, under which the statute or provision being construed is private property." I hereby reproduce the same by reference for brevity's sake. But since
defeated, or as otherwise expressed, nullified, destroyed, emasculated, repealed, we are reverting to the old above-cited established doctrine and precedents and
explained away, or rendered insignificant, meaningless, inoperative, or nugatory. If a discarding the Meralco and Iglesia ni Cristo cases which departed therefrom in the
statute is fairly susceptible of two constructions, one of which will give effect to the act, recent past, I feel constrained to write this concurrence in amplification of my views and
while the other will defeat it, the former construction is preferred. One part of a statute ratio decidendi.
may not be construed so as to render another part nugatory or of no effect. Moreover, Succeeding cases, of which only some need be mentioned, likeof Lacaste vs. Director
notwithstanding the general rule against the enlargement of extension of a statute by of Lands, 7 Mesina vs. Vda. de Sonza, 8 Manarpac vs. Cabanatuan, 9 Miguel vs. Court
construction, the meaning of a statute may be extended beyond the precise words used of Appeals 10 and Herico vs. Dar, supra, by invoking and affirming the Susi doctrine
in the law, and words or phrases may be altered or supplied, where this is necessary to have firmly rooted it in jurisprudence.
prevent a law from becoming a nullity. Wherever the provision of a statute is general
everything which is necessary to make such provision effectual is supplied by
implication. (Pliakos vs. Illinois Liquor Control Com. 11 III 2d 456, 143 NE2d 47; cited in
73 AM Jur. 2d pp. 422-423)
The statutory provision and the constitutional prohibition express a public policy. The
proper course for the Court to take is to promote in the fullest manner the policy thus
laid down and to avoid a construction which would alter or defeat that policy.
In fine, I confirm my adherence to the ruling of this Court in Meralco vs. Hon. Castro-
Bartolome, 114 SCRA 799 [1982] and related cases.
I am honored by my brethren's judgment at bar that my dissenting opinion in the June,
1982 Meralco and Iglesia ni Cristo cases, 1 which is herein upheld, "expressed what is
the better. . . . and indeed the correct view." My dissent was anchored on the landmark
1909 case of Carino 2 through the 1925 case of Susi 3 and the long line of cases cited
gemelina trees in addition to existing coconut trees which were then 50 to 60 years old,
and paid the corresponding taxes due on the subject land. At present, there are parcels
of land surrounding the subject land which have been issued titles by virtue of judicial
SECOND DIVISION decrees. Naguit and her predecessors-in-interest have occupied the land openly and in
the concept of owner without any objection from any private person or even the
[G.R. No. 144057. January 17, 2005] government until she filed her application for registration.
REPUBLIC OF THE PHILIPPINES, petitioner, vs. THE HONORABLE After the presentation of evidence for Naguit, the public prosecutor manifested that
COURT OF APPEALS and CORAZON NAGUIT, respondents. the government did not intend to present any evidence while oppositor Jose Angeles, as
representative of the heirs of Rustico Angeles, failed to appear during the trial despite
DECISION notice. On September 27, 1997, the MCTC rendered a decision ordering that the
subject parcel be brought under the operation of the Property Registration Decree or
TINGA, J.:
Presidential Decree (P.D.) No. 1529 and that the title thereto registered and confirmed
in the name of Naguit.[6]
This is a Petition for Review on Certiorari under Rule 45 of the 1997 Rules of
Civil Procedure, seeking to review the Decision[1] of the Sixth Division of the Court of The Republic of the Philippines (Republic), thru the Office of the Solicitor General
Appeals dated July 12, 2000 in CA-G.R. SP No. 51921. The appellate court affirmed the (OSG), filed a motion for reconsideration. The OSG stressed that the land applied for
decisions of both the Regional Trial Court (RTC),[2] Branch 8, of Kalibo, Aklan dated was declared alienable and disposable only on October 15, 1980, per the certification
February 26, 1999, and the 7th Municipal Circuit Trial Court (MCTC)[3] of Ibajay-Nabas, from Regional Executive Director Raoul T. Geollegue of the Department of Environment
Aklan dated February 18, 1998, which granted the application for registration of a parcel and Natural Resources, Region VI.[7] However, the court denied the motion for
of land of Corazon Naguit (Naguit), the respondent herein. reconsideration in an order dated February 18, 1998.[8]

Thereafter, the Republic appealed the decision and the order of the MCTC to the
The facts are as follows:
RTC, Kalibo, Aklan, Branch 8. On February 26, 1999, the RTC rendered its decision,
On January 5, 1993, Naguit, a Filipino citizen, of legal age and married to Manolito dismissing the appeal.[9]
S. Naguit, filed with the MCTC of Ibajay-Nabas, Aklan, a petition for registration of title
Undaunted, the Republic elevated the case to the Court of Appeals via Rule 42 of
of a parcel of land situated in Brgy. Union, Nabas, Aklan. The parcel of land is
the 1997 Rules of Civil Procedure. On July 12, 2000, the appellate court rendered a
designated as Lot No. 10049, Cad. 758-D, Nabas Cadastre, AP 060414-014779, and
decision dismissing the petition filed by the Republic and affirmed in toto the assailed
contains an area of 31,374 square meters. The application seeks judicial confirmation of
decision of the RTC.
respondents imperfect title over the aforesaid land.
Hence, the present petition for review raising a pure question of law was filed by
On February 20, 1995, the court held initial hearing on the application. The public
the Republic on September 4, 2000.[10]
prosecutor, appearing for the government, and Jose Angeles, representing the heirs of
Rustico Angeles, opposed the petition. On a later date, however, the heirs of Rustico The OSG assails the decision of the Court of Appeals contending that the
Angeles filed a formal opposition to the petition. Also on February 20, 1995, the court appellate court gravely erred in holding that there is no need for the governments prior
issued an order of general default against the whole world except as to the heirs of release of the subject lot from the public domain before it can be considered alienable
Rustico Angeles and the government. or disposable within the meaning of P.D. No. 1529, and that Naguit had been in
possession of Lot No. 10049 in the concept of owner for the required period.[11]
The evidence on record reveals that the subject parcel of land was originally
declared for taxation purposes in the name of Ramon Urbano (Urbano) in 1945 under Hence, the central question for resolution is whether is necessary under Section
Tax Declaration No. 3888 until 1991.[4] On July 9, 1992, Urbano executed a Deed of 14(1) of the Property Registration Decree that the subject land be first classified as
Quitclaim in favor of the heirs of Honorato Maming (Maming), wherein he renounced all alienable and disposable before the applicants possession under a bona fide claim of
his rights to the subject property and confirmed the sale made by his father to Maming ownership could even start.
sometime in 1955 or 1956.[5] Subsequently, the heirs of Maming executed a deed of
The OSG invokes our holding in Director of Lands v. Intermediate Appellate
absolute sale in favor of respondent Naguit who thereupon started occupying the same.
Court[12] in arguing that the property which is in open, continuous and exclusive
She constituted Manuel Blanco, Jr. as her attorney-in-fact and administrator. The
possession must first be alienable. Since the subject land was declared alienable only
administrator introduced improvements, planted trees, such as mahogany, coconut and
on October 15, 1980, Naguit could not have maintained a bona fide claim of ownership
since June 12, 1945, as required by Section 14 of the Property Registration Decree, is made, has not yet deemed it proper to release the property for alienation or
since prior to 1980, the land was not alienable or disposable, the OSG argues. disposition, the presumption is that the government is still reserving the right to utilize
the property; hence, the need to preserve its ownership in the State irrespective of the
Section 14 of the Property Registration Decree, governing original registration
length of adverse possession even if in good faith. However, if the property has already
proceedings, bears close examination. It expressly provides:
been classified as alienable and disposable, as it is in this case, then there is already an
SECTION 14. Who may apply. The following persons may file in the proper Court of intention on the part of the State to abdicate its exclusive prerogative over the property.
First Instance an application for registration of title to land, whether personally or
This reading aligns conformably with our holding in Republic v. Court of
through their duly authorized representatives:
Appeals.[14] Therein, the Court noted that to prove that the land subject of an
(1) those who by themselves or through their predecessors-in-interest have been in application for registration is alienable, an applicant must establish the existence of a
open, continuous, exclusive and notorious possession and occupation of alienable and positive act of the government such as a presidential proclamation or an executive
disposable lands of the public domain under a bona fide claim of ownership since June order; an administrative action; investigation reports of Bureau of Lands investigators;
12, 1945, or earlier. and a legislative act or a statute.[15] In that case, the subject land had been certified by
the DENR as alienable and disposable in 1980, thus the Court concluded that the
(2) Those who have acquired ownership over private lands by prescription under the alienable status of the land, compounded by the established fact that therein
provisions of existing laws. respondents had occupied the land even before 1927, sufficed to allow the application
.... for registration of the said property. In the case at bar, even the petitioner admits that
the subject property was released and certified as within alienable and disposable zone
There are three obvious requisites for the filing of an application for registration of in 1980 by the DENR.[16]
title under Section 14(1) that the property in question is alienable and disposable land of
the public domain; that the applicants by themselves or through their predecessors-in- This case is distinguishable from Bracewell v. Court of Appeals,[17] wherein the
interest have been in open, continuous, exclusive and notorious possession and Court noted that while the claimant had been in possession since 1908, it was only in
occupation, and; that such possession is under a bona fide claim of ownership since 1972 that the lands in question were classified as alienable and disposable. Thus, the
June 12, 1945 or earlier. bid at registration therein did not succeed. In Bracewell, the claimant had filed his
application in 1963, or nine (9) years before the property was declared alienable and
Petitioner suggests an interpretation that the alienable and disposable character of disposable. Thus, in this case, where the application was made years after the property
the land should have already been established since June 12, 1945 or earlier. This is had been certified as alienable and disposable, the Bracewell ruling does not apply.
not borne out by the plain meaning of Section 14(1). Since June 12, 1945, as used in
the provision, qualifies its antecedent phrase under a bonafide claim of ownership. A different rule obtains for forest lands,[18] such as those which form part of a
Generally speaking, qualifying words restrict or modify only the words or phrases to reservation for provincial park purposes[19] the possession of which cannot ripen into
which they are immediately associated, and not those distantly or remotely ownership.[20] It is elementary in the law governing natural resources that forest land
located.[13] Ad proximum antecedents fiat relation nisi impediatur sentencia. cannot be owned by private persons. As held in Palomo v. Court of Appeals,[21] forest
land is not registrable and possession thereof, no matter how lengthy, cannot convert it
Besides, we are mindful of the absurdity that would result if we adopt petitioners into private property, unless such lands are reclassified and considered disposable and
position. Absent a legislative amendment, the rule would be, adopting the OSGs view, alienable.[22] In the case at bar, the property in question was undisputedly classified as
that all lands of the public domain which were not declared alienable or disposable disposable and alienable; hence, the ruling in Palomo is inapplicable, as correctly held
before June 12, 1945 would not be susceptible to original registration, no matter the by the Court of Appeals.[23]
length of unchallenged possession by the occupant. Such interpretation renders
paragraph (1) of Section 14 virtually inoperative and even precludes the government It must be noted that the present case was decided by the lower courts on the
from giving it effect even as it decides to reclassify public agricultural lands as alienable basis of Section 14(1) of the Property Registration Decree, which pertains to original
and disposable. The unreasonableness of the situation would even be aggravated registration through ordinary registration proceedings. The right to file the application for
considering that before June 12, 1945, the Philippines was not yet even considered an registration derives from a bona fide claim of ownership going back to June 12, 1945 or
independent state. earlier, by reason of the claimants open, continuous, exclusive and notorious
possession of alienable and disposable lands of the public domain.
Instead, the more reasonable interpretation of Section 14(1) is that it merely
requires the property sought to be registered as already alienable and disposable at the A similar right is given under Section 48(b) of the Public Land Act, which reads:
time the application for registration of title is filed. If the State, at the time the application
Sec. 48. The following described citizens of the Philippines, occupying lands of the date later than June 12, 1945, and such possession being been open, continuous and
public domain or claiming to own any such land or an interest therein, but those titles exclusive, then the possessor may have the right to register the land by virtue of Section
have not been perfected or completed, may apply to the Court of First Instance of the 14(2) of the Property Registration Decree.
province where the land is located for confirmation of their claims and the issuance of a
The land in question was found to be cocal in nature, it having been planted with
certificate of title therefor, under the Land Registration Act, to wit:
coconut trees now over fifty years old.[27] The inherent nature of the land but confirms
xxx xxx xxx its certification in 1980 as alienable, hence agricultural. There is no impediment to the
application of Section 14(1) of the Property Registration Decree, as correctly
(b) Those who by themselves or through their predecessors in interest have been in
accomplished by the lower courts.
open, continuous, exclusive, and notorious possession and occupation of agricultural
lands of the public domain, under a bona fide claim of acquisition of ownership, for at The OSG posits that the Court of Appeals erred in holding that Naguit had been in
least thirty years immediately preceding the filing of the application for confirmation of possession in the concept of owner for the required period. The argument begs the
title except when prevented by war or force majeure. These shall be conclusively question. It is again hinged on the assertionshown earlier to be unfoundedthat there
presumed to have performed all the conditions essential to a Government grant and could have been no bona fide claim of ownership prior to 1980, when the subject land
shall be entitled to a certificate of title under the provisions of this chapter. was declared alienable or disposable.

When the Public Land Act was first promulgated in 1936, the period of possession We find no reason to disturb the conclusion of both the RTC and the Court of
deemed necessary to vest the right to register their title to agricultural lands of the public Appeals that Naguit had the right to apply for registration owing to the continuous
domain commenced from July 26, 1894. However, this period was amended by R.A. possession by her and her predecessors-in-interest of the land since 1945. The basis of
No. 1942, which provided that the bona fide claim of ownership must have been for at such conclusion is primarily factual, and the Court generally respects the factual
least thirty (30) years. Then in 1977, Section 48(b) of the Public Land Act was again findings made by lower courts. Notably, possession since 1945 was established through
amended, this time by P.D. No. 1073, which pegged the reckoning date at June 12, proof of the existence of 50 to 60-year old trees at the time Naguit purchased the
1945. This new starting point is concordant with Section 14(1) of the Property property as well as tax declarations executed by Urbano in 1945. Although tax
Registration Decree. declarations and realty tax payment of property are not conclusive evidence of
ownership, nevertheless, they are good indicia of the possession in the concept of
Indeed, there are no material differences between Section 14(1) of the Property
owner for no one in his right mind would be paying taxes for a property that is not in his
Registration Decree and Section 48(b) of the Public Land Act, as amended. True, the
actual or at least constructive possession. They constitute at least proof that the holder
Public Land Act does refer to agricultural lands of the public domain, while the Property
has a claim of title over the property. The voluntary declaration of a piece of property for
Registration Decree uses the term alienable and disposable lands of the public domain.
taxation purposes manifests not only ones sincere and honest desire to obtain title to
It must be noted though that the Constitution declares that alienable lands of the public
the property and announces his adverse claim against the State and all other interested
domain shall be limited to agricultural lands.[24] Clearly, the subject lands under Section
parties, but also the intention to contribute needed revenues to the Government. Such
48(b) of the Public Land Act and Section 14(1) of the Property Registration Decree are
an act strengthens ones bona fide claim of acquisition of ownership.[28]
of the same type.
Considering that the possession of the subject parcel of land by the respondent
Did the enactment of the Property Registration Decree and the amendatory P.D.
can be traced back to that of her predecessors-in-interest which commenced since
No. 1073 preclude the application for registration of alienable lands of the public
1945 or for almost fifty (50) years, it is indeed beyond any cloud of doubt that she has
domain, possession over which commenced only after June 12, 1945? It did not,
acquired title thereto which may be properly brought under the operation of the Torrens
considering Section 14(2) of the Property Registration Decree, which governs and
system. That she has been in possession of the land in the concept of an owner, open,
authorizes the application of those who have acquired ownership of private lands by
continuous, peaceful and without any opposition from any private person and the
prescription under the provisions of existing laws.
government itself makes her right thereto undoubtedly settled and deserving of
Prescription is one of the modes of acquiring ownership under the Civil protection under the law.
Code.[25] There is a consistent jurisprudential rule that properties classified as alienable
WHEREFORE, foregoing premises considered, the assailed Decision of the Court
public land may be converted into private property by reason of open, continuous and
of Appeals dated July 12, 2000 is hereby AFFIRMED. No costs.
exclusive possession of at least thirty (30) years.[26] With such conversion, such
property may now fall within the contemplation of private lands under Section 14(2), and SO ORDERED.
thus susceptible to registration by those who have acquired ownership through
Puno, (Chairman), Austria-Martinez, Callejo, Sr., and Chico-Nazario, JJ., concur.
prescription. Thus, even if possession of the alienable public land commenced on a
REPUBLIC OF THE PHILIPPINES, petitioner, vs. JEREMIAS AND DAVID
HERBIETO, respondents.

DECISION

CHICO-NAZARIO, J.:

Before this Court is a Petition for Review on Certiorari, under Rule 45 of the 1997
Rules of Civil Procedure, seeking the reversal of the Decision of the Court of Appeals in
CA-G.R. CV No. 67625, dated 22 November 2002,[1] which affirmed the Judgment of
the Municipal Trial Court (MTC) of Consolacion, Cebu, dated 21 December
1999,[2] granting the application for land registration of the respondents.

Respondents in the present Petition are the Herbieto brothers, Jeremias and
David, who filed with the MTC, on 23 September 1998, a single application for
registration of two parcels of land, Lots No. 8422 and 8423, located in Cabangahan,
Consolacion, Cebu (Subject Lots). They claimed to be owners in fee simple of the
Subject Lots, which they purchased from their parents, spouses Gregorio Herbieto and
Isabel Owatan, on 25 June 1976.[3] Together with their application for registration,
respondents submitted the following set of documents:

(a) Advance Survey Plan of Lot No. 8422, in the name of respondent Jeremias; and
Advance Survey Plan of Lot No. 8423, in the name of respondent David;[4]

(b) The technical descriptions of the Subject Lots;[5]

(c) Certifications by the Department of Environment and Natural Resources (DENR)


dispensing with the need for Surveyors Certificates for the Subject Lots;[6]

(d) Certifications by the Register of Deeds of Cebu City on the absence of certificates of
title covering the Subject Lots;[7]

(e) Certifications by the Community Environment and Natural Resources Office


(CENRO) of the DENR on its finding that the Subject Lots are alienable and disposable,
by virtue of Forestry Administrative Order No. 4-1063, dated 25 June 1963;[8]

(f) Certified True Copies of Assessment of Real Property (ARP) No. 941800301831, in
the name of Jeremias, covering Lot No. 8422, issued in 1994; and ARP No.
941800301833, in the name of David, covering Lot No. 8423, also issued in
1994;[9] and
SECOND DIVISION (g) Deed of Definite Sale executed on 25 June 1976 by spouses Gregorio Herbieto and
Isabel Owatan selling the Subject Lots and the improvements thereon to their sons and
respondents herein, Jeremias and David, for P1,000. Lot No. 8422 was sold to
Jeremias, while Lot No. 8423 was sold to David.[10]
[G.R. No. 156117. May 26, 2005]
On 11 December 1998, the petitioner Republic of the Philippines (Republic) filed
an Opposition to the respondents application for registration of the Subject Lots arguing
that: (1) Respondents failed to comply with the period of adverse possession of the
Subject Lots required by law; (2) Respondents muniments of title were not genuine and 1963 or from the time the subject lots had been classified as within the alienable and
did not constitute competent and sufficient evidence of bona fideacquisition of the disposable zone, still the argument of the appellant does not hold water.
Subject Lots; and (3) The Subject Lots were part of the public domain belonging to the
As earlier stressed, the subject property, being alienable since 1963 as shown by
Republic and were not subject to private appropriation.[11]
CENRO Report dated June 23, 1963, may now be the object of prescription, thus
The MTC set the initial hearing on 03 September 1999 at 8:30 a.m.[12] All owners susceptible of private ownership. By express provision of Article 1137, appellees are,
of the land adjoining the Subject Lots were sent copies of the Notice of Initial with much greater right, entitled to apply for its registration, as provided by Section 14(4)
Hearing.[13] A copy of the Notice was also posted on 27 July 1999 in a conspicuous of P.D. 1529 which allows individuals to own land in any manner provided by law.
place on the Subject Lots, as well as on the bulletin board of the municipal building of Again, even considering that possession of appelless should only be reckoned from
Consolacion, Cebu, where the Subject Lots were located.[14] Finally, the Notice was 1963, the year when CENRO declared the subject lands alienable, herein appellees
also published in the Official Gazette on 02 August 1999[15] and The Freeman Banat have been possessing the subject parcels of land in open, continuous, and in the
News on 19 December 1999.[16] concept of an owner, for 35 years already when they filed the instant application for
registration of title to the land in 1998. As such, this court finds no reason to disturb the
During the initial hearing on 03 September 1999, the MTC issued an Order of
finding of the court a quo.[20]
Special Default,[17] with only petitioner Republic opposing the application for
registration of the Subject Lots. The respondents, through their counsel, proceeded to The Republic filed the present Petition for the review and reversal of the Decision
offer and mark documentary evidence to prove jurisdictional facts. The MTC of the Court of Appeals, dated 22 November 2002, on the basis of the following
commissioned the Clerk of Court to receive further evidence from the respondents and arguments:
to submit a Report to the MTC after 30 days.
First, respondents failed to establish that they and their predecessors-in-interest
On 21 December 1999, the MTC promulgated its Judgment ordering the had been in open, continuous, and adverse possession of the Subject Lots in the
registration and confirmation of the title of respondent Jeremias over Lot No. 8422 and concept of owners since 12 June 1945 or earlier. According to the petitioner Republic,
of respondent David over Lot No. 8423. It subsequently issued an Order on 02 February possession of the Subject Lots prior to 25 June 1963 cannot be considered in
2000 declaring its Judgment, dated 21 December 1999, final and executory, and determining compliance with the periods of possession required by law. The Subject
directing the Administrator of the Land Registration Authority (LRA) to issue a decree of Lots were classified as alienable and disposable only on 25 June 1963, per CENROs
registration for the Subject Lots.[18] certification. It also alleges that the Court of Appeals, in applying the 30-year acquisitive
prescription period, had overlooked the ruling in Republic v. Doldol,[21] where this Court
Petitioner Republic appealed the MTC Judgment, dated 21 December 1999, to the
declared that Commonwealth Act No. 141, otherwise known as the Public Land Act, as
Court of Appeals.[19] The Court of Appeals, in its Decision, dated 22 November 2002,
amended and as it is presently phrased, requires that possession of land of the public
affirmed the appealed MTC Judgment reasoning thus:
domain must be from 12 June 1945 or earlier, for the same to be acquired through
In the case at bar, there can be no question that the land sought to be registered has judicial confirmation of imperfect title.
been classified as within the alienable and disposable zone since June 25, 1963. Article
Second, the application for registration suffers from fatal infirmity as the subject of
1113 in relation to Article 1137 of the Civil Code, respectively provides that All things
the application consisted of two parcels of land individually and separately owned by
which are within the commerce of men are susceptible of prescription, unless otherwise
two applicants. Petitioner Republic contends that it is implicit in the provisions of
provided. Property of the State or any of its subdivisions of patrimonial character shall
Presidential Decree No. 1529, otherwise known as the Property Registration Decree, as
not be the object of prescription and that Ownership and other real rights over
amended, that the application for registration of title to land shall be filed by a single
immovables also prescribe through uninterrupted adverse possession thereof for thirty
applicant; multiple applicants may file a single application only in case they are co-
years, without need of title or of good faith.
owners. While an application may cover two parcels of land, it is allowed only when the
As testified to by the appellees in the case at bench, their parents already acquired the subject parcels of land belong to the same applicant or applicants (in case the subject
subject parcels of lands, subject matter of this application, since 1950 and that they parcels of land are co-owned) and are situated within the same province. Where the
cultivated the same and planted it with jackfruits, bamboos, coconuts, and other trees authority of the courts to proceed is conferred by a statute and when the manner of
(Judgment dated December 21, 1999, p. 6). In short, it is undisputed that herein obtaining jurisdiction is mandatory, it must be strictly complied with or the proceedings
appellees or their predecessors-in-interest had occupied and possessed the subject will be utterly void. Since the respondents failed to comply with the procedure for land
land openly, continuously, exclusively, and adversely since 1950. Consequently, even registration under the Property Registration Decree, the proceedings held before the
assuming arguendo that appellees possession can be reckoned only from June 25, MTC is void, as the latter did not acquire jurisdiction over it.

I
Jurisdiction misjoinder of causes of action and parties. Instead of a single or joint application for
registration, respondents Jeremias and David, more appropriately, should have filed
separate applications for registration of Lots No. 8422 and 8423, respectively.
Addressing first the issue of jurisdiction, this Court finds that the MTC had no
jurisdiction to proceed with and hear the application for registration filed by the Misjoinder of causes of action and parties do not involve a question of jurisdiction
respondents but for reasons different from those presented by petitioner Republic. of the court to hear and proceed with the case.[26]They are not even accepted grounds
for dismissal thereof.[27] Instead, under the Rules of Court, the misjoinder of causes of
A. The misjoinder of causes of action and parties does not affect the jurisdiction action and parties involve an implied admission of the courts jurisdiction. It
of the MTC to hear and proceed with respondents application for registration. acknowledges the power of the court, acting upon the motion of a party to the case or
Respondents filed a single application for registration of the Subject Lots even on its own initiative, to order the severance of the misjoined cause of action, to be
though they were not co-owners. Respondents Jeremias and David were actually proceeded with separately (in case of misjoinder of causes of action); and/or the
seeking the individual and separate registration of Lots No. 8422 and 8423, dropping of a party and the severance of any claim against said misjoined party, also to
respectively. be proceeded with separately (in case of misjoinder of parties).

Petitioner Republic believes that the procedural irregularity committed by the The misjoinder of causes of action and parties in the present Petition may have
respondents was fatal to their case, depriving the MTC of jurisdiction to proceed with been corrected by the MTC motu propio or on motion of the petitioner Republic. It is
and hear their application for registration of the Subject Lots, based on this Courts regrettable, however, that the MTC failed to detect the misjoinder when the application
pronouncement in Director of Lands v. Court of Appeals,[22] to wit: for registration was still pending before it; and more regrettable that the petitioner
Republic did not call the attention of the MTC to the fact by filing a motion for severance
. . . In view of these multiple omissions which constitute non-compliance with the above- of the causes of action and parties, raising the issue of misjoinder only before this
cited sections of the Act, We rule that said defects have not invested the Court with the Court.
authority or jurisdiction to proceed with the case because the manner or mode of
obtaining jurisdiction as prescribed by the statute which is mandatory has not been B. Respondents, however, failed to comply with the publication requirements
strictly followed, thereby rendering all proceedings utterly null and void. mandated by the Property Registration Decree, thus, the MTC was not invested
with jurisdiction as a land registration court.
This Court, however, disagrees with petitioner Republic in this regard. This
procedural lapse committed by the respondents should not affect the jurisdiction of the Although the misjoinder of causes of action and parties in the present Petition did
MTC to proceed with and hear their application for registration of the Subject Lots. not affect the jurisdiction of the MTC over the land registration proceeding, this Court,
nonetheless, has discovered a defect in the publication of the Notice of Initial Hearing,
The Property Registration Decree[23] recognizes and expressly allows the which bars the MTC from assuming jurisdiction to hear and proceed with respondents
following situations: (1) the filing of a single application by several applicants for as long application for registration.
as they are co-owners of the parcel of land sought to be registered;[24] and (2) the filing
of a single application for registration of several parcels of land provided that the same A land registration case is a proceeding in rem,[28] and jurisdiction in rem cannot
are located within the same province.[25] The Property Registration Decree is silent, be acquired unless there be constructive seizure of the land through publication and
however, as to the present situation wherein two applicants filed a single application for service of notice.[29]
two parcels of land, but are seeking the separate and individual registration of the Section 23 of the Property Registration Decree requires that the public be given
parcels of land in their respective names. Notice of the Initial Hearing of the application for land registration by means of (1)
Since the Property Registration Decree failed to provide for such a situation, then publication; (2) mailing; and (3) posting. Publication of the Notice of Initial Hearing shall
this Court refers to the Rules of Court to determine the proper course of action. Section be made in the following manner:
34 of the Property Registration Decree itself provides that, [t]he Rules of Court shall, 1. By publication.
insofar as not inconsistent with the provisions of this Decree, be applicable to land
registration and cadastral cases by analogy or in a suppletory character and whenever Upon receipt of the order of the court setting the time for initial hearing, the
practicable and convenient. Commissioner of Land Registration shall cause a notice of initial hearing to be
published once in the Official Gazette and once in a newspaper of general circulation in
Considering every application for land registration filed in strict accordance with the Philippines: Provided, however, that the publication in the Official Gazette shall be
the Property Registration Decree as a single cause of action, then the defect in the joint sufficient to confer jurisdiction upon the court. Said notice shall be addressed to all
application for registration filed by the respondents with the MTC constitutes a persons appearing to have an interest in the land involved including the adjoining
owners so far as known, and to all whom it may concern. Said notice shall also require issued pursuant thereto. In fact, the MTC did issue an Order of Special Default on 03
all persons concerned to appear in court at a certain date and time to show cause why September 1999.
the prayer of said application shall not be granted.
The late publication of the Notice of Initial Hearing in the newspaper of general
Even as this Court concedes that the aforequoted Section 23(1) of the Property circulation is tantamount to no publication at all, having the same ultimate result. Owing
Registration Decree expressly provides that publication in the Official Gazette shall be to such defect in the publication of the Notice, the MTC failed to constructively seize the
sufficient to confer jurisdiction upon the land registration court, it still affirms its Subject Lots and to acquire jurisdiction over respondents application for registration
declaration in Director of Lands v. Court of Appeals[30] that publication in a newspaper thereof. Therefore, the MTC Judgment, dated 21 December 1999, ordering the
of general circulation is mandatory for the land registration court to validly confirm and registration and confirmation of the title of respondents Jeremias and David over Lots
register the title of the applicant or applicants. That Section 23 of the Property No. 8422 and 8423, respectively; as well as the MTC Order, dated 02 February 2000,
Registration Decree enumerated and described in detail the requirements of publication, declaring its Judgment of 21 December 1999 final and executory, and directing the LRA
mailing, and posting of the Notice of Initial Hearing, then all such requirements, Administrator to issue a decree of registration for the Subject Lots, are both null and
including publication of the Notice in a newspaper of general circulation, is essential and void for having been issued by the MTC without jurisdiction.
imperative, and must be strictly complied with. In the same case, this Court expounded
II
on the reason behind the compulsory publication of the Notice of Initial Hearing in a
newspaper of general circulation, thus

It may be asked why publication in a newspaper of general circulation should be Period of Possession
deemed mandatory when the law already requires notice by publication in the Official
Gazette as well as by mailing and posting, all of which have already been complied with
in the case at hand. The reason is due process and the reality that the Official Gazette Respondents failed to comply with the required period of possession of the Subject Lots
is not as widely read and circulated as newspaper and is oftentimes delayed in its for the judicial confirmation or legalization of imperfect or incomplete title.
circulation, such that the notices published therein may not reach the interested parties
on time, if at all. Additionally, such parties may not be owners of neighboring properties, While this Court has already found that the MTC did not have jurisdiction to hear
and may in fact not own any other real estate. In sum, the all encompassing in and proceed with respondents application for registration, this Court nevertheless
rem nature of land registration cases, the consequences of default orders issued deems it necessary to resolve the legal issue on the required period of possession for
acquiring title to public land.
against the whole world and the objective of disseminating the notice in as wide a
manner as possible demand a mandatory construction of the requirements for Respondents application filed with the MTC did not state the statutory basis for
publication, mailing and posting.[31] their title to the Subject Lots. They only alleged therein that they obtained title to the
Subject Lots by purchase from their parents, spouses Gregorio Herbieto and Isabel
In the instant Petition, the initial hearing was set by the MTC, and was in fact held,
Owatan, on 25 June 1976. Respondent Jeremias, in his testimony, claimed that his
on 03 September 1999 at 8:30 a.m. While the Notice thereof was printed in the issue of
the Official Gazette, dated 02 August 1999, and officially released on 10 August 1999, it parents had been in possession of the Subject Lots in the concept of an owner since
was published in The Freeman Banat News, a daily newspaper printed in Cebu City and 1950.[32]
circulated in the province and cities of Cebu and in the rest of Visayas and Mindanao, Yet, according to the DENR-CENRO Certification, submitted by respondents
only on 19 December 1999, more than three months after the initial hearing. themselves, the Subject Lots are within Alienable and Disposable, Block I, Project No.
Indubitably, such publication of the Notice, way after the date of the initial hearing, 28 per LC Map No. 2545 of Consolacion, Cebu certified under Forestry Administrative
Order No. 4-1063, dated June 25, 1963. Likewise, it is outside Kotkot-Lusaran Mananga
would already be worthless and ineffective. Whoever read the Notice as it was
published in The Freeman Banat News and had a claim to the Subject Lots was Watershed Forest Reservation per Presidential Proclamation No. 932 dated June 29,
1992.[33] The Subject Lots are thus clearly part of the public domain, classified as
deprived of due process for it was already too late for him to appear before the MTC on
the day of the initial hearing to oppose respondents application for registration, and to alienable and disposable as of 25 June 1963.
present his claim and evidence in support of such claim. Worse, as the Notice itself As already well-settled in jurisprudence, no public land can be acquired by private
states, should the claimant-oppositor fail to appear before the MTC on the date of initial persons without any grant, express or implied, from the government;[34] and it is
hearing, he would be in default and would forever be barred from contesting indispensable that the person claiming title to public land should show that his title was
respondents application for registration and even the registration decree that may be acquired from the State or any other mode of acquisition recognized by law.[35]
The Public Land Act, as amended, governs lands of the public domain, except Section 48(b) of the Public Land Act, as amended. Section 48(b), as amended, now
timber and mineral lands, friar lands, and privately-owned lands which reverted to the requires adverse possession of the land since 12 June 1945 or earlier. In the present
State.[36] It explicitly enumerates the means by which public lands may be disposed, as Petition, the Subject Lots became alienable and disposable only on 25 June 1963. Any
follows: period of possession prior to the date when the Subject Lots were classified as
alienable and disposable is inconsequential and should be excluded from the
(1) For homestead settlement;
computation of the period of possession; such possession can never ripen into
(2) By sale; ownership and unless the land had been classified as alienable and disposable, the
rules on confirmation of imperfect title shall not apply thereto.[41] It is very apparent
(3) By lease; then that respondents could not have complied with the period of possession required
(4) By confirmation of imperfect or incomplete titles; by Section 48(b) of the Public Land Act, as amended, to acquire imperfect or incomplete
title to the Subject Lots that may be judicially confirmed or legalized.
(a) By judicial legalization; or
The confirmation of respondents title by the Court of Appeals was based on the
(b) By administrative legalization (free patent).[37] erroneous supposition that respondents were claiming title to the Subject Lots under the
Property Registration Decree. According to the Decision of the Court of Appeals, dated
Each mode of disposition is appropriately covered by separate chapters of the Public
22 November 2002, Section 14(4) of the Property Registration Decree allows individuals
Land Act because there are specific requirements and application procedure for every
to own land in any other manner provided by law. It then ruled that the respondents,
mode.[38] Since respondents herein filed their application before the MTC,[39] then it
having possessed the Subject Lots, by themselves and through their predecessors-in-
can be reasonably inferred that they are seeking the judicial confirmation or legalization
interest, since 25 June 1963 to 23 September 1998, when they filed their application,
of their imperfect or incomplete title over the Subject Lots.
have acquired title to the Subject Lots by extraordinary prescription under Article 1113,
Judicial confirmation or legalization of imperfect or incomplete title to land, not in relation to Article 1137, both of the Civil Code.[42]
exceeding 144 hectares,[40] may be availed of by persons identified under Section 48
The Court of Appeals overlooked the difference between the Property Registration
of the Public Land Act, as amended by Presidential Decree No. 1073, which reads
Decree and the Public Land Act. Under the Property Registration Decree, there already
Section 48. The following-described citizens of the Philippines, occupying lands of the exists a title which is confirmed by the court; while under the Public Land Act, the
public domain or claiming to own any such lands or an interest therein, but whose titles presumption always is that the land applied for pertains to the State, and that the
have not been perfected or completed, may apply to the Court of First Instance of the occupants and possessors only claim an interest in the same by virtue of their imperfect
province where the land is located for confirmation of their claims and the issuance of a title or continuous, open, and notorious possession.[43] As established by this Court in
certificate of title thereafter, under the Land Registration Act, to wit: the preceding paragraphs, the Subject Lots respondents wish to register are
undoubtedly alienable and disposable lands of the public domain and respondents may
(a) [Repealed by Presidential Decree No. 1073].
have acquired title thereto only under the provisions of the Public Land Act.
(b) Those who by themselves or through their predecessors-in-interest have been in
However, it must be clarified herein that even though respondents may acquire
open, continuous, exclusive, and notorious possession and occupation of agricultural
imperfect or incomplete title to the Subject Lots under the Public Land Act, their
lands of the public domain, under a bona fide claim of acquisition of ownership, since
application for judicial confirmation or legalization thereof must be in accordance with
June 12, 1945, or earlier, immediately preceding the filing of the applications for
the Property Registration Decree, for Section 50 of the Public Land Act reads
confirmation of title, except when prevented by war or force majeure. These shall be
conclusively presumed to have performed all the conditions essential to a Government SEC. 50. Any person or persons, or their legal representatives or successors in right,
grant and shall be entitled to a certificate of title under the provisions of this chapter. claiming any lands or interest in lands under the provisions of this chapter, must in
every case present an application to the proper Court of First Instance, praying that the
(c) Members of the national cultural minorities who by themselves or through their
validity of the alleged title or claim be inquired into and that a certificate of title be issued
predecessors-in-interest have been in open, continuous, exclusive and notorious
to them under the provisions of the Land Registration Act.[44]
possession and occupation of lands of the public domain suitable to agriculture whether
disposable or not, under a bona fide claim of ownership since June 12, 1945 shall be Hence, respondents application for registration of the Subject Lots must have
entitled to the rights granted in subsection (b) hereof. complied with the substantial requirements under Section 48(b) of the Public Land Act
and the procedural requirements under the Property Registration Decree.
Not being members of any national cultural minorities, respondents may only be
entitled to judicial confirmation or legalization of their imperfect or incomplete title under
Moreover, provisions of the Civil Code on prescription of ownership and other real Malabanan, who had purchased the property from Eduardo Velazco, filed an application
rights apply in general to all types of land, while the Public Land Act specifically governs for land registration covering the property in the Regional Trial Court (RTC) in Tagaytay
lands of the public domain. Relative to one another, the Public Land Act may be City, Cavite, claiming that the property formed part of the alienable and disposable land
considered a special law[45] that must take precedence over the Civil Code, a general of the public domain, and that he and his predecessors-in-interest had been in open,
law. It is an established rule of statutory construction that between a general law and a continuous, uninterrupted, public and adverse possession and occupation of the land for
special law, the special law prevails Generalia specialibus non derogant.[46] more than 30 years, thereby entitling him to the judicial confirmation of his title.1
To prove that the property was an alienable and disposable land of the public domain,
WHEREFORE, based on the foregoing, the instant Petition is GRANTED. The Malabanan presented during trial a certification dated June 11, 2001 issued by the
Decision of the Court of Appeals in CA-G.R. CV No. 67625, dated 22 November 2002, Community Environment and Natural Resources Office (CENRO) of the Department of
is REVERSED. The Judgment of the MTC of Consolacion, Cebu in LRC Case No. N- Environment and Natural Resources (DENR), which reads:
75, dated 21 December 1999, and its Order, dated 02 February 2000 are declared This is to certify that the parcel of land designated as Lot No. 9864 Cad 452-D, Silang
NULL AND VOID. Respondents application for registration is DISMISSED. Cadastre as surveyed for Mr. Virgilio Velasco located at Barangay Tibig, Silang, Cavite
containing an area of 249,734 sq. meters as shown and described on the Plan Ap-04-
SO ORDERED. 00952 is verified to be within the Alienable or Disposable land per Land Classification
Puno, Acting C.J., (Chairman), Austria-Martinez, and Callejo, Sr., JJ., concur. Map No. 3013 established under Project No. 20-A and approved as such under FAO 4-
Tinga, J., out of the country. 1656 on March 15, 1982.2
After trial, on December 3, 2002, the RTC rendered judgment granting Malabanans
application for land registration, disposing thusly:
WHEREFORE, this Court hereby approves this application for registration and thus
places under the operation of Act 141, Act 496 and/or P.D. 1529, otherwise known as
Property Registration Law, the lands described in Plan Csd-04-0173123-D, Lot 9864-A
and containing an area of Seventy One Thousand Three Hundred Twenty Four (71,324)
Square Meters, as supported by its technical description now forming part of the record
of this case, in addition to other proofs adduced in the name of MARIO MALABANAN,
who is of legal age, Filipino, widower, and with residence at Munting Ilog, Silang, Cavite.
Once this Decision becomes final and executory, the corresponding decree of
Republic of the Philippines registration shall forthwith issue.
SUPREME COURT SO ORDERED.3
Manila The Office of the Solicitor General (OSG) appealed the judgment to the CA, arguing that
EN BANC Malabanan had failed to prove that the property belonged to the alienable and
G.R. No. 179987 September 3, 2013 disposable land of the public domain, and that the RTC erred in finding that he had
HEIRS OF MARIO MALABANAN, (Represented by Sally A. Malabanan), Petitioners, been in possession of the property in the manner and for the length of time required by
vs. law for confirmation of imperfect title.
REPUBLIC OF THE PHILIPPINES, Respondent. On February 23, 2007, the CA promulgated its decision reversing the RTC and
RESOLUTION dismissing the application for registration of Malabanan. Citing the ruling in Republic v.
BERSAMIN, J.: Herbieto (Herbieto),4 the CA declared that under Section 14(1) of the Property
For our consideration and resolution are the motions for reconsideration of the parties Registration Decree, any period of possession prior to the classification of the land as
who both assail the decision promulgated on April 29, 2009, whereby we upheld the alienable and disposable was inconsequential and should be excluded from the
ruling of the Court of Appeals (CA) denying the application of the petitioners for the computation of the period of possession. Noting that the CENRO-DENR certification
registration of a parcel of land situated in Barangay Tibig, Silang, Cavite on the ground stated that the property had been declared alienable and disposable only on March 15,
that they had not established by sufficient evidence their right to the registration in 1982, Velazcos possession prior to March 15, 1982 could not be tacked for purposes of
accordance with either Section 14(1) or Section 14(2) of Presidential Decree No. 1529 computing Malabanans period of possession.
(Property Registration Decree). Due to Malabanans intervening demise during the appeal in the CA, his heirs elevated
Antecedents the CAs decision of February 23, 2007 to this Court through a petition for review on
The property subject of the application for registration is a parcel of land situated in certiorari.
Barangay Tibig, Silang Cavite, more particularly identified as Lot 9864-A, Cad-452-D, The petitioners assert that the ruling in Republic v. Court of Appeals and Corazon
with an area of 71,324-square meters. On February 20, 1998, applicant Mario Naguit5 (Naguit) remains the controlling doctrine especially if the property involved is
agricultural land. In this regard, Naguit ruled that any possession of agricultural land Classifications of land according to ownership
prior to its declaration as alienable and disposable could be counted in the reckoning of Land, which is an immovable property, 10 may be classified as either of public dominion
the period of possession to perfect title under the Public Land Act (Commonwealth Act or of private ownership.11Land is considered of public dominion if it either: (a) is
No. 141) and the Property Registration Decree. They point out that the ruling in intended for public use; or (b) belongs to the State, without being for public use, and is
Herbieto, to the effect that the declaration of the land subject of the application for intended for some public service or for the development of the national wealth. 12 Land
registration as alienable and disposable should also date back to June 12, 1945 or belonging to the State that is not of such character, or although of such character but no
earlier, was a mere obiter dictum considering that the land registration proceedings longer intended for public use or for public service forms part of the patrimonial property
therein were in fact found and declared void ab initio for lack of publication of the notice of the State.13 Land that is other than part of the patrimonial property of the State,
of initial hearing. provinces, cities and municipalities is of private ownership if it belongs to a private
The petitioners also rely on the ruling in Republic v. T.A.N. Properties, Inc. 6 to support individual.
their argument that the property had been ipso jure converted into private property by Pursuant to the Regalian Doctrine (Jura Regalia), a legal concept first introduced into
reason of the open, continuous, exclusive and notorious possession by their the country from the West by Spain through the Laws of the Indies and the Royal
predecessors-in-interest of an alienable land of the public domain for more than 30 Cedulas,14 all lands of the public domain belong to the State. 15This means that the State
years. According to them, what was essential was that the property had been is the source of any asserted right to ownership of land, and is charged with the
"converted" into private property through prescription at the time of the application conservation of such patrimony.16
without regard to whether the property sought to be registered was previously classified All lands not appearing to be clearly under private ownership are presumed to belong to
as agricultural land of the public domain. the State. Also, public lands remain part of the inalienable land of the public domain
As earlier stated, we denied the petition for review on certiorari because Malabanan unless the State is shown to have reclassified or alienated them to private persons.17
failed to establish by sufficient evidence possession and occupation of the property on Classifications of public lands
his part and on the part of his predecessors-in interest since June 12, 1945, or earlier. according to alienability
Petitioners Motion for Reconsideration Whether or not land of the public domain is alienable and disposable primarily rests on
In their motion for reconsideration, the petitioners submit that the mere classification of the classification of public lands made under the Constitution. Under the 1935
the land as alienable or disposable should be deemed sufficient to convert it into Constitution,18 lands of the public domain were classified into three, namely,
patrimonial property of the State. Relying on the rulings in Spouses De Ocampo v. agricultural, timber and mineral.19 Section 10, Article XIV of the 1973 Constitution
Arlos,7 Menguito v. Republic8 and Republic v. T.A.N. Properties, Inc.,9 they argue that classified lands of the public domain into seven, specifically, agricultural, industrial or
the reclassification of the land as alienable or disposable opened it to acquisitive commercial, residential, resettlement, mineral, timber or forest, and grazing land, with
prescription under the Civil Code; that Malabanan had purchased the property from the reservation that the law might provide other classifications. The 1987 Constitution
Eduardo Velazco believing in good faith that Velazco and his predecessors-in-interest adopted the classification under the 1935 Constitution into agricultural, forest or timber,
had been the real owners of the land with the right to validly transmit title and ownership and mineral, but added national parks.20 Agricultural lands may be further classified by
thereof; that consequently, the ten-year period prescribed by Article 1134 of the Civil law according to the uses to which they may be devoted. 21 The identification of lands
Code, in relation to Section 14(2) of the Property Registration Decree, applied in their according to their legal classification is done exclusively by and through a positive act of
favor; and that when Malabanan filed the application for registration on February 20, the Executive Department.22
1998, he had already been in possession of the land for almost 16 years reckoned from Based on the foregoing, the Constitution places a limit on the type of public land that
1982, the time when the land was declared alienable and disposable by the State. may be alienated. Under Section 2, Article XII of the 1987 Constitution, only agricultural
The Republics Motion for Partial Reconsideration lands of the public domain may be alienated; all other natural resources may not be.
The Republic seeks the partial reconsideration in order to obtain a clarification with Alienable and disposable lands of the State fall into two categories, to wit: (a)
reference to the application of the rulings in Naguit and Herbieto. patrimonial lands of the State, or those classified as lands of private ownership under
Chiefly citing the dissents, the Republic contends that the decision has enlarged, by Article 425 of the Civil Code,23 without limitation; and (b) lands of the public domain, or
implication, the interpretation of Section 14(1) of the Property Registration Decree the public lands as provided by the Constitution, but with the limitation that the lands
through judicial legislation. It reiterates its view that an applicant is entitled to must only be agricultural. Consequently, lands classified as forest or timber, mineral, or
registration only when the land subject of the application had been declared alienable national parks are not susceptible of alienation or disposition unless they are
and disposable since June 12, 1945 or earlier. reclassified as agricultural.24 A positive act of the Government is necessary to enable
Ruling such reclassification,25 and the exclusive prerogative to classify public lands under
We deny the motions for reconsideration. existing laws is vested in the Executive Department, not in the courts. 26 If, however,
In reviewing the assailed decision, we consider to be imperative to discuss the different public land will be classified as neither agricultural, forest or timber, mineral or national
classifications of land in relation to the existing applicable land registration laws of the park, or when public land is no longer intended for public service or for the development
Philippines. of the national wealth, thereby effectively removing the land from the ambit of public
dominion, a declaration of such conversion must be made in the form of a law duly 2. The possession and occupation must be open, continuous, exclusive, and notorious;
enacted by Congress or by a Presidential proclamation in cases where the President is 3. The possession and occupation must be under a bona fide claim of acquisition of
duly authorized by law to that effect.27 Thus, until the Executive Department exercises ownership;
its prerogative to classify or reclassify lands, or until Congress or the President declares 4. The possession and occupation must have taken place since June 12, 1945, or
that the State no longer intends the land to be used for public service or for the earlier; and
development of national wealth, the Regalian Doctrine is applicable. 5. The property subject of the application must be an agricultural land of the public
Disposition of alienable public lands domain.
Section 11 of the Public Land Act (CA No. 141) provides the manner by which alienable Taking into consideration that the Executive Department is vested with the authority to
and disposable lands of the public domain, i.e., agricultural lands, can be disposed of, to classify lands of the public domain, Section 48(b) of the Public Land Act, in relation to
wit: Section 14(1) of the Property Registration Decree, presupposes that the land subject of
Section 11. Public lands suitable for agricultural purposes can be disposed of only as the application for registration must have been already classified as agricultural land of
follows, and not otherwise: the public domain in order for the provision to apply. Thus, absent proof that the land is
(1) For homestead settlement; already classified as agricultural land of the public domain, the Regalian Doctrine
(2) By sale; applies, and overcomes the presumption that the land is alienable and disposable as
(3) By lease; and laid down in Section 48(b) of the Public Land Act. However, emphasis is placed on the
(4) By confirmation of imperfect or incomplete titles; requirement that the classification required by Section 48(b) of the Public Land Act is
(a) By judicial legalization; or classification or reclassification of a public land as agricultural.
(b) By administrative legalization (free patent). The dissent stresses that the classification or reclassification of the land as alienable
The core of the controversy herein lies in the proper interpretation of Section 11(4), in and disposable agricultural land should likewise have been made on June 12, 1945 or
relation to Section 48(b) of the Public Land Act, which expressly requires possession by earlier, because any possession of the land prior to such classification or reclassification
a Filipino citizen of the land since June 12, 1945, or earlier, viz: produced no legal effects. It observes that the fixed date of June 12, 1945 could not be
Section 48. The following-described citizens of the Philippines, occupying lands of the minimized or glossed over by mere judicial interpretation or by judicial social policy
public domain or claiming to own any such lands or an interest therein, but whose titles concerns, and insisted that the full legislative intent be respected.
have not been perfected or completed, may apply to the Court of First Instance of the We find, however, that the choice of June 12, 1945 as the reckoning point of the
province where the land is located for confirmation of their claims and the issuance of a requisite possession and occupation was the sole prerogative of Congress, the
certificate of title thereafter, under the Land Registration Act, to wit: determination of which should best be left to the wisdom of the lawmakers. Except that
xxxx said date qualified the period of possession and occupation, no other legislative intent
(b) Those who by themselves or through their predecessors-in-interest have been in appears to be associated with the fixing of the date of June 12, 1945. Accordingly, the
open, continuous, exclusive, and notorious possession and occupation of alienable and Court should interpret only the plain and literal meaning of the law as written by the
disposable lands of the public domain, under a bona fide claim of acquisition of legislators.
ownership, since June 12, 1945, or earlier, immediately preceding the filing of the Moreover, an examination of Section 48(b) of the Public Land Act indicates that
applications for confirmation of title, except when prevented by war or force majeure. Congress prescribed no requirement that the land subject of the registration should
These shall be conclusively presumed to have performed all the conditions essential to have been classified as agricultural since June 12, 1945, or earlier. As such, the
a Government grant and shall be entitled to a certificate of title under the provisions of applicants imperfect or incomplete title is derived only from possession and occupation
this chapter. (Bold emphasis supplied) since June 12, 1945, or earlier. This means that the character of the property subject of
Note that Section 48(b) of the Public Land Act used the words "lands of the public the application as alienable and disposable agricultural land of the public domain
domain" or "alienable and disposable lands of the public domain" to clearly signify that determines its eligibility for land registration, not the ownership or title over it.
lands otherwise classified, i.e., mineral, forest or timber, or national parks, and lands of Alienable public land held by a possessor, either personally or through his
patrimonial or private ownership, are outside the coverage of the Public Land Act. What predecessors-in-interest, openly, continuously and exclusively during the prescribed
the law does not include, it excludes. The use of the descriptive phrase "alienable and statutory period is converted to private property by the mere lapse or completion of the
disposable" further limits the coverage of Section 48(b) to only the agricultural lands of period.29 In fact, by virtue of this doctrine, corporations may now acquire lands of the
the public domain as set forth in Article XII, Section 2 of the 1987 Constitution. Bearing public domain for as long as the lands were already converted to private ownership, by
in mind such limitations under the Public Land Act, the applicant must satisfy the operation of law, as a result of satisfying the requisite period of possession prescribed
following requirements in order for his application to come under Section 14(1) of the by the Public Land Act.30 It is for this reason that the property subject of the application
Property Registration Decree,28 to wit: of Malabanan need not be classified as alienable and disposable agricultural land of the
1. The applicant, by himself or through his predecessor-in-interest, has been in public domain for the entire duration of the requisite period of possession.
possession and occupation of the property subject of the application;
To be clear, then, the requirement that the land should have been classified as of private ownership that may be alienated or disposed through any of the modes of
alienable and disposable agricultural land at the time of the application for registration is acquiring ownership under the Civil Code. If the mode of acquisition is prescription,
necessary only to dispute the presumption that the land is inalienable. whether ordinary or extraordinary, proof that the land has been already converted to
The declaration that land is alienable and disposable also serves to determine the point private ownership prior to the requisite acquisitive prescriptive period is a condition sine
at which prescription may run against the State. The imperfect or incomplete title being qua non in observance of the law (Article 1113, Civil Code) that property of the State not
confirmed under Section 48(b) of the Public Land Act is title that is acquired by reason patrimonial in character shall not be the object of prescription.
of the applicants possession and occupation of the alienable and disposable To reiterate, then, the petitioners failed to present sufficient evidence to establish that
agricultural land of the public domain. Where all the necessary requirements for a grant they and their predecessors-in-interest had been in possession of the land since June
by the Government are complied with through actual physical, open, continuous, 12, 1945. Without satisfying the requisite character and period of possession -
exclusive and public possession of an alienable and disposable land of the public possession and occupation that is open, continuous, exclusive, and notorious since
domain, the possessor is deemed to have acquired by operation of law not only a right June 12, 1945, or earlier - the land cannot be considered ipso jure converted to private
to a grant, but a grant by the Government, because it is not necessary that a certificate property even upon the subsequent declaration of it as alienable and disposable.
of title be issued in order that such a grant be sanctioned by the courts. 31 Prescription never began to run against the State, such that the land has remained
If one follows the dissent, the clear objective of the Public Land Act to adjudicate and ineligible for registration under Section 14(1) of the Property Registration Decree.
quiet titles to unregistered lands in favor of qualified Filipino citizens by reason of their Likewise, the land continues to be ineligible for land registration under Section 14(2) of
occupation and cultivation thereof for the number of years prescribed by law32 will be the Property Registration Decree unless Congress enacts a law or the President issues
defeated. Indeed, we should always bear in mind that such objective still prevails, as a a proclamation declaring the land as no longer intended for public service or for the
fairly recent legislative development bears out, when Congress enacted legislation development of the national wealth.1wphi1
(Republic Act No. 10023)33in order to liberalize stringent requirements and procedures WHEREFORE, the Court DENIES the petitioners' Motion for Reconsideration and the
in the adjudication of alienable public land to qualified applicants, particularly residential respondent's Partial Motion for Reconsideration for their lack of merit.
lands, subject to area limitations.34 SO ORDERED.
On the other hand, if a public land is classified as no longer intended for public use or
for the development of national wealth by declaration of Congress or the President,
thereby converting such land into patrimonial or private land of the State, the applicable
provision concerning disposition and registration is no longer Section 48(b) of the Public
Land Act but the Civil Code, in conjunction with Section 14(2) of the Property
Registration Decree.35 As such, prescription can now run against the State.
To sum up, we now observe the following rules relative to the disposition of public land
or lands of the public domain, namely:
(1) As a general rule and pursuant to the Regalian Doctrine, all lands of the public
domain belong to the State and are inalienable. Lands that are not clearly under private
ownership are also presumed to belong to the State and, therefore, may not be
alienated or disposed;
(2) The following are excepted from the general rule, to wit:
(a) Agricultural lands of the public domain are rendered alienable and disposable
through any of the exclusive modes enumerated under Section 11 of the Public Land
Act. If the mode is judicial confirmation of imperfect title under Section 48(b) of the
Public Land Act, the agricultural land subject of the application needs only to be
classified as alienable and disposable as of the time of the application, provided the
applicants possession and occupation of the land dated back to June 12, 1945, or
earlier. Thereby, a conclusive presumption that the applicant has performed all the
conditions essential to a government grant arises, 36 and the applicant becomes the
owner of the land by virtue of an imperfect or incomplete title. By legal fiction, the land
has already ceased to be part of the public domain and has become private property. 37
(b) Lands of the public domain subsequently classified or declared as no longer
intended for public use or for the development of national wealth are removed from the
sphere of public dominion and are considered converted into patrimonial lands or lands
Significantly, on November 28, 1975, this Court already ruled in Director of Lands v.
Reyes[4] that the property subject of Garcias application was inalienable as it formed
part of a military reservation. Moreover, the existence of Possessory Information Title
No. 216 (allegedly registered in the name of a certain Melecio Padilla on March 5,
1895), on which therein respondent Paraaque Investment and Development
THIRD DIVISION Corporation anchored its claim on the land, was not proven. Accordingly, the decree of
registration issued in its favor was declared null and void.
FLORENCIA G. DIAZ, G.R. No. 181502
Reyes notwithstanding, the CFI ruled in Garcias favor in a decision[5] dated July 1,
Petitioner,
1981.
Present:
The Republic eventually appealed the decision of the CFI to the Court of Appeals (CA).
CORONA, J., Chairperson,
- v e r s u s - CARPIO , In its decision[6] dated February 26, 1992, penned by Justice Vicente V. Mendoza
(Mendoza decision),[7] the appellate court reversed and set aside the decision of the
VELASCO, JR.,
CFI. The CA found that Reyes was applicable to petitioners case as it involved the
NACHURA and
same property.
PERALTA, JJ.

REPUBLIC of the PHILIPPINES, The CA observed that Garcia also traced her ownership of the land in question to
Respondent. Possessory Information Title No. 216. As Garcias right to the property was largely
dependent on the existence and validity of the possessory information title the probative
Promulgated:
value of which had already been passed upon by this Court in Reyes, and inasmuch as
February 2, 2010
the land was situated inside a military reservation, the CA concluded that she did not
x - - - - - - - - - - - - - - - - - - - - - - - - - - - - - - - - - - - - - - - - - - - - - - - - - - -x validly acquire title thereto.
During the pendency of the case in the CA, Garcia passed away and was substituted by
RESOLUTION her heirs, one of whom was petitioner Florencia G. Diaz.[8]
CORONA, J.:
Petitioner filed a motion for reconsideration of the Mendoza decision. While the motion
was pending in the CA, petitioner also filed a motion for recall of the records from the
former CFI. Without acting on the motion for reconsideration, the appellate court, with
This is a letter-motion praying for reconsideration (for the third time) of the June 16,
Justice Mendoza as ponente, issued a resolution[9] upholding petitioners right to recall
2008 resolution of this Court denying the petition for review filed by petitioner Florencia
the records of the case.
G. Diaz.

Subsequently, however, the CA encouraged the parties to reach an amicable settlement


Petitioners late mother, Flora Garcia (Garcia), filed an application for registration of a
on the matter and even gave the parties sufficient time to draft and finalize the same.
vast tract of land[1] located in Laur, Nueva Ecija and Palayan City in the then Court of
First Instance (CFI), Branch 1, Nueva Ecija on August 12, 1976.[2] She alleged that she
The parties ultimately entered into a compromise agreement with the Republic
possessed the land as owner and worked, developed and harvested the agricultural
withdrawing its claim on the more or less 4,689 hectares supposedly outside the FMMR.
products and benefits of the same continuously, publicly and adversely for more or less
For her part, petitioner withdrew her application for the portion of the property inside the
26 years.
military reservation. They filed a motion for approval of the amicable settlement in the
CA.[10]
The Republic of the Philippines, represented by the Office of the Solicitor General
(OSG), opposed the application because the land in question was within the Fort
On June 30, 1999, the appellate court approved the compromise agreement.[11] On
Magsaysay Military Reservation (FMMR), established by virtue of Proclamation No. 237
January 12, 2000, it directed the Land Registration Administration to issue the
(Proclamation 237)[3] in 1955. Thus, it was inalienable as it formed part of the public
corresponding decree of registration in petitioners favor.[12]
domain.
However, acting on a letter written by a certain Atty. Restituto S. Lazaro, the OSG filed should have inhibited himself when the case reached the CA. His failure to do so, she
a motion for reconsideration of the CA resolution ordering the issuance of the decree of laments, worked an injustice against her constitutional right to due process. Thus, the
registration. The OSG informed the appellate court that the tract of land subject of the Mendoza decision should be declared null and void. The motion was denied.[14]
amicable settlement was still within the military reservation.
Thereafter, petitioner filed a petition for review on certiorari[15] in this Court. It was
On April 16, 2007, the CA issued an amended resolution (amended denied for raising factual issues.[16] She moved for reconsideration.[17] This motion
resolution)[13] annulling the compromise agreement entered into between the parties. was denied with finality on the ground that there was no substantial argument
The relevant part of the dispositive portion of the resolution read: warranting a modification of the Courts resolution. The Court then ordered that no
ACCORDINGLY, the Court resolves to: further pleadings would be entertained. Accordingly, we ordered entry of judgment to be
(1) x x x x x x made in due course.[18]
(2) x x x x x x Petitioner, however, insisted on filing a motion to lift entry of judgment and motion for
(3) x x x x x x leave to file a second motion for reconsideration and to refer the case to the Supreme
(4) x x x x x x Court en banc.[19] The Court denied[20] it considering that a second motion for
(5) x x x x x x reconsideration is a prohibited pleading.[21] Furthermore, the motion to refer the case to
(6) REVERSE the Resolution dated June 30, 1999 of this Court the banc was likewise denied as the banc is not an appellate court to which decisions or
approving the Amicable Settlement dated May 18, 1999 executed resolutions of the divisions may be appealed.[22] We reiterated our directive that no
between the Office of the Solicitor General and Florencia Garcia further pleadings would be entertained and that entry of judgment be made in due
Diaz[;] course.
(7) ANNUL and SET ASIDE the Amicable Settlement dated May
18, 1999 executed between the Office of the Solicitor General and Not one to be easily deterred, petitioner wrote identical letters, first addressed to Justice
Florencia Garcia Diaz; the said Amicable Settlement is Leonardo A. Quisumbing (then Acting Chief Justice) and then to Chief Justice Reynato
hereby DECLARED to be without force and effect; S. Puno himself.[23] The body of the letter, undoubtedly in the nature of a third motion
(8) GRANT the Motion for Reconsideration filed by the Office of for reconsideration, is hereby reproduced in its entirety:
the Solicitor General and, consequently, SET ASIDE the Resolution
dated January 12, 2000 which ordered, among other matters, that a This is in response to your call for Moral Forces in order to redirect
certificate of title be issued in the name of plaintiff-appellee the destiny of our country which is suffering from moral decadence,
Florencia Garcia Diaz over the portion of the subject property in that to your mind, is the problem which confronts us. (Inquirer,
consonance with the Amicable Settlement dated May 18, 1999 January 15, 2009, page 1)[.]
approved by the Court in its Resolution dated June 30, 1999;
(9) SET ASIDE the Resolution dated June 30, 1999 approving the I recently lost my case with the Supreme Court, G.R. N[o]. 181502,
May 18, 1999 Amicable Settlement and the Resolution dated and my lawyer has done all that is humanly possible to convince the
September 20, 1999 amending the aforesaid June 30, 1999 court to take a second look at the miscarriage of justice that will result
Resolution; and from the implementation of the DISMISSAL in a MINUTE
(10) REINSTATE the Decision dated February 26, 1992 RESOLUTION of our Petition for Review.
dismissing applicant-appellee Diaz registration herein.
Pending before your Division (First Division) is a last plea for
SO ORDERED. justice so that the case may be elevated to the Supreme Court en
banc. I hope the Court exercises utmost prudence in resolving
(Emphasis supplied) the last plea. For ready reference, a copy of the Motion is hereto
attached as Annex A.

Petitioner moved for reconsideration. For the first time, she assailed the validity of the The issue that was brought before the Honorable Supreme Court
Mendoza decision the February 26, 1992 decision adverted to in the CAs amended involves the Decision of then Justice Vicente Mendoza of the Court of
resolution. She alleged that Justice Mendoza was the assistant solicitor general during Appeals, which is NULL and VOID, ab initio.
the initial stages of the land registration proceedings in the trial court and therefore
It is null and void because destiny placed Hon. Justice Vicente honor an otherwise legally infirm compromise agreement, at the risk of being vilified in
Mendoza in a position in which it became possible for him to the media and by the public.
discharge the minimum requirement of due process, [i.e.] the ability of
the court to render impartial justice, because Mr. Justice Mendoza This Court will not be cowed into submission. We deny petitioners letter/third motion for
became the ponente of the Court of Appeals Decision, reversing the reconsideration.
findings of the trial court, notwithstanding the fact that he, as Assistant
Solicitor General, was the very person who appeared on behalf of the APPLICABILITY
Republic, as the oppositor in the very same land registration OF REYES
proceedings in which he lost.

In other words, he discharged the duties of prosecutor and judge in The Court agrees with the Republics position that Reyes is applicable to this case.
the very same case.
To constitute res judicata, the following elements must concur:
In the case of the Alabang Boys[,] the public was outraged by the (1) the former judgment or order must be final;
actions of Atty. Verano who admitted having prepared a simple (2) the judgment or order must be on the merits;
resolution to be signed by the Secretary of Justice. (3) it must have been rendered by a court
having jurisdiction over the subject matter and
In my case, the act complained of is the worst kind of violation of my parties; and
constitutional right. It is simply immoral, illegal and unconstitutional, (4) there must be between the first and
for the prosecutor to eventually act as the judge, and reverse the very second actions, identity of parties, of subject
decision in which he had lost. matter, and of causes of action. [24]

If leaked to the tri-media[,] my case will certainly evoke even greater The first three requisites have undoubtedly been complied with. However, petitioner
spite from the public, and put the Supreme Court in bad light. I must takes exception to the fourth requisite, particularly on the issue of identity of parties. In
confess that I was tempted to pursue such course of action. I however her petition for review filed in this Court, she contends that since the applicants in the
believe that such an action will do more harm than good, and even two cases are different, the merits of the two cases should, accordingly, be determined
destroy the good name of Hon. Justice Mendoza. independently of each other.[25]
This contention is erroneous.
I fully support your call for moral force that will slowly and eventually
lead our country to redirect its destiny and escape from this moral The facts obtaining in this case closely resemble those in Aquino v. Director of
decadence, in which we all find ourselves. Lands.[26] In that case, Quintin Taedo endeavored to secure title to a considerable tract
of land by virtue of his possession thereof under CA 141. When the case eventually
I am content with the fact that at least, the Chief Justice continues to reached this Court, we affirmed the trial courts decision to dismiss the proceedings as
fight the dark forces that surround us everyday. the property in question was part of the public domain. Quintins successor-in-interest,
Florencia Taedo, who despite knowledge of the proceedings did not participate therein,
I only ask that the Supreme Court endeavor to ensure that cases thereafter sold the same property to Benigno S. Aquino. The latter sought to have it
such as mine do not happen again, so that the next person who registered in his name. The question in that case, as well as in this one, was whether
seeks justice will not experience the pain and frustration that I our decision in the case in which another person was the applicant constituted res
suffered under our judicial system. judicataas against his successors-in-interest.

Thank you, and more power to you, SIR. (Emphasis in the original). We ruled there, and we so rule now, that in registration cases filed under the provisions
of the Public Land Act for the judicial confirmation of an incomplete and imperfect title,
an order dismissing an application for registration and declaring the land as part of the
The language of petitioners letter/motion is unmistakable. It is a thinly veiled threat public domain constitutes res judicata, not only against the adverse claimant, but also
precisely worded and calculated to intimidate this Court into giving in to her demands to against all persons.[27]
By way of a background, we recognized in Reyes that the property where the military
We also declared in Aquino that: reservation is situated is forest land. Thus:

From another point of view, the decision in the first action has Before the military reservation was established, the evidence is
become the law of the case or at least falls within the rule of stare inconclusive as to possession, for it is shown by the evidence that the
decisis. That adjudication should be followed unless manifestly land involved is largely mountainous and forested. As a matter of
erroneous. It was taken and should be taken as the authoritative view fact, at the time of the hearing, it was conceded that approximately
of the highest tribunal in the Philippines. It is indispensable to the due 13,957 hectares of said land consist of public forest. x x x
administration of justice especially by a court of last resort that a (Emphasis supplied)[32]
question once deliberately examined and decided should be
considered as settled and closed to further argument. x x x[28]
Concomitantly, we stated therein, and we remind petitioner now, that forest lands are
Be that as it may, the fact is that, even before the CFI came out with its decision in favor not registrable under CA 141.
of petitioner on July 1, 1981, this Court, in Reyes, already made an earlier ruling on
November 28, 1975 that the disputed realty was inalienable as it formed part of a [E]ven more important, Section 48[b] of CA No. 141, as amended,
military reservation. Thus, petitioners argument that the findings of fact of the trial court applies exclusively to public agricultural land. Forest lands or area
on her registrable title are binding on us on the principle that findings of fact of lower covered with forest are excluded. It is well-settled that forest land
courts are accorded great respect and bind even this Court is untenable. Rather, it was is incapable of registration; and its inclusion in a title, whether
incumbent upon the court a quo to respect this Courts ruling in Reyes, and not the other such title be one issued using the Spanish sovereignty or under
way around. the present Torrens system of registration, nullifies the
title. (Emphasis supplied).[33]
However, despite having been apprised of the Court's findings in Reyes (which should
have been a matter of judicial notice in the first place), the trial court still insisted on its
divergent finding and disregarded the Court's decision in Reyes, declaring the subject However, it is true that forest lands may be registered when they have been reclassified
land as forming part of a military reservation, and thus outside the commerce of man. as alienable by the President in a clear and categorical manner (upon the
recommendation of the proper department head who has the authority to classify the
By not applying our ruling in Reyes, the trial judge virtually nullified the decision of this lands of the public domain into alienable or disposable, timber and mineral
Court and therefore acted with grave abuse of discretion.[29] Notably, a judgment lands)[34] coupled with possession by the claimant as well as that of her predecessors-
rendered with grave abuse of discretion is void and does not exist in legal in-interest. Unfortunately for petitioner, she was not able to produce such evidence.
contemplation.[30] Accordingly, her occupation thereof, and that of her predecessors-in-interest, could not
have ripened into ownership of the subject land. This is because prior to the conversion
All lower courts, especially the trial court concerned in this case, ought to be reminded of forest land as alienable land, any occupation or possession thereof cannot be
that it is their duty to obey the decisions of the Supreme Court. A conduct becoming of counted in reckoning compliance with the thirty-year possession requirement under
inferior courts demands a conscious awareness of the position they occupy in the Commonwealth Act 141 (CA 141) or the Public Land Act.[35] This was our ruling
interrelation and operation of our judicial system. As eloquently declared by Justice J.B. in Almeda v. CA.[36] The rules on the confirmation of imperfect titles do not apply
L. Reyes, "There is only one Supreme Court from whose decision all other courts unless and until the land classified as forest land is released through an official
should take their bearings."[31] proclamation to that effect. Then and only then will it form part of the disposable
agricultural lands of the public domain.[37]
ACQUISITION OF
PRIVATE RIGHTS Coming now to petitioners contention that her private rights to the property, meaning her
and her predecessors possession thereof prior to the establishment of the FMMR, must
be respected, the same is untenable. As earlier stated, we had already recognized the
Petitioner, however, argues that Proclamation 237 itself recognizes that its effectivity is same land to be public forest even before the FMMR was established. To reiterate:
subject to private rights, if any there be.
Before the military reservation was established, the evidence is xxx
inconclusive as to possession, for it is shown by the evidence that the
land involved is largely mountainous and forested. As a matter of fact,
at the time of the hearing, it was conceded that approximately 13,957 Furthermore, the fact that the possessory information title on which petitioner also
hectares of said land consist of public forest. x x x bases her claim of ownership was found to be inexistent in Reyes,[39] thus rendering its
probative value suspect, further militates against granting her application for
registration.
Therefore, even if possession was for more than 30 years, it could never ripen to
ownership. NULLITY OF COMPROMISE
But even assuming that the land in question was alienable land before it was AGREEMENT
established as a military reservation, there was nevertheless still a dearth of evidence
with respect to its occupation by petitioner and her predecessors-in-interest for more
than 30 years. In Reyes, we noted: On the compromise agreement between the parties, we agree with the CA that the
same was null and void.
Evidently, Melecio Padilla, having died on February 9, 1900, barely
five (5) years after the inscription of the informacion possessoria, An amicable settlement or a compromise agreement is in the nature of a contract and
could not have converted the same into a record of ownership twenty must necessarily comply with the provisions of Article 1318 of the New Civil Code which
(20) years after such inscription, pursuant to Article 393 of the provides:
Spanish Mortgage Law.
Art. 1318. There is no contract unless the following requisites concur:
xxx (1) Consent of the contracting parties;
(2) Object certain which is the subject matter of the contract;
During the lifetime of Melecio Padilla, only a small portion thereof was (3) Cause of the obligation which is established.
cleared and cultivated under the kaingin system, while some portions
were used as grazing land. After his death, his daughter, Maria Petitioner was not able to provide any proof that the consent of the Republic, through
Padilla, caused the planting of vegetables and had about forty (40) the appropriate government agencies, i.e. the Department of Environment and Natural
tenants for the purpose. During the Japanese occupation, Maria Resources, Land Management Bureau, Land Registration Authority, and the Office of
Padilla died. x x x the President, was secured by the OSG when it executed the agreement with
xxx her.[40] The lack of authority on the part of the OSG rendered the compromise
agreement between the parties null and void because although it is the duty of the OSG
A mere casual cultivation of portions of the land by the claimant, and to represent the State in cases involving land registration proceedings, it must do so
the raising thereon of cattle, do not constitute possession under claim only within the scope of the authority granted to it by its principal, the Republic of the
of ownership. In that sense, possession is not exclusive and notorious Philippines.[41]
as to give rise to a presumptive grant from the State. While grazing
livestock over land is of course to be considered with other acts of In this case, although the OSG was authorized to appear as counsel for respondent, it
dominion to show possession, the mere occupancy of land by grazing was never given the specific or special authority to enter into a compromise agreement
livestock upon it, without substantial inclosures, or other permanent with petitioner. This is in violation of the provisions of Rule 138 Section 23, of the Rules
improvements, is not sufficient to support a claim of title thru of Court which requires special authority for attorneys to bind their clients.
acquisitive prescription. The possession of public land, however long
the period may have extended, never confers title thereto upon the Section 23. Authority of attorneys to bind clients. Attorneys have
possessor because the statute of limitations with regard to public land authority to bind their clients in any case by any agreement in relation
does not operate against the State unless the occupant can prove thereto made in writing, and in taking appeals, and in all matters of
possession and occupation of the same under claim of ownership for ordinary judicial procedure. But they cannot, without special
the required number of years to constitute a grant from the State.[38] authority, compromise their clients litigation, or receive anything
in discharge of a clients claim but the full amount in cash. (Emphasis
supplied). I recently lost my case with the Supreme Court, G.R. N[o]. 181502,
and my lawyer has done all that is humanly possible to convince the
court to take a second look at the miscarriage of justice that will
Moreover, the land in question could not have been a valid subject matter of a contract result from the implementation of the DISMISSAL in a MINUTE
because, being forest land, it was inalienable. Article 1347 of the Civil Code provides: RESOLUTION of our Petition for Review.

Art. 1347. All things which are not outside the commerce of men, Pending before your Division (First Division) is a last plea for
including future things, may be the object of a contract. All rights justice so that the case may be elevated to the Supreme Court en
which are not intransmissible may also be the object of contracts. banc. I hope the Court exercises utmost prudence in resolving
No contract may be entered into upon future inheritance except in the last plea. For ready reference, a copy of the Motion is hereto
cases expressly authorized by law. attached as Annex A.
All services which are not contrary to law, morals, good customs,
public order or public policy may likewise be the object of a contract. The issue that was brought before the Honorable Supreme Court
(Emphasis supplied) involves the Decision of then Justice Vicente Mendoza of the Court of
Appeals, which is NULL and VOID, ab initio.

Finally, the Court finds the cause or consideration of the obligation contrary to law and It is null and void because destiny placed Hon. Justice Vicente
against public policy. The agreement provided that, in consideration of petitioners Mendoza in a position in which it became possible for him to
withdrawal of her application for registration of title from that portion of the property discharge the minimum requirement of due process, [i.e.] the ability of
located within the military reservation, respondent was withdrawing its claim on that part the court to render impartial justice, because Mr. Justice Mendoza
of the land situated outside said reservation. The Republic could not validly enter into became the ponente of the Court of Appeals Decision, reversing the
such undertaking as the subject matter of the agreement was outside the commerce of findings of the trial court, notwithstanding the fact that he, as Assistant
man. Solicitor General, was the very person who appeared on behalf of the
Republic, as the oppositor in the very same land registration
proceedings in which he lost. (Emphasis supplied).
PETITIONERS CONTEMPT
OF COURT
Petitioner then indirectly hints that, when push comes to shove, she has no choice but
to expose the irregularity concerning the Mendoza decision to the media. This is evident
This Court, being the very institution that dispenses justice, cannot reasonably be in her arrogant declaration that:
expected to just sit by and do nothing when it comes under attack.
If leaked to the tri-media[,] my case will certainly evoke even greater
That petitioners letter-motion constitutes an attack against the integrity of this Court spite from the public, and put the Supreme Court in bad light.
cannot be denied. Petitioner started her letter innocently enough by stating:
But she hastens to add in the same breath that:
This is in response to your call for Moral Forces in order to redirect I must confess that I was tempted to pursue such course of action. I
the destiny of our country which is suffering from moral decadence, however believe that such an action will do more harm than good,
that to your mind, is the problem which confronts us. (Inquirer, and even destroy the good name of Hon. Justice Mendoza.
January 15, 2009, page 1)[.]

Petitioner ends her letter by taking this Court to task:


It, however, quickly progressed into a barely concealed resentment for what she
perceived as this Courts failure to exercise utmost prudence in rendering impartial
justice in deciding her case. Petitioner recounted:
. . . endeavor to ensure that cases such as mine do not happen again, upon and sufficiently addressed by the appellate court and this Court in their respective
so that the next person who seeks justice will not experience the pain resolutions.
and frustration that I suffered under our judicial system.
As to petitioners complaint regarding this Courts denial of her petition through a mere
minute resolution (which allegedly deprived her of due process as the Court did not
When required to show cause why she should not be cited for contempt for her issue a full-blown decision stating the facts and applicable jurisprudence), suffice it to
baseless charges and veiled threats, petitioner answered: say that the Court is not duty-bound to issue decisions or resolutions signed by the
justices all the time. It has ample discretion to formulate ponencias, extended
xxx resolutions or even minute resolutions issued by or upon its authority, depending on its
evaluation of a case, as long as a legal basis exists. When a minute resolution (signed
The Letter of January 26, 2009 is not a veiled threat[.] It was written in by the Clerk of Court upon orders of the Court) denies or dismisses a petition or motion
response to the call of the Chief Justice for a moral revolution. for reconsideration for lack of merit, it is understood that the assailed decision or order,
Juxtaposed against the factual backdrop of the Alabang Boys case together with all its findings of fact and legal conclusions, are deemed sustained.[42]
and the Meralco [c]ase, involving Mr. Justice Jose L. Sabio which also Furthermore, petitioner has doggedly pursued her case in this Court by filing three
enjoyed wide publicity over the tri-media, petitioner felt that the facts successive motions for reconsideration, including the letter-motion subject of this
of the said cases pale in comparison to the facts of her case where resolution. This, despite our repeated warnings that no further pleadings shall be
the lawyer of her opponent eventually became justice of the appellate entertained in this case. Her unreasonable persistence constitutes utter defiance of this
court and ended up reversing the very decision in which he lost, in Courts orders and an abuse of the rules of procedure. This, alongside her thinly veiled
clear violation of her [c]onstitutional [r]ight to fundamental fair play for threats to leak her case to the media to gain public sympathy although the tone of
no contestant in any litigation can ever serve as a judge without petitioners compliance with our show-cause resolution was decidedly subdued
transgression of the due process clause. This is basic. compared to her earlier letters constitutes contempt of court.

Petitioner confesses that she may have been emotional in the In Republic v. Unimex,[43] we held:
delivery of her piece, because correctly or incorrectly[,] she believes
they are irrefutable. If in the course of that emotional delivery, she has A statement of this Court that no further pleadings would be
offended your honors sensibilities, she is ready for the punishment, entertained is a declaration that the Court has already considered all
and only prays that his Court temper its strike with compassion as her issues presented by the parties and that it has adjudicated the case
letter to the Chief Justice was never written with a view of threatening with finality. It is a directive to the parties to desist from filing any
the Court. further pleadings or motions. Like all orders of this Court, it must be
strictly observed by the parties. It should not be circumvented by filing
xxx motions ill-disguised as requests for clarification.

Petitioner wrote the Chief Justice in order to obtain redress and


correction of the inequity bestowed upon her by destiny. It was never A FEW OBSERVATIONS
meant as a threat.
If petitioner was, as she adamantly insists, only guarding her constitutional right to due
process, then why did she question the validity of the Mendoza decision late in the
The Court now puts an end to petitioners irresponsible insinuations and threats of going proceedings, that is, only after her motion for reconsideration in the CA (for its
public with this case. We are not blind to petitioners clever and foxy interplay of threats subsequent annulment of the compromise agreement) was denied? It is obvious that it
alternating with false concern for the reputation of this Court. was only when her case became hopeless that her present counsel frantically searched
for some ground, any ground to resuscitate his clients lost cause, subsequently raising
It is well to remind petitioner that the Court has consistently rendered justice with neither the issue. This is evident from a statement in her petition to this Court that:
fear nor favor. The disposition in this case was arrived at after a careful and thorough
deliberation of the facts of this case and all the matters pertaining thereto. The records It is this fresh discovery by the undersigned counsel of the
of the case, in fact, show that all the pertinent issues raised by petitioner were passed nullity of the proceedings of the Court of Appeals that places in
doubt the entire proceedings it previously conducted, which led to the DECISION
rendition of the February 26, 1992 Decision, a fact that escaped the
BRION, J.:
scrutiny of applicant for registration Flora L. Garcia, as well as
her lawyer, Atty. Cayetano Dante Diaz, who died in 1993, and the Before the Court is a petition for review on certiorari1 filed by the petitioners, spouses
late Justice Fernando A. Santiago, who stood as counsel Antonio and Erlinda Fortuna, assailing the decision dated May 16, 20052 and the
for Flora L. Garcias successor-in-interest, herein petitioner, resolution dated June 27, 20063 of the Court of Appeals (CA) in CA-G.R. CV No.
Florencia G. Garcia.[44] (Emphasis supplied). 71143. The CA reversed and set aside the decision dated May 7, 20014 of the Regional
Trial Court (RTC) of San Fernando, La Union, Branch 66, in Land Registration Case
The above cited statement does not help petitioners cause at all. If anything, it only (LRC) No. 2372.
proves how desperate the case has become for petitioner and her counsel.
THE BACKGROUND FACTS
WHEREFORE, the letter-motion dated January 26, 2009 of petitioner is NOTED and is In December 1994, the spouses Fortuna filed an application for registration of a 2,597-
hereby treated as a third motion for reconsideration. The motion is DENIED considering square meter land identified as Lot No. 4457, situated in Bo. Canaoay, San Fernando,
that a third motion for reconsideration is a prohibited pleading and the plea utterly lacks La Union. The application was filed with the RTC and docketed as LRC No. 2372.
merit.
The spouses Fortuna stated that Lot No. 4457 was originally owned by Pastora
Petitioner is found GUILTY of contempt of court. Accordingly, a FINE of Five Thousand Vendiola, upon whose death was succeeded by her children, Clemente and Emeteria
Pesos is hereby imposed on her, payable within ten days from receipt of this resolution. Nones. Through an affidavit of adjudication dated August 3, 1972, Emeteria renounced
She is hereby WARNED that any repetition hereof shall be dealt with more severely. all her interest in Lot No. 4457 in favor of Clemente. Clemente later sold the lot in favor
of Rodolfo Cuenca on May 23, 1975. Rodolfo sold the same lot to the spouses Fortuna
through a deed of absolute sale dated May 4, 1984.
Treble costs against petitioner.
The spouses Fortuna claimed that they, through themselves and their predecessors-in-
interest, have been in quiet, peaceful, adverse and uninterrupted possession of Lot No.
SO ORDERED.
4457 for more than 50 years, and submitted as evidence the lots survey plan, technical
description, and certificate of assessment.

Although the respondent, Republic of the Philippines (Republic), opposed the


application,5 it did not present any evidence in support of its opposition. Since no
private opposition to the registration was filed, the RTC issued an order of general
default on November 11, 1996 against the whole world, except the Republic.6

In its Decision dated May 7, 2001,7 the RTC granted the application for registration in
favor of the spouses Fortuna. The RTC declared that "[the spouses Fortuna] have
established [their] possession, including that of their predecessors-in-interest of the land
sought to be registered, has been open, continuous, peaceful, adverse against the
Republic of the Philippines whole world and in the concept of an owner since 1948, or for a period of over fifty (50)
SUPREME COURT years."8
Manila The Republic appealed the RTC decision with the CA, arguing that the spouses Fortuna
SECOND DIVISION did not present an official proclamation from the government that the lot has been
classified as alienable and disposable agricultural land. It also claimed that the spouses
G.R. No. 173423 March 5, 2014 Fortunas evidence Tax Declaration No. 8366 showed that possession over the lot
dates back only to 1948, thus, failing to meet the June 12, 1945 cut-off period provided
SPS. ANTONIO FORTUNA and ERLINDA FORTUNA, Petitioners,
under Section 14(1) of Presidential Decree (PD) No. 1529 or the Property Registration
vs.
Decree (PRD).
REPUBLIC OF THE PHILIPPINES, Respondent.
In its decision dated May 16, 2005,9 the CA reversed and set aside the RTC decision. lots on her way to school since 1938. She knew the property was owned by Pastora
Although it found that the spouses Fortuna were able to establish the alienable and because the latters family had constructed a house and planted fruit-bearing trees
disposable nature of the land,10 they failed to show that they complied with the length thereon; they also cleaned the area. On the basis of Macarias testimony and the other
of possession that the law requires, i.e., since June 12, 1945. It agreed with the evidence presented in LRC No. 2373, the RTC granted the spouses Fortunas
Republics argument that Tax Declaration No. 8366 only showed that the spouses application for registration of Lot Nos. 4462, 27066, and 27098 in its decision of January
Fortunas predecessor-in-interest, Pastora, proved that she had been in possession of 3, 2005.16 The RTCs decision has lapsed into finality unappealed.
the land only since 1948.
The spouses Fortuna claim that Macarias testimony in LRC No. 2373 should be
The CA denied the spouses Fortunas motion for reconsideration of its decision in its considered to prove Pastoras possession prior to 1948. Although LRC No. 2373 is a
resolution dated June 27, 2006.11 separate registration proceeding, it pertained to lots adjacent to the subject property, Lot
No. 4457, and belonged to the same predecessor-in-interest. Explaining their failure to
THE PARTIES ARGUMENTS
present Macaria in the proceedings before the RTC in LRC No. 2372, the spouses
Through the present petition, the spouses Fortuna seek a review of the CA rulings. Fortuna said "it was only after the reception of evidence x x x that [they] were able to
trace and establish the identity and competency of Macaria[.]"17
They contend that the applicable law is Section 48(b) of Commonwealth Act No. 141 or
the Public Land Act (PLA), as amended by Republic Act (RA) No. 1942. RA No. 1942 Commenting on the spouses Fortunas petition, the Republic relied mostly on the CAs
amended the PLA by requiring 30 years of open, continuous, exclusive, and notorious ruling which denied the registration of title and prayed for the dismissal of the petition.
possession to acquire imperfect title over an agricultural land of the public domain. This
THE COURTS RULING
30-year period, however, was removed by PD No. 1073 and instead required that the
possession should be since June 12, 1945. The amendment introduced by PD No. 1073 We deny the petition for failure of the spouses Fortuna to sufficiently prove their
was carried in Section 14(1) of the PRD.12 compliance with the requisites for the acquisition of title to alienable lands of the public
domain.
The spouses Fortuna point out that PD No. 1073 was issued on January 25, 1977 and
published on May 9, 1977; and the PRD was issued on June 11, 1978 and published on The nature of Lot No. 4457 as alienable and
January 2, 1979. On the basis of the Courts ruling in Taada, et al. v. Hon. Tuvera, disposable public land has not been sufficiently
etc., et al.,13 they allege that PD No. 1073 and the PRD should be deemed effective established
only on May 24, 1977 and January 17, 1979, respectively. By these dates, they claim to
The Constitution declares that all lands of the public domain are owned by the
have already satisfied the 30-year requirement under the RA No. 1942 amendment
State.18 Of the four classes of public land, i.e., agricultural lands, forest or timber lands,
because Pastoras possession dates back, at the latest, to 1947.
mineral lands, and national parks, only agricultural lands may be alienated.19 Public
They allege that although Tax Declaration No. 8366 was made in 1948, this does not land that has not been classified as alienable agricultural land remains part of the
contradict that fact that Pastora possessed Lot No. 4457 before 1948. The failure to inalienable public domain. Thus, it is essential for any applicant for registration of title to
present documentary evidence proving possession earlier than 1948 was explained by land derived through a public grant to establish foremost the alienable and disposable
Filma Salazar, Records Officer of the Provincial Assessors Office, who testified that the nature of the land. The PLA provisions on the grant and disposition of alienable public
records were lost beyond recovery due to the outbreak of World War II. lands, specifically, Sections 11 and 48(b), will find application only from the time that a
public land has been classified as agricultural and declared as alienable and
Notwithstanding the absence of documents executed earlier than 1948, the spouses
disposable.
Fortuna contend that evidence exists indicating that Pastora possessed the lot even
before 1948. First, Tax Declaration No. 8366 does not contain a statement that it is a Under Section 6 of the PLA,20 the classification and the reclassification of public lands
new tax declaration. Second, the annotation found at the back of Tax Declaration No. are the prerogative of the Executive Department. The President, through a presidential
8366 states that "this declaration cancels Tax Nos. 10543[.]"14 Since Tax Declaration proclamation or executive order, can classify or reclassify a land to be included or
No. 8366 was issued in 1948, the cancelled Tax Declaration No. 10543 was issued, at excluded from the public domain. The Department of Environment and Natural
the latest, in 1947, indicating that there was already an owner and possessor of the lot Resources (DENR) Secretary is likewise empowered by law to approve a land
before 1948. Third, they rely on the testimony of one Macaria Flores in LRC No. 2373. classification and declare such land as alienable and disposable.21
LRC No. 2373 was also commenced by the spouses Fortuna to register Lot Nos. 4462,
Accordingly, jurisprudence has required that an applicant for registration of title acquired
27066, and 27098,15 which were also originally owned by Pastora and are adjacent to
through a public land grant must present incontrovertible evidence that the land subject
the subject Lot No. 4457. Macaria testified that she was born in 1926 and resided in a
of the application is alienable or disposable by establishing the existence of a positive
place a few meters from the three lots. She stated that she regularly passed by these
act of the government, such as a presidential proclamation or an executive order; an Although the above finding that the spouses Fortuna failed to establish the alienable
administrative action; investigation reports of Bureau of Lands investigators; and a and disposable character of Lot No. 4457 serves as sufficient ground to deny the
legislative act or a statute. petition and terminate the case, we deem it proper to continue to address the other
important legal issues raised in the petition.
In this case, the CA declared that the alienable nature of the land was established by
the notation in the survey plan,22 which states: As mentioned, the PLA is the law that governs the grant and disposition of alienable
agricultural lands. Under Section 11 of the PLA, alienable lands of the public domain
This survey is inside alienable and disposable area as per Project No. 13 L.C. Map No.
may be disposed of, among others, by judicial confirmation of imperfect or incomplete
1395 certified August 7, 1940. It is outside any civil or military reservation.23
title. This mode of acquisition of title is governed by Section 48(b) of the PLA, the
It also relied on the Certification dated July 19, 1999 from the DENR Community original version of which states:
Environment and Natural Resources Office (CENRO) that "there is, per record, neither
Sec. 48. The following-described citizens of the Philippines, occupying lands of the
any public land application filed nor title previously issued for the subject
public domain or claiming to own any such lands or an interest therein, but whose titles
parcel[.]"24 However, we find that neither of the above documents is evidence of a
have not been perfected or completed, may apply to the Court of First Instance of the
positive act from the government reclassifying the lot as alienable and disposable
province where the land is located for confirmation of their claims and the issuance of a
agricultural land of the public domain.
certificate of title therefor, under the Land Registration Act, to wit:
Mere notations appearing in survey plans are inadequate proof of the covered
xxxx
properties alienable and disposable character.25 These notations, at the very least,
only establish that the land subject of the application for registration falls within the (b) Those who by themselves or through their predecessors-in- interest have been in
approved alienable and disposable area per verification through survey by the proper open, continuous, exclusive, and notorious possession and occupation of agricultural
government office. The applicant, however, must also present a copy of the original lands of the public domain, under a bona fide claim of acquisition or ownership, except
classification of the land into alienable and disposable land, as declared by the DENR as against the Government, since July twenty-sixth, eighteen hundred and ninety- four,
Secretary or as proclaimed by the President.26 In Republic v. Heirs of Juan except when prevented by war or force majeure. These shall be conclusively presumed
Fabio,27 the Court ruled that [t]he applicant for land registration must prove that the to have performed all the conditions essential to a government grant and shall be
DENR Secretary had approved the land classification and released the land of the entitled to a certificate of title under the provisions of this chapter. [emphasis supplied]
public domain as alienable and disposable, and that the land subject of the application
On June 22, 1957, the cut-off date of July 26, 1894 was replaced by a 30-year period of
for registration falls within the approved area per verification through survey by the
possession under RA No. 1942. Section 48(b) of the PLA, as amended by RA No. 1942,
PENRO28 or CENRO. In addition, the applicant must present a copy of the original
read:
classification of the land into alienable and disposable, as declared by the DENR
Secretary, or as proclaimed by the President. (b) Those who by themselves or through their predecessors in interest have been in
open, continuous, exclusive and notorious possession and occupation of agricultural
The survey plan and the DENR-CENRO certification are not proof that the President or
lands of the public domain, under a bona fide claim of acquisition of ownership, for at
the DENR Secretary has reclassified and released the public land as alienable and
least thirty years immediately preceding the filing of the application for confirmation of
disposable. The offices that prepared these documents are not the official repositories
title, except when prevented by war or force majeure. [emphasis and underscore ours]
or legal custodian of the issuances of the President or the DENR Secretary declaring
the public land as alienable and disposable.29 On January 25, 1977, PD No. 1073 replaced the 30-year period of possession by
requiring possession since June 12, 1945. Section 4 of PD No. 1073 reads:
For failure to present incontrovertible evidence that Lot No. 4457 has been reclassified
as alienable and disposable land of the public domain though a positive act of the SEC. 4. The provisions of Section 48(b) and Section 48(c), Chapter VIII of the Public
Executive Department, the spouses Fortunas claim of title through a public land grant Land Act are hereby amended in the sense that these provisions shall apply only to
under the PLA should be denied. alienable and disposable lands of the public domain which have been in open,
continuous, exclusive and notorious possession and occupation by the applicant himself
In judicial confirmation of imperfect
or thru his predecessor-in-interest, under a bona fide claim of acquisition of ownership,
or incomplete title, the period of
since June 12, 1945. [emphasis supplied]
possession should commence, at the
latest, as of May 9, 1947 Under the PD No. 1073 amendment, possession of at least 32 years from 1945 up to
its enactment in 1977 is required. This effectively impairs the vested rights of
applicants who had complied with the 30-year possession required under the RA No. nature of Pastoras possession. Notably, Section 48(b) of the PLA speaks of possession
1942 amendment, but whose possession commenced only after the cut-off date of June and occupation. "Since these words are separated by the conjunction and, the clear
12, 1945 was established by the PD No. 1073 amendment. To remedy this, the Court intention of the law is not to make one synonymous with the other. Possession is
ruled in Abejaron v. Nabasa30that "Filipino citizens who by themselves or their broader than occupation because it includes constructive possession. When, therefore,
predecessors-in-interest have been, prior to the effectivity of P.D. 1073 on January 25, the law adds the word occupation, it seeks to delimit the all encompassing effect of
1977, in open, continuous, exclusive and notorious possession and occupation of constructive possession. Taken together with the words open, continuous, exclusive
agricultural lands of the public domain, under a bona fide claim of acquisition of and notorious, the word occupation serves to highlight the fact that for an applicant to
ownership, for at least 30 years, or at least since January 24, 1947 may apply for qualify, his possession must not be a mere fiction."35 Nothing in Tax Declaration No.
judicial confirmation of their imperfect or incomplete title under Sec. 48(b) of the [PLA]." 8366 shows that Pastora exercised acts of possession and occupation such as
January 24, 1947 was considered as the cut-off date as this was exactly 30 years cultivation of or fencing off the land. Indeed, the lot was described as "cogonal."36
counted backward from January 25, 1977 the effectivity date of PD No. 1073.
The spouses Fortuna seeks to remedy the defects of Tax Declaration No. 8366 by
It appears, however, that January 25, 1977 was the date PD No. 1073 was enacted; relying on Macarias testimony in a separate land registration proceeding, LRC No.
based on the certification from the National Printing Office,31 PD No. 1073 was 2373. Macaria alleged that she passed by Pastoras lots on her way to school, and she
published in Vol. 73, No. 19 of the Official Gazette, months later than its enactment or saw Pastoras family construct a house, plant fruit-bearing trees, and clean the area.
on May 9, 1977. This uncontroverted fact materially affects the cut-off date for However, the Court is not convinced that Macarias testimony constituted as the "well-
applications for judicial confirmation of incomplete title under Section 48(b) of the PLA. nigh incontrovertible evidence" required in cases of this nature.

Although Section 6 of PD No. 1073 states that "[the] Decree shall take effect upon its The records disclose that the spouses Fortuna acquired adjoining parcels of land, all of
promulgation," the Court has declared in Taada, et al. v. Hon. Tuvera, etc., et which are claimed to have previously belonged to Pastora. These parcels of land were
al.32 that the publication of laws is an indispensable requirement for its effectivity. "[A]ll covered by three separate applications for registration, to wit:
statutes, including those of local application and private laws, shall be published as a
a. LRC No. N-1278, involving Lot Nos. 1 and 2, with a total area of 2,961 sq.
condition for their effectivity, which shall begin fifteen days after publication unless a
m., commenced by Emeteria;
different effectivity date is fixed by the legislature."33 Accordingly, Section 6 of PD No.
1073 should be understood to mean that the decree took effect only upon its b. LRC No. 2373, involving Lot Nos. 4462, 27066, and 27098, with a total area
publication, or on May 9, 1977. This, therefore, moves the cut-off date for applications of 4,006 sq. m., commenced by the spouses Fortuna; and
for judicial confirmation of imperfect or incomplete title under Section 48(b) of the PLA to
May 8, 1947. In other words, applicants must prove that they have been in open, c. LRC No. 2372 (the subject case), involving Lot No. 4457, with a total area of
continuous, exclusive and notorious possession and occupation of agricultural lands of 2,597 sq. m.
the public domain, under a bona fide claim of acquisition of ownership, for at least 30 As these cases involved different but adjoining lots that belonged to the same
years, or at least since May 8, 1947. predecessor-in-interest, the spouses Fortuna alleged that the final rulings in LRC Nos.
The spouses Fortuna were unable to prove N-1278 and 2373,37 upholding Pastoras ownership, be taken into account in resolving
that they possessed Lot No. 4457 since May 8, 1947 the present case.

Even if the Court assumes that Lot No. 4457 is an alienable and disposable agricultural Notably, the total land area of the adjoining lots that are claimed to have previously
land of the public domain, the spouses Fortunas application for registration of title belonged to Pastora is 9,564 sq. m. This is too big an area for the Court to consider that
would still not prosper for failure to sufficiently prove that they possessed the land since Pastoras claimed acts of possession and occupation (as testified to by Macaria)
May 8, 1947. encompassed the entirety of the lots. Given the size of the lots, it is unlikely that
Macaria (age 21 in 1947) could competently assess and declare that its entirety
The spouses Fortunas allegation that: (1) the absence of a notation that Tax belonged to Pastora because she saw acts of possession and occupation in what must
Declaration No. 8366 was a new tax declaration and (2) the notation stating that Tax have been but a limited area. As mentioned, Tax Declaration No. 8366 described Lot
Declaration No. 8366 cancels the earlier Tax Declaration No. 10543 both indicate that No. 4457 as "cogonal," thus, Macaria could not have also been referring to Lot No. 4457
Pastora possessed the land prior to 1948 or, at the earliest, in 1947. We also observe when she said that Pastora planted fruit-bearing trees on her properties.
that Tax Declaration No. 8366 contains a sworn statement of the owner that was
subscribed on October 23, 1947.34 While these circumstances may indeed indicate The lower courts' final rulings in LRC Nos. N-1278 and 2373, upholding Pastora's
possession as of 1947, none proves that it commenced as of the cut-off date of May 8, possession, do not tie this Court's hands into ruling in favor of the spouses Fortuna.
1947. Even if the tax declaration indicates possession since 1947, it does not show the Much to our dismay, the rulings in LRC Nos. N-1278 and 2373 do not even show that
the lots have been officially reclassified as alienable lands of the public domain or that
Present:
the nature and duration of Pastora's occupation met the requirements of the PLA, thus,
failing to convince us to either disregard the rules of evidence or consider their merits. In PUNO, C.J.,
this regard, we reiterate our directive in Santiago v. De las Santos:38 QUISUMBING,
Both under the 193 5 and the present Constitutions, the conservation no less than the YNARES-SANTIAGO,
utilization of the natural resources is ordained. There would be a failure to abide by its - versus - CARPIO,
command if the judiciary does not scrutinize with care applications to private ownership AUSTRIA-MARTINEZ,
of real estate. To be granted, they must be grounded in well-nigh incontrovertible
evidence. Where, as in this case, no such proof would be forthcoming, there is no
CORONA,
justification for viewing such claim with favor. It is a basic assumption of our polity that CLT REALTY DEVELOPMENT CARPIO MORALES,
lands of whatever classification belong to the state. Unless alienated in accordance with CORPORATION, TINGA,
law, it retains its rights over the same as do minus.
Respondent. CHICO-NAZARIO,
WHEREFORE, the petition is DENIED. The decision dated May 16, 2005 and the VELASCO, JR.,
resolution dated June 27, 2006 of the Court of Appeals in CA-G.R. CV No. 71143 are
AFFIRMED insofar as these dismissed the spouses Antonio and Erlinda Fortuna's
NACHURA,
application of registration of title on the basis of the grounds discussed above. Costs LEONARDO-DE CASTRO
against the spouses Fortuna. BRION, and
SO ORDERED. PERALTA, JJ.

Promulgated:

March 31, 2009

x-------------------------------------------------------------------------
-------- x

ARANETA INSTITUTE OF AGRI- G.R. No. 134385


CULTURE, INC.,
Petitioner,

- versus -
EN BANC
HEIRS OF JOSE B. DIMSON, REPRESENTED BY
MANOTOK REALTY, INC. and G.R. No. 123346 HIS COMPULSORY HEIRS: HIS SURVIVING
MANOTOK ESTATE CORPORATION,
Petitioners,
SPOUSE, ROQUETA R. DIMSON AND THEIR findings and recommended conclusions within three (3) months
CHILDREN, NORMA AND CELSA TIRADO, ALSON from finality of this Resolution.
AND VIRGINIA DIMSON, LINDA AND CARLOS
In ascertaining which of the conflicting claims of title should
LAGMAN, LERMA AND RENE POLICAR, AND prevail, the Special Division is directed to make the following
ESPERANZA R. DIMSON; AND THE REGISTER determinations based on the evidence already on record and
OF DEEDS OF MALABON, such other evidence as may be presented at the proceedings
Respondents. before it, to wit:

x-------------------------------------------------------------------------
-------x

RESOLUTION

TINGA, J.: i. Which of the contending parties are able to


trace back their claims of title to OCT No. 994
dated 3 May 1917?
[1]
In the Courts Resolution dated 14 December 2007, ii. Whether the imputed flaws in the titles of the
Manotoks and Araneta, as recounted in the 2005
the Court constituted a Special Division of the Court of Decision, are borne by the evidence? Assuming
they are, are such flaws sufficient to defeat the
Appeals to hear the instant case on remand. The claims of title of the Manotoks and Araneta?
Special Division was composed of three Associate iii. Whether the factual and legal bases of
Justices of the Court of Appeals, with Justice Josefina 1966 Order of Judge Muoz-Palma and the 1970
Order of Judge Sayo are true and valid. Assuming
Guevara-Salonga as Chairperson; Justice Lucas they are, do these orders establish a superior right
to the subject properties in favor of the Dimsons
Bersamin as Senior Member; and Associate Justice and CLT as opposed to the claims of Araneta and
the Manotoks?
Japar B. Dimaampao as Junior Member. We instructed
the Special Division to proceed as follows: iv. Whether any of the subject properties had
been the subject of expropriation proceedings at
any point since the issuance of OCT No. 994 on 3
May 1917, and if so what are those proceedings,
The Special Division is tasked to hear and receive evidence, what are the titles acquired by the Government
conclude the proceedings and submit to this Court a report on its and whether any of the parties is able to trace its
title to the title acquired by the Government preliminary matter. On February 17, 2009, the Manotoks
through expropriation.
filed a motion beseeching that copies of the report be
v. Such other matters necessary and proper in
ascertaining which of the conflicting claims of title furnished the parties so that they may submit their
should prevail. comments and objections thereon in accord with the
WHEREFORE, the instant cases are hereby principle contained in Sec. 10, Rule 32 of the Rules of
REMANDED to the Special Division of the Court of
Appeals for further proceedings in accordance with Parts Court. We deny the motion.
VI, VII and VIII of this Resolution.

[2] It is incorrect to presume that the earlier referral of these


SO ORDERED.
cases to the Court of Appeals for reception of evidence
The Special Division proceeded to conduct hearings in was strictly in accordance with Rule 32. Notably, Section
accordance with the Resolution. The parties to these 1 of said Rule authorizes the referral of the case to a
cases, namely CLT Realty Development Corporation commissioner by written consent of both parties,
(CLT), Manotok Realty Inc. and Manotok Estate whereas in the cases at bar, the Court did not endeavor
Corporation (the Manotoks), the Heirs of Jose B. to secure the consent of the parties before effectuating
Dimson (Heirs of Dimson), and Araneta Institute of the remand to the Court of Appeals. Nonetheless, our
Agriculture, Inc. (Araneta), were directed by the Special earlier advertence to Rule 32 remains proper even if the
Division to present their respective evidence to the adopted procedure does not hew strictly to that Rule,
Court of Appeals. Thereafter, the Special Division owing to our power under Section 6, Rule 135 to adopt

[3] any suitable process or mode of proceeding which


rendered a 70-page Report (Report) on 26 November
appears conformable to the spirit of the Rules to carry
2008. The Special Division submitted the sealed Report into effect all auxiliary processes and other means
to this Court. necessary to carry our jurisdiction into effect.

Before taking action on the Report itself, we dispose of a


Moreover, furnishing the parties with copies of the then Court of First Instance [CFI] of Rizal a
complaint for Recovery of Possession and
Sealed Report would not serve any useful purpose. It Damages against ARANETA. On 7 May 1980,
DIMSON amended his complaint and included
would only delay the promulgation of the Courts action Virgilio L. Enriquez [ENRIQUEZ] as his co-plaintiff.
on the Sealed Report and the adjudication of these
In said Amended Complaint, DIMSON claimed
cases. In any event, the present Resolution quotes that he is the absolute owner of a 50-hectare land
located in Bo. Potrero, Malabon, Metro Manila
extensively from the sealed Report and discusses its covered by TCT No. R-15169, [Lot25-A-2] of the
Caloocan Registry of Deeds. Allegedly, DIMSON
other substantive segments which are not quoted. had transferred the subject property to ENRIQUEZ
by way of an absolute and irrevocable sale on 14
November 1979.Unfortunately though, DIMSON
The Report is a commendably exhaustive and pellucid and ENRIQUEZ discovered that the subject
property was being occupied by ARANETA
analysis of the issues referred to the Special Division. It wherein an agricultural school house is erected
and that despite repeated demands, the latter
is a more than adequate basis for this Court to make the refused to vacate the parcel of land and remove
following final dispositions in these cases. the improvements thereon.

I.
ARANETA, for its part, refuted said allegations
and countered that it is the absolute owner of the
We adopt the succeeding recital of operative land being claimed by DIMSON and that the real
properties in the Araneta Compound are properly
antecedents made by the Special Division in its Report. documented and validly titled. It maintained that it
had been in possession of the subject parcel of
land since 1974. For this reason, the claims of
DIMSON and ENRIQUEZ were allegedly barred
THE PROCEDURAL ANTECEDENTS by prescription.

DIMSON v. ARANETA During the trial, counsel for ARANETA marked in


CA-G.R. CV. NO. 41883 & CA-G.R. SP No. evidence, among others, certifications from the
34819 Land Registration Commission attesting that TCTs
[SC-G.R. No. 134385] Nos. 13574 and 26538, covering the disputed
property, are in the names of ARANETA and Jose
Rato, respectively. ARANETA also offered TCT
On 18 December 1979, DIMSON filed with the No. 7784 in evidence to prove that it is the
registered owner of the land described therein.

On 28 May 1993, the trial court rendered a


Decision upholding the title of DIMSON over the disputed
property xxx Consequently, ARANETA filed a petition before
the Supreme Court. Refuting the factual finding of
Undaunted, ARANETA interposed an appeal to the trial court and the Court of Appeals, ARANETA
the Court of Appeals, docketed as CA-G.R. CV contended that there in only one OCT 994
No. 41883, which was later consolidated with CA- covering the Maysilo Estate issued on 3 May 1917
GR. SP No. 34819 in view of the inter-related pursuant to the Decree No. 36455 issued by the
issues of the two cases. Court of Land Registration on 19 April 1917 and
added that there were subsequent certifications
In its 30 May 1997 Decision, the Court of Appeals, issued by the government officials, notably from
in CA-G.R. CV No. 41883, sustained the RTC the LRS, the DOJ Committee Report and the
Decision in favor of DIMSON finding that the title Senate Committees Joint Report which attested
of ARANETA to the disputed land in a nullity. In that there is only one OCT 994, that which had
CA-GR. SP No. 34819, the Court of Appeals been issued on 3 May 1917.
likewise invalidated the titles of ARANETA, relying
on the Supreme Court ruling in Metropolitan
Waterworks and Sewerage System v. Court of CLT v. MANOTOK
Appeals, which declared null and void the CA-G.R. CV. No. 45255
certificates of title derived from OCT No. 994 [SC-G.R. No. 123346]
registered on 3 may 1917. It was also held that
ARANETA failed to sufficiently show that the
Order sought to be nullified was obtained through On 10 August 1992, CLT filed with the Regional
extrinsic fraud that would warrant the annulment Trial Court [RTC] A COMPLAINT FOR Annulment
thereof. of Transfer Certificates of Title, Recovery of
Possession and Damages against the
Dissatisfied still, ARANETA filed a Motion for MANOTOKS and the Registry of Deeds of Metro
Reconsideration And/Or New Trial espousing Manila District II (Calookan City, Metro Manila)
therein as basis for its entreaty the various letters [CALOOCAN RD].
from different government agencies and
Department order No. 137 of the Department of In its Complaint, CLT alleged that it is the
Justice, among others. registered owner of Lot 26 of the Maysilo Estate
located in Caloocan City and covered by Transfer
On 16 July 1998, the various Motions of Certificate of Title No. T- 177013, a derivative title
ARANETA were denied by the Court of Appeals. of OCT No. 994. As a basis of its proprietary
Nonetheless, the Court ordered DIMSON to claim, CLT averred that on 10 December 1988, it
maintain status quo until the finality of the had acquired Lot 26 from its former registered
aforesaid judgment. owner, Estelita I. Hipolito [HIPOLITO], by virtue of
a Deed of Sale with Real Estate Mortgage.
HIPOLITOs title was , in turn, a direct transfer from married to Rufina Narciso.
DIMSON, the registered owner of TCT No. 15166,
the latter having acquired the same by virtue of a Thereafter, TCT No. T-35485, canceling TCT No.
Court Order dated 13 June 1966 issued by the T-5261, was issued to Rufina Narcisa Vda. de
Court of First Instance of Rizal in Civil Case No. Gonzales which was later replaced with the names
4557. of Gonzales six (6) children. The property was
then subdivided and as a result of which, seven
On the other hand, the MANOTOKS maintained (7) certificates of titles were issued, six (6),under
the validity of their titles, which were all derivatives the names of each of the children while the
of OCT No. 994 covering over twenty (20) parcels remaining title was held by all of them as co-
of land located over a portion of Lot 26 in the owners.
Maysilo Estate. In substance, it was contented that
the title of CLT was an offspring of an ineffective Eventually, the properties covered by said seven
grant of an alleged undisputed portion of Lot 26 by certificates of title were expropriated by the
way of attorneys fees to its predecessor-in- Republic of the Philippines. These properties were
interest, Jose B. Dimson. The MANOTOKS, in this then later subdivided by the National Housing
connection, further contended that the portion of Authority [NHA], into seventy-seven (77) lots and
Lot 26, subject of the present controversy, had thereafter sold to qualified vendees. As it turned
long been disposed of in favor of Alejandro Ruiz out, a number of said vendees sold nineteen (19)
and Mariano Leuterio and hence, there was of these lots to Manotok Realty, Inc. while one (1)
nothing more in said portion of Lot 26 that could lot was purchased by the Manotok Estate
have been validly conveyed to Dimson. Corporation.

During the pre-trial conference, the trial court,


upon agreement of the parties, approved the
creation of a commission composed of three
commissioners tasked to resolve the conflict in
Tracing the legitimacy of their certificates of titles, their respective titles. Accordingly, the created
the MANOTOKS alleged that TCT No. 4210, Commission convened on the matter in dispute.
which cancelled OCT No. 994, had been issued in
the names of Alejandro Ruiz and Mariano Leuterio On 8 October 1993, Ernesto Erive and Avelino
on Sept ember 1918 by virtue of an Escritura De San Buenaventura submitted an exhaustive Joint
Venta executed by Don Tomas Arguelles and Don Final Report [THE MAJORITY REPORT] finding
Enrique Lopes on 21 August 1918. TCT No. 4210 that there were inherent technical infirmities or
allegedly covered an approximate area of defects on the face of TCT No. 4211, from which
19,565.43 square meters of Lot 26. On even date, the MANOTOKS derived their titles (also on TCT
TCT No. 4211 was transferred to Francisco No. 4210), TCT No. 5261 and TCT No. 35486.
Gonzales on the strength of an Escritura de Venta Teodoro Victoriano submitted his Individual Final
dated 3 March 1920 for which TCT No. T-5261, Report [THE MINORITY REPORT] dated 23
covering an area of 871,982 square meters was October 1993.
issued in the name of one Francisco Gonzales,
Committee dated 28 August 1997 and the Senate
Committee Report No. 1031 dated 25 May 1998
which concluded that there was only one OCT No.
994 issued, transcribed and registered on 3 May
1917.

After the conduct of a hearing on these reports, THE SUPREME COURT DECISION
the parties filed their respective
comments/objections thereto. Upon order of the In its Decision dated 29 November 2005 [THE
trial court, the parties filed their respective SUPREME COURT 2005 DECISION], the
memoranda. Supreme Court, through its Third Division,
affirmed the RTC Decision and Resolutions of the
Adopting the findings contained in the Majority Court of Appeals, which declared the titles of CLT
Report, the RTC, on 10 May 1994, rendered a and DIMSON as valid.
Decision, in favor of CLT and ordered, among
others, the cancellation of the certificates of title In invalidating the respective titles of the
issued in the name of the MANOTOKS. MANOTOKS and ARANETA, the Supreme Court,
in turn, relied on the factual and legal findings of
The MANOTOKS elevated the adverse RTC the trial courts, which had heavily hinged on the
Decision on appeal before the Court of Appeals. In imputed flaws in said titles. Considering that these
its Decision dated 28 September 1995, the Court trial court findings had been affirmed by the Court
of Appeals affirmed the RTC Decision, except as of Appeals, the Supreme Court highlighted the fact
to the award of damages which was deleted. The that the same were accorded the highest degree
MANOTOKS then moved for reconsideration, but of respect and, generally, should not be disturbed
said motion was denied by said appellate court in on appeal.
its Resolution dated 8 January 1996. After the
denial of their Motion for Reconsideration, the
MANOTOKS filed a Petition for Review before the
Supreme Court.

PROCEEDINGS BEFORE THE SUPREME


COURT Emphasis was also made on the settled rule that
because the Supreme Court was not a trier of
Before the Supreme Court, the Petitioners for facts, it was not within its function to review factual
Review, separately filed by the MANOTOKS, issues and examine, evaluate or weigh the
ARANETA and Sto. Nio Kapitbahayan probative value of the evidence presented by the
Association, Inc., [STO. NIO], were consolidated. parties.

Also submitted for consideration of the Supreme THE SUPEME COURT RESOLUTION
Court were the report of the Fact Finding
Expectedly, the MANOTOKS and ARANETA filed title is inexistent. The fact that the
their respective Motions for Reconsideration of the Dimson and CLT titles made
Supreme Court 2005 Decision. specific reference to an OCT No.
994 dated (19) April 1917 casts
Resolving said motions for reconsideration, with doubt on the
the Office of the Solicitor General [OSG]
intervening on behalf of the Republic, the
Supreme Court, in its Resolution of 14 December
2007 [THE SUPREME CCOURT 2007 validity of such titles since they refer
RESOLUTION] reversed and nullified its 2005 to an inexistent OCT. This error
Decision and categorically invalidated OCT No. alone is, in fact, sufficient to
994 dated 19 April 1917, which was the basis of invalidate the Dimson and CLT
the propriety claims of CLT and DIMSON. claims over the subject property if
However, the Supreme Court resolved to remand singular reliance is placed by them
the cases to this Special Division of the Court of on the dates appearing on their
Appeals for reception of evidence. respective titles.

To guide the proceedings before this Special Third. The decision of this Court in
Division of the Court of Appeals, the Supreme MWSS v. Court of Appeals and
Court made the following binding conclusions: Gonzaga v. Court of Appeals
cannot apply to the cases at bar,
First, there is only one OCT 994. As especially in regard to their
it appears on the record, that recognition of an OCT No. 994
mother title was received for dated 19 April 1917, a title which we
transcription by the Register of now acknowledge as inexistent.
Deeds on 3 May 1917, and that Neither could the conclusions in
should be the date which should be MWSS or Gonzaga with respect to
reckoned as the ate of registration an OCT No. 994 dated 19 April
of the title. It may also be 1917 bind any other case operating
acknowledged, as appears on the under the factual setting the same
title, that OCT No. 994 resulted from [4]
as or similar to that at bar.
the issuance of the decree of
*
registration on (19) April 1917,
although such dated cannot be
considered as the date of the title or
II.
the date when the title took effect.

Second. Any title that traces its The parties were afforded the opportunity to present
source to OCT No. 994 dated (19)
April 1917 is void, for such mother their evidence before the Special Division. The Report
names the evidence submitted to the Special Division [PHILVILLE], in Civil Case No. 15045; this Court of
Appeals Decision in CA-G.R. CV. No. 52606
for its evaluation: between CLT and PHILVILLE; the Orders of
Judge Palma dated 13 June 1966 and 16 August
1966 in Case No. 4557 and the billing statements
CLT EVIDENCE of SSHG Law Office. They also submitted in
evidence the Affidavits and Supplemental
[5] Affidavits of Rosa R. Manotok and Luisa T.
In its Offer of Evidence,[ ] CLT adopted the
documentary exhibits and testimonial evidence of Padora; Affidavits of Atty. Felix B. Lerio, Atty. Ma.
witnesses submitted in the case filed by CLT P.G. Ongkiko and Engineer Jose Marie P.
against STO. NIO in Civil Case No. C-15491, Bernabe; a copy of a photograph of BM No. 9;
[CLT-STO NIO CASE]. These pieces of evidence certified true copy of coordinates and reference
include, among others, the Majority and Minority point of L.M. No. 1 and BM No. 1 to 10 of Piedad
Reports, the Formal Offer of Evidence in the [6]
Estate and TCT No. 177013 of CLT.[ ]
presentation of the evidence-in-chief and rebuttal
evidence in the CLT-STO NIO CASE consisting of
various certificates of titles, plans by geodetic DIMSON EVIDENCE
engineer, tax declarations, chemistry report,
specimen signatures and letters of In their Consolidated Formal Offer of
correspondence. [7]
Evidence,[ ] DIMSON submitted the previous
decisions and resolutions passed relative to these
cases, various certifications of different
government agencies, OCT 994, subdivision plan
of Lot 25-A-2, observations of Geodetic Engineer
Reggie P. Garcia showing the relative positions of
properties within Lot 25-A; the Novation of
Contract/Deed of Sale and Mortgage dated 15
MANOTOKS EVIDENCE January 1948 between Rato, Don Salvador
Araneta and Araneta Institute of Agriculture;
The MANOTOKS sought admission of the copies of various certificates of titles to dispute
following evidence: Senate and DOJ Committee some of the titles held by ARANETA; several
Reports; certificates of title issued to them and letter-requests and official receipts.
their vendees/assignees, i.e., Republic of the
Philippines, the Gonzalezes, Alejandro Ruiz and ARANETA EVIDENCE
Mariano Leuterio, Isabel Gil del Sola and Estelita
Hipolito; deeds of absolute sale; contracts to sell; ARANETA, in turn, offered in evidence various
tax declarations and real property tax receipts; the certificates of title, specifically, OCT No. 994, TCT
Formal Officer of Evidence of Philville No. 8692; TCT No. 21857; TCT No. 26538; TCT
Development & Housing Corporation; No. 26539; TCT No. (7784)-738 and TCT no.
13574. It also marked in evidence the certified true flaws in the titles of the Manotoks
copies of Decree No. 36577; the DOJ and Senate and Araneta, as recounted in the
Reports; letters of correspondence to the Land
Supreme Court 2005 Decision, are
Registration Commission and the Register of
Deeds of Malabon City; survey plans of Lot 25-A borne by the evidence.Assuming
and TCT r-15169 of Dimson and; the affidavit of they are, are such flaws sufficient to
Engineer Felino M. Cortez and his curriculum defeat said claims?
vitae.ARANETA also offered the certified true copy
of TCT No. 6196 in the name of Victoneta, Inc.; iii. Whether the factual and legal
TCT No. 13574 in the name of ARANETA;
bases of the 1966 Order of Judge
certifications issued by Atty. Josephine H.
Ponciano, Acting Register of Deeds of Malabon Muoz-Palma and the 1970 Order of
city-Navotas; certified true copy of Judge Palmas Judge Sayo are true and valid.
Order dated 16 August 1966 in Case No. 4557; Assuming they are, do these orders
Circular No. 17 (which pertains to the rules on establish a superior right to the
reconstitution of titles as of 19 February 1947) and subject properties in favor of the
its official receipt and; the owners duplicate copy
Dimsons and CLT as opposed to the
[8] [9]
of OCT No. 994.[ ] claims of the Araneta and the
Manotoks?

III. iv. Whether any of the subject


properties had been the subject of
expropriation proceedings at any
We now turn to the evaluation of the evidence engaged point since the issuance of OCT No.
994 on 3 May 1917, and if so, what
in by the Special Division. To repeat, the Special
are those proceedings, what are the
Division was tasked to determine the following issues titles acquired by the Government,
and is any of the parties able to
based on the evidence:
trace its title acquired by the
government through expropriation?
i. Which of the contending parties
are able to trace back their claims to v. Such other matters necessary and
Original Certificate of Title (OCT) proper in ascertaining which of the
No. 994 dated 3 May 1917: conflicting claims of title should
prevail.

ii. Whether the respective imputed


The ultimate purpose of the inquiry undertaken by the then and still is in possession of the property. The
Court of Appeals was to ascertain which of the four Araneta titles state, as their mother title, OCT No. 994
groups of claimants were entitled to claim ownership dated 3 May 1917. Consequently, Dimson filed an
over the subject properties to which they claimed title action for recovery of possession against Araneta.
thereto. One set of properties was disputed between
CLT and the Manotoks, while the other set was disputed
between Araneta and the Heirs of Dimson.
Another property in Dimsons name, apparently taken
As can be gleaned from the Report, Jose Dimson was from Lot 26 of the Maysilo Estate, was later sold to
able to obtain an order in 1977 issued by Judge Estelita Hipolito, who in turn sold the same to CLT. Said
Marcelino Sayo of the Court of First Instance (CFI) of property was registered by CLT under TCT No. T-
Caloocan Cityon the basis of which he was able to 177013, which also reflected, as its mother title, OCT
register in his name properties belonging to the Maysilo [12]
No. 994 dated 19 April 1917. Said property claimed
Estate. Judge Sayos order in turn was sourced from a
by CLT encroached on property covered by titles in the
1966 Order issued by Judge (later Supreme Court
name of the Manotoks. The Manotoks traced their titles
Associate Justice) Cecilia Muoz-Palma of the CFI of
to TCT Nos. 4210 and 4211, both issued in 1918 and
Rizal. Dimsons titles reflected, as their mother title, OCT
both reflecting, as their mother title, OCT No. 994 dated
[10]
No. 994 dated 19 April 1917. Among these 3 May 1917.
properties was a fifty (50)-hectare property covered by
Transfer Certificate of Title (TCT) No. 151169, which It is evident that both the Heirs of Dimson and CLT had
apparently overlapped with the property of Araneta primarily relied on the validity of OCT No. 994 dated 19
[11] April 1917 as the basis of their claim of ownership.
covered by TCT No. 13574 and 26538. Araneta was
However, the Court in its 2007 Resolution held that OCT
No. 994 dated 19 April 1917 was inexistent. The
proceedings before the Special Division afforded the A.
Heirs of Dimson and CLT alike the opportunity to prove
the validity of their respective claims to title based on We begin with the Heirs of Dimson. The Special Division
evidence other than claims to title the inexistent 19 April made it clear that the Heirs of Dimson were heavily
1917 OCT No. 994. Just as much was observed by the reliant on the OCT No. 994 dated 19 April 1917.
Special Division:
[DIMSON], on the strength of Judge Sayos Order
dated 18 October dated 18 October 1977, was
Nonetheless, while the respective certificates of
issued separate certificates of title, i.e., TCT Nos.
title of DIMSON and CLT refer to OCT 994 issued
15166, 15167, 15168 and 15169, covering
on 19 April 1917 and that their previous
portions of the Maysilo Estate. Pertinently, with
postulations in the present controversies had been
respect to TCT No. 15169 of DIMSON, which
anchored on the supposed validity of their titles,
covers Lot 25-A-2 of the said estate, the following
that which emanated from OCT 994 of 19 April
were inscribed on the face of the instrument.
1917, and conversely the invalidity of the 3 May
1917 OCT 994, the Supreme Court has yet again
IT IS FURTHER CERTIFIED
allowed them to substantiate their claims on the
that said land was originally
basis of other evidentiary proofs:
registered on the 19th day of April in
the year nineteen hundred and
seventeen in the Registration Book
of the Office of the Register of
Otherwise stated, both DIMSON and CLT bear the
Deeds of Rizal, Volume NA page
onus of proving in this special proceedings, by
NA , as Original Certificate of Title
way of the evidence already presented before and
No. 994 pursuant to Decree No.
such other forms of evidence that are not yet of
36455 issued in L.R.C. Case No.
record, that either there had only been an error in
4429 Record No. ______
the course of the transcription or registration of
their derivative titles, or that other factual and legal
This Certificate is a transfer from
bases existed to validate or substantiate their titles
Original Certificate of Title No.
aside from the OCT No. 994 issued on 19 April
994/NA, which is cancelled by virtue
[13] hereof in so far as the above-
1917.
[14]
described land is concerned.[ ]
Were they able to discharge such burden?
explanation as to what led to the erroneous entry
of the registration dated of OCT 994, DIMSON are
left without any recourse but to substantiate their
From the above accounts, it is clear that the claim on the basis of other evidence not presented
mother title of TCT no. 15169, the certificate of title during the proceedings below, which would
of DIMSON covering the now disputed Lot 25-A-2, effectively prove that they had a valid proprietary
is OCT No. 994 registered on 19 April 1917. claim over the disputed properties. This is
Manifestly, the certificate of title issued to specifically true because DIMSON had previously
DIMSON, and as a matter of course, the derivative placed reliance on the MWSS doctrine to prove
title later issued to CLT, should both be voided [17]
the validity of their title.
inasmuch as the OCT which they emanated had
[15]
already been declared inexistent.

The Special Division noted that the Heirs of Dimson did Absent such explanation, the Heirs of Dimson were
not offer any explanation why their titles reflect the particularly constrained to rely on the 1977 Order of
erroneous date of 19 April 1917. At the same time, it Judge Sayo, which was allegedly sourced from the 1966
rejected CLTs explanation that the transcription of the Order of Judge Muoz Palma. On that issue, the Special
erroneous date was a typographical error. Division made the following determinations:

As can be gleaned from the records, both DIMSON and their It should be recalled that in their appellees
successor-in-interest CLT, had failed to present evidence before brief in CA-G.R.CV No. 41883, therein appellee
this Court to prove that there had been a mere typographical error Jose Dimson specifically denied the falsity of
in the transcription of their respective titles with regard to the date TCT No. R-15169 alleging that the contention is
of registration of OCT No. 994. CLT specifically harps on this already moot and can be determined by a
assertion that there had only been a typographical error in the [18]
[16] controlling decision.[ ] Jose Dimson
transcription of its title.[ ] On the other hand, while DIMSON
expounded on his reliance as follows:
had refused to categorically assert that there had been such a
typographical error causing the invalidity of their title, their failure In Metropolitan Waterworks &
to proffer any reason or argument which would otherwise justify Sewerage System (for brevity
why their title reflects 19 April 1917 and not 3 May 1917 leads this MWSS) case, Jose B. Dimsons (as
Court to conclude that they simply had no basis to support their private respondent) title TCT No.
proprietary claim. 15167 issued for Lot 28 on June 8,
1978 derived from OCT No. 994
Thus, without proffering any plausible registered on April 19, 1917, is
overlapping with MWSS title TCT Assuming they are, do these
No. 41028 issued on July 29, 1940 orders establish a superior right
derived from the same OCT 994, to the subject properties in favor
registered on May 3, 1917. of the Dimsons and CLT as
opposed to the claims of Araneta
(Same facts in the case at bar; Jose and the Manotoks?
B. Dimson (plaintiff-appellee) title
TCT No. R-15169 issued for Lot 25- As it is, in contending that their certificates of title
A-2, on June 8, 1978, is overlapping could be validly traced from the 3 May 1917 OCT
with defendant-appellants title TCT No. 994, DIMSON point out that their title was
Nos. 13574 and 21343, not derived issued pursuant to a court order issued by Judge
[19] Palma in Case No. 4557 and entered in the
from OCT No. 994.[ ]
memorandum of Encumbrance of OCT No. 994.
DIMSON also insist that TCT Nos. 8692, 21857
and 26538 were mere microfilmed or certified
So viewed, sans any proof of a mechanical error copies and, therefore, inadmissible. Lastly,
in the transcription or annotation on their DIMSON reiterated the flaws and irregularities
respective certificates of title, the present inquiry which voided the titles of the ARANETA in the
then hinges on whether the Order dated 13 June previous proceedings and focused on the burden
1966 issued by then Judge Cecilia Muoz-Palma of ARANETA to present evidence to defeat their
of the Court of First Instance of Rizal in Civil titles.
Case No. 4557 [PALMA ORDER] and Judge
Sayos Order dated 18 October 1977 [SAYOS 18 The foregoing contentions of DIMSON find to
OCTOBER 1977 ORDER], can be validated and factual and legal basis. As we see it, Sayos 18
authenticated. It is so since the brunt of the October 1977 Order, which apparently confirmed
proprietary claims of both DIMSON and CLT has Palmas 13 June 1966 Order, raised serious
its roots on said Orders. questions as to the validity of the manner by
which it was arrived at.

It is worthy to note that as early as 25 August


1981, counsel for the ARANETA applied for a
subpoena duces tecum addressed to the Clerk of
Court of CFI Pasig for the production of
Perforce, in consideration of the foregoing, this therecords of LRC Case No. 4557 for purposes
leads Us to the THIRD ISSUE as presented by of determining the genuineness and authenticity
the Supreme Court, to wit: of the signature of Judge Palma and also of her
Order granting the confirmation. A certain Atty.
Whether the factual and legal Contreras, Officer-in-Charge of the said court,
bases of Palmas 13 June 1966 appeared and manifested in open court that the
Order and Sayos 18 October 1977 records pertaining to the petition for Substitution
Order are true and valid. of names of Bartolome Rivera, et al. could no
longer be located inasmuch as they had passed That is the reason why we want to
hands from one court to another. see the original.

What is perplexing to this Court is not only the Court:


loss of the entire records of Case No. 4557 but
the admission of Judge Sayo that he had not I did not see the original also. When
seen the original of the Palma Order. Neither was the records of this case was brought
the signature of Judge Palma on the Order duly here, I checked the records, there
proven because all that was presented was an were so many pages missing and the
unsigned duplicate copy with a stamped notation pages were re-numbered but then I
of original signed. Equally perplexing is that while saw the duplicate original and there
CFI Pasig had a Case No. 4557 on file, said file is a certification of a woman clerk of
pertained not to an LRC case but to a simple civil Court, Atty. Molo.
[20]
case.[ ] Thus:
Atty. Directo:
That is the reason why we want to
see this document, we are surprised
why it is missing.
Atty. Directo:
Court:
The purpose of this subpoena duces
tecum is to present your Honor the We are surprised also. You better
Order Order (sic) of Judge Palma in ask Judge Muoz Palma.
order to determine the genuineness
and authenticity of the signature of
Judge Palma in this court order and
which order was a basis of a petition
in this court to be confirmed. That is
the reason why we want to see the
genuineness of the signature of Atty. Contreras:
Judge Palma.
May I make of record that in verifying
COURT: our records, we found in our original
vault LRC application no. N-4557 but
No signature of Judge Palma was the applications were certain
presented in this court. it was a Feliciano Manuel and Maria Leao
duplicate copy not signed. There is a involving Navotas property because I
stamp only of original signed. was wondering why they have the
same number. There should be only
Atty. Directo: one.
Atty. Directo:

Aside from that, are there other


cases of the same number?

Atty. Contreras: Be that as it may, even if We are to consider that


no Recall Order was ever issued by then Judge
No, there should be only number for Palma, the validity of the DIMSON titles over the
a particular case; that must be a properties in the Maysilo Estate becomes
petition after decree record. doubtful in light of the fact that the supposed
share went beyond what was actually due to
Atty. Ignacio: Jose Dimson under the Compromise Agreement
with Rivera. It should be recalled that Palmas 13
This 4557 is not an LRC Case, it is a June 1966 Order approved only the conveyance
simple civil case. to Jose Dimson of 25% of whatever share of
xxxxxx Bartolome Rivera has over Lots 25, 26, 27, 28-B
and 29 of OCT 994 x x x subject to availability of
[22]
undisposed portion of the said lots.[ ]
Moreover, both the MANOTOKS and
ARANETA insist that Palmas 13 June 1966
Order had been recalled by a subsequent Order
[21] In relation to this, We find it significant to
dated 16 August 1966, [RECALL ORDER],[ ] note the observations contained in the Senate
wherein the trial court dismissed the motion filed Committee Report No. 1031 that, based on the
by DIMSON on the courts findings that x x x assumption that the value of the lots were equal,
whatever portion of the property covered by OCT and (C)onsidering that the share of Maria de la
994 which has not been disposed of by the Concepcion Vidal was only 1-189/1000 percent
previous registered owners have already been of the Maysilo Estate, the Riveras who claimed to
assigned and adjudicated to Bartolome Rivera be the surviving heirs of Vidal will inherit only
and his assignees, as a result of which there is 197, 405.26 square meters (16,602,629.53 m2 x
no portion that is left to be given to the herein [23]
1.1890%) or 19.7 hectares as their share.[ ]
supposed assignee Jose Dimson.
Even if we are to base the 25% of Jose Dimson
However, We are reluctant to recognize on the 19.7 hectares allotted to the Riveras, it
the existence and due execution of the Recall would appear that Jose Dimson would only be
Order considering that its original or even a entitled to more or less five (5)hectares of the
certified true copy thereof had not been Maysilo Estate. Obviously, basing only on TCT
submitted by either of the two parties relying on it No. 15169 of Dimson which covered a land area
despite having been given numerous [24]
of 50 hectares (500,000 square meters),[ ] it
opportunities to do so. is undisputable that the total properties
eventually transferred to Jose Dimson went over
and beyond his supposed 25% share.
[Firstly], OCT No. 994 showed that Lot 25-A
What is more, Palmas 13 June 1966 Order of the Maysilo Estate was originally surveyed on
specifically required that x x x whatever title is to September 8-27, 1911, October 4-21 and
be issued herein in favor of Jose Dimson, the November 17-18, 1911. Yet, in said TCT No. R-
same shall be based on a subdivision plan duly 15169, the date of the original survey is reflected
certified by the Land Registration Commission as [26]
correct and in accordance with previous orders as Sept. 8-27, 1911 and nothing more.[ ] The
issued in this proceedings, said plan to be variation in date is revealing considering that
submitted to this court for final approval. DIMSONs titles are all direct transfers from OCT
No. 994 and, as such, would have faithfully
Interestingly however, despite such requirement, adopted the mother lots data. Unfortunately, no
DIMSON did not submit Survey Plan LRC explanation for the variance was ever offered.
(GLRO) Rec. No. 4429 SWO-5268 which
allegedly was the basis of the segregation of the Equally worthy of consideration is the fact
lands, if only to prove that the same had been that TCT No. 15169 indicates that not only was the
duly approved and certified correct by the Land date of original registration inexistent, but the
Registration Commission. What was submitted remarks thereon tend to prove that OCT No. 994
before the RTC and this Court was only the had not been presented prior to the issuance of
Subdivision Plan of Lot 25-A-2 which notably the said transfer certificate. This manifest from the
does not bear the stamp of approval of the LRC. notations NA on the face of DIMSONs title
Even an inspection of the exhibit for CLT does meaning, not available. It bears emphasizing that
not bear this Survey Plan, which could have, at the issuance of a transfer certificate of title to the
the very least, proven the authenticity of the purchaser without the production of the owners
DIMSON title. duplicate is illegal (Rodriguez v. Llorente, 49 Phil.
826) and does not confer any right to the
Indeed, We find the absence of this piece of purchaser (Philippine National Bank vs.
evidence as crucial in proving the validity of the Fernandez, 61 Phil. 448 [1935]). The Registrar of
titles of DIMSON in view of the allegation of Deeds must, therefore, deny registration of any
contending parties that since the survey plan deed or voluntary instrument if the owners
upon which the land titles were based contained duplicate is not presented in connection therewith.
the notation SWO, meaning that the subdivision (Director of Lands vs. Addison, 40 Phil. 19 [1926];
plan was only a product of a special work order, Hodges vs. Treasurer of the Phil. 50 Phil. 16
the same could not have passed the LRC. [27]
[1927].[ ]
[25]
Neither was it duly certified by the said office.

In addition, the Special Division took note of other


irregularities attending Dimsons TCT No. R-15169. In has also been held that, in cases where
transfer certificates of title emanating from one Judge Sayo to seek confirmation of Palmas Order
common original certificate of title were issued on dated 13 June 1966.
different dates to different persons or entities
covering the same land, it would be safe to So viewed the general rule proscribing the
conclude that the transfer certificate issued at an application of laches or the statute of limitations in
earlier date along the line should prevail, barring [30]
land registration cases,[ ] as well as Section 6,
[28]
anomaly in the process of registration.[ ] Thus, Rule 39 of the Rules of Court, in relation to its
(w)here two certificates purport to include the provisions on revival of judgment applies only to
same land, the earlier in date prevails. X x x. In ordinary civil actions and not to other or
successive registration, where more than one extraordinary proceedings such as land
certificate is issued in respect of a particular estate registration cases, is clearly not applicable in the
or interest in land, the person is deemed to hold present case. The legal consequences of laches
under the prior certificate who is the holder or as committed by DIMSON and their failure to
whose claim is derived directly from the person observe the provisions of Rule 39 should,
who was the holder of the earliest certificate therefore, find application in this case and thus,
[29] the confirmation of DIMSONs title, if any, should
issued in respect thereof. x x x[ ]
fail.

Parenthetically, the allegations of DIMSON


xxx would further show that they derive the validity of
Still another indication of irregularity of the their certificates of title from the decreased Jose
DIMSON title over Lot No. 25-A is that the Dimsons 25% share in the alleged hereditary
issuance of the Sayo Order allegedly confirming rights of Bartolome Rivera [RIVERA] as an alleged
the Palma Order was in itself suspect.Gleaning grandson of Maria Concepcion Vidal [VIDAL].
from the records, DIMSON filed the Motion only on However, the records of these cases would
10 October 1977, or eleven (11) years after somehow negate the rights of Rivera to claim from
obtaining the supposed sanction for the issuance Vidal. The Verification Report of the Land
of titles in this name. Besides, what was lodged by Registration
Jose Dimson before the sala of then Judge Palma Commission dated 3 August 1981 showed that
was not a simple land registration case wherein Rivera was 65 years old on 17 May 1963 (as
the only purpose of Jose Dimson was to establish gathered from the records of Civil Case Nos. 4429
his ownership over the subject parcels of land, but, [31]
as reflected in the Palma Order, the subject of the and 4496).[ ] It can thus be deduced that, if
case was the confirmation of Jose Dimsons claim Rivera was already 65 years old in 1963, then he
over the purported rights of Rivera in the disputed must have been born around 1898. On the other
properties. The case did not partake of the nature hand, Vidal was only nine (9) years in 1912;
of a registration proceeding and thus, evidently did hence, she could have been born only on
not observe the requirements in land registration 1905.This alone creates an unexplained
cases. Unlike in a land registration case, therefore, anomalous, if not ridiculous, situation wherein
Jose Dimson needed to file an action before Vidal, Riveras alleged grandmother, was seven (7)
years younger than her alleged grandson. Serious However, an examination of the annotation on
doubts existed as to whether Rivera was in fact an OCT No. 994, particularly the following entries,
heir of Vidal, for him to claim a share in the showed:
[32]
disputed portions of the Maysilo Estate.
AP-6665/0-994 Venta: Queda
cancelado el presente Certificado
en cuanto a una extencion
These findings are consonant with the observations superficial de 3,052.93 metros
cuadrados y 16,512.50 metros
raised by Justice Renato Corona in his Concurring and cuadrados, y descrita en el lote no.
26, vendida a favor de Alejandro
Dissenting Opinion on our 2007 Resolution. To wit: Ruiz y Mariano P Leuterio, el primer
casado con Deogracias Quinones el
Segundo con Josefa Garcia y se ha
TCT No. T-177013 covers Lot 26 of the Maysilo expedido el certificado de Titulo No;
Estate with an area of 891,547.43 sq. m. It was a 4210, pagina 163 Libro T-22.
transfer from TCT No. R-17994 issued in the
name of Estelita I. Hipolito. On the other hand, Fecha del instrumento Agosto 29,
TCT No. R-17994 was a transfer from TCT No. R- 1918
15166 in the name of Jose B. Dimson which, in
turn, was supposedly a direct transfer from OCT Fecha de la inscripcion September
No. 994 registered on April 19, 1917. 9, 1918
10.50 AM
Annotations at the back of Hipolito's title revealed
that Hipolito acquired ownership by virtue of a
court order dated October 18, 1977 approving the AP-6665/0-994 Venta: Queda
compromise agreement which admitted the sale cancelado el presente Certficado el
made by Dimson in her favor on September 2, cuanto a una extencion superficial
1976. Dimson supposedly acquired ownership by de 871,982.00 metros cuadrados,
virtue of the order dated June 13, 1966 of the CFI descrita en el lote no. 26, vendida a
of Rizal, Branch 1 in Civil Case No. 4557 awarding favor de Alejandro Ruiz y Mariano
him, as his attorney's fees, 25% of whatever P. Leuterio, el primer casado con
remained of Lots 25-A, 26, 27, 28 and 29 that Deogracias Quinones el segundo
were undisposed of in the intestate estate of the con Josefa Garcia y se ha expedido
decedent Maria de la Concepcion Vidal, one of the el certificado de Titulo No 4211,
registered owners of the properties covered by pagina 164, Libro T-22.
OCT No. 994. This order was confirmed by the
CFI of Caloocan in a decision dated October 13, Fecha del instrumento Agosto 25,
1977 and order dated October 18, 1977 in SP 1918
Case No. C-732.
Fecha de la inscripcion September
9, 1918 Division on the validity of Jose Dimsons titles, which he
10:50- AM obtained consequent to the 1977 Order of Judge Sayo.

Based on the description of Lot No. 26 in OCT No. Consequently, we cannot give due legal recognition to
994, it has an area of 891,547.43 sq. m. which any and all titles supposedly covering the Maysilo Estate
corresponds to the total area sold in 1918
pursuant to the above-cited entries. Inasmuch as, obtained by Dimson upon the authority of either the
at the time the order of the CFI of Rizal was made
on June 13, 1966, no portion of Lot No. 26 purported 1966 Order of Judge Muoz-Palma or the 1977
remained undisposed of, there was nothing for the
heirs of Maria de la Concepcion Vidal to convey to Order of Judge Sayo.
Dimson. Consequently, Dimson had nothing to
convey to Hipolito who, by logic, could not transmit
anything to CLT.
B.

Indubitably, as between the titles of ARANETA and the


Moreover, subdivision plan Psd-288152 covering MANOTOKS and their predecessors-in-interest, on one
Lot No. 26 of the Maysilo Estate described in
Hipolito's certificate of title was not approved by
hand, and those of DIMSON, on the other, the titles held
the chief of the Registered Land Division as it by ARANETA and the MANOTOKS must prevail
appeared to be entirely within Pcs-1828, Psd-
5079, Psd-5080 and Psd-15345 of TCT Nos. 4210 considering that their titles were issued much earlier
and 4211. How Hipolito was able to secure TCT
No. R-17994 was therefore perplexing, to say the than the titles of the latter.
least.

All these significant facts were conveniently


brushed aside by the trial and appellate courts.
The circumstances called for the need to preserve
and protect the integrity of the Torrenssystem.
However, the trial and appellate courts simply
[33]
disregarded them. Our findings regarding the titles of Jose Dimson
necessarily affect and even invalidate the claims of all
persons who seek to derive ownership from the Dimson
The Court thus adopts these findings of the Special
titles. These include CLT, which acquired the properties
they laid claim on from Estelita Hipolito who in turn expropriation undertaken by the Government as a
means of staking their claims.
acquired the same from Jose Dimson. Just as much
was concluded by the Special Division as it evaluated
CLTs claims.

For its part, CLT contended that even at the trial


court level, it maintained that there was only one
OCT No. 994 from where its claim emanates. It To restate, CLT claims the 891,547.43 square
argued that its case against the MANOTOKS, [36]
meters of land covered by TCT No. T-177013[ ]
including that of STO. NIO, was never decided
based on the doctrines laid down in Metropolitan located in Malabon, Caloocan City and designated
Waterworks and Sewerage System v. Court of as Lot 26, Maysilo Estate, LRC Swo-5268. TCT
[34] No. T-177013 shows that its mother titles is OCT
Appeals[ ] and Heirs of Gonzaga v. Court of No. 994 registered on 19 April 1917. Tracing said
[35] claim, Estelita Hipoloto executed a Deed of Sale
Appeals.[ ]
with Real Estate Mortgage in favor of CLT on 10
December 1988. By virtue of this transfer,
Before this Special Division, CLT insists that the [37]
MANOTOKS failed to submit new competent Hipolitos TCT No. R-17994[ ] was cancelled
evidence and, therefore, dwelling on the alleged and in lieu thereof, CLTs TCT No. 223677/R-
flaws of the MANOTOKs titles, the findings and 17994 of TCT No. R-17994. Hipolito, on the other
conclusions of the court-appointed commissioners hand, was a transferee of the deceased Dimson
as adopted by the trial court, then upheld by the who was allegedly the registered owner of the
Honorable Court in its Decision dated 28 subject land on the basis of TCT No. 15166.
September 1995 and finally affirmed in the
Supreme Courts Decision dated 29 November In view of the foregoing disquisitions, invalidating
2005, therefore stand, as there is no reason to the titles of DIMSON, the title of CLT should also
disturb them. be declared a nullity inasmuch as the nullity of the
titles of DIMSON necessarily upended CLTs
Furthermore, CLT contends that the Orders of propriety claims. As earlier highlighted, CLT had
Judge Palma and Judge Sayo are no longer open anchored its claim on the strength of Hipolitos title
to attack in view of their finality. Lastly, CLT and that of DIMSONs TCT No. 15166.
asserts that the properties covered by the Remarkably and curiously though, TCT No. 15166
MANOTOKS titles and those covered by the was never presented in evidence for purposes of
expropriation proceedings did not property pertain tracing the validity of titles of CLT. On this basis
to and were different from Lot 26 owned by CLT. alone, the present remand proceedings remain
Thus, it maintains that the MANOTOKS cannot damning to CLTs claim of ownership.
use as basis for the validity of their titles the
Moreover, considering that the land title of CLT We begin by evaluating the Araneta titles. The Special
carried annotations identical to those of DIMSON
and consequently included the defects in Division quoted the observations of the trial court, which
DIMSONs title, the fact that whatever
typographical errors were not at anytime cured by upheld Dimsons claim over that of Araneta, citing the
subsequent compliance with the administrative following perceived flaws of TCT Nos. 26538 and
requirements or subjected to administrative
correction bolsters the invalidity of the CLT title 26539, from which Araneta derived its titles, thus:
due to its complete and sole dependence on the
[38]
void DIMSON title.
Let us now examine TCT 26538 and TCT 26539
both in the name of Jose Ma. Rato from where
defendant was said to have acquired TCT 13574
IV. and TCT 7784 now TCT 21343 in the name of
The task of the Special Division was not limited to Araneta and the other documents related thereto:
assessing the claims of the Heirs of Dimson and CLT. 1) Perusal of TCT 26538 shows that its Decree
We likewise tasked the Special Division to ascertain as No. and Record No. are both 4429. In the same vein,
TCT 26539 also shows that it has Decree No. 4429 and
well the validity of the titles held by the Manotoks and Record No. 4429.

Araneta, titles which had been annulled by the courts However, Decree No. 4429 was issued by the
Court of First Instance, Province of Isabela
below. Facially, these titles of the Manotoks and Araneta (Exhibit I) and Record No. 4429, issued for
reflect, as their valid mother title, OCT No. 994 dated 3 Ordinary Land Registration Case, was issued on
March 31, 1911 in CLR No. 5898, Laguna (Exhibit
May 1917. Nonetheless, particular issues were raised 8, 8-A Bartolome Rivera et al.)

as to the validity of the Manotok and Araneta titles How then could TCT No. 26538 and TCT No.
26539 both have Decree No. 4429 and Record
independent of their reliance on the 3 May 1917 OCT No. 4429, which were issued in Court of First
No. 994 vis--vis the inexistent 19 April 1917 OCT No. Instance, Province of Isabela and issued in
Laguna, respectively.
994.
2) TCT no. 26538 and TCT No. 26539 in the name
of Jose Ma. Rato are not annotated in the Original
Certificate of Title 994, where they were said to
A. have originated.

3) The Escritura de Incorporacion de Philippine


Land Improvement Company (Exhibit I) executed Under Sec. 55 of Land Registration Act (Act No.
on April 8, 1925 was only registered and was 496) now Sec. 53 of Presidential Decree No.
stamped received by the Office of the Securities 1529, no new certificate of title shall be entered,
and Exchange Commission only April 29, 1953 no memorandum shall be made upon any
when the Deed of Sale & Mortgage was executed certificate of title by the register of deeds, in
on August 23, 1947 (Exh. 5 defendant) and the pursuance of any deed or other voluntary
Novation of Contract, Deed of Sale and Mortgage instrument, unless the owners duplicate certificate
executed on November 13, 1947 (Exh. M0. So, is presented for such endorsement.
that when Philippine Land Improvement was
allegedly given a special power of attorney by 8) The sale by Jose Ma. Rato in favor of defendant
Jose Ma. Rato to represent him in the execution of Araneta is not reflected on the Memorandum of
the said two (2) documents, the said Philippine Encumbrances of TCT 26538 (Exhibit 7-
Land Improvement Company has not yet been defendant) meaning that TCT 26538 still exists
duly registered. and intact except for the encumbrances annotated
in the Memorandum of Encumbrances affecting
4) TCT 26538 and 26538 and TCT 26539 both in the said title (Exhibits 16, 16-A and 16-N David &
the name of Jose Ma. Rato, both cancel 21857 Santos)
which was never presented in Court if only to have
a clear tracing back of the titles of defendant 9) In the encumbrances annotated at the back of
Araneta. TCT 26539 (Exhibit 4-defendant) there appears
under entry No. 450 T 6196 Victoneta,
5) If the subject matter of the Deed of Sale & Incorporated covering parcel of land canceling
Mortgage (Exhibit 5 defendant) is TCT 26539, why said title (TCT 26539) and TCT 6196 was issued (
is it that TCT 13574 of defendant Araneta cancels x x x) which could have referred to the Deed (sic)
TCT 6196 instead of TCT 26539.That was never of Sale and Mortgage of 8-23-47 (Exhibit 5-
explained. TCT 6196 was not even presented in defendant) entered before Entry 5170 T-8692
Court. Convenio Philippine Land Improvement Company,
with Date of Instrument: 1-10-29, and Date of
6) How come TCT 26538 of Jose Ma. Rato with an Inscription: 9-21-29.
area of 593,606.90 was cancelled by TCT 7784
with an area of only 390,282 sq.m.

7) How was defendant Araneta able to have TCT In TCT 26838 this Entry 5170 T-8692 Convenio
7784 issued in its name, when the registration of Philippine Land Improvement Company (Exhibit
the document entitled Novation of Contract, Deed 16-J-1) appears, but the document, Novation of
of Sale & Mortgage (Exhibit M) was Contract, Deed of Sale & Mortgage dated
suspended/denied (Exhibit N) and no title was November 13, 1947 (Exhibit M) does not appear.
received by the Register of Deeds of Pasig at the
time the said document was filed in the said Office Entry marked Exhibit 16-J-1 on TCT 26538 shows
on March 4, 1948 (Exhibit N and N-1). only the extent of the value of P42,000.00 invested
by Jose Ma. Rato in the Philippine Land
Improvement Company. Said entry was also Tuazon [RATO], one of the co-heirs named in
entered on TCT 26539. OCT No. 994, who was allotted the share of nine
and five hundred twelve one thousandths (9-
The Court also wonders why it would seem that all 512/1000) percent share of the Maysilo
the documents presented by defendant Araneta [41]
Estate.[ ] For this reason, to ascertain the
are not in possession of said defendant, for
according to witness Zacarias Quintan, the real legitimacy of the derivative title of ARANETA, the
estate officer of the said defendant Araneta since origin and authenticity of the title of RATO need to
1970, his knowledge of the land now in be reassessed.
possession of defendant Araneta was acquired by
him from all its documents marked in evidence Verily, attesting to RATOs share on the
which were obtained only lately when they needed property, Entry No. 12343/O-994 of the Owners
[39] [40] Duplicate Copy of OCT no. 994, records the
for presentation before this Court.[ ] following:

12343/O-994 Auto: Jose Rato y


Tuason - - - Queda cancelado el
The Special Division then proceeded to analyze these
presente seartificado en cuanto a
factual contentions, and ultimately concluded that the una estension superficial de
1,405,725.90 metro Cuadrados mas
Araneta claim to title was wholly valid. We adopt in full o menos descrita en el Lote No. 25-
A-3, an virtud del auto dictado por el
the following factual findings of the Special Division, Juzgado de Primera Instancia de
Riza, de fecha 28 de Julio de 1924,
thus:
y que en au lugar se had expedido
el Certificados de Titulo No. 8692,
folio 492 del Tomo T-35 del Libro de
As for the proprietary claim of ARANETA, it Certicadads de Transferencia.
maintains that it has established by direct
evidence that its titles were validly derived from Date of Instrument Julio 28, 1924.
OCT No. 994 dated 3 May 1917. With regard to
the imputed flaws, it asseverates that these were Date of Inscription Agosto 1, 1024
unfounded and thus, labored to refute all of them. 10:19 a.m.
ARANETA further expounded on the nullity of the
Palma and Sayo Orders which was the basis of SGD. GLICERIO OPINION,
DIMSONs titles. Register of deeds
[42]
Agosto 19, 1924[ ]
The documentary exhibits it proffered traced
its certificates of title to OCT No. 994 registered on In accordance with the decree, RATO was
3 May 1917. From the titles submitted, its [43]
predecessor-in-interest was Jose Ma. Rato y issued on 1 August 1924, TCT No. 8692[ ]
which covers Lote No. 25 A-3 del plano del registered owner appears to be a certain
subdivision, parte del Lote No. 25-A, plano Psu- Victoneta, Inc. This parcel of land has an area of
(not legible), Hacienda de Maysilo, situado en el 581,872 square meters designated as section No.
Munisipio de Caloocan, Provincia del Rizal x x 2 of subdivision plan Psd-10114, being a portion of
[44] Lot 25-A-3-C.
x.[ ] The parcel of land covers an approximate
area of UN MILLION CUATROCIENTOS CINCO As shown on its face, TCT No. 6196 issued
MIL SETECIENTOS VEINTICINCO metros on 18 October 1947 in the name of Victoneta, Inc.
cuadrados con NOVENTA decimetros cuadrados and its mother title were traced from OCT No. 994
(1,405,725.90) mas o menos. As reflected under registered on 3 May 1917. Later, TCT No. 6196
[45] was cancelled, and in lieu thereof, TCT No. 13574
Entry No. 14517.T-8692,[ ] the parcel of land
covered under this certificate of title was was issued in favor of Araneta Institute of
subdivided into five (5) lots under subdivision plan [53]
Agriculture on 20 May 1949.[ ] It covers a
Psd-6599 as per Order of the court of First parcel of land designated as section No. 2 of
Instance of Rizal. Consequently, TCT Nos. 21855, subdivision plan Psd-10114, being a portion of Lot
21856, 21857, 21858 and 21859 were issued. 25-A-3-C. It has an aggregate area of 581,872
square meters.
Focusing on TCT No. 21857 issued on 23
May 1932, this certificate of title issued in RATOs
[46] [47] On the other hand, appearing under Entry
name,[ ] cancelled TCT No. 8692[ ] with
respect to the property it covers. On its face, TCT No. 16086/T-No. 13574 of TCT No. 6196 is the
[48] following:
No. 21857,[ ] was a derivative of OCT No. 994
registered on 3 May 1917. It covers Lot No. 25 A- Entry No. 16086/T-No. 13574 SALE
3-C of subdivision plan Psd-6589, being a portion in favor of the ARANETA INSTITUTE
of Lot No. 25-A-3, G.L.R.O Record No. 4429. OF AGRICULTURE, vendee:
Thereafter, TCT No. 21857 was cancelled by TCT Conveying the property described in
[49] [50] this certificate of title which is hereby
No. 26538[ ] and TCT No. 26539[ ] which
cancelled and issuing in lieu thereof
were both issued in the name of Jose Ma. Rato y Transfer Certificate of Title No.
Tuazon on 17 September 1934. 13574, page 74, Book T-345 in the
name of the vendee. (Doc. No. 149,
With respect to TCT No. 26539, the page 98, Book II, S. of 1949 of
certificate of title showed that it covered a parcel of Notary Public for Manila, Hospicio B.
land designated as Section No. 2 of the Bias).
subdivision plan Psd-10114, being a portion of Lot
25-A-3-C having an approximate area of 581,872 Date of Instrument May 18, 1949
[51] Date of the Inscription May 30, 1949
square meters.[ ] Thereafter, TCT No. 26539
[52] [54]
was cancelled by TCT No. 6196[ ] whose at 11:00 a.m.[ ]
25 A-3-C. In all his certificates of title, including
those that ultimately passed ownership to
[55] ARANETA, the designation of the lot as either
TCT No. 26538[ ] in turn showed on its
belonging to or portions of Lot 25-A-3 was
face that it covers a parcel of land designated as retained, thereby proving identity of the land.
Section 1 of subdivision plan Psd-10114 being a
portion of Lot 25-A-3-C having an area of More importantly, the documentary trail of
[56] land titles showed that all of them were derived
592,606.90 square meters.[ ]
from OCT No. 994 registered on 3 May 1917. For
On 4 March 1948, TCT No. 26538 was purposes of tracing ARANETAs titles to Oct No.
cancelled by TCT No. 7784, which was issued in 994, it would appear that the evidence presented
favor of Araneta Institute of Agriculture. TCT No. ultimately shows a direct link of TCT Nos. 7784
7784 covers four (4) parcels of land with an and 13574 to said mother title. Suffice it to state,
[57] the origin and legitimacy of the proprietary claim of
aggregate area of 390,282 square meters.[ ] It ARANETA had been well substantiated by the
would appear from the records of CA-G.R. SP No. evidence on record and on this note, said titles
34819 consolidated with CA-G.R. CV No. 41883 deserve validation.
that TCT No. 7784 was eventually cancelled by
[58] Under the guidelines set, we shall now
TCT No. 21343.[ ] As per attachment of
proceed to evaluate the imputed flaws which had
ARANETA in its Answer dated 6 march 1980 filed been the previous bases of the trial court in
in Civil Case No. 8050, a mere copy of TCT No. invalidating ARANETAs titles.
21343 showed that it covers a parcel of land
designated as Lot 6-B of the subdivision plan Psd- One of the flaws observed on the titles of
24962 being a portion of Lot 6, described as plan ARANETAs predecessor-in-interest was that TCT
Psd-21943, G.L.R.O. Record No. 4429 with an No. 26538 and TCT No. 26539 in Ratos name
[59] refer to Decree No. 4429 and Record No.4429, as
approximate area of 333,377 square meters.[ ]
However, for reasons unknown, a copy of TCT No. basis of their issuance. This is being questioned
21343, whether original or certified true copy inasmuch as Decree No. 4429 refers to a decree
thereof, was not submitted before this Court. issued by the CFI of Isabela while Record No.
4429 was issued for ordinary Land Registration
In summation, ARANETA had shown that Case No. 31 March 1911 in CLR No. 5898 of
RATO, as one of the co-owners of the property Laguna.
covered by OCT NO. 994, was assigned Lot No.
25-A-3. His evidence of ownership is reflected on Explaining this discrepancy, ARANETA
TCT No. 8692 issued in his name. RATO held title insisted that the same was a mere typographical
to these parcels of land even after its subdivision error and did not have any effect on the validity of
in the 1930s. Further subdividing the property, their title. It further contended that the number
RATO was again issued TCT No. 21857, and later 4429 was the case number of Decree No. 36455
TCT Nos. 26538 and 26539, still covering Lot No. and was used interchangeably as the record
number.
the land, as stated on the face of the title, had not
been shown to be erroneous or otherwise
inconsistent with the source of titles. In
ARANETAs case, all the titles pertaining to Lot No.
25 had been verified to be an offshoot of Decree
No. 36455 and are all located in Tinajeros,
Malabon. At any rate, despite the incorrect entries
This Court finds that the incorrect entry with on the title, the properties, covered by the subject
respect to the Decree and Record Number certificates of title can still be determined with
appearing on the title of ARANETAs predecessor- sufficient certainty.
in-interest cannot, by itself, invalidate the titles of
ARANETAs predecessors-in-interest and It was also opined that TCT No. 26538 and
ultimately, that of ARANETA. To the mind of this TCT No. 26539 in the name of RATO had not
Court, the incorrect entries alluded to would not been annotated on OCT No. 994 from which said
have the effect of rendering the previous titles void titles had supposedly originated. It should be
sans any strong showing of fraudulent or stressed that what partially cancelled OCT No.
intentional wrongdoing on the part of the person 994 with respect to this subject lot were not TCT
making such entries. Fraud is never presumed but Nos. 26538 and 26539 but TCT No. 8692 issued
must be established by clear and convincing on 1 August 1924. In fact, TCT Nos.26538 and
[60] 26539 are not even the immediate predecessors
evidence.[ ] The strongest suspicion cannot
of OCT No. 994 but were mere derivatives of TCT
sway judgment or overcome the presumption of No. 21857. Logically therefore, these two
regularity. The sea of suspicion has no shore, and certificates of title could not have been annotated
the court that embarks upon it is without rudder or on OCT No. 994, they not being the preceding
[61] titles.
compass.[ ]

The Supreme Court, in Encinas v. National In any case, a perusal of OCT No. 994
[62] shows an entry, which pertains to Jose Ma. Rato
Bookstore, Inc.[ ] acknowledged that certain but, on account of the physical condition of the
defects on a certificate of title, specifically, the copy submitted to this Court, the entry remains
interchanging of numbers, may occur and it is [64]
illegible for us to make a definite conclusion.[ ]
certainly believable that such variance in the
copying of entries could be merely a typographical On the other hand, Entry No. 12343/O-994 found
or clerical error. In such cases, citing with approval on the Owners Duplicate Copy of OCT No. 994
the decision of the appellate court, the technical specifically recorded the issuance of TCT No.
description in the title should prevail over the [65]
8692 over Lot No. 25-A-3.[ ]
[63] The other flaws noted on ARANETAs
record number.[ ]
certificates of title pertained to its failure to present
Thus, what is of utmost importance is that TCT Nos. 21857, 6196 and 21343. As we have
the designation and the technical description of discussed, ARANETA offered in evidence a
certified microfilm copy of TCT No. 21857 and a
certified true copy of TCT No. 6196 marked as
Exhibits 5-A1A and 19-A1A, respectively.
However, it failed to submit a copy of said TCT
No. 21343. Be that as it may, we will not hasten to The trial court, relying on Exhibit N, further
declare void TCT No. 7784 as a consequence of asserted that ARANETA should not have been
such omission, especially so since TCT No. 21343 issued TCT No. 7784 considering that the
appears to be a mere derivative of TCT No. 7784. registration of the Novation of Contract, deed of
Given that the validity of TCT No. 7784 had been Sale & Mortgage was suspended/denied and no
preponderantly proven in these proceedings, the title was received by the Register of Deeds of
authenticity of said title must be sustained. Pasig at the time the said document was filed in
Besides, ARANETAs failure to submit TCT No. the said Office on march 4, 1948. A perusal of
21343 had never been put into issue in these Exhibit N submitted before the trial court, shows
proceedings. that the suspension or denial was merely
conditional considering that the person seeking
With respect to the difference in the area of registration had give days within which to correct
more than 200,0000 square meters between TCT the defects before final denial thereof. As we see
No. 7784 and TCT No. 26538, we find that the trial it, the Notice merely contained a warning
court failed to consider the several conveyances of regarding the denial of the registration of the
portions of TCT No. 26538 before they finally voluntary deed but, in no way, did it affect the
passed on to ARANETA. Thus, on the vested rights of ARANETA to be land. The fact
Memorandum of Encumbrance of TCT No. 26538, that the title to the land was subsequently issued
it is apparent that portions of this piece of land had free from any notation of the alluded defect
been sold to various individuals before the same creates a reasonable presumption that ARANETA
were transferred to ARANETA on 4 march 1948. was in fact able to comply with the condition
Naturally, since the subject land had been partially imposed. This is especially true since the notice
cancelled with respect to the portion disposed of, it itself contained a note, Just Completed, written
could not be expected that the area of TCT No. across the face of the letter.
26538 will remain the same at the time of its
transfer to ARANETA. Even assuming that the Records also reveal the RTCs observation
entire area covered by TCT No. 26538 had been with regard to Aranetas failure to disprove the
disposed of, this fact alone, cannot lend us to result of the plotting made on the subject land
conclude that the conveyance was irregular. An (Exhibit K) to the effect that TCT 26538 overlaps
anomaly exists if the area covered under the portion of TCT 15159 and TCT 26539 also
derivative title will be much more than its overlaps the other portion of said TCT R-15169.
predecessor-in-interest. Evidently, this is not so in The trial court further noted that TCT R-15169
the case before us. (Jose Dimson) and TCT 26539 (Jose Rato) and
TCT 21343 (Araneta) are overlapping each other
within Lot 25-A. That portion of TCT R-15169
(Jose Dimson) along bearing distance points to 17
to 18 to 19 to 20 to 21 to 1 and 2 shaded in yellow
color in the Plan is not covered by TCT 21343 of TCT No. 26538 inasmuch as TCT No. 7784
[66] cancelled the former certificate of title to the extent
(Araneta).[ ]
only of Three Hundred Ninety Thousand Two
Hundred Eighty Two (390,282) square meters.
Scrutinizing Exhibit K, it becomes apparent
that the said evidence relied upon was only a Notably also, with the evident intent to
private survey conducted by Geodetic Engineer discredit and refute the title of ARANETA,
Reggie P. Garcia which had not been duly [69]
approved by the Bureau of Lands and was based DIMSON submitted TCT Nos. 26538[ ] and
[67] [70]
only on photocopies of relevant land titles.[ ] 21857,[ ] which are both derivatives of OCT
What is more, said geodetic engineer also failed to No.994 registered on 3 May 1917 and cover
adequately explain his observations, approach parcels of land located in Malabon, Rizal.
and manner of plotting the relative positions of the However, these certificates of title reflect different
[68] registered owners and designation of the land
lots.[ ] From all indications, the conclusions
covered.
reached by said geodetic engineer were anchored
on unfounded generalizations. Pertinently, Exhibit M-Dimson relating to
TCT No. 26538, registered on 12 June 1952,
points to one Angela Bautista de Alvarez as the
registered owner of a 240 square meter of land
designated as Lot No. 19, Block 14 of the
subdivision plan Psd-5254 being a portion of Lot
Another defect cited on ARANETAs title was No. 7-A-1-A. This certificate of title cancels TCT
the absence of any entry on the Memorandum of No. 14112/T-348 and refers to a certain TCT No.
Encumbrances of TCT No. 26538 of the alleged 30473 on the inscriptions.
sale between RATO and ARANETA.As pointed
out by ARANETA, the copy of TCT No. 26538 Exhibit N-Dimson, on the other hand,
submitted to the trial court contained entries only pertaining to TCT No. 21857 was issued on 30
up to the year 1947, thus, explaining the (1) lack of March 1951 to one Angela I. Tuason de Perez
entry with regard to the issuance of TCT No. 7784 married to Antonio Perez. This certificate of Title
in favor of ARANETA considering that the same covers a parcel of land described as Lot No. 21,
was issued a year later and; (2) entry pertaining to Block 16 of the consolidation and subdivision plan
Convenio Philippine Land Improvement Company Pcs-140, G.L.R.O. Record No. 4429. It ahs an
which was entered way back on 21 August 1929. area of 436 square meters and cancels TCT No.
21856.
Nonetheless, it still cannot be denied that
Rato and ARANETA together with Don Salvador
Araneta, entered into a voluntary agreement with
the intention of transferring the ownership of the
subject property. Moreover, no conclusion should
have been reached regarding the total cancellation
Republic were numbered
[71] consecutively and the titles issued
Exhibit Q-Dimson[ ] consisting of TCT
after the inauguration were
No. 8692 covers two parcels of land designated as numbered also consecutively starting
Lot Nos. 1 and 2 of Block No. 44 of the with No. 1, so that eventually, the
consolidation Subdivision Plan Pcs-188 with a titles issued before the inauguration
total area of 3,372 square meters. It was issued to were duplicated by titles issued after
Gregorio Araneta, Incorporated on 7 May 1948. the inauguration of the Philippine
This certificate of title cancelled TCT No. 46118. Republic x x x.
Comparing these titles to those of the
Parenthetically, in their Motion for Partial
ARANETA, it is apparent that no identity of the
Reconsideration of this Courts Resolution dated
land could be found. The Supreme Court, in the
30 October 2008, DIMSON objected to the
case of Alonso v. Cebu City Country Club,
admissibility of Exhibits 4-A1A to 7-A1A on the
[72] ground that ARANETA failed to submit the original
Inc.[ ] agreeing with the Court of Appeals
dissertation in said case, ruled that there is copies of these certificates of title and contended
nothing fraudulent for a certificate of title to bear that the originals contain different contents from
the same number as another title to another land. [73]
their own Exhibits M, N and Q.[ ]The fact that
On this score, the Supreme Court elucidated as
the entries contained in ARANETAs pieces of
follows:
evidence are different from that of DIMSONs do
not automatically make ARANETAs exhibits
inferior replications or a confirmation of their
On the question that TCT No.
falsity. Interestingly, the objection regarding the
RT-1310 (T-1151) bears the same
non-submission of the original copy had not been
number as another title to another
raised by DIMSON in their Comments/Objections
land, we agree with the Court of
to Consolidated Formal Offer of Evidence (Of
Appeals that there is nothing
fraudulent with the fact that Cebu
[74]
Araneta Institute of Agriculture, Inc.).[ ] In any
Country Club, Inc.s reconstituted title case, we find the objections unwarranted
bears the same number as the title of considering that certified true copies or certified
another parcel of land. This came microfilm copies of Exhibits 4-A1A to 7-A1A had
about because under General Land been submitted by ARANETA in these
Registration Office (GLRO) Circular proceedings.
No. 17, dated February 19, 1947,
and Republic Act No. 26 and Circular Lastly, on the alleged non-registration of
No. 6, RD 3, dated August 5, 1946, Philippine Land Improvement Company at the time
which were in force at the time the the special power of attorney was executed by
title was reconstituted on July 26, Jose Ma. Rato to represent him in the execution of
1946, the titles issued before the the deed of conveyances, the same only proves
inauguration of the Philippine that Philippine Land Improvement Company was
not yet registered and this does not go as far as designation and the technical description of the land on
proving the existence or non-existence of the
company at which time it was executed. In effect, those titles not having been shown to be erroneous or
the company was not precluded to enter into
contracts and be bound by them but it will do so at variant with the source title. The Special Division also
the risk of the adverse effects of non-registration correctly considered that the trial court had failed to take
under the law.
into account the several conveyances of TCT No. 26538
Ultimately, the question of whether the
aforesaid certificates of title constitute as clouds before it was ultimately transferred to Araneta in 1948,
on ARANETAs titles are not for this Court to rule
upon for purposes of the present which explain the difference in area between TCT No.
remand.Needless to state, it is not for the Heirs of 7784 and TCT No. 26538. The imputed overlap of TCT
Dimson to rely on the weakness of ARANETAs
titles and profit from it. Rather, they should have No. 26538 and TCT No. 26539 with the titles held by
focused on the strength of their own titles since it
is not within our office to decide in whose hands Dimson was based on a private survey which had not
the contested lands should go, our task being
merely to trace back the parties claims to OCT No. been duly approved by the Bureau of Lands. The
[75] alleged absence of any entry on the Memorandum of
994 dated 3 May 1917.
Encumbrances of TCT No. 26538 of the sale of the
property between Rato and Araneta did not, according
There is no question that the Araneta titles were derived
to the Special Division, discount the fact that Rato and
from OCT No. 994 dated 3 May 1917, particularly from
Araneta entered into a voluntary agreement with the
the share of Jose Ma. Rato y Tuazon, one of the co-
intention of transferring the ownership of the subject
heirs named in OCT No. 994. The Special Division
property. Finally, the Special Division noted that the
correctly assessed, among others, the reference to
titles derived from OCT No. 994, which Dimson had
Decree No. 4429 and Record No. 4429 in some of the
submitted as evidence to discredit the Araneta claim,
[76]
antecedent titles of Araneta as mere clerical errors pertain to properties wholly different from those covered
that could not have invalidated said titles, 4429 being by the Araneta titles.
the case number of Decree No. 36455, and the
There is no cause to dispute the factual findings and
Ap. 6655/O-994 Venta: Queda
conclusions of the Special Division on the validity of the Cancelado el presente Certificado
en cuanto a una extension
Araneta titles, and we affirm the same. superficial de 3,052.93 Metros
cuadrados y 16,512.50 metros
Cuadrados y descrita en elLote No.
B. 26 vendida a favor de Alejandro
Ruis y Mariano P. Leuterio, el
primar casado con Diogracias
It appears that the claim to title of the Manotoks is Quinones y el Segundo con Josefa
Garcia y se be expedido el
somewhat more controversial. The Special Division did Certificado de Titulo No. 4210,
Pagina 163, Libro T-22.
not discount the fact that there could have been flaws in
some of the intervening titles between the 3 May 1917 Date of the Instrument Aug. 29,
1918
OCT No. 994 and the present titles of the Manotoks. Date of Inscription Sept. 9, 1918
10:50 a.m.
However, the significant event was the expropriation (GD) L. GARDUNIO, Register of
Deeds
proceedings undertaken by the Republic of the
Philippines sometime in 1947. At least some of the titles Ap. 6665/O-994-Venta: Queda
Cancelado el presente Cerficiado
in the name of the Manotoks were sourced from the en cuanto a una extension
superficial de 871,982.00 metros
titles issued to and subsequently distributed by the cuadrados, descrita en el Lote No.
26, vendida a favor de Alejandro
Republic. The Special Division explained the milieu in Ruiz y Mariano P. Leuterio, el
full: primar casado con Deogracias
Quinones y el Segundo con Josefa
Garcia y se be expedido el
Certificado de Titulo No. 4211,
VALIDITY OF THE MANOTOK TITLES Pagina 164, Libro T-No. 22.

The notation under Entry No. 6655/O-994, found Date of Instrument Aug. 21, 1918
on page 17 of OCT 994 of the Owners Duplicate Date of Inscription Sept. 9, 1918
Copy, shows that Lot No. 26 had been a subject of 10:50 a.m.
sale in favor of Alejandro Ruiz and Mariano P. (SGD.) L. GARDUNIO, Register of
[77] Deeds
Leuterio.[ ] The notations reads:
Gonzales, No. 49034, se cancela el
presente certificado de tituto y se
As a result, TCT No. 4211 was cancelled by TCT expide otre a hombre decha Rufina
No. 5261 which was issued in the name of Narciso, con (not legible) No. 35486,
Francisco Gonzales. Inscribed on the folio 86, Tomo T-168 del libro de
Memorandum of the Incumbrances Affecting the transferencias, archivando se la
Property Described in this Certificate was the sale copia de dicha orden da que se ha
executed in favor of heche referencia en al Legajo T-No.
Francisco Gonzales dated 3 March 1920. Thus, on 35486.
6 April 1920, TCT No. 5261 was issued in the
[78] (SGD) TEODORO GONZALES,
name of Francisco Gonzales.[ ]
Registrado de Titulos.
On 22 August 1938, TCT No. 5261 was cancelled
by TCT No. 35486 in the names of Jose Gonzales
y Narciso married to Maria P. Gutierrez, Consuelo The property was later subdivided into seven lots
Susana Gonzales y Narciso married to Alfonso D. in accordance with subdivision plan Psd-
Prescilla; Juana Francisco Gonzales y Narciso [80]
married to Fortunato de Leon; Maria Clara 21154.[ ] Partitioning the lots among the co-
Gonzales y Narciso married to Delfin Hilario; owners, TCT No. 35486 was eventually cancelled
Francisco Felipe Gonzales y Narciso married to and in lieu thereof six (6) certificates of titles were
Pilar Narciso, and Concepcion Andrea Gonzales y [81]
individually issued[ ] to Francisco Gonzaless
Narciso married to Melquiades M. Virata, Jr.
six (6) children, specifically, TCT Nos. 1368-1373
Appearing on the Memorandum of TCT No. 5261 while TCT No. 1374 was issued in favor of all the
[79] [82]
is NOTA: Ap 2111 which reads as follows:[ ] children.[ ]

A/2111 Adjudicado el torreno As previously mentioned, the properties covered


descrito en este certificado de titulo, by TCT Nos. 1368-1374 were expropriated by the
a Rufina Narciso Vda. de Gonzales, Republic of the Philippines and were eventually
a cuenta de la participacion de osia subdivided and sold to various vendees. Eighteen
esta en (not legible) los tienes de la (18) lots were obtained by MRI from the years
eseledad de genanciales. Habida 1965 to 1974, while it acquired the lot covered by
entre la misma y el finado Francisco TCT No. 165119 in 1988. On the other hand, MEC
J. Gonzales, per una orden del Hon. acquired from PhilVille Development Housing
Fernando Jugo, Juez del Juzgado de Corporation Lot No. 19-B by virtue of Deed of
Primera Instancia de Manila Sala II, Exchange executed in its favor for which, TCT No.
dienada el 20 de Septiembre de 19 232568 was issue don 9 May 1991.
(not legible), en el Expidiente de
intestado del nombrado Francisco J. The 20 certificates of titles were traced by the
MANOTOKS, as follows:
name of Filemon Custodio, a transferee of
1) TCT No. 7528 registered in the name of Jose Dionisio, who was issued TCT No. 9853.
MRI covers Lot No. 2 of consolidation- Dionisios title in turn cancelled the Republics
subdivision plan (LRC) Pcs-1828 which has an [88]
TCT No. 36657-63.[ ]
area of 4,988 square meters. MRI purchased
this lot from one Basilio Caina who was issued
TCT No. 7526 which cancelled TCT Nos. 5) TCT No. 21107 issued to MRI covers
36657-62 registered in the name of the Lot 22 with an approximate area of 2,557
[83] square meters. MRI acquired the same by
Republic of the Philippines.[ ] virtue of sale between him and Francisco
Custodio, holder of TCT No. 21040. Francisco
2) TCT No. 7762, covering Lot 1-C, was Custodio was a transferee of Lorenzo Caina,
obtained by MRI from one Narcisa registered owner of TCT No. 21039 as
Buenaventura. The Parcel of land has an evidenced by a Deed of Sale between Caina
approximate area of 2,876 square meters. and the PHHC, the latters certificate of title
Buenaventuras ownership was evidenced by canceling TCT No. 36557-63 of the
[84] [89]
TCT No. 7525,[ ] deriving the same from Republic.[ ]
[85]
TCT No. 36657-63.[ ]
6) TCT No. 21485 was issued to MRI by
virtue of sale between it and Francisco
Custodio, registered owner of TCT No. 21484.
The certificate of title covers Lot 20 with an
approximate area of 25,276 square meters
3) TCT No. 8012 in the name of MRI Custodio was in turn a transferee of Lorenzo
covers Lot No. 12-1 having an area of 20,000 Caina, the latter being the registered owner of
[86] TCT No. 21013 by reason of sale between him
square meters.[ ] This certificate of title
was traced from one Filemon Custodio who [90]
and PHHC.[ ] Under Entry No. 6277/T-
held TCT No. 7792. Custodio was in turn a 21485, it would appear that portions of the
transferee of Guillermo Rivera, the latter property covered under TCT No. 21485 and
having been issued TCT No. 7760 by virtue of TCT No. 232568 had been subject of an
sale between him and then Peoples Homesite expropriation proceedings to which the
and Housing Corporation [PHHC]. The latter Manotok Estate Corporation, et al. interposed
title eventually cancelled TCT No. 36557-63 of no objections subject to the payment of just
[87] [91]
the Republic.[ ] compensation.[ ]

4) TCT No. 9866 issued to MRI covers Lot


No. 21 and has an approximate area of 23,979
square meters. MRIs certificate of title was [92] [93]
derived from TCT No. 9854 registered in the 7) TCT Nos. 26405[ ] and 26406,[ ]
both registered in the name of MRI, cancelled This certificate of title cancels TCT No. 36557-
TCT Nos. 9773 and 9774, respectively. TCT [98]
63 of the Republic.[ ]
Nos. 9773 and 9774 were registered in the
names of Romulo, Rosalina, Lucila, Felix and
Emilia all surnamed Jacinto, [JACINTOS],
before the same were transferred to MRI by
reason of sale in favor of the latter. The
JACINTOS certificates of title were in turn
derived from TCT Nos. 8014 and 8015 issued
[94]
in the name of Filemon Custodio[ ] Both
TCT Nos. 8014 and 8015 cancelled TCT [99]
11) TCT No. 254875[ ] bears MRI as the
7792/T-39. However, for purposes of tracing
TCT No. 7792/T-39 to the Republics certificate registered owner of Lot 55-A with an area of
of titles, this certificate of title was not approximately 1,910 square meters. This
submitted in evidence. certificate of title cancelled TCT No. 41956
which covers Lot 55, also registered in the
[95] name of MRI. It would appear that MRI
8) TCT No. 26407[ ] issued to MRI was acquired the lot covered under TCT No. 41956
traced back to the title of Lourdes Mercado from one Joaquin Caina who was the
Cloribel who was the registered owner of TCT registered owner of TCT No. 25715 being a
No. 8404 by virtue of sale between the two, [100]
vendee of PHHC.[ ]
thereby transferring ownership to MRI. On the
fact of TCT No. 8404, it would show that it
cancelled TCT No. 8013/T41 but there is no 12) TCT No. 53268 of MRI covered Lot No.
showing in whose name TCT No. 8013 was [101]
15,[ ] which was purchased by MRI from
registered and what certificate of title it one Maria V. Villacorta who held TCT No.
cancelled. 53155. Villacorta in turn acquired the same
land from one Eufrocina Mackay whose TCT
[96] No. 7827 was eventually cancelled by
9) TCT No. 33904[ ] of MRI cancelled
TCT No. 8017 of Filemon Custodio by virtue of [102]
Villacortas land title.[ ] It would appear
[97] that TCT No. 7827 cancelled TCT No. 7826/T-
sale between the latter and MRI.[ ] We
note that TCT No. 8017 cancelled TCT 40 but there is no trace to whom the latter title
No.7792/T-39 but there is no showing whether was registered and what certificate of title it
the same could be traced back to the cancelled.
Republics certificates of title.
13) TCT No. 55897 shows MRI as the
10) TCT No. 34255, covering Lot No. 11- registered owner of Lot 3 of the consolidation-
Bm, Psd-75797 with an area of 11,000 square subdivision plan (LRC) Pcs-1828 of the
meters, reflects MRI as the registered owner. Maysilo Estate covering an area of more or
less 20,531 square meters. This certificate of was subdivided into two lots, namely, Lot Nos.
title cancelled TCT No. 53122 in the names of 56-A and 56-B. TCT No. 25146 cancelled TCT
MRI (19,531 square meters) and one Silvestre No. 25145 registered in the name of Quirino
Domingo (1,000 square meters). TCT No. Labing-isa by virtue of sale in favor of MRI. In
53122 in turn cancelled TCT No. 21347 turn, TCT No. 21545 cancelled TCT Nos.
registered in the names of Jesus Hipona [106]
(36557) 12836 to (36563) 12842.[ ]
(19,531 square meters) and Silvestre Domingo
(1,000 square meters). Notably, TCT No.
21347 cancelled TCT No. 21315/T-107 but 16) TCT No. T-121428, registered in the
there is no indication to whom TCT No. 21315 name of MRI covers Lot No. 5-C of subdivision
was registered and what certificate of title it plan (LRC) psd-315272 which has an
[103] approximate area of 4,650 square meters. It
cancelled.[ ] was previously registered in the names of MRI
(4,650 square meters), Ricardo Cruz (941
14) TCT No. C-17272 reflects MRI as the square meters) and Conchita Umali (1,000
registered owner of Lot 6-C which has an square meters) under TCT No. 53123 by order
approximate area of 27,850 square meters. of the Court of First Instance of Rizal,
MRIs certificate of title cancelled TCT No. C- Caloocan City, Branch XII and as per
17234 registered in the names of MRI (27,750 agreement of the parties in Civil Case No. C-
square meters), Roberto S. David (3,0000 424. TCT No. 53123 in turn cancelled TCT No.
square meters) and Jose Madulid (500 square 21346 whose registered owners were
meters). It would appear that TCT No. C- Conchita Umali (1,000 square meters),
17234 cancelled TCT No. 53124 registered in Ricardo Cruz (941 square meters) and Jesus
the names of MRI, Spouses Priscila and [107]
Hipona (4,650 square meters).[ ] Like
[104]
Antonio Sebastian and Jose Madulid.[ ] some of the other titles, TCT No.21346
MRI also submitted in evidence a Deed of cancelled TCT No. 21316 but there is no trace
Partition between itself, Roberto David and of this latter certificate of title.
Madulid thereby subdividing the property into
Lots 6-A, 6-B and 6-C as per subdivision plan 17) TCT No. 163902, registered in the
[105] name of MRI, covers Lot No. 4-B-2 and has an
(LRC) Psd-277091.[ ] Again, we note that
area of more or less 6,354 square meters and
TCT No. 53124 cancelled TCT No. 21350/T- a by-product of TCT No. 9022, also in the
107 but the records are bereft of any indication name of MRI, after the same was subdivided
what certificate of title it cancelled and to under subdivision plan (LRC) Psd-334454.
whom the same was registered. TCT No. 9022, in turn, cancelled TCT No.
8994/T-45 registered in the name of Filemon
15) TCT No. C-35267, covering Lot 56-B of S. Custodio whose ownership thereon was
subdivision plan (LRC) Psd-292683 with an transferred to MRI by virtue of a voluntary
approximate area of 9,707 square meters, was [108]
a by-product of TCT No. 25146, also sale.[ ] TCT No. 8894 cancelled TCT No.
registered in the name of MRI, after the same 8846/T-45 but this latter certificate of title was
not submitted in evidence for purposes of expropriation proceedings sometime in 1947
tracing back to the Republics title. under Commonwealth Act No. 539 for resale to
tenants is beyond question, as also enunciated by
[109] the Supreme Court in Republic of the Philippines
18) TCT No. 165119[ ] was issued to
v. Jose Leon Gonzales, et al. To bolster this fact,
MRI by virtue of a Deed of Sale between paragraph r of the Majority Report noted that the
Spouses Francisca Labing-isa and Juan seven properties covered by TCT Nos.1368 to
Ignacio [SPOUSES IGNACIO] and MRI, as a 1374 were expropriated by the Republic from the
result of which, TCT No. C-36960 of the Gonzalezes.
[110]
SPOUSES IGNACIO was cancelled.[ ] It
would appear that TCT No. C-39690 cancelled The fact that these lots were subjected to
TCT No. 35266/T-173 but TCT No. 35266/T- expropriation proceedings sometime in 1947
173 was not submitted in evidence. under Commonwealth Act No. 539 for resale to
tenants is beyond question, as also enunciated by
19) TCT No. T-232568 of the Manotok the Supreme Court in Republic of the Philippines
Estate Corporation, covering Lot No. 19-B of vs. Jose Leon Gonzaels, et al. To bolster this fact,
subdivision plan Psd-13011152 with an area paragraph r of the Majority Report noted that the
of 23,206 square meters, was derived from the seven properties covered by TCT Nos.1368 to
certificate of title held by PhiVille Development 1374 were expropriated by the Peoples Homesite
and Housing Corporation under TCT No. and Housing Corporation which were later
197357. MEC acquired the subject parcel of consolidated and subdivided into 77 lots for resale
land by virtue of Deed of Exchange between it to tenants. No sign of protest was ever raised by
[111] [112]
and PHILVILLE DATED 9 May 1991.[ ] CLT on this point.
TCT No. 197357 cancelled TCT No.
195730/T-974 but there is no trace what
certificate of title the latter title cancelled. The fact of expropriation is extremely significant,
By and large, all the certificates of title for titles acquired by the State by way of expropriation
submitted by the MANOTOKS, including their
derivative titles, were all traced to OCT No. 994 are deemed cleansed of whatever previous flaws may
registered on 3 May 1917. Likewise, they declared have attended these titles. As Justice Vitug explained in
all the lots covered by such titles for taxation
purposes. Without doubt, MRI had successfully [113]
traced back some of their certificates of title to the Republic v. Court of Appeals, and then Associate
valid OCT No. 994, they having acquired the lots
from some of the vendees of the PHHC after the Justice (now Chief Justice) Puno reiterated in Reyes v.
same were expropriated by the Republic from the
Gonzalezes. [114]
NHA: In an rem proceeding, condemnation acts
The fact that these lots were subjected to
upon the property. After condemnation, the paramount September of 1918. However, TCT No. 4211 was
title is in the public under a new and independent title; issued decades before the property was expropriated.
thus, by giving notice to all claimants to a disputed title, Thus, any and all defects that may have attended that
condemnation proceedings provide a judicial process for particular title would have been purged when the
securing better title against all the world than may be property covered by it was subsequently acquired by the
[115] State through eminent domain. The Special Division
obtained by voluntary conveyance. This doctrine
noted as much:
was derived from the opinion of then Chief Judge (now
U.S. Supreme Court Justice) Stephen Breyer in As it is, the validity of most of MRIs
certificates of title should be upheld because they
[116] were derived from the Republics valid certificates
Cadorette v. U.S., which in turn cited the
of title. In fact, some of the MANOTOKS titles can
be traced back to the Governments titles as a
pronouncement of the U.S. Supreme Court in U.S. v. result of the expropriation in 1947.
[117]
Carmack that [b]y giving notice to all claimants to a Relevantly, the titles of the Republic, as the
predecessor-in-interest of the MANOTOKS, are
disputed title, condemnation proceedings provide a presumed valid by virtue of their acquisition
resulting from the exercise of its inherent power of
judicial process for securing better title against all the eminent domain that need not be granted even by
the fundamental law. Thus, the alleged flaws
world than may be obtained by voluntary concerning the certificates of title issued previous
to the exercise of the State of its inherent power
[118] did not affect or render invalid the subsequent
conveyance.
transfers after the forced sale. Indeed, when land
has been acquired for public use in fee simple
unconditionally, either by the exercise of eminent
domain or by purchase, the former owner retains
no rights in the land, and the public use may be
abandoned, or the land may be devoted to a
different use, without any impairment of the estate
or title acquired or any reversion to the former
In annulling the Manotok titles, focus was laid on [119]
owner.
the alleged defects of TCT No. 4211 issued in
title.
The Special Division also took exception to the
With respect to the imputed flaws on the
majority report of the Commissioners (Majority Report) MANOTOKS titles which were based on the
Majority Report, we find that the bases of the
who had been tasked by the trial court to examine the alleged defects proceeded from unreliable sources
thus, tainting the veracity of the said report.
validity of the Manotok titles. The Majority Report
The records of the case between CLT and
the MANOTOKS reveal that the parties approved
the creation of a commission to resolve only these
two issues, to wit:
had arrived at several conclusions with respect to the
xxx
[120]
TCTs from which the Manotok titles were derived.
These issues to be resolved by the
3 Commissioners are as follows:
The Special Division, however, concluded that such
report was in fact tainted by the fact that it was 1) Whether or not the
property covered by the Transfer
determined outside the scope of the issues framed and Certificates of Title of defendants
pertain to or involve Lot No. 26 of
agreed upon by the parties. To wit: the Maysilo Estate presently titled in
the name of the plaintiff; and
In meeting the issue, the MANOTOKS
disproved the opinion with regard to the alleged 2) Whether or not the
defects of their titles inasmuch as the majority property covered by the title of the
report submitted before the trial court was made plaintiff and the property covered by
outside the scope of the tasks which the trial court the titles of the defendants
confined them to perform. The MANOTOKS also [121]
overlap.[ ]
argued that before this proceeding on remand,
CLT failed to introduce evidence of such flaws Scrutinizing the Majority Report upon which
neither were the concerned geodetic engineers the trial courts conclusions were based, it would
presented as witnesses. Moreover, the appear that the findings therein were outside the
MANOTOKS further maintained that CLT failed to scope of the issues framed and agreed upon by
submit any factual or legal bases to prove the the parties. Specifically, the deductions with
authenticity and validity of the Palma and Sayo regard to the technical infirmities and defects of
Orders. They insisted that the Palma Order was a TCT Nos. 4211, 4210, 5261 and 35486 do not
void one for being conditional and having resulted involve the question of whether or not the subject
to the issuance of duplicate certificates of land properties were identified as Lot No. 26 of the
Maysilo estate or whether there was overlapping
of titles. Records bear out that the MANOTOKS Although the MANOTOKS had traced their
took exception to the procedure taken citing title from the vendees of PHHC, there are,
therein the ultra vires acts of the two however, some certificates of title which could not
Commissioners. be traced back to the titles previously held by the
Republic specifically, MRIs TCT Nos. 26405 and
In addition, the majority report focused on 26406, 26407, 33904, 53268, 55897, C-17272, T-
the alleged flaws and inherent technical defects of 121428, 163903, 165119 and MECs TCT No. T-
TCT Nos. 4211, 5261 and 35486, ranging from the 232568. As to these certificates of title, the
language of the technical descriptions, absence of MANOTOKS failed to make any specific reference
subdivision plan, lot number and survey plan. to the preceding certificates of title which they
Evidently, these defects go only as far as the cancelled and to whose names they were
certificates of title issued prior to those of the subsequently transferred and registered.Thus, we
Republic. Remarkably, no specific flaw was found find no sufficient basis to make a conclusion as to
on the MANOTOKS titles indicating any irregularity [125]
on their issuance. In fact, the Commissioners who their origins.
signed the majority report even concluded that
only TCT Nos. 4211, 4210, 5261, 35486, 1368
[122] V.
thru 1324 (sic)[ ] were irregularly and
questionably issued without any reference to the
[123]
MANOTOKS certificates of title.[ ] Otherwise
The Special Division supplied the following precise
stated, the imputed flaws affect only those
certificates of title issued prior to those registered and concise summary of its conclusions:
in the name of the Republic. No flaw had been
specifically identified or established in the
proceedings below, which would taint the titles In prcis, the factual milieu of the present
held by the MANOTOKS in so far as the regularity controversy and the evidence on record clearly
[124] establish the failure of DIMSON and CLT to
of their issuance is concerned. substantiate their titles and overcome the onus of
proving that said titles are derivatives of OCT 994
registered on 3 May 1917, and not 19 April 1917,
At the same time, the Special Division was not as what is reflected in their titles. In contrast, the
MANOTOKS and ARANETA, both of which had
prepared to uphold the validity of all of the Manotok consistently anchored their proprietary claims on
OCT No. 994 registered on 3 May 1917, have, in
titles. It took issue with the particular titles which could this remand proceeding, been able to support their
claims of ownership over the respective portions of
not be retraced to the titles acquired by the Republic of the Maysilo Estate. Except in the case of the
the Philippines by way of expropriation. MANOTOKS which had failed to substantiate the
validity of some of their certificates of title, the
MANOTOKS and ARANETA presented evidence and CLT cannot rightly insist on the validity of their
proving the identity, the extent and the origin of titles. Such flaws are hard to overcome as they
their titles. delve into the substance of their proprietary
claims. As stated, DIMSON and CLT miserably
Answering the issues assigned by the failed to overcome their onus and instead opted to
Supreme Court relative to the tenability of the hap on the supposed flaws of the adverse parties.
respective imputed flaws in the titles of the For these reasons, the titles of DIMSON and CLT
MANOTOKS and ARANETA and whether such should be declared a nullity.
flaws are sufficient to defeat said claims, this Court
finds that, as discussed above, such flaws are xxx
inconsequential and ineffectual in invalidating the
MANOTOKS and ARANETA titles. From the foregoing evaluation and in
conformity with the Supreme Court 2007
Significantly, since the respective Resolution, this Court arrived at the following
certificates of title of herein contending parties are conclusions as to the status of the original title and
contradictory to each other and stand to refute the its subsequent conveyances:
validity of their opposing titles, it cannot be
gainsaid that said certificates of title have 1. As categorically declared by the Supreme
correspondingly been subjected to dispute on the Court, there is only one OCT 994, the registration
basis of separate and distinct imputed flaws. Still, date of which had already been decisively settled
the crucial difference between the imputed flaws as 3 May 1917 and not 19 April 1917.OCT 994
allegedly tainting said contending titles, DIMSON which reflects the date of 19 April 1917 as its
and CLT on one hand, and the MANOTOKS and registration date is null and void.
ARANETA, on the other, is that the imputed flaws
purportedly beleaguering the respective
certificates of title of the MANOTOKS and
ARANETA relate to the mechanical and technical
aspect of the transcription of their titles and are
therefore inconsequential to the import and validity 2. In view thereof and in addition to other
thereof. Said imputed flaws do not depart from the grounds we have already discussed, the
fact that the predecessors-in-interest of the certificates of title of the deceased Jose Dimson
MANOTOKS and ARANETA had been clothed and his successor-in-interest, CLT, having been
with the right of ownership over the disputed traced back to OCT 994 dated 19 April 1917, are
portions of the Maysilo Estate. NULL and VOID and thus vest no legal right or
claim in favor of DIMSON and CLT.
On the other hand, the flaws attending the
titles of DIMSON and CLT primarily stem from 3. The 13 June 1966 Palma Order and the
infirmities attending or otherwise affecting the very 18 October 1977 Sayo Order, on which DIMSON
crux of their claim of ownership. Having derived and CLT anchor the validity of their respective
their titles from RIVERA, whose title is titles, do not substantiate their proprietary claims.
questionable and dubious to the core, DIMSON
While the existence of said Orders are admitted, evaluation of the Special Division, we likewise adopt the
the legal import thereof nonetheless fails to confer
a semblance of legality on the titles of DIMSON above conclusions. As we earlier stated, it was
and consequently, of CLT, more so, a superior
right to defeat the titles of the MANOTOKS and incumbent on the Heirs of Dimson and/or CLT to
ARANETA, respectively. establish their claim to title for reasons other than the
4. Portions of Lot No. 26 pertinent to this fact that OCT No. 994 dated 19 April 1917 is extant.
controversy, particularly that being disputed by the
MANOTOKs and CLT, were expropriated by the They failed to do so. It should be noted that the instant
Republic of the Philippines sometime in 1947
under Commonwealth Act No. 539 for resale to cases arose from separate actions filed by Jose Dimson
tenants. The MANOTOKS, thus as successor-in- and CLT seeking the recovery of possession and/or
interest of the Republic, were able to establish that
some of their certificates of title had indeed annulment of title against Araneta and the Manotok
originated or were derived from said expropriated
parcels of land. Group. Thus, the burden of evidence was on Dimson

5. The evidence on record confirm that the and CLT to establish the strength of their respective
certificates of title covering the land being claimed claims of ownership, and not merely to rely upon
by ARANETA were derived from OCT NO. 994
registered on 3 May 1917 thereby ultimately whatever weaknesses in the claims of the Manotoks and
showing a direct link of TCT Nos. 7784 and 13574
to said mother title. By reason of which, that is Araneta for their causes of action to prosper. The well-
either belonging to or portions of Lot 25-A-3 as
previously owned by RATO, had been well settled legal principle in actions for annulment or
substantiated and proven to be superior to that of reconveyance of title is that a party seeking it should
DIMSON.
establish not merely by a preponderance of evidence
6. For reasons above-stated and in view of
the established rights of ownership of both the but by clear and convincing evidence that the land
MANOTOKS and ARANETA over the contested
properties, we find that the imputed flaws on their [127]
sought to be reconveyed is his. In an action to
titles cannot defeat the valid claims of the
MANOTOKS and ARANETA over the disputed
recover, the property must be identified, and the plaintiff
[126]
portions of the Maysilo Estate.
must rely on the strength of his title and not on the

Inasmuch as we agree with the factual findings and


[128]
weakness of the defendant's claim.
meters.

We now proceed to tackle the recommendations e) TCT No. 21107 covering Lot 22
with an approximate area of 2,557 square meters.
submitted by the Special Division. They are as follows:
f) TCT No. 21485 covering Lot 20
RECOMMENDATIONS with an approximate area of 25,276 square
meters.

Apropos to said conclusions, this Court g) TCT No. 34255 covering Lot No.
hereby respectfully makes the following 11-Bm, Psd-75797 with an area of 11,000 square
recommendations regarding the validity of the meters.
conflicting proprietary claims as interposed by the
herein contending parties: h) TCT No. 254875 covering Lot 55-
A with an area of approximately 1,910 square
1. To declare with finality that the certificates meters.
of title of DIMSON and CLT including other
derivative titles issued to their successors-in- i) TCT No. C-35267 covering Lot 56-
interest, if any, are NULL and VOID, thus B of subdivision plan (LRC) Psd-292683 with an
invalidating their legal claims over the subject approximate area of 9,707 square meters.
parcels of land.
With regard to the following certificates of
2. To declare LEGAL and VALID the title, namely:
proprietary claims the MANOTOKS over the
parcels of land covered by the following 3.A. MANOTOK REALTY INC.
certificates of title:

a) TCT No. 7528 registered in the a) TCT No. 26405 covering Lot No. 12-E
name of MRI covers Lot No. 2 of consolidation- with an area of 1,0000 square meters.
subdivision plan (LRC) Pcs-1828 which has an
area of 4,988 square meters. b) TCT No. 26406 covering Lot No. 12-F
with an area of 1,000 square meters.
b) TCT No. 7762 covering Lot 1-C,
with an approximate area of 2,287 square meters. c) TCT No. 26407 covering Lot No. 12-B
with an area of 1,000 square meters.
c) TCT No. 8012 covering Lot No.
12-1 having an area of 20,000 square meters. d) TCT No. 33904 covering Lot No. 12-H
with an area of 1,802 square meters.
d) TCT No. 9866 covering Lot No. 21
and has an approximate area of 23,979 square e) TCT No. 53268 covering Lot No. 15
purchased by MRI from one Maria V. Villacorta
with an approximate area of 3,163 square meters. the following certificates of title:

f) TCT No. 55897 covering Lot 3 of a) TCT No. 13574 covering a parcel of
consolidation-subdivision plan (LRC) Pcs-1828 of land designated as Section No. 2 of subdivision
the Maysilo Estate covering an area of more or plan Psd-10114, being a portion of Lot 25-A-3-C
less 20,531 square meters. with an aggregate area of 581,872 square meters;

g) TCT No. C-17272 covering Lot 6-C b) TCT No. 7784 covering four (4) parcels
which has an approximate area of 27,850 square of land with an aggregate area of 390,383 square
meters. [129]
meters.
h) TCT No. T-121428 covering Lot No. 5-
C of subdivision plan (LRC) psd-315278, which
has an approximate area of 4,650 square meters.

i) TCT No. 163902 covering Lot No. 4-B-


2 with an area of more or less 6,354 square
meters allegedly a by-product of TCT No. 9022, The first, second and fourth recommendations are
which in turn, cancelled TCT No. 8994/T-45
registered in the name of Filemon S Custodio. well taken as they logically arise from the facts and
j) TCT No. 165119 which allegedly conclusions, as determined by the Special Division,
cancelled TCT No. C-36960 of the SPOUSES
which this Court adopts.
IGNACIO by virtue of a Deed of Sale between said
Spouses and MRI.

3.B. MANOTOK ESTATE CORPORATION The third recommendation that eleven (11) of the
a) TCT No. T-232568 covering Lot No. 19-B of titles held by the Manotoks be declared null and void or
subdivision plan Psd-13011152 with an area of
subjected to further technical verification warrants some
23,206 square meters.
analysis.
The foregoing certificates of title (3.A and
3.B), failing to make specific references to the
particular certificates of title which they cancelled
and in whose name they were registered, may be The Court has verified that the titles mentioned in
declared NULL and VOID, or in the alternative,
the third recommendation do not, as stated by the
subject the same to further technical verification.
Special Division, sufficiently indicate that they could be
4. To declare LEGAL and VALID the title of
ARANETA respecting parcels of land covered by traced back to the titles acquired by the Republic when
it expropriated portions of the Maysilo Estate in the It is worth mentioning that the Special Division
1940s. On the other hand, the Manotok titles that were refused to adopt the Majority Report earlier rendered in
affirmed by the Special Division are traceable to the the case between the Manotoks and CLT, said report
titles of the Republic and thus have benefited, as they having exhaustively listed the perceived flaws in the
should, from the cleansing effect the expropriation had antecedent TCTs from which the Manotoks derived their
on whatever flaws that attached to the previous titles. claim. The Special Division concluded that such findings
However, although the Special Division did not concede had been reached by the Commissioners in excess of
the same benefit to the other Manotok titles named in their original mandate and, thus, ultra vires. Assuming
the third recommendation, at the same time it did not that such flaws were extant, they existed on the titles
conclude that such titles were false or fraudulently and anteceded the expropriation of the properties by the
acquired. Absent such a finding, we are disinclined to Government. As stated earlier, such expropriation would
take the ultimate step of annulling those titles. have cleansed the titles of the prior flaws. But even if
the Manotok titles enumerated in the third
Said titles have as their origin what we have recommendation could not be sourced from the titles
acknowledged to be a valid mother title OCT No. 994 acquired by the Republic through expropriation, still the
dated 3 May 1917. This is in stark contrast with the titles rejection of the Majority Report signifies that the flaws
of CLT, the oppositors to the Manotoks, which all advert adverted to therein could not form the basis for the
to an inexistent mother title. On their face, the Manotok annulment of the titles involved. Indeed, the Special
titles do not reflect any error or fraud, and certainly the Divisions rejection of the Majority Report further
Special Division do not point to any such flaw in these diminishes any ground to annul the Manotok titles
titles. Nothing on the face of the titles gives cause for referred to in the third recommendation.
the Court to annul the same.
Yet, the Court is cognizant that the inability to
trace the Manotok titles specified in the third court.
recommendation to those titles acquired by the
Government through expropriation puts such titles in WHEREFORE, the Court hereby adopts the
doubt somehow. In addition, the Court is aware that the Report of the Special Division and issues the following
ground utilized by the Special Division in rejecting the reliefs:
Majority Report that the determinations were made
outside the scope of the issues framed and agreed upon 1) The certificates of title of the DIMSONs and CLT
by the parties -- does not categorically refute the including other derivative titles issued to their
technical findings made therein. Those circumstances, successors-in-interest, if any, are declared NULL and
while insufficient for now to annul the Manotoks titles VOID, thus invalidating their legal claims over the subject
listed in the third recommendation, should be sufficiently parcels of land;
made public.
2. The proprietary claims of the MANOTOKS over the
parcels of land covered by the following certificates of
Hence, in lieu of annulling the Manotok titles per
title are declared LEGAL and VALID, to wit:
the Special Divisions third recommendation, the Court
deems it sufficient to require the Registers of Deeds
a) TCT No. 7528 registered in the
concerned to annotate this Resolution on said titles so name of MRI covers Lot No. 2 of
consolidation-subdivision plan
as to sufficiently notify the public of their unclear status,
(LRC) Pcs-1828 which has an
more particularly the inability of the Manotoks to trace area of 4,988 square meters.
the titles without any gap back to OCT No. 994 issued
on 3 May 1917. If there should be any cause for the
annulment of those titles from a proper partys end, then
b) TCT No. 7762 covering Lot 1-
let the proper case be instituted before the appropriate C, with an approximate area of
2,287 square meters.
c) TCT No. 8012 covering Lot No. a) TCT No. 13574 covering a
12-1 having an area of 20,000 parcel of land designated as
square meters. Section No. 2 of subdivision plan
Psd-10114, being a portion of Lot
d) TCT No. 9866 covering Lot No. 25-A-3-C with an aggregate area of
21 and having an approximate 581,872 square meters;
area of 23,979 square meters.
b) TCT No. 7784 covering four (4)
e) TCT No. 21107 covering Lot parcels of land with an aggregate
22 with an approximate area of area of 390,383 square meters.
2,557 square meters.

f) TCT No. 21485 covering Lot 20


with an approximate area of 4) On the following titles in the name of
25,276 square meters.
Manotok Realty, Inc. or Manotok Estate Corporation, to
g) TCT No. 34255 covering Lot wit:
No. 11-Bm, Psd-75797 with an
area of 11,000 square meters.
a) TCT No. 26405 covering Lot
h) TCT No. 254875 covering Lot No. 12-E with an area of 1,0000
55-A with an area of approximately square meters;
1,910 square meters.
b) TCT No. 26406 covering Lot
i) TCT No. C-35267 covering Lot No. 12-F with an area of 1,000
56-B of subdivision plan (LRC) square meters;
Psd-292683 with an approximate
area of 9,707 square meters. c) TCT No. 26407 covering Lot
No. 12-B with an area of 1,000
3) The following certificates of titles in the name of square meters;
ARANETA are hereby declared LEGAL and VALID, to
d) TCT No. 33904 covering Lot
wit: No. 12-H with an area of 1,802
square meters;
36960 of the SPOUSES IGNACIO
e) TCT No. 53268 covering Lot by virtue of a Deed of Sale
No. 15 purchased by MRI from between said spouses and MRI;
one Maria V. Villacorta with an
approximate area of 3,163 square k) TCT No. T-232568 covering
meters; Lot No. 19-B of subdivision plan
Psd-13011152 with an area of
f) TCT No. 55897 covering Lot 3 23,206 square meters.
of consolidation-subdivision plan
(LRC) Pcs-1828 of the Maysilo
Estate covering an area of more or the Registers of Deeds concerned are ordered to
less 20,531 square meters;
annotate that as determined in the foregoing Resolution,
g) TCT No. C-17272 covering Lot the registered owners of the said titles failed to make
6-C which has an approximate
any specific reference to the preceding certificates of
area of 27,850 square meters;
title which they cancelled and to whose names they
h) TCT No. T-121428 covering
were subsequently transferred and registered, thereby
Lot No. 5-C of subdivision plan
(LRC) psd-315278, which has an leading the Supreme Court to find no sufficient basis to
approximate area of 4,650 square
[130]
meters; make a conclusion as to their origins.

i) TCT No. 163902 covering Lot


No. 4-B-2 with an area of more or
Costs against private respondents.
less 6,354 square meters allegedly
a by-product of TCT No. 9022,
which in turn, cancelled TCT No.
SO ORDERED.
8994/T-45 registered in the name
of Filemon S. Custodio;

j) TCT No. 165119 which


allegedly cancelled TCT No. C-
ANTONIO T. CARPIO MA. ALICIA AUS
Associate Justice Associa
DANTE O. TINGA
Associate Justice

WE CONCUR:

RENATO C. CORONA CONCHITA CA


Associate Justice Associate Justice

REYNATO S. PUNO
Chief Justice

MINITA V. CHICO-NAZARIO PREBITERO J


Associate Justice Associa

LEONARDO A. QUISUMBING CONSUELO YNARES-SANTIAGO


Associate Justice Associate Justice
NIO EDUARDO B. NACHURA TERESITA J. LEONARDO
[1] DE CASTRO
See also 540 SCRA 304.
Associate Justice Associate Justice
[2]
Manotok Realty v. CLT Realty, G.R. Nos. 123346 & 134385, 14 December
2007, 540 SCRA 304.

[3]
Hereinafter, Report. Penned by Associate Justice J. Guevara-Salonga,
concurred in by Associate Justices L. Bersamin and J. Dimaampao.

*
Through advertence, the number 17 appeared in the original.

[4]
Report, pp. 5-16.
ARTURO D. BRION DIOSDADO M. PERALTA
[5]
Associate Justice Associate Justice
Rollo of the Special Division, Vol. I, pp. 771-809.

[6]
Rollo of the Special Division, Consolidated Offer of Evidence, Vol. II, pp.
1584-1619.

[7]
Id. at 1626-1638.

CERTIFICATION [8]
Id. at 1541-1581.

Pursuant to Article VIII, Section 13 of the Constitution, it [9]


Report, pp. 19-21.

is hereby certified that the conclusions in the above [10]


See id. at 26-27.

Resolution were reached in consultation before the case [11]


See id. at 6.

was assigned to the writer of the opinion of the Court. [12]


See id. at 12-13.

[13]
Id. at 24-25.

[14]
Rollo of the Special Division, DIMSONs Exhibit J-Dimson.

REYNATO S. PUNO [15]


Id. at 23-24.
Chief Justice
[16]
Transcript of Stenographic Notes, in RTC Civil Case No. C-8050, 10 July
2008, pp. 14-15. [33]
J. Corona, Concurring and Dissenting Opinion, Manotok Realty v. CLT,
[17] supra note 2 at 412-414.
Id. at 25-26.
[34]
G.R. No. 103558, 17 November 1992, 215 SCRA 783.
[18]
Brief for Plaintiff-Appellee, rollo, SC-G.R. No. 134385, p. 266.
[35]
G.R. No. 96259, 3 September 1996, 261 SCRA 327.
[19]
Id. at 266-267.
[36]
Rollo of the Special Division, CLTs Exhibit 3-A-CLT.
[20]
Transcript of Stenographic Notes, in RTC Civil Case No. C-8050, 25
August 1981, pp. 4-5, 7. [37]
Id.
[21]
Rollo of the Special Division, MANOTOKS Exhibit 63. [38]
Report, pp. 35-36.
[22]
Id., MANOTOKS Exhibit 64. [39]
Original records, SC-G.R. No. 134385, Vol. II, RTC Decision, pp. 337-339.
[23]
Id., ARANETAs Exhibit 12-AIA. [40]
Report, pp. 8-11.
[24]
Id., DIMSONs Exhibit J-Dimson. [41]
Rollo of the Special Division, ARANETAs Exhibit 24-A-AIA.
[25]
Report, pp. 26-32. [42]
Id.
[26]
Rollo of the Special Division, MANOTOKS Exhibit 41. [43]
Rollo of the Special Division, ARANETAs Exhibit 4-AIA.
[27]
REGISTRATION OF LAND TITLES AND DEEDS, NOBLEJAS AND [44]
Another TCT No. 8692, as per certification of Acting Register of Deeds of
NOBLEJAS, 1992 Revised Ed., p. 292.
Malabon City, Navotas, Josephine Ponciano, surfaced during the hearing upon
[28] a subpoena duces tecum applied for by the counsel for the Heirs of Dimson.
REGISTRATION OF LAND TITLES AND DEEDS, PEA, PEA, JR., PEA, This TCT No. 8692 is registered under the name of Gregorio Araneta,
1994 Revised Ed., p. 144. Incorporated and located at Tinajeros, Malabon, Rizal, designated as Lot Nos.
1 and 2, Block No. 44 of the consolidation and subdivision plan Pcs-188. It also
[29] showed that it cancelled TCT No. 46118 and its mother title was traced back to
Alzate v. Philippine National Bank, L-20068, 26 June 1967, 20 SCRA 422. OCT NO. 994 registered on 3 May 1917. Rollo of the Special Division, Vol. I,
pp. 1229-1230.
[30]
Republic v. Lourdes Abiera Nillas, G.R. No. 159595, 23 January 2007, 512
SCRA 286.
[45]
Rollo of the Special Division, ARANETAs Exhibit 4-G-AIA.
[31]
Records of CA-G.R. SP No. 34819, Vol. I, pp. 94-97.
[46]
Id., ARANETAs Exhibit 5-E-AIA.
[32]
Report, pp. 32-34.
[47]
Entry No. 12343 of the owners duplicate copy of OCT NO. 994 makes a
[60]
reference to TCT No. 8692 and Lot No. 25-A-3, Exhibit 24-A-AIA. MC Engineering, Inc. v. Court of Appeals, G.R. No. 104047, 3 April, 2002,
380 SCRA 116.
[48]
The lot area could not be determined from the certificate of title submitted.
[61]
Alonso v. Cebu Country Club, Inc., G.R. No. 130876, 31 January 2002,
[49] 375 SCRA 390.
Exhibit 7-AIA.
[62]
[50] G.R. No. 162704, 19 November 2004, 443 SCRA 293.
Exhibit 8-AIA.
[63]
[51] Encinas v. National Bookstore, Inc., id.
Rollo of the Special Division, ARANETAs Exhibit 8-A-AIA and 8-C-AIA.
[64]
[52] Rollo of the Special Division, ARANETAs Exhibit 2-G-AIA.
Id., ARANETAs Exhibit 19-AIA.
[65]
[53] Id., ARANETAs Exhibit 24-A-AIA.
Id., ARANETAs Exhibit 21-AIA.
[66]
[54] Original Records, SC-G.R. No. 134385, Vol. II, RTC Decision, p. 337.
Id., ARANETAs Exhibit 19-AIA.
[67]
[55] Exhibit K, Folder of Exhibits, RTC Civil Case No. C-8050.
As per certification of Reynaldo S. Vergara, Acting Register of Deeds,
upon the request of one Crisanta Santos appearing on the dorsal portion of [68]
Exhibit 7-AIA. Exhibit L, Folder of Exhibits, RTC Civil Case No. C-8050.

[56] [69]
Rollo of the Special Division, ARANETAs Exhibit 7-AIA. Rollo of the Special Division, DIMSONs Exhibit M-Dimson.

[57] [70]
Covering (1) Lot No. 1, Block No. 127 of the subdivision plan Psd-20096 Id., DIMSONs Exhibit N-Dimson.
being a portion of Lot No. 2, Block No. 100 of subdivision plan Psd-17729,
G.L.R.O. Record No. 4429 with an area of 5,625 square meters; (2) Lot No. 2, [71]
Rollo of the Special Division, DIMSONs Exhibit Q-Dimson.
Block No. 130 of the subdivision plan Psd-20096 being a portion of Lot No. 2,
Block No. 100 of the subdivision plan Psd-17729, G.L.R.O. Record No. 4429
with an area of 3,440 square meters; (3) Block No. 131 of the subdivision plan [72]
G.R. No. 130876, 31 January 2002, 375 SCRA 390.
Psd-20096 being a portion of Lot No. 2, Block No. 100 of the subdivision plan
Psd-17729, G.L.R.O. Record No. 4429 with an area of 7,840 square meters;
[73]
and (4) Lot No. 6 of the subdivision plan Psd-21943, being a portion of Block Rollo of the Special Division, Vol. II, pp. 2433-2436.
No. 132 of the subdivision plan Psd-20096, G.L.R.O. Record No. 4429 with an
area of 373,377 square meters. [74]
Id. at 1664-1675.
[58]
CA Decision, CA-G.R. SP No. 34819 and CA-G.R. SP No. 41883, Vol. II,
pp. 898-899. [75]
Report, pp. 51-63.

[59] [76]
Original Records, RTC Civil Case No. C-8050, p. 42. Particularly TCT Nos. 26538 and TCT Nos. 263539.
[92]
Covers Lot No. 12-E with an area of 1,000 square meters. Rollo of the
[77]
Rollo of the Special Division, MANOTOKS Exhibit 41. Special Division, MANOTOKS Exhibit 9.

[78] [93]
Id., CLTs Exhibit 3-F-1-CLT. Covers Lot No. 12-F with an area of 1,000 square meters. Rollo of the
Special Division, MANOTOKS Exhibit 10.
[79]
Id., CLTs Exhibit 3-F-CLT.
[94]
Rollo of the Special Division, MANOTOKS Exhibits 9C and 10B.
[80]
As per Entry No. 1368 appearing on TCT No. 35486, id., MANOTOKS
[95]
Exhibit 37. Covers Lot 12-B with an area of 1,000 square meters. Id., MANOTOKS
Exhibit 11.
[81]
Entry No. 3730/T-No. 1368 appearing on TCT No. 35486.
[96]
Covers Lot No. 12-H with an area of 1,802 square meters.
[82]
TCT Nos. 1368 to 1374, Rollo of the Special Division, MANOTOKS
[97]
Exhibits 30 to 36. Rollo of the Special Division, MANOTOKS Exhibit 12B.

[83] [98]
Rollo of the Special Division, MANOTOKS Exhibit 3. Id., MANOTOKS Exhibit 13B.

[84] [99]
The parcel of land was subdivided into three (3) lots, namely, Lot Nos. 1-A, Lot No. 55-A with an area of 1,910 square meters.
1-B and 1-C, under subdivision plan (LRC) Psd-42090.
[100]
Rollo of the Special Division, MANOTOKS Exhibit 14B.
[85]
Rollo of the Special Division, MANOTOKS Exhibit 4.
[101]
With an approximate area of 3,163 square meters.
[86]
Per Deed of Sale between Custodio and MRI, id., MANOTOKS Exhibit 5B.
[102]
Rollo of the Special Division, MANOTOKS Exhibits 15A and 15C.
[87]
Id., MANOTOKS Exhibit 5E.
[103]
Id., MANOTOKS Exhibits 16 and 16B.
[88]
Id., MANOTOKS Exhibit 6.
[104]
Id., MANOTOKS Exhibits 17 and 17A.
[89]
Id., MANOTOKS Exhibit 7.
[105]
Id., MANOTOKS Exhibit 17C.
[90]
TCT 21013 was not submitted in evidence but appears only in the Deed of
Absolute Sale between Custodio and Caina. The Deed of Sale between [106]
Custodio and PHHC was also submitted. Id., MANOTOKS Exhibit 8. Id., MANOTOKS Exhibits 18A and 18B.

[91] [107]
TCT No. 21485, attached to the Manifestation of MRI, submitted on 29 Id., MANOTOKS Exhibits 19, 19A and 19B.
October 2008 to the Special Division. Rollo of the Special Division, Vol. II, pp.
2144-2146. [108]
Id., MANOTOKS Exhibits 20A and 20B.
[109] conclusion that TCT Nos. 4211, 5261 and 35486 could not have been derived
Lot No. 56-A with an area of 415 square meters.
from OCT-994. It is the established procedure to always indicate in the
certificate of title, whether original or transfer certificates, the date of the
[110] original survey of the mother title together with the succeeding date of
Rollo of the Special Division, MANOTOKS Exhibit 21A.
subdivision or consolidation. Thus, in the absence of the original survey dates
of OCT-994 on TCT Nos. 4211, 5261 and 35486, then OCT-994 is not the
[111]
Id., MANOTOKS Exhibits 22, 22A and 22B. mother title of TCT Nos. 4211, 5261 and 35486, not only because the original
survey dates are different but because the date of original survey is always
[112] earlier than the date of the issuance of the original title. OCT-994 was issued
Report, pp. 37-47. on May 3, 1917 and this is much ahead of the date of survey indicated on TCT
Nos. 4210 and 4211 which is December 22, 1917;
[113]
433 Phil. 106 (2002).
c. Granting that the date December 22, 1917 is the date of a subdivision
survey leading to the issuance of TCT Nos. 4210 and 4211, there are,
[114] however, no indications on the face of the titles themselves which show that a
443 Phil. 604 (2003).
verified and approved subdivision of Lot 26 took place. In subdividing a lot, the
[115] resulting parcels are always designated by the lot number of the subdivided lot
See Republic v. Court of Appeals, supra at 121-122; Reyes v. NHA, followed by letters of the alphabet starting from the letter "A" to designate the
supra at 614. first resultant lot, etc., for example, if Lot 26 is subdivided into three (3) lots,
these lots will be referred to as Lot 26-A, Lot 26-N and Lot 26-C followed by a
[116] survey number such as "Psd-_____" or "(LRC) Psd-_____." However, the lots
988 F2d 215 (1993).
on TCT Nos. 4210 and 4211 do not contain such descriptions. In fact, the
parcels of land covered by TCT Nos. 4210 and 4211 are not even described by
[117] lot number, and this is again technically irregular and defective because the
329 U.S. 230 (1946).
designation of lots by Lot Number was already a practice at that time as
[118] exemplified by the technical descriptions of some sub-lots covered by OCT-
Id. at 239. 994, i.e., 23-A, 25-A, 25-D, etc.;

[119] d. That TCT Nos. 4210 and 4211 which allegedly was the result of a
Report, pp. 47-48.
subdivision of Lot 26 should not have been issued without a subdivision plan
approved by the Director of Lands or the Chief of the General Land
[120] Registration Office. Republic Act No. 496 which took effect on November 6,
In the light of the foregoing facts, the undersigned Commissioners have
come to the following conclusions: 1902, particularly Section 58 thereof, provided that the Registry of Deeds shall
not enter the transfer certificate to the grantee until a plan of such land showing
a. There are inherent technical infirmities or defects on the face of TCT all the portions or lots into which it has been subdivided, and the technical
Nos. 4211 (also on TCT No. 4210), 5261 and 35486. The fact that the technical description of each portion or lot, have been verified and approved by the
descriptions in TCT Nos. 4211, 5261 and 35486 are written in Spanish while Director of Lands. . .' and as corroborated by Section 44, Paragraph 2, and that
those on the alleged mother title, OCT-994, were already in English, is the plan has been approved by the Chief of the General Land Registration
abnormal and contrary to the usual practice in the issuance of titles. If OCT- Office, or by the Director of Lands as provided in Section fifty-eight of this Act,
994 is the mother title of TCT Nos. 4211, 5261 and 35486, then said titles the Registry of Deeds may issue new certificates of title for any lot in
should also be written in English because OCT-994 is already in English. It is accordance with said subdivision plan;'
possible that an ascendant title be written in Spanish and the descendant title
in English, the language now officially used, but the reverse is highly e. The absence of a lot number and survey plan number in the technical
improbable and irregular. description inscribed on TCT Nos. 4210 and 4211, and the absence of a
subdivision survey plan for Lot 26 at the records of the Bureau of Lands or the
b. Also, the fact that the original survey dates of OCT-994 (September 8- Land Registration Authority lead to the conclusion that there was no verified
27, October 4-21 and November 17-18, 1911) are not indicated on the and approved subdivision survey plan of Lot 26, which is a compulsory
technical descriptions on TCT Nos. 4211, 5261 and 35486, but an entirely requirement needed in the issuance of said titles;
different date, December 22, 1917, is instead indicated, likewise leads to the
f. Similarly, the absence of plan Psd-21154 from the files of the Bureau of
Lands, the official depository of survey plans, is another indication that the titles
covered by TCT Nos. 1368 thru 1374 which were derived from TCT No. 4211
are again doubtful and questionable;

g. Moreover, the changing of the tie points in the technical descriptions on


TCT Nos. 1368 thru 1374 from that of the mother lot's tie point which is BLLM
No. 1, Caloocan City to different location monuments of adjoining Piedad
Estate which resulted in the shifting of the position of the seven (7) lots in
relation to the mother lot defeats the very purpose of tie points and tie lines
since the accepted practice is to adopt the mother lot's tie point in order to fix
the location of the parcels of land being surveyed on the earth's surface. FIRST DIVISION

h. Based on the foregoing, it is the conclusion of the undersigned


Commissioners that defendants' (Manotok Realty, Inc. and Manotok Estate FIDELA R. ANGELES,
Corporation) titles overlap portions of plaintiff's (CLT Realty Development
Corporation's) title, which overlapping is due to the irregular and questionable Petitioner, G. R. No. 142549
issuance of TCT Nos. 4211 (also of TCT No. 4210), 5261, 35486, 1368 to
1374. The inherent technical defects on TCT No. 4211 (from where defendants
derived their titles) and TCT No. 4210 which were exhaustively elucidated
above, point to the fact that there was no approved subdivision of Lot 26 which
served as legal basis for the regular issuance of TCT Nos. 4210 and 4211.
Thus, as between plaintiff's title, which was derived from regularly issued titles,
and defendants' titles, which were derived from irregularly issued titles, - versus -
Present:
plaintiff's title which pertains to the entire Lot 26 of the Maysilo Estate should
prevail over defendants' titles. 18 (Underscoring supplied) See G.R. No. PUNO, C.J.,
123346, rollo, pp. 268-275. Chairperson,
CARPIO MORALES,
[121] LEONARDO-DE CASTRO,
RTC Order dated 2 July 1993, Original Records, CV No. C-15539, BERSAMIN, and
THE SECRETARY OF JUSTICE, THE
Vol. I, pp. 245-246. VILLARAMA, JR., JJ.
ADMINISTRATOR, LAND REGISTRATION
Promulgated:
AUTHORITY, THE REGISTER OF DEEDS
[122] OF QUEZON CITY, and SENATOR
Should have been 1368 thru 1374. TEOFISTO T. GUINGONA, JR.,
March 9, 2010

[123] Respondents.
Majority Report, paragraph hBased on the foregoing, it is the
conclusion of the undersigned Commissioners that defendants titles overlap
portions of plaintiffs title which overlapping is due to the irregular and x----------------------------------------------------x
questionable issuance of TCT Nos. 4211 (also TCT No. 4210), 5261, 35486,
1368 thru 1324 (sic). The inherent technical defects on TCT No. 4211 (from
where defendants derive their titles) and TCT No. 4210, which were
exhaustively elucidated above, point to the fact that there was no approved DECISION
subdivision of Lot 26 which served as legal basis for the regular issuance of
TCT Nos. 4210 and 4211. Thus, as between plaintiffs title which was derived
from regularly issued tiles and defendants titles which were derived from LEONARDO-DE CASTRO, J.:
irregularly issued titles, plaintiffs title which pertains to the entire Lot 26 of the
Maysilo Estate should prevail over defendants titles.
The property involved in this case is covered by Original Certificate of Title (OCT) No.
994, which encompasses One Thousand Three Hundred Forty-Two (1,342) hectares of
the Maysilo Estate, previously described by this Court En Banc as a vast tract of land In the RTC Order sought to be implemented, Judge Jaime D. Discaya granted the
[that] stretches over three cities, comprising an area larger than the sovereign states of partition and accounting prayed for by plaintiffs in that case; directed the respective
[1]
Monaco and the Vatican. What we have before us now is touted as one of the biggest Registers of Deeds of Caloocan City and Quezon City to issue transfer certificates of
and most extensive land-grabbing incidents in recent history.[2] title in the names of all the co-owners, including petitioner, for twelve (12) parcels of
land with an aggregate area of One Hundred Five Thousand and Nine Hundred Sixty-
The existence of several cases already decided by this Court dealing with this infamous Nine square meters (105,969 sq. m.), more or less; and ordered that said parcels of
estate has made the job of deciding this particular petition easy, on one hand, as there land be sold, subject to the confirmation of the Court, and the proceeds be divided
are cases squarely on point and at the outset, applicable; but complicated, on the other among the plaintiffs in proportion to their respective interests in the property.
hand, as such applicability must be determined with thoroughness and accuracy to
come up with a just, equitable, and fair conclusion to a controversy that has now lasted The dispositive portion of said Order reads as follows:
for almost forty-five (45) years.
WHEREFORE, premises considered, the recommendation of the Commissioners in
their Joint Commissioners Report dated October 21, 1997 and Supplemental
Submitted for Decision is a petition for mandamus seeking respondents Secretary of
Commissioners Report dated December 30, 1997 that the following lots with transfer
Justice, the Administrator of the Land Registration Authority (LRA), and the Register of certificates of title to be issued by the Register of Deeds of Caloocan City in the names
of all co-owners be sold and the proceeds thereof divided among themselves in
Deeds of Quezon City to comply with the Order[3] dated January 8, 1998 issued by the
proportion to their respective interest in the property, is approved.
Regional Trial Court (RTC) of Caloocan City in Civil Case No. C-424, entitled
The Register of Deeds of Caloocan City and of Quezon City are hereby directed to
Bartolome Rivera, et al. v. Isabel Gil de Sola, et al. (the RTC Order), which was issued
issue transfer certificates of title in the names of all the co-owners for the following lots,
a Certificate of Finality on March 12, 1998. namely:

xxxx
On May 3, 1965, petitioner, together with other individuals, all of them claiming to be the
Any sale of above-mentioned lots shall be subject to confirmation by this Court pursuant
heirs of a certain Maria de la Concepcion Vidal, and alleging that they are entitled to
to Section 11, Rule 69 of the Rules of Civil Procedure. [6]
inherit her proportional share in the parcels of land located in Quezon City and in the
municipalities of Caloocan and Malabon, Province of Rizal, commenced a special civil
Petitioner alleges that the respective Registers of Deeds of Caloocan City and Quezon
action for partition and accounting of the property otherwise known as Maysilo
City refused to comply with the RTC Order because they were still awaiting word from
Estate covered by OCT No. 994, allegedly registered on April 19, 1917 with the
the LRA Administrator before proceeding. Counsel for petitioner then requested the
Registry of Deeds of Caloocan City. This was docketed as Civil Case No. C-424 in the
LRA Administrator to direct said Registers of Deeds to comply with the Order.
RTC of Caloocan City, Branch 120.

The LRA Administrator, Mr. Alfredo R. Enriquez, sent counsel for petitioner a letter-
Some of said alleged heirs were able to procure Transfer Certificates of Title (TCTs)
reply[7] dated March 27, 2000, with two attachments: 1) the 1 st Indorsement[8] dated
over portions of the Maysilo Estate. They also had led this Court to believe that OCT
September 22, 1997 (the 1st Indorsement) issued by then Department of Justice (DOJ)
No. 994 was registered twice, thus, in Metropolitan Waterworks and Sewerage Systems
Secretary Teofisto T. Guingona, Jr. (respondent Guingona), and 2) LRA Circular No.
(MWSS) v. Court of Appeals,[4] reiterated in Heirs of Luis J. Gonzaga v. Court Of
97-11[9] issued to all Registers of Deeds. The letter-reply reads in part:
Appeals,[5] the Court held that OCT No. 994 dated April 19, 1917, and not May 3, 1917,
was the valid title by virtue of the prior registration rule.
We regret to inform you that your request cannot be granted in view of the directive of
the Department of Justice in its 1st Indorsement dated 22 September 1997, copy
enclosed, as a result of the inquiry conducted by the Composite Fact-Finding
Committee (created under DOJ Department Order No. 137) finding that there is only
one OCT No. 994 which was issued by the Rizal Register of Deeds on 3 May 1917 As found by the Senate Committees, the mess caused by the former Register of Deeds
(and not on 19 April 1919) pursuant to Decree No. 36455 in Land Registration Case and Deputy Register of Deeds in making it appear that OCT No. 994 was issued in 19
No. 4429. Pursuant to this DOJ directive, this Authority issued LRA Circular No. 97-11 April 1917, thus giving the wrong impression that there were two (2) OCT No. 994,
to all Registers of Deeds, copy attached, stating the following: resulted in the double, if not multiple, issuance of transfer certificates of title covering
the subdivided portions of the Maysilo Estate, including the parcels of land mentioned in
xxxx the subject Order dated 8 January 1998. Our Authority, as the protector of the integrity
of the Torrens title is mandated to prevent anomalous titling of real properties and put a
stop to further erode the confidence of the public in the Torrens system of land
In compliance with the DOJ directive, this Authority, in its 1 st Indorsement dated 27 registration.
March 1998, x x x had recommended to the Office of the Solicitor General the filing of
an appropriate pleading relative to the said Order dated 8 January 1998. With due respect, the Order dated 8 January 1998 which directs the issuance of transfer
certificates of title as direct transfer from OCT No. 994, suffers from certain deficiencies,
The findings of the DOJ on OCT No. 994 are in fact sustained by the Senate Committee to wit:OCT No. 994 had long been cancelled totally by the issuance of various
on Justice and Human Rights and Urban Planning in its Senate Committee Report No. certificates of title in the names of different persons; and that the plan and descriptions
1031 dated 25 May 1998 x x x.[10] (Emphasis ours.) of the lands were not based on a subdivision plan duly approved by the proper
government agency but merely sketch plans, in violation of Section 50 of PD 1529.
Obviously, compliance with the Order will result to duplication of certificates of title
The LRA Administrator likewise wrote that in Senate Committee Report No. 1031 dated covering land previously registered in the names of other persons. Besides, in MWSS
vs. CA, the Supreme Court did not declare the nullity of the certificates of title which
May 25, 1998, the Senate Committees on Justice and Human Rights and Urban emanated from OCT No. 994 issued on 3 May 1917. It merely invalidates the title of
Planning came up with the following findings: MWSS and recognizes as valid the title of Jose B. Dimson. There was no such
declaration as to the various transfer certificates of title emanating from OCT No. 994.
Under the law, there must be a separate action in court for the declaration of nullity of
i. There is only one Original Certificate of Title (OCT) No. 994 and this was issued certificates of title pursuant to the due process clause of the Constitution.
or registered on May 3, 1917[.]
As observed by the Supreme Court in Republic vs. Court of Appeals (94 SCRA 874),
ii. The [OCT] No. 994 dated April 19, 1917 is non-existent. It was a fabrication there are too many fake titles being peddled around and it behooves every official of the
perpetrated by Mr. Norberto Vasquez, Jr., former Deputy Registrar of Deeds of government whose functions concern the issuance of legal titles to see to it that this
Caloocan City. plague that has made a mockery of the Torrens system is eradicated right now through
their loyalty, devotion, honesty and integrity, in the interest of our country and people at
iii. The alleged surviving heirs could not have been the true and legal heirs of the late large.[12]
Maria de la Concepcion Vidal as government findings showed the physical and genetic
impossibility of such relationship[.]
Petitioner avers that respondent Guingona, in issuing the 1 st Indorsement,[13] made a
iv. Mr. Norberto Vasquez, Jr., former Deputy Registrar of Deeds of Caloocan City, substantive modification of the ruling made by this Court in MWSS v. Court of Appeals
acted maliciously, fraudulently and in bad faith, by issuing certifications and/or written
statements to the effect that OCT No. 994 was issued or registered on April 19, 1917 and Heirs of Luis Gonzaga v. Court of Appeals. She further avers that [n]ot even the
when in truth and in fact it was issued or registered on May 3, 1917. Secretary of Justice has the power or authority to set aside or alter an established ruling

v. Atty. Yolanda O. Alfonso, Registrar of Deeds of Caloocan City, likewise acted made by the highest Court of the land. According to petitioner, respondent Guingona
maliciously, fraudulently and in bad faith, when she signed the TCTs issued in the name claimed to have made his own finding that there is only one OCT No. 994 which was
of Eleuteria Rivera which bear a wrong date of the registration of OCT No. 994. Malice
was evident because she had previously issued certificates of title in the names of other issued by the Register of Deeds of Rizal on May 3, 1917, and not on April 19, 1917, and
individuals which were derived from OCT No. 994 dated May 3, 1917 and she had in this finding is a reversal of the decisions of this Court on what is the valid OCT No.
fact questioned the falsity of April 19, 1917 as the correct date of the registration of OCT
No. 994.[11] (Underscoring in the original.) 994.Petitioner contends that [t]he rule is well settled that once a decision becomes
final[,] the Court can no longer amend, modify, much less set aside the same and that

The letter-reply further stated that OCT No. 994 was intact and was being kept in the respondent Guingona usurped judicial functions and did a prohibited act which rendered

LRA to prevent its alteration and tampering. We quote the last portion of said letter- the Order of no effect.[14]

reply:
Petitioner claims that respondent Guingona was the one who caused the issuance by Respondent Guingona avers that he was prompted to issue DOJ Department Order No.
the LRA Administrator of Circular No. 97-11 dated October 3, 1997, which had the same 137 dated April 13, 1997 creating a committee due to several complaints received by
legal effect on other cases similarly situated without hearing or notice to the parties-in- the Office of the Secretary of Justice in February 1997. Among others, the complaints
interest, and that this was contemptuous and contumacious and calls for condemnation prayed for the investigation of certain actions taken by the LRA officials and personnel
and reproof of the highest degree.[15] in connection with transactions involving the Maysilo Estate. According to him, the
committee was tasked for the purpose of initiating a fact-finding inquiry:
Petitioner alleges that compliance with a final judicial order is a purely ministerial duty,
that she and her co-plaintiffs in Civil Case No. C-424 cannot avail of the benefits (1) to ascertain the circumstances surrounding the issuance of original Certificate(s) of
Title (OCT) No. 994 of the Registry of Deeds of Rizal purporting to cover a mass of land
granted to them by the Order, and that she has no plain, speedy and adequate remedy
encompassing Malabon, Caloocan City and Quezon City as well as the issuance and
in the ordinary course of law, other than this action. regularity of Transfer Certificates of Titles (TCTs) derived therefrom; (2) in the event of a
finding of the irregular issuance of any such [TCTs], (a) to determine the involvement of
and to recommend the actions to be taken against person(s) and/or officials and
In his Comment,[16] respondent Guingona raises the following grounds for denial of the employees of this Department or its agencies who may appear to have participated
therein, and (b) to recommend the administrative and/or judicial actions, if any, that may
petition:
directly be undertaken by this Department, the Office of the Solicitor General, the Land
Registration Authority, and other units and attached agencies of this Department, with
respect to such irregularly issued Transfer Certificates of Title, taking into account the
1. Petitioner has no cause of action against respondent Guingona in that the latter is
final decisions of the courts affecting the Maysilo Estate. [18]
no longer the Secretary of Justice.

Respondent Guingona contends that it can be gleaned from the purpose of the creation
2. The issuance of the 1st Indorsement dated September 22, 1997 was pursuant to of the committee that its fact-finding investigation was merely administrative to formulate
the report dated August 27, 1997 made by the committee created by Department Order
No. 137 dated April 23, 1997 after conducting an independent fact-finding investigation. and recommend policies, procedures and courses of action which the DOJ, the LRA,
It did not in any way alter or modify any judgment of this Honorable Court. the Office of the Solicitor General and other agencies of the DOJ can adopt with regard
to the problem of the proliferation of fake land titles, including those that relate to the
Maysilo Estate. He alleges that based on this committees report dated August 27, 1997,
3. Petitioner was not denied due process as her rights, if any, under the Order dated he issued the subject 1st Indorsement which spelled out the policies, procedures, and
January 18, 1998 were not yet in existence at the time the 1st Indorsement was issued.
courses of action which the LRA, an agency under the DOJ, must follow not only with
respect to OCT No. 994 and its derivative titles covering the Maysilo Estate but to all
other original or transfer certificates of title as well. He contends that the 1 st Indorsement
4. Mandamus is not the appropriate remedy to enforce claims of damages.[17] was merely an administrative issuance of the DOJ; thus, it could not be said that it
altered or supplanted any judgment of this Court.

Respondent Guingona contends that he was no longer the Secretary of Justice,


Respondent Guingona further states that the 1 st Indorsement dated September 22,
therefore, he did not anymore possess the mandatory duties being compelled to be
1997 was issued long before the Order dated January 18, 1998, thus it could not be
performed in this case by way of a writ of mandamus; he had no more duty resulting
said that petitioner was denied due process as her rights and interests were non-
from the said position and could not perform an act that pertained to said duty, even if
existent at that time. Furthermore, respondent Guingona alleges that petitioner was
he wanted to; and since he did not have the powers and duties of the Secretary of
accorded due process when the LRA Administrator gave an opportunity to petitioners
Justice, he was therefore not a real party-in-interest in this case.
counsel to present petitioners case to the LRA legal staff. Respondent Guingona claims
that such opportunity to be heard satisfies the requirements of due process, as the RULE 65
[19]
essence of due process is simply the opportunity to be heard.
xxxx

With regard to the claim for damages, respondent Guingona argues that it is a factual SECTION 9. Service and enforcement of order or judgment. A certified copy of the
judgment rendered in accordance with the last preceding section shall be served upon
issue which the petitioner must prove in the course of a trial where petitioners claim for the court, quasi-judicial agency, tribunal, corporation, board, officer or person concerned
damages can be fully litigated. This Honorable Court, however, is not a trier of facts. in such manner as the court may direct, and disobedience thereto shall be punished as
contempt. An execution may issue for any damages or costs awarded in accordance
Such being the case, it is inappropriate for petitioner to include in her petition for with Section 1 of Rule 39.
mandamus a claim for damages the amount of which she did not even specify. As it is,
such claim should be denied by this Honorable Court. There is also no showing that RULE 39
petitioner paid the required docket fees for her claims for damages. On this score alone,
SECTION 1. Execution upon final judgments or orders. Execution shall issue as a
such a claim should be outrightly dismissed.[20]
matter of right, on motion, upon a judgment or order that disposes of the action or
proceeding upon the expiration of the period to appeal therefrom if no appeal has been
duly perfected.
In her Reply,[21] petitioner contends that former DOJ Secretary Guingona has to be
named as private respondent because he was the cause of public respondents failure to If the appeal has been duly perfected and finally resolved, the execution may forthwith
comply with their ministerial duty. A private respondent is the person interested in be applied for in the court of origin, on motion of the judgment obligee, submitting
therewith certified true copies of the judgment or judgments or final order or orders
sustaining the proceedings in the court; and it shall be the duty of such private sought to be enforced and of the entry thereof, with notice to the adverse party.
respondent to appear and defend, both in his own behalf and in behalf of the public
The appellate court may, on motion in the same case, when the interest of justice so
respondents affected by the proceedings x x x. He is not charged with any improper act, requires, direct the court of origin to issue the writ of execution.
but he is a necessary party as the grant of relief prayed for by petitioner shall require
private respondents active participation. [22] Petitioner avers that private respondent seemed to assume a function that did not
belong to the Executive Department, because he had caused the issuance of an LRA
Anent private respondents argument that the 1st Indorsement did not in any way alter or Circular that forbade compliance with a court order that had already become final and
modify any judgment of this Honorable Court, petitioner counters that the executory. Petitioner likewise avers that the doctrine of separation of powers called for
1stIndorsement and pertinent acts of private respondent x x x resulted in the altering or each branch of government to be left alone to discharge its functions within its
supplanting of a judgment of this Court. The complaints praying that an investigation be jurisdiction, as it saw fit.[23]
conducted on the irregular issuance of titles in the Maysilo Estate were made to the
private respondent by parties who held titles derived from OCT No. 994 on May 3, 1917, Public respondents Secretary of Justice, the Administrator of the Land Registration
after the Supreme Court had rendered its decision in MWSS v. Court of Appeals and Authority, and the Register of Deeds of Quezon City filed their Comment[24] on
Heirs of Gonzaga v. Court of Appeals. November 16, 2000. Public respondents claim that petitioner and her co-plaintiffs are
not the rightful owners of the property subject of said complaint for partition. Their
Petitioner argues that contrary to private respondents claim, she is entitled to file a allegation in the complaint that they are the heirs and successors-in-interest of the late
petition for mandamus as she and her co-plaintiffs in Civil Case No. C-424 has been Maria de la Concepcion Vidal, co-owner of the parcels of land described in OCT No.
suffering from damages and losses incapable of quantification, because of the wrongful 994, and are therefore entitled to the proportionate share, ownership, and possession of
act of the respondents. Petitioner cites the following provisions of the Rules of Court in the parcels of land described in paragraphs XI to XV of the complaint, is an untrue
support of her argument: statement made with intent to deceive. This is because the findings embodied in the
Report of the Fact Finding Committee created by the DOJ, which are the result of the
joint undertaking of the Department proper, the Office of the Solicitor General, and the Considering the factual background and recent jurisprudence related to this controversy
LRA, support the conclusion that petitioner and her co-plaintiffs are not entitled to the as will be discussed below, we find that it was not unlawful for public respondents to
[25]
issuance of new transfer certificates of title in their names. refuse compliance with the RTC Order, and the act being requested of them is not their
ministerial duty; hence, mandamus does not lie and the petition must be dismissed.
Public respondents claim the following as facts:
Rule 65 of the 1997 Rules of Civil Procedure provides:
The DOJ Report became the subject of [a] Senate investigation. On May 25, 1998, the
Honorable Senate of the Tenth Congress of the Republic of the Philippines reached the
SECTION 3. Petition for mandamus. When any tribunal, corporation, board, officer or
conclusion that petitioner and her co-plaintiffs are not and cannot be true heirs of the
person unlawfully neglects the performance of an act which the law specifically enjoins
late Maria de la Concepcion Vidal (par. 3, p. 33, Senate Report). x x x.
as a duty resulting from an office, trust, or station, or unlawfully excludes another from
the use and enjoyment of a right or office to which such other is entitled, and there is no
As early as 1917, subject property of the instant case had already been partitioned and
other plain, speedy and adequate remedy in the ordinary course of law, the person
divided among the true owners, namely, Gonzalo Tuason y Patino, Jose Rato y Tuason,
aggrieved thereby may file a verified petition in the proper court, alleging the facts with
Luis Vidal y Tuason, Concepcion Vidal y Tuason, Pedro Baos, Maria de la Concepcion
certainty and praying that judgment be rendered commanding the respondent,
Vidal, Trinidad Jurado, Bernardino Hernandez, Esperanza Tuason Chua Jap, Isabel
immediately or at some other time to be specified by the court, to do the act required to
Tuason Chua, Juan Jose Tuason de la Paz, Maria Teresa Tuason y de la Paz, Mariano
be done to protect the rights of the petitioner, and to pay the damages sustained by the
Severo Tuason y de la Paz, Demetrio Asuncion Tuason y de la Paz, Augusto Hoberto
petitioner by reason of the wrongful acts of the respondent.
Tuason y de la Paz, Maria Soterrana Tuason y de la Paz, Benito Legarda y de la Paz,
Consuelo Legarda y de la Paz, Rita Legarda y de la Paz, Benito Legarda y Tuason,
Emilia Tuason y Patio, Maria Rocha de Despujols, Sofia OFarrell y Patio, German
It is settled that mandamus is employed to compel the performance, when refused, of a
Franco y Gonzales, Concepcion Franco y Gonzales, Domingo Franco y Gonzales,
Guillerma Ferrer y Tuason, Vicente Ferrer y Tuason, Josefa Tuason vda. de Flores, and ministerial duty, but not to compel the performance of a discretionary duty.Mandamus
heirs of Filemon Tuazon in proportion to their respective shares, as evidenced by the
will not issue to enforce a right which is in substantial dispute or to which a substantial
document entitled PROYECTO DE PARTICION DE LA HACIENDA DE MAYSILO
(PARTITION PLAN OF HACIENDA MAYSILO) consisting of fifty-two (52) pages which doubt exists.[27] It is nonetheless likewise available to compel action, when refused, in
is attached as Annex D, and its faithful translation into English consisting of forty-nine
matters involving judgment and discretion, but not to direct the exercise of judgment or
(49) pages attached as Annex E, and both made integral parts hereof.
discretion in a particular way or the retraction or reversal of an action already taken in
As a result of said partition, transfer certificates of titles covering the same subject
the exercise of either.[28]
parcels of land were legally issued in the names of above-enumerated true owners.

The Register of Deeds of Quezon City and Caloocan City, through the undersigned
Therefore, we must look into the alleged right of petitioner and see if compliance with
counsel, filed the aforestated Motion for Reconsideration of the questioned Order of the
lower court. the RTC Order is compellable by mandamus; or, in the alternative, find out if substantial
doubt exists to justify public respondents refusal to comply with said Order. Did public
The resolution of said motion and other incidents in related cases pending before the
lower court has been held in abeyance to await the resolution by higher courts of other respondents have sufficient legal basis to refuse to grant petitioners request?
cases involving the Maysilo Estate.[26]
In this regard, we find our discussion in Laburada v. Land Registration Authority[29]
instructive, to wit:
We are thus faced with the issue of whether public respondents unlawfully
neglected to perform their duties by their refusal to issue the questioned transfer
That the LRA hesitates in issuing a decree of registration is understandable. Rather
certificates of title to petitioner and her co-plaintiffs (in Civil Case No. C-424) or have than a sign of negligence or nonfeasance in the performance of its duty, the LRA's
reaction is reasonable, even imperative. Considering the probable duplication of
unlawfully excluded petitioner from the use and enjoyment of whatever claimed
titles over the same parcel of land, such issuance may contravene the policy and
right, as would warrant the issuance of a writ of mandamus against said public the purpose, and thereby destroy the integrity, of the Torrens system of
registration.
respondents.
xxxx
x x x Likewise, the writ of mandamus can be awarded only when the petitioners' legal
right to the performance of the particular act which is sought to be compelled is clear
and complete. Under Rule 65 of the Rules of Court, a clear legal right is a right which is The determinative test to resolve whether the prior decision of this Court should be
indubitably granted by law or is inferable as a matter of law. If the right is clear and the affirmed or set aside is whether or not the titles invoked by the respondents are valid. If
case is meritorious, objections raising merely technical questions will be disregarded. these titles are sourced from the so-called OCT No. 994 dated 17 April 1917, then such
But where the right sought to be enforced is in substantial doubt or dispute, as in this titles are void or otherwise should not be recognized by this Court. Since the true basic
case, mandamus cannot issue.[30](Emphasis ours.) factual predicate concerning OCT No. 994 which is that there is only one such OCT
differs from that expressed in the MWSS and Gonzaga decisions, said rulings have
become virtually functus officio except on the basis of the "law of the case" doctrine,
As can be gleaned from the above discussion, the issuance by the LRA officials of a and can no longer be relied upon as precedents. [35]
decree of registration is not a purely ministerial duty in cases where they find that such
would result to the double titling of the same parcel of land. In the same vein, we find Specifically, petitioner cannot anymore insist that OCT No. 994 allegedly issued on April
that in this case, which involves the issuance of transfer certificates of title, the Register 19, 1917 validly and actually exists, given the following conclusions made by this Court
of Deeds cannot be compelled by mandamus to comply with the RTC Order since there in the 2007 Manotok case:
were existing transfer certificates of title covering the subject parcels of land and there
was reason to question the rights of those requesting for the issuance of the TCTs.
Neither could respondent LRA Administrator be mandated by the Court to require the First, there is only one OCT No. 994. As it appears on the record, that mother title
was received for transcription by the Register of Deeds on 3 May 1917, and that
Register of Deeds to comply with said Order, for we find merit in the explanations of should be the date which should be reckoned as the date of registration of the
respondent LRA Administrator in his letter-reply that cites the 1st Indorsement issued by title. It may also be acknowledged, as appears on the title, that OCT No. 994 resulted
from the issuance of the decree of registration on [19] April 1917, although such date
respondent Guingona, LRA Circular No. 97-11, and Senate Committee Report No. cannot be considered as the date of the title or the date when the title took effect.
1031, as reasons for his refusal to grant petitioners request. [31] There was, therefore,
Second. Any title that traces its source to OCT No. 994 dated [19] April 1917 is
sufficient basis for public respondents to refuse to comply with the RTC Order, given the void, for such mother title is inexistent. The fact that the Dimson and CLT titles
finding, contained in the cited documents, that OCT No. 994 dated April 19, 1917, on made specific reference to an OCT No. 994 dated [19] April 1917 casts doubt on the
validity of such titles since they refer to an inexistent OCT. x x x.
which petitioner and her co-plaintiffs in the civil case clearly anchored their rights, did
not exist. Third. The decisions of this Court in MWSS v. Court of Appeals and Gonzaga v.
Court of Appeals cannot apply to the cases at bar, especially in regard to their
recognition of an OCT No. 994 dated 19 April 1917, a title which we now
It is important to emphasize at this point that in the recent case resolved by this Court acknowledge as inexistent. Neither could the conclusions in MWSS or Gonzaga
with respect to an OCT No. 994 dated 19 April 1917 bind any other case operating
En Banc in 2007, entitled Manotok Realty, Inc. v. CLT Realty Development under the factual setting the same as or similar to that at bar.[36] (Emphases
Corporation[32] (the 2007 Manotok case), as well as the succeeding resolution[33] in the supplied.)
same case dated March 31, 2009 (the 2009 Manotok case), the controversy
surrounding the Maysilo Estate and the question of the existence of another OCT No. To be sure, this Court did not merely rely on the DOJ and Senate reports regarding
994 have been finally laid to rest. All other cases involving said estate and OCT No. OCT No. 994. In the 2007 Manotok case, this Court constituted a Special Division of the
994, such as the case at bar, are bound by the findings and conclusions set forth in said Court of Appeals to hear the cases on remand, declaring as follows:
resolutions.
Since this Court is not a trier of fact[s], we are not prepared to adopt the findings made
by the DOJ and the Senate, or even consider whether these are admissible as
As stated earlier, petitioner anchors her claim on previous cases decided by this evidence, though such questions may be considered by the Court of Appeals upon the
Court[34] which have held that there are two existing OCT No. 994, dated differently, and initiative of the parties. x x x The reports cannot conclusively supersede or overturn
judicial decisions, but if admissible they may be taken into account as evidence on the
the one from which she and her co-plaintiffs (in Civil Case No. C-424) derived their
same level as the other pieces of evidence submitted by the parties. The fact that they
rights was dated earlier, hence, was the superior title. Regrettably, petitioners claim no were rendered by the DOJ and the Senate should not, in itself, persuade the courts to
accept them without inquiry. The facts and arguments presented in the reports must still
longer has a leg to stand on. As we held in the 2007 Manotok case:
undergo judicial scrutiny and analysis, and certainly the courts will have the discretion to
accept or reject them.

There are many factual questions looming over the properties that could only be
threshed out in the remand to the Court of Appeals. x x x.

xxxx

The Special Division is tasked to hear and receive evidence, conclude the proceedings
and submit to this Court a report on its findings and recommended conclusions within
three (3) months from finality of this Resolution.[37]

Thus, in the 2009 Manotok case, this Court evaluated the evidence engaged in by said
Special Division, and adopted the latters conclusions as to the status of the original title
and its subsequent conveyances. This case affirmed the earlier finding that there is only
one OCT No. 994, the registration date of which had already been decisively settled as
3 May 1917 and not 19 April 1917 and categorically concluded that OCT No. 994 which
reflects the date of 19 April 1917 as its registration date is null and void.

In the case at bar, petitioner is the last surviving co-plaintiff in Civil Case No. C-424
originally filed on May 3, 1965. The records bear several attempts of different individuals
to represent her as counsel, a matter that could be attributed to her advanced age and
potential access to a vast sum of money, should she get a favorable decision from this
case. It appears, however, that the partition and accounting of a portion of the Maysilo
Estate that she and her co-plaintiffs prayed for can no longer prosper because of the Republic of the Philippines
conclusive findings quoted above that the very basis of their claim, a second, albeit
SUPREME COURT
earlier registered, OCT No. 994, does not exist.
Manila
The requirements under Rule 65 for the issuance of the writ of mandamus not having
FIRST DIVISION
been proven by petitioner to exist, we dismiss the petition for lack of merit.

G.R. No. 83609 October 26, 1989


WHEREFORE, premises considered, the petition is hereby DISMISSED.
DIRECTOR OF LANDS, petitioner,

SO ORDERED.
vs.

COURT OF APPEALS, IBARRA BISNAR and AMELIA BISNAR, respondents.

Ibarra L. Bisnar for himself and for and in behalf of co-private respondent Amelia Bisnar.
GRIO-AQUINO, J.: introduced improvements on the lands by planting coconuts, bamboos and other plants,
and converted a part of the land into productive fishponds (p. 68, Rollo).
Petitioner Director of Lands, through the Solicitor General, seeks a review of the
decision dated May 27, 1988, of the Court of Appeals in CA-G.R. CV No. 66426, On appeal, the Appellate Court affirmed the trial court's decision. It held that the
entitled "Ibarra Bisnar, et al. vs. Director of Lands," affirming in totothe decision of the classification of the lots as timberland by the Director of Forestry cannot prevail in the
Court of First Instance of Capiz, granting the private respondents' application for absence of proof that the said lots are indeed more valuable as forest land than as
confirmation and registration of their title to two (2) parcels of land in LRC Cad. Rec. agricultural land, citing as authority the case of Ankron vs. Government of the Philippine
1256. Islands (40 Phil. 10). In this petition, the government alleges that:

In their joint application for registration of title to two (2) parcels of land filed on July 1. the classification or reclassification of public lands into alienable or disposable
20,1976, the applicants Ibarra and Amelia Bisnar claimed to be the owners in fee simple agricultural land, mineral land or forest land is a prerogative of the Executive
of Lots 866 and 870 of the Pilar Cadastre Plan AP-06-000869, respectively containing Department of the government and not of the courts;
an area of 28 hectares (284,424 sq. m.) and 34 hectares (345,385 sq. m.) situated in
barrio Gen. Hizon, Municipality of President Roxas, Province of Capiz (p. 14, Rollo). 2. that possession of forest lands, no matter how long, cannot ripen into private
The applicants alleged that they inherited those parcels of land (p. 41, Rollo) and they ownership; and
had been paying the taxes thereon (p. 40, Rollo).
3. that an applicant for registration of title has the burden of proving that he meets
On December 16,1976, the Director of Lands and the Director of the Bureau of Forest the requirements of Section 48 of Com. Act No. 141, as amended. (p. 19, Rollo.)
Development, opposed the application on the grounds that:
The principal issue in this appeal is whether the lots in question may be registered
1. Neither the applicants nor their predecessors-in-interest possess sufficient title under Section 48 (b) of CA 141, as amended.
to acquire ownership in fee simple of the land or lots applied for, the same not having
been acquired by any of the various types of title issued by the Spanish Government,
such as, (1) 'titulo real' or royal grant, (2) the 'concession especial' or special grant, (3) The petition is impressed with merit.
the 'composicion con el estado titulo' or adjustment title, (4) the 'titulo de compra 'or title
by purchase, and (5) the 'informacion possessoria' or possessory information under the In the case of Bureau of Forestry vs. Court of Appeals, 153 SCRA 351, we ruled:
Royal Decree of 13 February 1894, or any other recognized mode of acquisition of title
over realty under pertinent applicable laws. As provided for under Section 6 of Commonwealth Act 141, which was lifted from Act
2874, the classification or reclassification of public lands into alienable or disposable,
2. Neither the applicants nor their predecessors-in-interest have been in open, mineral or forest lands is now a prerogative of the Executive Department of the
continuous, exclusive and notorious possession and occupation of the land in question government and not the courts. With these rules, there should be no more room for
for at least thirty (30) years immediately preceding the filing of the application. doubt that it is not the court which determines the classification of lands of the public
domain into agricultural, forest or mineral but the Executive Branch of the government,
3. The properties in question are a portion of the public domain belonging to the through the Office of the President. Hence, it was grave error and/or abuse of discretion
Republic of the Philippines, not subject to private appropriation, (pp. 17-19, Record on for respondent court to ignore the uncontroverted facts that (1) the disputed area is
Appeal). (pp. 14-15, Rollo.) within a timberland block, and (2) as certified to by the then Director of Forestry, the
area is needed for forest purposes. (pp. 21-22, Rollo.)
On February 24,1977, the applicants filed an amended application, which was approved
on March 14, 1977, and included the following allegation: It bears emphasizing that a positive act of the government is needed to declassify land
which is classified as forest and to convert it into alienable or disposable land for
agricultural or other purposes (Republic vs. Animas, 56 SCRA 499). Unless and until
Should the Land Registration Act invoked be not applicable to the case, they hereby the land classified as forest is released in an official proclamation to that effect so that it
apply for the benefits of Chapter 8, Commonwealth Act 141, as amended, as they and may form part of the disposable agricultural lands of the public domain, the rules on
their predecessors-in-interest have been in possession of the land as owners for more confirmation of imperfect title do not apply (Amunategui vs. Director of Forestry, 126
than fifty (50) years. (p. 16, Rollo.) SCRA 69; Director of Lands vs. Court of Appeals, 129 SCRA 689; Director of Lands vs.
Court of Appeals, 133 SCRA 701; Republic vs. Court of Appeals, 148 SCRA 480;
After hearing, the trial court ordered the registration of the title of the lots in the names Vallarta vs. Intermediate Appellate Court, 151 SCRA 679).
of the applicants, herein private respondents. It found that applicants and their
predecessors- in-interest have been in open, public, continuous, peaceful and adverse Thus, possession of forest lands, however long, cannot ripen into private ownership
possession of the subject parcels of land under bona fide claims of ownership for more (Vano vs. Government, 41 Phil. 161 [1920]; Adorable vs. Director of Forestry, 107 Phil.
than eighty (80) years (not only 30) prior to the filing of the application for registration, 401 [1960]). A parcel of forest land is within the exclusive jurisdiction of the Bureau of
Forestry and beyond the power and jurisdiction of the cadastral court to register under
the Torrens System (Republic vs. Court of Appeals, 89 SCRA 648; Republic vs. Vera, DEPARTMENT OF TOURISM CHICO-NAZARIO,
120 SCRA 210 [1983]; Director of Lands vs. Court of Appeals, 129 SCRA 689 [1984]). SECRETARY, DIRECTOR OF VELASCO, JR.,
PHILIPPINE TOURISM NACHURA,**
Section 48 (b) of Commonwealth Act No. 141, as amended, applies exclusively to public AUTHORITY, REYES,
agricultural land. Forest lands or areas covered with forests are excluded (p. 26, Rollo).
We reiterate our ruling in Amunategui that: Petitioners, LEONARDO-DE CASTRO, and

In confirmation of imperfect title cases, the applicant shoulders the burden of proving BRION, JJ.
that he meets the requirements of Section 48, Commonwealth Act No. 141, as
amended by Republic Act 1942. He must overcome the presumption that the land he is - versus -
applying for is part of the public domain but that he has an interest therein sufficient to
warrant registration in his name because of an imperfect title such as those derived
from old Spanish grants or that he has had continuous, open and notorious possession
and occupation of agricultural lands of the public domain under a bona fide claim of
acquisition of ownership for at least thirty (30) years preceding the filing of his MAYOR JOSE S. YAP, LIBERTAD
application. (Heirs of Amunategui vs. Director of Forestry, 126 SCRA 69.) TALAPIAN, MILA Y. SUMNDAD, and
ANICETO YAP, in their behalf and Promulgated:
in behalf of all those similarly situated,
WHEREFORE, the appealed decision is reversed and set aside. The application for Respondents. October 8, 2008
registration in LRC Cad. Rec. 1256 of the former Court of First Instance, is hereby
dismissed without costs.
x--------------------------------------------------x
SO ORDERED.
DR. ORLANDO SACAY and G.R. No. 173775
WILFREDO GELITO, joined by
THE LANDOWNERS OF
BORACAY SIMILARLY
SITUATED NAMED IN A LIST,
ANNEX A OF THIS PETITION,
Petitioners,
Republic of the Philippines
Supreme Court
Manila - versus -

EN BANC
THE SECRETARY OF THE
DEPARTMENT OF ENVIRONMENT
THE SECRETARY OF THE G.R. No. 167707 AND NATURAL RESOURCES, THE
DEPARTMENT OF ENVIRONMENT REGIONAL TECHNICAL
AND NATURAL RESOURCES, THE DIRECTOR FOR LANDS, LANDS
REGIONAL EXECUTIVE Present: MANAGEMENT BUREAU,
DIRECTOR, DENR-REGION VI, REGION VI, PROVINCIAL
REGIONAL TECHNICAL PUNO, C.J., ENVIRONMENT AND NATURAL
RESOURCES OFFICER, KALIBO,
DIRECTOR FOR LANDS, QUISUMBING, AKLAN,
Respondents.
LANDS MANAGEMENT BUREAU, YNARES-SANTIAGO,
REGION VI PROVINCIAL CARPIO,
ENVIRONMENT AND NATURAL AUSTRIA-MARTINEZ, x--------------------------------------------------x
RESOURCES OFFICER OF KALIBO, CORONA,*
AKLAN, REGISTER OF DEEDS, CARPIO MORALES, DECISION
DIRECTOR OF LAND AZCUNA,
REGISTRATION AUTHORITY, TINGA,
REYES, R.T., J.: Claiming that Proclamation No. 1801 and PTA Circular No 3-82 precluded them from
filing an application for judicial confirmation of imperfect title or survey of land for titling
AT stake in these consolidated cases is the right of the present occupants of Boracay purposes, respondents-claimants
Island to secure titles over their occupied lands. Mayor Jose S. Yap, Jr., Libertad Talapian, Mila Y. Sumndad, and Aniceto Yap filed a
petition for declaratory relief with the RTC in Kalibo, Aklan.

There are two consolidated petitions. The first is G.R. No. 167707, a petition for review
on certiorari of the Decision[1] of the Court of Appeals (CA) affirming that[2] of the In their petition, respondents-claimants alleged that Proclamation No. 1801 and PTA

Regional Trial Court (RTC) in Kalibo, Aklan, which granted the petition for declaratory Circular No. 3-82 raised doubts on their right to secure titles over their occupied lands.

relief filed by respondents-claimants Mayor Jose Yap, et al. and ordered the survey of They declared that they themselves, or through their predecessors-in-interest, had been

Boracay for titling purposes. The second is G.R. No. 173775, a petition for prohibition, in open, continuous, exclusive, and notorious possession and occupation in Boracay

mandamus, and nullification of Proclamation No. 1064 [3] issued by President Gloria since June 12, 1945, or earlier since time immemorial. They declared their lands for tax

Macapagal-Arroyo classifying Boracay into reserved forest and agricultural land. purposes and paid realty taxes on them.[10]

The Antecedents Respondents-claimants posited that Proclamation No. 1801 and its implementing
Circular did not place Boracay beyond the commerce of man. Since the Island was

G.R. No. 167707 classified as a tourist zone, it was susceptible of private ownership. Under Section 48(b)
of Commonwealth Act (CA) No. 141, otherwise known as the Public Land Act, they had

Boracay Island in the Municipality of Malay, Aklan, with its powdery white sand beaches the right to have the lots registered in their names through judicial confirmation of

and warm crystalline waters, is reputedly a premier Philippine tourist destination.The imperfect titles.

island is also home to 12,003 inhabitants[4] who live in the bone-shaped islands three
barangays.[5] The Republic, through the Office of the Solicitor General (OSG), opposed the petition
for declaratory relief. The OSG countered that Boracay Island was an unclassified

On April 14, 1976, the Department of Environment and Natural Resources (DENR) land of the public domain. It formed part of the mass of lands classified as public forest,

approved the National Reservation Survey of Boracay which was not available for disposition pursuant to Section 3(a) of Presidential Decree

Island,[6] which identified several lots as being occupied or claimed by named persons. [7] (PD) No. 705 or the Revised Forestry Code,[11] as amended.

On November 10, 1978, then President Ferdinand Marcos issued Proclamation No. The OSG maintained that respondents-claimants reliance on PD No. 1801 and PTA

1801[8] declaring Boracay Island, among other islands, caves and peninsulas in the Circular No. 3-82 was misplaced. Their right to judicial confirmation of title was

Philippines, as tourist zones and marine reserves under the administration of the governed by CA No. 141 and PD No. 705. Since Boracay Island had not been classified

Philippine Tourism Authority (PTA). President Marcos later approved the issuance of as alienable and disposable, whatever possession they had cannot ripen into

PTACircular 3-82[9] dated September 3, 1982, to implement Proclamation No. 1801. ownership.

During pre-trial, respondents-claimants and the OSG stipulated on the following facts:
(1) respondents-claimants were presently in possession of parcels of land in Boracay
Island; (2) these parcels of land were planted with coconut trees and other natural lands in Boracay and that only those forested areas in public lands were declared as
growing trees; (3) the coconut trees had heights of more or less twenty (20) meters and part of the forest reserve.[22]
were planted more or less fifty (50) years ago; and (4) respondents-claimants declared
the land they were occupying for tax purposes.[12] The OSG moved for reconsideration but its motion was denied. [23] The Republic then
appealed to the CA.
The parties also agreed that the principal issue for resolution was purely legal: whether
Proclamation No. 1801 posed any legal hindrance or impediment to the titling of the On December 9, 2004, the appellate court affirmed in toto the RTC decision, disposing
lands in Boracay. They decided to forego with the trial and to submit the case for as follows:
resolution upon submission of their respective memoranda.[13]
WHEREFORE, in view of the foregoing premises, judgment is hereby rendered by us
DENYING the appeal filed in this case and AFFIRMING the decision of the lower
The RTC took judicial notice[14] that certain parcels of land in Boracay Island, more court.[24]
particularly Lots 1 and 30, Plan PSU-5344, were covered by Original Certificate of Title
No. 19502 (RO 2222) in the name of the Heirs of Ciriaco S. Tirol. These lots were
involved in Civil Case Nos. 5222 and 5262 filed before the RTC of Kalibo, Aklan. [15]The The CA held that respondents-claimants could not be prejudiced by a declaration that

titles were issued on the lands they occupied since time immemorial were part of a forest reserve.

August 7, 1933.[16]
Again, the OSG sought reconsideration but it was similarly denied. [25] Hence, the

RTC and CA Dispositions present petition under Rule 45.

On July 14, 1999, the RTC rendered a decision in favor of respondents-claimants, with G.R. No. 173775

a fallo reading:
On May 22, 2006, during the pendency of G.R. No. 167707, President Gloria

WHEREFORE, in view of the foregoing, the Court declares that Proclamation No. 1801 Macapagal-Arroyo issued Proclamation No. 1064[26] classifying Boracay Island into four
and PTA Circular No. 3-82 pose no legal obstacle to the petitioners and those similarly hundred (400) hectares of reserved forest land (protection purposes) and six hundred
situated to acquire title to their lands in Boracay, in accordance with the applicable laws
and in the manner prescribed therein; and to have their lands surveyed and approved twenty-eight and 96/100 (628.96) hectares of agricultural land (alienable and
by respondent Regional Technical Director of Lands as the approved survey does not in
itself constitute a title to the land. disposable). The Proclamation likewise provided for a fifteen-meter buffer zone on each
side of the centerline of roads and trails, reserved for right-of-way and which shall form
SO ORDERED.[17]
part of the area reserved for forest land protection purposes.

The RTC upheld respondents-claimants right to have their occupied lands titled in their
name. It ruled that neither Proclamation No. 1801 nor PTA Circular No. 3-82 mentioned On August 10, 2006, petitioners-claimants Dr. Orlando Sacay,[27] Wilfredo Gelito,[28] and

that lands in Boracay were inalienable or could not be the subject of disposition.[18] The other landowners[29] in Boracay filed with this Court an original petition for prohibition,
Circular itself recognized private ownership of lands. [19] The trial court cited Sections mandamus, and nullification of Proclamation No. 1064. [30] They allege that the
87[20] and 53[21] of the Public Land Act as basis for acknowledging private ownership of Proclamation infringed on their prior vested rights over portions of Boracay. They have
been in continued possession of their respective lots in Boracay since time immemorial.
They have also invested billions of pesos in developing their lands and building
internationally renowned first class resorts on their lots.[31] I.
AT THE TIME OF THE ESTABLISHED POSSESSION OF PETITIONERS IN
CONCEPT OF OWNER OVER THEIR RESPECTIVE AREAS IN BORACAY, SINCE
TIME IMMEMORIAL OR AT THE LATEST SINCE 30 YRS. PRIOR TO THE FILING OF
THE PETITION FOR DECLARATORY RELIEF ON NOV. 19, 1997, WERE THE
Petitioners-claimants contended that there is no need for a proclamation reclassifying AREAS OCCUPIED BY THEM PUBLIC AGRICULTURAL LANDS AS DEFINED BY
LAWS THEN ON JUDICIAL CONFIRMATION OF IMPERFECT TITLES OR PUBLIC
Boracay into agricultural land. Being classified as neither mineral nor timber land, the
FOREST AS DEFINED BY SEC. 3a, PD 705?
island is deemed agricultural pursuant to the Philippine Bill of 1902 and Act No. 926,
II.
known as the first Public Land Act.[32] Thus, their possession in the concept of owner for HAVE PETITIONERS OCCUPANTS ACQUIRED PRIOR VESTED RIGHT OF
the required period entitled them to judicial confirmation of imperfect title. PRIVATE OWNERSHIP OVER THEIR OCCUPIED PORTIONS OF BORACAY LAND,
DESPITE THE FACT THAT THEY HAVE NOT APPLIED YET FOR JUDICIAL
CONFIRMATION OF IMPERFECT TITLE?
Opposing the petition, the OSG argued that petitioners-claimants do not have a vested III.
right over their occupied portions in the island. Boracay is an unclassified public forest IS THE EXECUTIVE DECLARATION OF THEIR AREAS AS ALIENABLE AND
DISPOSABLE UNDER SEC 6, CA 141 [AN] INDISPENSABLE PRE-REQUISITE FOR
land pursuant to Section 3(a) of PD No. 705. Being public forest, the claimed portions of PETITIONERS TO OBTAIN TITLE UNDER THE TORRENS SYSTEM?
the island are inalienable and cannot be the subject of judicial confirmation of imperfect IV.
title. It is only the executive department, not the courts, which has authority to reclassify IS THE ISSUANCE OF PROCLAMATION 1064 ON MAY 22, 2006, VIOLATIVE OF
THE PRIOR VESTED RIGHTS TO PRIVATE OWNERSHIP OF PETITIONERS OVER
lands of the public domain into alienable and disposable lands. There is a need for a THEIR LANDS IN BORACAY, PROTECTED BY THE DUE PROCESS CLAUSE OF
THE CONSTITUTION OR IS PROCLAMATION 1064 CONTRARY TO SEC. 8, CA 141,
positive government act in order to release the lots for disposition.
OR SEC. 4(a) OF RA 6657.

V.
On November 21, 2006, this Court ordered the consolidation of the two petitions as they CAN RESPONDENTS BE COMPELLED BY MANDAMUS TO ALLOW THE SURVEY
principally involve the same issues on the land classification of Boracay Island.[33] AND TO APPROVE THE SURVEY PLANS FOR PURPOSES OF THE APPLICATION
FOR TITLING OF THE LANDS OF PETITIONERS IN BORACAY?[35] (Underscoring
supplied)
Issues
In capsule, the main issue is whether private claimants (respondents-claimants in G.R.
No. 167707 and petitioners-claimants in G.R. No. 173775) have a right to secure titles
G.R. No. 167707
over their occupied portions in Boracay. The twin petitions pertain to their right, if any, to
judicial confirmation of imperfect title under CA No. 141, as amended. They do not
The OSG raises the lone issue of whether Proclamation No. 1801 and PTA Circular No.
involve their right to secure title under other pertinent laws.
3-82 pose any legal obstacle for respondents, and all those similarly situated, to acquire
title to their occupied lands in Boracay Island.[34]
Our Ruling

Regalian Doctrine and power of the executive


to reclassify lands of the public domain
G.R. No. 173775
Private claimants rely on three (3) laws and executive acts in their bid for judicial

Petitioners-claimants hoist five (5) issues, namely: confirmation of imperfect title, namely: (a) Philippine Bill of 1902 [36] in relation to Act No.
926, later amended and/or superseded by Act No. 2874 and CA No. 141;[37] (b) Our present land law traces its roots to the Regalian Doctrine. Upon the Spanish
Proclamation No. 1801[38] issued by then President Marcos; and (c) Proclamation No. conquest of the Philippines, ownership of all lands, territories and possessions in the
1064[39] issued by President Gloria Macapagal-Arroyo. We shall proceed to determine Philippines passed to the Spanish Crown.[50] The Regalian doctrine was first introduced
their rights to apply for judicial confirmation of imperfect title under these laws and in the Philippines through the Laws of the Indies and the Royal Cedulas, which laid the
executive acts. foundation that all lands that were not acquired from the Government, either by
purchase or by grant, belong to the public domain. [51]
But first, a peek at the Regalian principle and the power of the executive to reclassify
lands of the public domain. The Laws of the Indies was followed by the Ley Hipotecaria or the Mortgage Law of
1893. The Spanish Mortgage Law provided for the systematic registration of titles and
The 1935 Constitution classified lands of the public domain into agricultural, forest or deeds as well as possessory claims.[52]
timber.[40] Meanwhile, the 1973 Constitution provided the following divisions:
agricultural, industrial or commercial, residential, resettlement, mineral, timber or forest The Royal Decree of 1894 or the Maura Law[53] partly amended the Spanish Mortgage
and grazing lands, and such other classes as may be provided by law, [41] giving the Law and the Laws of the Indies. It established possessory information as the method of
government great leeway for classification.[42] Then the 1987 Constitution reverted to legalizing possession of vacant Crown land, under certain conditions which were set
the 1935 Constitution classification with one addition: national parks.[43] Of these, forth in said decree.[54] Under Section 393 of the Maura Law, an informacion posesoria
onlyagricultural lands may be alienated.[44] Prior to Proclamation No. 1064 of May 22, or possessory information title,[55] when duly inscribed in the Registry of Property, is
2006, Boracay Island had never been expressly and administratively classified under converted into a title of ownership only after the lapse of twenty (20) years of
any of these grand divisions. Boracay was an unclassified land of the public domain. uninterrupted possession which must be actual, public, and adverse, [56] from the date of
its inscription.[57] However, possessory information title had to be perfected one year
The Regalian Doctrine dictates that all lands of the public domain belong to the State, after the promulgation of the Maura Law, or until April 17, 1895. Otherwise, the lands
that the State is the source of any asserted right to ownership of land and charged with would revert to the State.[58]
the conservation of such patrimony.[45] The doctrine has been consistently adopted
under the 1935, 1973, and 1987 Constitutions.[46] In sum, private ownership of land under the Spanish regime could only be founded on
royal concessions which took various forms, namely: (1) titulo real or royal grant; (2)
All lands not otherwise appearing to be clearly within private ownership are presumed to concesion especial or special grant; (3) composicion con el estado or adjustment title;
belong to the State.[47] Thus, all lands that have not been acquired from the government, (4) titulo de compra or title by purchase; and (5) informacion posesoria or possessory
either by purchase or by grant, belong to the State as part of the inalienable public information title.[59]
domain.[48] Necessarily, it is up to the State to determine if lands of the public domain
will be disposed of for private ownership. The government, as the agent of the state, is The first law governing the disposition of public lands in the Philippines under American
possessed of the plenary power as the persona in law to determine who shall be the rule was embodied in the Philippine Bill of 1902.[60] By this law, lands of the public
favored recipients of public lands, as well as under what terms they may be granted domain in the Philippine Islands were classified into three (3) grand divisions, to wit:
such privilege, not excluding the placing of obstacles in the way of their exercise of what agricultural, mineral, and timber or forest lands.[61] The act provided for, among others,
otherwise would be ordinary acts of ownership.[49] the disposal of mineral lands by means of absolute grant (freehold system) and by lease
(leasehold system).[62] It also provided the definition by exclusion of agricultural public
lands.[63] Interpreting the meaning of agricultural lands under the Philippine Bill of 1902, Section 48(b) of CA No. 141 retained the requirement under Act No. 2874 of possession
the Court declared in Mapa v. Insular Government:[64] and occupation of lands of the public domain since time immemorial or since July 26,
1894. However, this provision was superseded by Republic Act (RA) No. 1942, [72] which
provided for a simple thirty-year prescriptive period for judicial confirmation of imperfect
x x x In other words, that the phrase agricultural land as used in Act No. 926 means title. The provision was last amended by PD No. 1073,[73] which now provides for
those public lands acquired from Spain which are not timber or mineral lands. x x
x[65](Emphasis Ours) possession and occupation of the land applied for since June 12, 1945, or earlier.[74]

On February 1, 1903, the Philippine Legislature passed Act No. 496, otherwise known The issuance of PD No. 892[75] on February 16, 1976 discontinued the use of Spanish
as the Land Registration Act. The act established a system of registration by which titles as evidence in land registration proceedings. [76] Under the decree, all holders of
recorded title becomes absolute, indefeasible, and imprescriptible. This is known as the Spanish titles or grants should apply for registration of their lands under Act No. 496
Torrens system.[66] within six (6) months from the effectivity of the decree on February 16, 1976. Thereafter,
the recording of all unregistered lands[77] shall be governed by Section 194 of the
Concurrently, on October 7, 1903, the Philippine Commission passed Act No. 926, Revised Administrative Code, as amended by Act No. 3344.
which was the first Public Land Act. The Act introduced the homestead system and
made provisions for judicial and administrative confirmation of imperfect titles and for On June 11, 1978, Act No. 496 was amended and updated by PD No. 1529, known as
the sale or lease of public lands. It permitted corporations regardless of the nationality of the Property Registration Decree. It was enacted to codify the various laws relative to
persons owning the controlling stock to lease or purchase lands of the public domain. [67] registration of property.[78] It governs registration of lands under the Torrens system as
Under the Act, open, continuous, exclusive, and notorious possession and occupation well as unregistered lands, including chattel mortgages. [79]
of agricultural lands for the next ten (10) years preceding July 26, 1904 was sufficient
for judicial confirmation of imperfect title.[68] A positive act declaring land as alienable and disposable is required. In keeping
with the presumption of State ownership, the Court has time and again emphasized that
On November 29, 1919, Act No. 926 was superseded by Act No. 2874, otherwise there must be a positive act of the government, such as an official proclamation,[80]
known as the second Public Land Act. This new, more comprehensive law limited the declassifying inalienable public land into disposable land for agricultural or other
exploitation of agricultural lands to Filipinos and Americans and citizens of other purposes.[81] In fact, Section 8 of CA No. 141 limits alienable or disposable lands only to
countries which gave Filipinos the same privileges. For judicial confirmation of title, those lands which have been officially delimited and classified. [82]
possession and occupation en concepto dueo since time immemorial, or since July 26,
1894, was required.[69] The burden of proof in overcoming the presumption of State ownership of the lands of
the public domain is on the person applying for registration (or claiming ownership), who
After the passage of the 1935 Constitution, CA No. 141 amended Act No. 2874 on must prove that the land subject of the application is alienable or disposable. [83] To
December 1, 1936. To this day, CA No. 141, as amended, remains as the existing overcome this presumption, incontrovertible evidence must be established that the land
general law governing the classification and disposition of lands of the public domain subject of the application (or claim) is alienable or disposable. [84] There must still be a
other than timber and mineral lands,[70] and privately owned lands which reverted to the positive act declaring land of the public domain as alienable and disposable. To prove
State.[71] that the land subject of an application for registration is alienable, the applicant must
establish the existence of a positive act of the government such as a presidential
proclamation or an executive order; an administrative action; investigation reports of Spouses Pedro S. Palanca and Soterranea Rafols Vda. De Palanca v. Republic,[92] in
Bureau of Lands investigators; and a legislative act or a statute.[85] The applicant may which it stated, through Justice Adolfo Azcuna, viz.:
also secure a certification from the government that the land claimed to have been
possessed for the required number of years is alienable and disposable.[86] x x x Petitioners furthermore insist that a particular land need not be formally released
by an act of the Executive before it can be deemed open to private ownership, citing the
cases of Ramos v. Director of Lands and Ankron v. Government of the Philippine
In the case at bar, no such proclamation, executive order, administrative action, report, Islands.

statute, or certification was presented to the Court. The records are bereft of evidence xxxx
showing that, prior to 2006, the portions of Boracay occupied by private claimants were
Petitioners reliance upon Ramos v. Director of Lands and Ankron v. Government is
subject of a government proclamation that the land is alienable and disposable.Absent misplaced. These cases were decided under the Philippine Bill of 1902 and the first
Public Land Act No. 926 enacted by the Philippine Commission on October 7, 1926,
such well-nigh incontrovertible evidence, the Court cannot accept the submission that under which there was no legal provision vesting in the Chief Executive or President of
lands occupied by private claimants were already open to disposition before 2006. the Philippines the power to classify lands of the public domain into mineral, timber and
agricultural so that the courts then were free to make corresponding classifications in
Matters of land classification or reclassification cannot be assumed. They call for justiciable cases, or were vested with implicit power to do so, depending upon the
preponderance of the evidence.[93]
proof.[87]

To aid the courts in resolving land registration cases under Act No. 926, it was then
Ankron and De Aldecoa did not make the whole of Boracay Island, or portions of necessary to devise a presumption on land classification. Thus evolved the dictum in
it, agricultural lands. Private claimants posit that Boracay was already an agricultural Ankron that the courts have a right to presume, in the absence of evidence to the
land pursuant to the old cases Ankron v. Government of the Philippine Islands (1919) [88] contrary, that in each case the lands are agricultural lands until the contrary is shown. [94]
and De Aldecoa v. The Insular Government (1909).[89] These cases were decided under
the provisions of the Philippine Bill of 1902 and Act No. 926. There is a statement in
these old cases that in the absence of evidence to the contrary, that in each case the
lands are agricultural lands until the contrary is shown.[90] But We cannot unduly expand the presumption in Ankron and De Aldecoa to an
argument that all lands of the public domain had been automatically reclassified as
Private claimants reliance on Ankron and De Aldecoa is misplaced. These cases did not disposable and alienable agricultural lands. By no stretch of imagination did the
have the effect of converting the whole of Boracay Island or portions of it into presumption convert all lands of the public domain into agricultural lands.
agricultural lands. It should be stressed that the Philippine Bill of 1902 and Act No. 926
merely provided the manner through which land registration courts would classify lands If We accept the position of private claimants, the Philippine Bill of 1902 and Act No.
of the public domain. Whether the land would be classified as timber, mineral, or 926 would have automatically made all lands in the Philippines, except those already
agricultural depended on proof presented in each case. classified as timber or mineral land, alienable and disposable lands. That would take
these lands out of State ownership and worse, would be utterly inconsistent with and
Ankron and De Aldecoa were decided at a time when the President of the Philippines totally repugnant to the long-entrenched Regalian doctrine.
had no power to classify lands of the public domain into mineral, timber, and
agricultural. At that time, the courts were free to make corresponding classifications in The presumption in Ankron and De Aldecoa attaches only to land registration cases
justiciable cases, or were vested with implicit power to do so, depending upon the brought under the provisions of Act No. 926, or more specifically those cases dealing
preponderance of the evidence.[91] This was the Courts ruling in Heirs of the Late
with judicial and administrative confirmation of imperfect titles. The presumption applies Since 1919, courts were no longer free to determine the classification of lands from the
to an applicant for judicial or administrative conformation of imperfect title under Act No. facts of each case, except those that have already became private lands. [96] Act No.
926. It certainly cannot apply to landowners, such as private claimants or their 2874, promulgated in 1919 and reproduced in Section 6 of CA No. 141, gave the
predecessors-in-interest, who failed to avail themselves of the benefits of Act No. 926. Executive Department, through the President, the exclusive prerogative to classify or
As to them, their land remained unclassified and, by virtue of the Regalian doctrine, reclassify public lands into alienable or disposable, mineral or forest.96-a Since then,
continued to be owned by the State. courts no longer had the authority, whether express or implied, to determine the
classification of lands of the public domain.[97]
In any case, the assumption in Ankron and De Aldecoa was not absolute. Land
classification was, in the end, dependent on proof. If there was proof that the land was Here, private claimants, unlike the Heirs of Ciriaco Tirol who were issued their title in
better suited for non-agricultural uses, the courts could adjudge it as a mineral or timber 1933,[98] did not present a justiciable case for determination by the land registration
land despite the presumption. In Ankron, this Court stated: court of the propertys land classification. Simply put, there was no opportunity for the
courts then to resolve if the land the Boracay occupants are now claiming were
In the case of Jocson vs. Director of Forestry (supra), the Attorney-General admitted in agricultural lands. When Act No. 926 was supplanted by Act No. 2874 in 1919, without
effect that whether the particular land in question belongs to one class or another is a
question of fact. The mere fact that a tract of land has trees upon it or has mineral within an application for judicial confirmation having been filed by private claimants or their
it is not of itself sufficient to declare that one is forestry land and the other, mineral land. predecessors-in-interest, the courts were no longer authorized to determine the
There must be some proof of the extent and present or future value of the forestry and
of the minerals. While, as we have just said, many definitions have been given for propertys land classification. Hence, private claimants cannot bank on Act No. 926.
agriculture, forestry, and mineral lands, and that in each case it is a question of fact, we
think it is safe to say that in order to be forestry or mineral land the proof must show that
it is more valuable for the forestry or the mineral which it contains than it is for We note that the RTC decision[99] in G.R. No. 167707 mentioned Krivenko v. Register of
agricultural purposes. (Sec. 7, Act No. 1148.) It is not sufficient to show that there exists
some trees upon the land or that it bears some mineral. Land may be classified as Deeds of Manila,[100] which was decided in 1947 when CA No. 141, vesting the
forestry or mineral today, and, by reason of the exhaustion of the timber or mineral, be Executive with the sole power to classify lands of the public domain was already in
classified as agricultural land tomorrow. And vice-versa, by reason of the rapid growth
of timber or the discovery of valuable minerals, lands classified as agricultural today effect. Krivenko cited the old cases Mapa v. Insular Government,[101] De Aldecoa v. The
may be differently classified tomorrow. Each case must be decided upon the proof in
that particular case, having regard for its present or future value for one or the Insular Government,[102] and Ankron v. Government of the Philippine Islands.[103]
other purposes. We believe, however, considering the fact that it is a matter of public
knowledge that a majority of the lands in the Philippine Islands are agricultural lands
that the courts have a right to presume, in the absence of evidence to the contrary, that Krivenko, however, is not controlling here because it involved a totally different issue.
in each case the lands are agricultural lands until the contrary is shown. Whatever the
land involved in a particular land registration case is forestry or mineral land The pertinent issue in Krivenko was whether residential lots were included in the
must, therefore, be a matter of proof. Its superior value for one purpose or the general classification of agricultural lands; and if so, whether an alien could acquire a
other is a question of fact to be settled by the proof in each particular case. The
fact that the land is a manglar [mangrove swamp] is not sufficient for the courts to residential lot. This Court ruled that as an alien, Krivenko was prohibited by the 1935
decide whether it is agricultural, forestry, or mineral land. It may perchance belong to
Constitution[104] from acquiring agricultural land, which included residential lots. Here,
one or the other of said classes of land. The Government, in the first instance, under the
provisions of Act No. 1148, may, by reservation, decide for itself what portions of public the issue is whether unclassified lands of the public domain are automatically deemed
land shall be considered forestry land, unless private interests have intervened before
such reservation is made. In the latter case, whether the land is agricultural, forestry, or agricultural.
mineral, is a question of proof. Until private interests have intervened, the Government,
by virtue of the terms of said Act (No. 1148), may decide for itself what portions of the
public domain shall be set aside and reserved as forestry or mineral land. (Ramos vs.
Director of Lands, 39 Phil. 175; Jocson vs. Director of Forestry, supra)[95] (Emphasis
ours) Notably, the definition of agricultural public lands mentioned in Krivenko relied on the
old cases decided prior to the enactment of Act No. 2874, including Ankron and De
Aldecoa.[105] As We have already stated, those cases cannot apply here, since they PD No. 705 issued by President Marcos categorized all unclassified lands of the public
were decided when the Executive did not have the authority to classify lands as domain as public forest. Section 3(a) of PD No. 705 defines a public forest as a mass of
agricultural, timber, or mineral. lands of the public domain which has not been the subject of the present system of
classification for the determination of which lands are needed for forest purpose and
Private claimants continued possession under Act No. 926 does not create a
which are not. Applying PD No. 705, all unclassified lands, including those in Boracay
presumption that the land is alienable. Private claimants also contend that their
Island, are ipso facto considered public forests. PD No. 705, however, respects titles
continued possession of portions of Boracay Island for the requisite period of ten (10)
already existing prior to its effectivity.
years under Act No. 926[106] ipso facto converted the island into private ownership.
Hence, they may apply for a title in their name.
The Court notes that the classification of Boracay as a forest land under PD No. 705

A similar argument was squarely rejected by the Court in Collado v. Court of may seem to be out of touch with the present realities in the island. Boracay, no doubt,

Appeals.[107] Collado, citing the separate opinion of now Chief Justice Reynato S. Puno has been partly stripped of its forest cover to pave the way for commercial

in Cruz v. Secretary of Environment and Natural Resources,107-a ruled: developments. As a premier tourist destination for local and foreign tourists, Boracay
appears more of a commercial island resort, rather than a forest land.

Act No. 926, the first Public Land Act, was passed in pursuance of the provisions of the
Philippine Bill of 1902. The law governed the disposition of lands of the public domain. It Nevertheless, that the occupants of Boracay have built multi-million peso beach resorts
prescribed rules and regulations for the homesteading, selling and leasing of portions of
the public domain of the Philippine Islands, and prescribed the terms and conditions to on the island;[111] that the island has already been stripped of its forest cover; or that the
enable persons to perfect their titles to public lands in the Islands. It also provided for
implementation of Proclamation No. 1064 will destroy the islands tourism industry, do
the issuance of patents to certain native settlers upon public lands, for the
establishment of town sites and sale of lots therein, for the completion of imperfect titles, not negate its character as public forest.
and for the cancellation or confirmation of Spanish concessions and grants in the
Islands. In short, the Public Land Act operated on the assumption that title to public
lands in the Philippine Islands remained in the government; and that the governments Forests, in the context of both the Public Land Act and the Constitution [112] classifying
title to public land sprung from the Treaty of Paris and other subsequent treaties
between Spain and the United States. The term public land referred to all lands of the lands of the public domain into agricultural, forest or timber, mineral lands, and national
public domain whose title still remained in the government and are thrown open to
private appropriation and settlement, and excluded the patrimonial property of the parks, do not necessarily refer to large tracts of wooded land or expanses covered by
government and the friar lands. dense growths of trees and underbrushes.[113] The discussion in Heirs of Amunategui v.
Thus, it is plain error for petitioners to argue that under the Philippine Bill of 1902 Director of Forestry[114] is particularly instructive:
and Public Land Act No. 926, mere possession by private individuals of lands
creates the legal presumption that the lands are alienable and disposable.[108]
(Emphasis Ours) A forested area classified as forest land of the public domain does not lose such
classification simply because loggers or settlers may have stripped it of its forest cover.
Except for lands already covered by existing titles, Boracay was an unclassified Parcels of land classified as forest land may actually be covered with grass or planted to
crops by kaingin cultivators or other farmers. Forest lands do not have to be on
land of the public domain prior to Proclamation No. 1064. Such unclassified lands mountains or in out of the way places. Swampy areas covered by mangrove trees, nipa
palms, and other trees growing in brackish or sea water may also be classified as forest
are considered public forest under PD No. 705. The DENR[109] and the National land. The classification is descriptive of its legal nature or status and does not
Mapping and Resource Information Authority[110] certify that Boracay Island is an have to be descriptive of what the land actually looks like. Unless and until the land
classified as forest is released in an official proclamation to that effect so that it may
unclassified land of the public domain. form part of the disposable agricultural lands of the public domain, the rules on
confirmation of imperfect title do not apply.[115] (Emphasis supplied)
There is a big difference between forest as defined in a dictionary and forest or timber Therefore, Proclamation No. 1801 cannot be deemed the positive act needed to classify
land as a classification of lands of the public domain as appearing in our statutes. One Boracay Island as alienable and disposable land. If President Marcos intended to
is descriptive of what appears on the land while the other is a legal status, a classify the island as alienable and disposable or forest, or both, he would have
classification for legal purposes.[116] At any rate, the Court is tasked to determine the identified the specific limits of each, as President Arroyo did in Proclamation No. 1064.
legal status of Boracay Island, and not look into its physical layout. Hence, even if its This was not done in Proclamation No. 1801.
forest cover has been replaced by beach resorts, restaurants and other commercial
establishments, it has not been automatically converted from public forest to alienable The Whereas clauses of Proclamation No. 1801 also explain the rationale behind the
agricultural land. declaration of Boracay Island, together with other islands, caves and peninsulas in the
Philippines, as a tourist zone and marine reserve to be administered by the PTA to
Private claimants cannot rely on Proclamation No. 1801 as basis for judicial ensure the concentrated efforts of the public and private sectors in the development of
confirmation of imperfect title. The proclamation did not convert Boracay into an the areas tourism potential with due regard for ecological balance in the marine
agricultural land. However, private claimants argue that Proclamation No. 1801 issued environment. Simply put, the proclamation is aimed at administering the islands for
by then President Marcos in 1978 entitles them to judicial confirmation of imperfect title. tourism and ecological purposes. It does not address the areas alienability.[119]
The Proclamation classified Boracay, among other islands, as a tourist zone. Private
claimants assert that, as a tourist spot, the island is susceptible of private ownership. More importantly, Proclamation No. 1801 covers not only Boracay Island, but sixty-four
(64) other islands, coves, and peninsulas in the Philippines, such as Fortune and Verde
Proclamation No. 1801 or PTA Circular No. 3-82 did not convert the whole of Boracay Islands in Batangas, Port Galera in Oriental Mindoro, Panglao and Balicasag Islands in
into an agricultural land. There is nothing in the law or the Circular which made Boracay Bohol, Coron Island, Puerto Princesa and surrounding areas in Palawan, Camiguin
Island an agricultural land. The reference in Circular No. 3-82 to private lands[117] and Island in Cagayan de Oro, and Misamis Oriental, to name a few. If the designation of
areas declared as alienable and disposable [118] does not by itself classify the entire Boracay Island as tourist zone makes it alienable and disposable by virtue of
island as agricultural. Notably, Circular No. 3-82 makes reference not only to private Proclamation No. 1801, all the other areas mentioned would likewise be declared wide
lands and areas but also to public forested lands. Rule VIII, Section 3 provides: open for private disposition. That could not have been, and is clearly beyond, the intent
of the proclamation.
No trees in forested private lands may be cut without prior authority from the PTA. All
forested areas in public lands are declared forest reserves. (Emphasis supplied)
It was Proclamation No. 1064 of 2006 which positively declared part of Boracay as

Clearly, the reference in the Circular to both private and public lands merely recognizes alienable and opened the same to private ownership. Sections 6 and 7 of CA No.

that the island can be classified by the Executive department pursuant to its powers 141[120] provide that it is only the President, upon the recommendation of the proper

under CA No. 141. In fact, Section 5 of the Circular recognizes the then Bureau of department head, who has the authority to classify the lands of the public domain into

Forest Developments authority to declare areas in the island as alienable and alienable or disposable, timber and mineral lands. [121]

disposable when it provides:


In issuing Proclamation No. 1064, President Gloria Macapagal-Arroyo merely exercised
the authority granted to her to classify lands of the public domain, presumably subject to
Subsistence farming, in areas declared as alienable and disposable by the Bureau of
Forest Development. existing vested rights. Classification of public lands is the exclusive prerogative of the
Executive Department, through the Office of the President. Courts have no authority to
do so.[122] Absent such classification, the land remains unclassified until released and In Heirs of the Late Spouses Pedro S. Palanca and Soterranea Rafols v. Republic,[124]
rendered open to disposition.[123] the Court stated that unclassified lands are public forests.

Proclamation No. 1064 classifies Boracay into 400 hectares of reserved forest land and
628.96 hectares of agricultural land. The Proclamation likewise provides for a 15-meter While it is true that the land classification map does not categorically state that
the islands are public forests, the fact that they were unclassified lands leads to
buffer zone on each side of the center line of roads and trails, which are reserved for the same result.In the absence of the classification as mineral or timber land, the land
right of way and which shall form part of the area reserved for forest land protection remains unclassified land until released and rendered open to disposition. [125]
(Emphasis supplied)
purposes.
Contrary to private claimants argument, there was nothing invalid or irregular, much less Moreover, the prohibition under the CARL applies only to a reclassification of land. If the
unconstitutional, about the classification of Boracay Island made by the President land had never been previously classified, as in the case of Boracay, there can be no
through Proclamation No. 1064. It was within her authority to make such classification, prohibited reclassification under the agrarian law. We agree with the opinion of the
subject to existing vested rights. Department of Justice[126] on this point:

Proclamation No. 1064 does not violate the Comprehensive Agrarian Reform Law.
Private claimants further assert that Proclamation No. 1064 violates the provision of the
Indeed, the key word to the correct application of the prohibition in Section 4(a) is the
Comprehensive Agrarian Reform Law (CARL) or RA No. 6657 barring conversion of
word reclassification. Where there has been no previous classification of public forest
public forests into agricultural lands. They claim that since Boracay is a public forest [referring, we repeat, to the mass of the public domain which has not been the subject
of the present system of classification for purposes of determining which are needed for
under PD No. 705, President Arroyo can no longer convert it into an agricultural land forest purposes and which are not] into permanent forest or forest reserves or some
without running afoul of Section 4(a) of RA No. 6657, thus: other forest uses under the Revised Forestry Code, there can be no reclassification of
forest lands to speak of within the meaning of Section 4(a).

Thus, obviously, the prohibition in Section 4(a) of the CARL against the reclassification
SEC. 4. Scope. The Comprehensive Agrarian Reform Law of 1988 shall cover, of forest lands to agricultural lands without a prior law delimiting the limits of the public
regardless of tenurial arrangement and commodity produced, all public and private domain, does not, and cannot, apply to those lands of the public domain, denominated
agricultural lands as provided in Proclamation No. 131 and Executive Order No. 229, as public forest under the Revised Forestry Code, which have not been previously
including other lands of the public domain suitable for agriculture. determined, or classified, as needed for forest purposes in accordance with the
provisions of the Revised Forestry Code.[127]
More specifically, the following lands are covered by the Comprehensive Agrarian
Reform Program:
Private claimants are not entitled to apply for judicial confirmation of imperfect
(a) All alienable and disposable lands of the public domain devoted to or suitable for
agriculture. No reclassification of forest or mineral lands to agricultural lands shall be title under CA No. 141. Neither do they have vested rights over the occupied
undertaken after the approval of this Act until Congress, taking into account ecological, lands under the said law. There are two requisites for judicial confirmation of imperfect
developmental and equity considerations, shall have determined by law, the specific
limits of the public domain. or incomplete title under CA No. 141, namely: (1) open, continuous, exclusive, and
notorious possession and occupation of the subject land by himself or through his
predecessors-in-interest under a bona fide claim of ownership since time immemorial or
That Boracay Island was classified as a public forest under PD No. 705 did not bar the
from June 12, 1945; and (2) the classification of the land as alienable and disposable
Executive from later converting it into agricultural land. Boracay Island still remained an
land of the public domain.[128]
unclassified land of the public domain despite PD No. 705.
As discussed, the Philippine Bill of 1902, Act No. 926, and Proclamation No. 1801 did The continued possession and considerable investment of private claimants do not
not convert portions of Boracay Island into an agricultural land. The island remained an automatically give them a vested right in Boracay. Nor do these give them a right to
unclassified land of the public domain and, applying the Regalian doctrine, is apply for a title to the land they are presently occupying. This Court is constitutionally
considered State property. bound to decide cases based on the evidence presented and the laws applicable. As
the law and jurisprudence stand, private claimants are ineligible to apply for a judicial
Private claimants bid for judicial confirmation of imperfect title, relying on the Philippine confirmation of title over their occupied portions in Boracay even with their continued
Bill of 1902, Act No. 926, and Proclamation No. 1801, must fail because of the absence possession and considerable investment in the island.
of the second element of alienable and disposable land. Their entitlement to a
government grant under our present Public Land Act presupposes that the land One Last Note
possessed and applied for is already alienable and disposable. This is clear from the
wording of the law itself.[129] Where the land is not alienable and disposable, possession
of the land, no matter how long, cannot confer ownership or possessory rights. [130] The Court is aware that millions of pesos have been invested for the development of
Boracay Island, making it a by-word in the local and international tourism industry. The

Neither may private claimants apply for judicial confirmation of imperfect title under Court also notes that for a number of years, thousands of people have called the island

Proclamation No. 1064, with respect to those lands which were classified as agricultural their home. While the Court commiserates with private claimants plight, We are bound

lands. Private claimants failed to prove the first element of open, continuous, exclusive, to apply the law strictly and judiciously. This is the law and it should prevail. Ito ang

and notorious possession of their lands in Boracay since June 12, 1945. batas at ito ang dapat umiral.

We cannot sustain the CA and RTC conclusion in the petition for declaratory relief that
private claimants complied with the requisite period of possession.
All is not lost, however, for private claimants. While they may not be eligible to apply for
judicial confirmation of imperfect title under Section 48(b) of CA No. 141, as amended,
The tax declarations in the name of private claimants are insufficient to prove the first
this does not denote their automatic ouster from the residential, commercial, and other
element of possession. We note that the earliest of the tax declarations in the name of
areas they possess now classified as agricultural. Neither will this mean the loss of their
private claimants were issued in 1993. Being of recent dates, the tax declarations are
substantial investments on their occupied alienable lands. Lack of title does not
not sufficient to convince this Court that the period of possession and occupation
necessarily mean lack of right to possess.
commenced on June 12, 1945.

Private claimants insist that they have a vested right in Boracay, having been in
possession of the island for a long time. They have invested millions of pesos in For one thing, those with lawful possession may claim good faith as builders of
developing the island into a tourist spot. They say their continued possession and improvements. They can take steps to preserve or protect their possession. For
investments give them a vested right which cannot be unilaterally rescinded by another, they may look into other modes of applying for original registration of title, such
Proclamation No. 1064. as by homestead[131] or sales patent,[132] subject to the conditions imposed by law.
More realistically, Congress may enact a law to entitle private claimants to acquire title WHEREFORE, judgment is rendered as follows:
to their occupied lots or to exempt them from certain requirements under the present
land laws. There is one such bill[133] now pending in the House of Representatives.
Whether that bill or a similar bill will become a law is for Congress to decide. 1. The petition for certiorari in G.R. No. 167707 is GRANTED and the Court of Appeals
Decision in CA-G.R. CV No. 71118 REVERSED AND SET ASIDE.

2. The petition for certiorari in G.R. No. 173775 is DISMISSED for lack of merit.
In issuing Proclamation No. 1064, the government has taken the step necessary to
open up the island to private ownership. This gesture may not be sufficient to appease SO ORDERED.
some sectors which view the classification of the island partially into a forest reserve as
absurd. That the island is no longer overrun by trees, however, does not becloud the
vision to protect its remaining forest cover and to strike a healthy balance between
progress and ecology. Ecological conservation is as important as economic progress.

To be sure, forest lands are fundamental to our nations survival. Their promotion and
protection are not just fancy rhetoric for politicians and activists. These are needs that
become more urgent as destruction of our environment gets prevalent and difficult to
control. As aptly observed by Justice Conrado Sanchez in 1968 in Director of Forestry
v. Munoz:[134]

The view this Court takes of the cases at bar is but in adherence to public policy that
should be followed with respect to forest lands. Many have written much, and many
more have spoken, and quite often, about the pressing need for forest preservation,
conservation, protection, development and reforestation. Not without justification. For,
forests constitute a vital segment of any country's natural resources. It is of common FIRST DIVISION
knowledge by now that absence of the necessary green cover on our lands produces a
number of adverse or ill effects of serious proportions. Without the trees, watersheds [G.R. NO. 154953 : June 26, 2008]
dry up; rivers and lakes which they supply are emptied of their contents. The fish
disappear. Denuded areas become dust bowls. As waterfalls cease to function, so will REPUBLIC OF THE PHILIPPINES, Petitioner, v. T.A.N. PROPERTIES,
hydroelectric plants. With the rains, the fertile topsoil is washed away; geological INC.,Respondent.
erosion results. With erosion come the dreaded floods that wreak havoc and destruction
to property crops, livestock, houses, and highways not to mention precious human lives.
Indeed, the foregoing observations should be written down in a lumbermans DECISION
decalogue.[135]
CARPIO, J.:

The Case
Before the Court is a Petition for Review 1 assailing the 21 August 2002 Decision 2 of the In its 16 December 1999 Decision, the trial court adjudicated the land in favor of
Court of Appeals in CA-G.R. CV No. 66658. The Court of Appeals affirmed in toto the respondent.
16 December 1999 Decision3 of the Regional Trial Court of Tanauan, Batangas, Branch
6 (trial court) in Land Registration Case No. T-635. The trial court ruled that a juridical person or a corporation could apply for registration of
land provided such entity and its predecessors-in-interest have possessed the land for
The Antecedent Facts 30 years or more. The trial court ruled that the facts showed that respondent's
predecessors-in-interest possessed the land in the concept of an owner prior to 12 June
This case originated from an Application for Original Registration of Title filed by T.A.N. 1945, which possession converted the land to private property.
Properties, Inc. covering Lot 10705-B of the subdivision plan Csd-04-019741 which is a
portion of the consolidated Lot 10705, Cad-424, Sto. Tomas Cadastre. The land, with The dispositive portion of the trial court's Decision reads:
an area of 564,007 square meters, or 56.4007 hectares, is located at San Bartolome,
Sto. Tomas, Batangas. WHEREFORE, and upon previous confirmation of the Order of General Default, the
Court hereby adjudicates and decrees Lot 10705-B, identical to Lot 13637, Cad-424,
On 31 August 1999, the trial court set the case for initial hearing at 9:30 a.m. on 11 Sto. Tomas Cadastre, on plan Csd-04-019741, situated in Barangay of San Bartolome,
November 1999. The Notice of Initial Hearing was published in the Official Gazette, 20 Municipality of Sto. Tomas, Province of Batangas, with an area of 564,007 square
September 1999 issue, Volume 95, No. 38, pages 6793 to 6794, 4 and in the 18 October meters, in favor of and in the name of T.A.N. Properties, Inc., a domestic corporation
1999 issue of People's Journal Taliba, 5 a newspaper of general circulation in the duly organized and existing under Philippine laws with principal office at 19 th Floor,
Philippines. The Notice of Initial Hearing was also posted in a conspicuous place on the PDCP Bank Building, 8737 Paseo de Roxas, Makati City.
bulletin board of the Municipal Building of Sto. Tomas, Batangas, as well as in a
conspicuous place on the land.6 All adjoining owners and all government agencies and Once this Decision shall have become final, let the corresponding decree of registration
offices concerned were notified of the initial hearing.7 be issued.

On 11 November 1999, when the trial court called the case for initial hearing, there was SO ORDERED.12
no oppositor other than the Opposition dated 7 October 1999 of the Republic of the
Philippines represented by the Director of Lands (petitioner). On 15 November 1999,
the trial court issued an Order8 of General Default against the whole world except as Petitioner appealed from the trial court's Decision. Petitioner alleged that the trial court
against petitioner. erred in granting the application for registration absent clear evidence that the applicant
and its predecessors-in-interest have complied with the period of possession and
occupation as required by law. Petitioner alleged that the testimonies of Evangelista and
During the hearing on 19 November 1999, Ceferino Carandang (Carandang) appeared Torres are general in nature. Considering the area involved, petitioner argued that
as oppositor. The trial court gave Carandang until 29 November 1999 within which to additional witnesses should have been presented to corroborate Evangelista's
file his written opposition.9 Carandang failed to file his written opposition and to appear testimony.
in the succeeding hearings. In an Order10 dated 13 December 1999, the trial court
reinstated the Order of General Default.
The Ruling of the Court of Appeals
During the hearings conducted on 13 and 14 December 1999, respondent presented
three witnesses: Anthony Dimayuga Torres (Torres), respondent's Operations Manager In its 21 August 2002 Decision, the Court of Appeals affirmed in toto the trial court's
and its authorized representative in the case; Primitivo Evangelista (Evangelista), a 72- Decision.
year old resident of San Bartolome, Sto. Tomas, Batangas since birth; and Regalado
Marquez, Records Officer II of the Land Registration Authority (LRA), Quezon City. The Court of Appeals ruled that Evangelista's knowledge of the possession and
occupation of the land stemmed not only from the fact that he worked there for three
The testimonies of respondent's witnesses showed that Prospero Dimayuga (Kabesang years but also because he and Kabesang Puroy were practically neighbors. On
Puroy) had peaceful, adverse, open, and continuous possession of the land in the Evangelista's failure to mention the name of his uncle who continuously worked on the
concept of an owner since 1942. Upon his death, Kabesang Puroy was succeeded by land, the Court of Appeals ruled that Evangelista should not be faulted as he was not
his son Antonio Dimayuga (Antonio). On 27 September 1960, Antonio executed a Deed asked to name his uncle when he testified. The Court of Appeals also ruled that at the
of Donation covering the land in favor of one of his children, Fortunato Dimayuga outset, Evangelista disclaimed knowledge of Fortunato's relation to Kabesang Puroy,
(Fortunato). Later, however, Antonio gave Fortunato another piece of land. Hence, on but this did not affect Evangelista's statement that Fortunato took over the possession
26 April 1961, Antonio executed a Partial Revocation of Donation, and the land was and cultivation of the land after Kabesang Puroy's death. The Court of Appeals further
adjudicated to one of Antonio's children, Prospero Dimayuga (Porting). 11 On 8 August ruled that the events regarding the acquisition and disposition of the land became public
1997, Porting sold the land to Respondent. knowledge because San Bartolome was a small community. On the matter of additional
witnesses, the Court of Appeals ruled that petitioner failed to cite any law requiring the
corroboration of the sole witness' testimony.
The Ruling of the Trial Court
The Court of Appeals further ruled that Torres was a competent witness since he was City,16certified that "lot 10705, Cad-424, Sto. Tomas Cadastre situated at Barangay San
only testifying on the fact that he had caused the filing of the application for registration Bartolome, Sto. Tomas, Batangas with an area of 596,116 square meters falls within the
and that respondent acquired the land from Porting. ALIENABLE AND DISPOSABLE ZONE under Project No. 30, Land Classification Map
No. 582 certified [on] 31 December 1925." The second certification 17 in the form of a
Petitioner comes to this Court assailing the Court of Appeals' Decision. Petitioner raises memorandum to the trial court, which was issued by the Regional Technical Director,
the following grounds in its Memorandum: Forest Management Services of the DENR (FMS-DENR), stated "that the subject area
falls within an alienable and disposable land, Project No. 30 of Sto. Tomas, Batangas
certified on Dec. 31, 1925 per LC No. 582."
The Court of Appeals erred on a question of law in allowing the grant of title to applicant
corporation despite the following:
The certifications are not sufficient. DENR Administrative Order (DAO) No. 20, 18 dated
30 May 1988, delineated the functions and authorities of the offices within the DENR.
1. Absence of showing that it or its predecessors-in-interest had open, continuous, Under DAO No. 20, series of 1988, the CENRO issues certificates of land classification
exclusive, and notorious possession and occupation in the concept of an owner since status for areas below 50 hectares. The Provincial Environment and Natural Resources
12 June 1945 or earlier; andcralawlibrary Offices (PENRO) issues certificate of land classification status for lands covering over
50 hectares. DAO No. 38,19 dated 19 April 1990, amended DAO No. 20, series of 1988.
2. Disqualification of applicant corporation to acquire the subject tract of land.13 DAO No. 38, series of 1990 retained the authority of the CENRO to issue certificates of
land classification status for areas below 50 hectares, as well as the authority of the
The Issues PENRO to issue certificates of land classification status for lands covering over 50
hectares.20 In this case, respondent applied for registration of Lot 10705-B. The area
covered by Lot 10705-B is over 50 hectares (564,007 square meters). The CENRO
The issues may be summarized as follows: certificate covered the entire Lot 10705 with an area of 596,116 square meters which,
as per DAO No. 38, series of 1990, is beyond the authority of the CENRO to certify as
1. Whether the land is alienable and disposable; alienable and disposable.

2. Whether respondent or its predecessors-in-interest had open, continuous, exclusive, The Regional Technical Director, FMS-DENR, has no authority under DAO Nos. 20 and
and notorious possession and occupation of the land in the concept of an owner since 38 to issue certificates of land classification. Under DAO No. 20, the Regional Technical
June 1945 or earlier; andcralawlibrary Director, FMS-DENR:

3. Whether respondent is qualified to apply for registration of the land under the Public 1. Issues original and renewal of ordinary minor products (OM) permits except rattan;
Land Act.
2. Approves renewal of resaw/mini-sawmill permits;
The Ruling of this Court
3. Approves renewal of special use permits covering over five hectares for public
The petition has merit. infrastructure projects; andcralawlibrary

Respondent Failed to Prove 4. Issues renewal of certificates of registration for logs, poles, piles, and lumber dealers.

that the Land is Alienable and Disposable Under DAO No. 38, the Regional Technical Director, FMS-DENR:

Petitioner argues that anyone who applies for registration has the burden of overcoming 1. Issues original and renewal of ordinary minor [products] (OM) permits except rattan;
the presumption that the land forms part of the public domain. Petitioner insists that
respondent failed to prove that the land is no longer part of the public domain. 2. Issues renewal of certificate of registration for logs, poles, and piles and lumber
dealers;
The well-entrenched rule is that all lands not appearing to be clearly of private dominion
presumably belong to the State.14 The onus to overturn, by incontrovertible evidence, 3. Approves renewal of resaw/mini-sawmill permits;
the presumption that the land subject of an application for registration is alienable and
disposable rests with the applicant.15
4. Issues public gratuitous permits for 20 to 50 cubic meters within calamity declared
areas for public infrastructure projects; andcralawlibrary
In this case, respondent submitted two certifications issued by the Department of
Environment and Natural Resources (DENR). The 3 June 1997 Certification by the
Community Environment and Natural Resources Offices (CENRO), Batangas
5. Approves original and renewal of special use permits covering over five hectares for the facts stated therein. All other public documents are evidence, even against a third
public infrastructure projects. person, of the fact which gave rise to their execution and of the date of the latter.

Hence, the certification issued by the Regional Technical Director, FMS-DENR, in the The CENRO and Regional Technical Director, FMS-DENR, certifications do not fall
form of a memorandum to the trial court, has no probative value. within the class of public documents contemplated in the first sentence of Section 23 of
Rule 132. The certifications do not reflect "entries in public records made in the
Further, it is not enough for the PENRO or CENRO to certify that a land is alienable and performance of a duty by a public officer," such as entries made by the Civil Registrar 22
disposable. The applicant for land registration must prove that the DENR Secretary had in the books of registries, or by a ship captain in the ship's logbook. 23 The certifications
approved the land classification and released the land of the public domain as alienable are not the certified copies or authenticated reproductions of original official records in
and disposable, and that the land subject of the application for registration falls within the legal custody of a government office. The certifications are not even records of
the approved area per verification through survey by the PENRO or CENRO. In public documents.24 The certifications are conclusions unsupported by adequate proof,
addition, the applicant for land registration must present a copy of the original and thus have no probative value.25 Certainly, the certifications cannot be considered
classification approved by the DENR Secretary and certified as a true copy by the legal prima facieevidence of the facts stated therein.
custodian of the official records. These facts must be established to prove that the land
is alienable and disposable. Respondent failed to do so because the certifications The CENRO and Regional Technical Director, FMS-DENR, certifications do not prove
presented by respondent do not, by themselves, prove that the land is alienable and that Lot 10705-B falls within the alienable and disposable land as proclaimed by the
disposable. DENR Secretary. Such government certifications do not, by their mere issuance, prove
the facts stated therein.26 Such government certifications may fall under the class of
Only Torres, respondent's Operations Manager, identified the certifications submitted by documents contemplated in the second sentence of Section 23 of Rule 132. As such,
respondent. The government officials who issued the certifications were not presented the certifications are prima facie evidence of their due execution and date of issuance
before the trial court to testify on their contents. The trial court should not have accepted but they do not constitute prima facie evidence of the facts stated therein.
the contents of the certifications as proof of the facts stated therein. Even if the
certifications are presumed duly issued and admissible in evidence, they have no The Court has also ruled that a document or writing admitted as part of the testimony of
probative value in establishing that the land is alienable and disposable. a witness does not constitute proof of the facts stated therein. 27 Here, Torres, a private
individual and respondent's representative, identified the certifications but the
Public documents are defined under Section 19, Rule 132 of the Revised Rules on government officials who issued the certifications did not testify on the contents of the
Evidence as follows: certifications. As such, the certifications cannot be given probative value. 28 The contents
of the certifications are hearsay because Torres was incompetent to testify on the
veracity of the contents of the certifications. 29 Torres did not prepare the certifications,
(a) The written official acts, or records of the official acts of the sovereign authority, he was not an officer of CENRO or FMS-DENR, and he did not conduct any verification
official bodies and tribunals, and public officers, whether of the Philippines, or of a survey whether the land falls within the area classified by the DENR Secretary as
foreign country; alienable and disposable.

(b) Documents acknowledged before a notary public except last wills and testaments; Petitioner also points out the discrepancy as to when the land allegedly became
andcralawlibrary alienable and disposable. The DENR Secretary certified that based on Land
Classification Map No. 582, the land became alienable and disposable on 31 December
(c) Public records, kept in the Philippines, of private documents required by law to be 1925. However, the certificate on the blue print plan states that it became alienable and
entered therein. disposable on 31 December 1985.

Applying Section 24 of Rule 132, the record of public documents referred to in Section We agree with petitioner that while the certifications submitted by respondent show that
19(a), when admissible for any purpose, may be evidenced by an official publication under the Land Classification Map No. 582, the land became alienable and disposable
thereof or by a copy attested by the officer having legal custody of the record, or on 31 December 1925, the blue print plan states that it became alienable and
by his deputy x x x. The CENRO is not the official repository or legal custodian of the disposable on 31 December 1985. Respondent alleged that "the blue print plan merely
issuances of the DENR Secretary declaring public lands as alienable and disposable. serves to prove the precise location and the metes and bounds of the land described
The CENRO should have attached an official publication 21 of the DENR Secretary's therein x x x and does not in any way certify the nature and classification of the land
issuance declaring the land alienable and disposable. involved."30 It is true that the notation by a surveyor-geodetic engineer on the survey
plan that the land formed part of the alienable and disposable land of the public domain
Section 23, Rule 132 of the Revised Rules on Evidence provides: is not sufficient proof of the land's classification. 31 However, respondent should have at
least presented proof that would explain the discrepancy in the dates of classification.
Marquez, LRA Records Officer II, testified that the documents submitted to the court
Sec. 23. Public documents as evidence. Documents consisting of entries in public consisting of the tracing cloth plan, the technical description of Lot 10705-B, the
records made in the performance of a duty by a public officer are prima facieevidence of approved subdivision plan, and the Geodetic Engineer's certification were faithful
reproductions of the original documents in the LRA office. He did not explain the
discrepancy in the dates. Neither was the Geodetic Engineer presented to explain why were only paid starting 1955 considering the claim that the Dimayugas were allegedly in
the date of classification on the blue print plan was different from the other certifications possession of the land before 1945. The payment of the realty taxes starting 1955 gives
submitted by Respondent. rise to the presumption that the Dimayugas claimed ownership or possession of the
land only in that year.
There was No Open, Continuous, Exclusive, and Notorious
Land Application by a Corporation
Possession and Occupation in the Concept of an Owner
Petitioner asserts that respondent, a private corporation, cannot apply for registration of
Petitioner alleges that the trial court's reliance on the testimonies of Evangelista and the land of the public domain in this case.
Torres was misplaced. Petitioner alleges that Evangelista's statement that the
possession of respondent's predecessors-in-interest was open, public, continuous, We agree with petitioner.
peaceful, and adverse to the whole world was a general conclusion of law rather than
factual evidence of possession of title. Petitioner alleges that respondent failed to Section 3, Article XII of the 1987 Constitution provides:
establish that its predecessors-in-interest had held the land openly, continuously, and
exclusively for at least 30 years after it was declared alienable and disposable.
Sec. 3. Lands of the public domain are classified into agricultural, forest or timber,
mineral lands, and national parks. Agricultural lands of the public domain may be further
We agree with petitioner. classified by law according to the uses to which they may be devoted. Alienable lands of
the public domain shall be limited to agricultural lands. Private corporations or
Evangelista testified that Kabesang Puroy had been in possession of the land before associations may not hold such alienable lands of the public domain except by lease,
1945. Yet, Evangelista only worked on the land for three years. Evangelista testified that for a period not exceeding twenty-five years, renewable for not more than twenty-five
his family owned a lot near Kabesang Puroy's land. The Court of Appeals took note of years, and not to exceed one thousand hectares in area. Citizens of the Philippines may
this and ruled that Evangelista's knowledge of Kabesang Puroy's possession of the land lease not more than five hundred hectares, or acquire not more than twelve hectares
stemmed "not only from the fact that he had worked thereat but more so that they were thereof by purchase, homestead or grant.
practically neighbors."32 The Court of Appeals observed:
Taking into account the requirements of conservation, ecology, and development, and
In a small community such as that of San Bartolome, Sto. Tomas, Batangas, it is not subject to the requirements of agrarian reform, the Congress shall determine, by law,
difficult to understand that people in the said community knows each and everyone. the size of lands of the public domain which may be acquired, developed, held, or
And, because of such familiarity with each other, news or events regarding the leased and the conditions therefor.
acquisition or disposition for that matter, of a vast tract of land spreads like wildfire, thus,
the reason why such an event became of public knowledge to them. 33 The 1987 Constitution absolutely prohibits private corporations from acquiring any kind
of alienable land of the public domain. In Chavez v. Public Estates Authority,35 the Court
Evangelista testified that Kabesang Puroy was succeeded by Fortunato. However, he traced the law on disposition of lands of the public domain. Under the 1935 Constitution,
admitted that he did not know the exact relationship between Kabesang Puroy and there was no prohibition against private corporations from acquiring agricultural land.
Fortunato, which is rather unusual for neighbors in a small community. He did not also The 1973 Constitution limited the alienation of lands of the public domain to individuals
know the relationship between Fortunato and Porting. In fact, Evangelista's testimony is who were citizens of the Philippines. Under the 1973 Constitution, private corporations,
contrary to the factual finding of the trial court that Kabesang Puroy was succeeded by even if wholly owned by Filipino citizens, were no longer allowed to acquire alienable
his son Antonio, not by Fortunato who was one of Antonio's children. Antonio was not lands of the public domain. The present 1987 Constitution continues the prohibition
even mentioned in Evangelista's testimony. against private corporations from acquiring any kind of alienable land of the public
domain.36 The Court explained in Chavez:
The Court of Appeals ruled that there is no law that requires that the testimony of a
single witness needs corroboration. However, in this case, we find Evangelista's The 1987 Constitution continues the State policy in the 1973 Constitution banning
uncorroborated testimony insufficient to prove that respondent's predecessors-in- private corporations from acquiring any kind of alienable land of the public domain.
interest had been in possession of the land in the concept of an owner for more than 30 Like the 1973 Constitution, the 1987 Constitution allows private corporations to hold
years. We cannot consider the testimony of Torres as sufficient corroboration. Torres alienable lands of the public domain only through lease. x x x x
testified primarily on the fact of respondent's acquisition of the land. While he claimed to
be related to the Dimayugas, his knowledge of their possession of the land was [I]f the constitutional intent is to prevent huge landholdings, the Constitution could have
hearsay. He did not even tell the trial court where he obtained his information. simply limited the size of alienable lands of the public domain that corporations could
acquire. The Constitution could have followed the limitations on individuals, who could
The tax declarations presented were only for the years starting 1955. While tax acquire not more than 24 hectares of alienable lands of the public domain under the
declarations are not conclusive evidence of ownership, they constitute proof of claim of 1973 Constitution, and not more than 12 hectares under the 1987 Constitution.
ownership.34 Respondent did not present any credible explanation why the realty taxes
If the constitutional intent is to encourage economic family-size farms, placing the land time; and registration thereunder would not confer title, but simply recognize a title
in the name of a corporation would be more effective in preventing the break-up of already vested. The proceedings would not originally convert the land from public to
farmlands. If the farmland is registered in the name of a corporation, upon the death of private land, but only confirm such a conversion already effected by operation of law
the owner, his heirs would inherit shares in the corporation instead of subdivided from the moment the required period of possession became complete.
parcels of the farmland. This would prevent the continuing break-up of farmlands into
smaller and smaller plots from one generation to the next. x x x [A]lienable public land held by a possessor, personally or through his
predecessors-in-interest, openly, continuously and exclusively for the prescribed
In actual practice, the constitutional ban strengthens the constitutional limitation on statutory period of (30 years under The Public Land Act, as amended) is converted to
individuals from acquiring more than the allowed area of alienable lands of the public private property by the mere lapse or completion of said period, ipso jure. Following that
domain. Without the constitutional ban, individuals who already acquired the maximum rule and on the basis of the undisputed facts, the land subject of this appeal was
area of alienable lands of the public domain could easily set up corporations to acquire already private property at the time it was acquired from the Infiels by Acme.
more alienable public lands. An individual could own as many corporations as his Acme thereby acquired a registrable title, there being at the time no prohibition
means would allow him. An individual could even hide his ownership of a corporation by against said corporation's holding or owning private land. x x x. 40 (Emphasis
putting his nominees as stockholders of the corporation. The corporation is a convenient supplied)cralawlibrary
vehicle to circumvent the constitutional limitation on acquisition by individuals of
alienable lands of the public domain. Director of Lands is not applicable to the present case. In Director of Lands, the "land x
x x was already private property at the time it was acquired x x x by Acme." In this
The constitutional intent, under the 1973 and 1987 Constitutions, is to transfer case, respondent acquired the land on 8 August 1997 from Porting, who, along with his
ownership of only a limited area of alienable land of the public domain to a qualified predecessors-in-interest, has not shown to have been, as of that date, in open,
individual. This constitutional intent is safeguarded by the provision prohibiting continuous, and adverse possession of the land for 30 years since 12 June 1945. In
corporations from acquiring alienable lands of the public domain, since the vehicle to short, when respondent acquired the land from Porting, the land was not yet private
circumvent the constitutional intent is removed. The available alienable public lands are property.
gradually decreasing in the face of an ever-growing population. The most effective way
to insure faithful adherence to this constitutional intent is to grant or sell alienable lands For Director of Lands to apply and enable a corporation to file for registration of
of the public domain only to individuals. This, it would seem, is the practical benefit alienable and disposable land, the corporation must have acquired the land when its
arising from the constitutional ban.37 transferor had already a vested right to a judicial confirmation of title to the land by
virtue of his open, continuous and adverse possession of the land in the concept of an
In Director of Lands v. IAC,38 the Court allowed the land registration proceeding filed by owner for at least 30 years since 12 June 1945. Thus, in Natividad v. Court of
Acme Plywood & Veneer Co., Inc. (Acme) for five parcels of land with an area of Appeals,41 the Court declared:
481,390 square meters, or 48.139 hectares, which Acme acquired from members of the
Dumagat tribe. The issue in that case was whether the title could be confirmed in favor Under the facts of this case and pursuant to the above rulings, the parcels of land in
of Acme when the proceeding was instituted after the effectivity of the 1973 Constitution question had already been converted to private ownership through acquisitive
which prohibited private corporations or associations from holding alienable lands of the prescription by the predecessors-in-interest of TCMC when the latter purchased them in
public domain except by lease not to exceed 1,000 hectares. The Court ruled that the 1979. All that was needed was the confirmation of the titles of the previous owners or
land was already private land when Acme acquired it from its owners in 1962, and predecessors-in-interest of TCMC.
thus Acme acquired a registrable title. Under the 1935 Constitution, private corporations
could acquire public agricultural lands not exceeding 1,024 hectares while individuals
could acquire not more than 144 hectares.39 Being already private land when TCMC bought them in 1979, the prohibition in the 1973
Constitution against corporations acquiring alienable lands of the public domain except
through lease (Article XIV, Section 11, 1973 Constitution) did not apply to them for they
In Director of Lands, the Court further ruled that open, exclusive, and undisputed were no longer alienable lands of the public domain but private property.
possession of alienable land for the period prescribed by law created the legal fiction
whereby the land, upon completion of the requisite period, ipso jure and without the
need of judicial or other sanction ceases to be public land and becomes private What is determinative for the doctrine in Director of Lands to apply is for the corporate
property. The Court ruled: applicant for land registration to establish that when it acquired the land, the same was
already private land by operation of law because the statutory acquisitive prescriptive
period of 30 years had already lapsed. The length of possession of the land by the
Nothing can more clearly demonstrate the logical inevitability of considering possession corporation cannot be tacked on to complete the statutory 30 years acquisitive
of public land which is of the character and duration prescribed by statute as the prescriptive period. Only an individual can avail of such acquisitive prescription since
equivalent of an express grant from the State than the dictum of the statute itself that both the 1973 and 1987 Constitutions prohibit corporations from acquiring lands of the
the possessor(s) "x x x shall be conclusively presumed to have performed all the public domain.
conditions essential to a Government grant and shall be entitled to a certificate of title x
x x." No proof being admissible to overcome a conclusive presumption, confirmation
proceedings would, in truth be little more than a formality, at the most limited to Admittedly, a corporation can at present still apply for original registration of land under
ascertaining whether the possession claimed is of the required character and length of the doctrine in Director of Lands. Republic Act No. 917642 (RA 9176) further amended
the Public Land Act43 and extended the period for the filing of applications for judicial
confirmation of imperfect and incomplete titles to alienable and disposable lands of the
public domain until 31 December 2020. Thus:

Sec. 2. Section 47, Chapter VIII of the same Act, as amended, is hereby further
amended to read as follows:

Sec. 47. The persons specified in the next following section are hereby granted time,
not to extend beyond December 31, 2020 within which to avail of the benefits of this
Chapter: Provided, That this period shall apply only where the area applied for does not
exceed twelve (12) hectares: Provided, further, That the several periods of time
designated by the President in accordance with Section Forty-five of this Act shall apply
also to the lands comprised in the provisions of this Chapter, but this Section shall not
be construed as prohibiting any of said persons from acting under this Chapter at any
time prior to the period fixed by the President.

Sec. 3. All pending applications filed before the effectivity of this amendatory Act shall
be treated as having been filed in accordance with the provisions of this Act.

Under RA 9176, the application for judicial confirmation is limited only to 12 hectares,
consistent with Section 3, Article XII of the 1987 Constitution that a private individual
may only acquire not more than 12 hectares of alienable and disposable land. Hence,
respondent, as successor-in-interest of an individual owner of the land, cannot apply for
registration of land in excess of 12 hectares. Since respondent applied for 56.4007
hectares, the application for the excess area of 44.4007 hectares is contrary to law, and
thus void ab initio. In applying for land registration, a private corporation cannot have
any right higher than its predecessor-in-interest from whom it derived its right. This Republic of the Philippines
assumes, of course, that the corporation acquired the land, not exceeding 12 hectares,
when the land had already become private land by operation of law. In the present SUPREME COURT
case, respondent has failed to prove that any portion of the land was already private
land when respondent acquired it from Porting in 1997. Manila

WHEREFORE, we SET ASIDE the 21 August 2002 Decision of the Court of Appeals in FIRST DIVISION
CA-G.R. CV No. 66658 and the 16 December 1999 Decision of the Regional Trial Court
of Tanauan, Batangas, Branch 6 in Land Registration Case No. T-635. We DENY the
application for registration filed by T.A.N. Properties, Inc. G.R. No. 160453 November 12, 2012

SO ORDERED. REPUBLIC OF THE PHILIPPINES, Petitioner,

vs.

ARCADIO IVAN A. SANTOS III, and ARCADIO C. SANTOS, JR., Respondents.

DECISION

BERSAMIN, J.:

By law, accretion - the gradual and imperceptible deposit made through the effects of
the current of the water- belongs to the owner of the land adjacent to the banks of rivers
where it forms. The drying up of the river is not accretion. Hence, the dried-up river bed
belongs to the State as property of public dominion, not to the riparian owner, unless a I
law vests the ownership in some other person.
THE TRIAL COURT ERRED IN RULING THAT THE PROPERTY SOUGHT TO BE
Antecedents REGISTERED IS AN ACCRETION TO THE ADJOINING PROPERTY OWNED BY
APPELLEES DESPITE THE ADMISSION OF APPELLEE ARCADIO C. SANTOS JR.
Alleging continuous and adverse possession of more than ten years, respondent THAT THE SAID PROPERTY WAS NOT FORMED AS A RESULT OF THE GRADUAL
Arcadio Ivan A. Santos III (Arcadio Ivan) applied on March 7, 1997 for the registration of FILLING UP OF SOIL THROUGH THE CURRENT OF THE RIVER.
Lot 4998-B (the property) in the Regional Trial Court (RTC) in Parafiaque City. The
property, which had an area of 1,045 square meters, more or less, was located in II
Barangay San Dionisio, Paraaque City, and was bounded in the Northeast by Lot 4079
belonging to respondent Arcadio C. Santos, Jr. (Arcadio, Jr.), in the Southeast by the THE TRIAL COURT ERRED IN GRANTING THE APPLICATION FOR LAND
Paraaque River, in the Southwest by an abandoned road, and in the Northwest by Lot REGISTRATION DESPITE APPELLEES FAILURE TO FORMALLY OFFER IN
4998-A also owned by Arcadio Ivan.1 EVIDENCE AN OFFICIAL CERTIFICATION THAT THE SUBJECT PARCEL OF LAND
IS ALIENABLE AND DISPOSABLE.
On May 21, 1998, Arcadio Ivan amended his application for land registration to include
Arcadio, Jr. as his co-applicant because of the latters co-ownership of the property. He III
alleged that the property had been formed through accretion and had been in their joint
open, notorious, public, continuous and adverse possession for more than 30 years. 2
THE TRIAL COURT ERRED IN RULING THAT APPELLEES HAD SUFFICIENTLY
ESTABLISHED THEIR CONTINUOUS, OPEN, PUBLIC AND ADVERSE
The City of Paraaque (the City) opposed the application for land registration, stating OCCUPATION OF THE SUBJECT PROPERTY FOR A PERIOD OF MORE THAN
that it needed the property for its flood control program; that the property was within the THIRTY (30) YEARS.
legal easement of 20 meters from the river bank; and that assuming that the property
was not covered by the legal easement, title to the property could not be registered in
favor of the applicants for the reason that the property was an orchard that had dried up On May 27, 2003, the CA affirmed the RTC.6
and had not resulted from accretion.3
The Republic filed a motion for reconsideration, but the CA denied the motion on
Ruling of the RTC October 20, 2003.7

On May 10, 2000,4 the RTC granted the application for land registration, disposing: Issues

WHEREFORE, the Court hereby declares the applicants, ARCADIO IVAN A. SANTOS, Hence, this appeal, in which the Republic urges that:8
III and ARCADIO C. SANTOS, JR., both Filipinos and of legal age, as the TRUE and
ABSOLUTE OWNERS of the land being applied for which is situated in the Barangay of I
San Dionisio, City of Paraaque with an area of one thousand forty five (1045) square
meters more or less and covered by Subdivision Plan Csd-00-000343, being a portion RESPONDENTS CLAIM THAT THE SUBJECT PROPERTY IS AN ACCRETION TO
of Lot 4998, Cad. 299, Case 4, Paraaque Cadastre, LRC Rec. No. and orders the THEIR ADJOINING LAND THAT WOULD ENTITLE THEM TO REGISTER IT UNDER
registration of Lot 4998-B in their names with the following technical description, to wit: ARTICLE 457 OF THE NEW CIVIL CODE IS CONTRADICTED BY THEIR OWN
EVIDENCE.
xxxx
II
Once this Decision became (sic) final and executory, let the corresponding Order for the
Issuance of the Decree be issued. ASSUMING THAT THE LAND SOUGHT TO BE REGISTERED WAS "PREVIOUSLY A
PART OF THE PARAAQUE RIVER WHICH BECAME AN ORCHARD AFTER IT
SO ORDERED. DRIED UP," THE REGISTRATION OF SAID PROPERTY IN FAVOR OF
RESPONDENTS CANNOT BE ALTERNATIVELY JUSTIFIED UNDER ARTICLE 461
The Republic, through the Office of the Solicitor General (OSG), appealed. OF THE CIVIL CODE.

Ruling of the CA III

In its appeal, the Republic ascribed the following errors to the RTC, 5 to wit:
THE COURT OF APPEALS COMMITTED REVERSIBLE ERROR IN NOT RULING adjoins the same property is owned by the applicant which was obtained by the latter
THAT THE FAILURE OF RESPONDENTS TO FORMALLY OFFER IN EVIDENCE AN from his mother (Decision, p. 3; p. 38 Rollo).10
OFFICIAL CERTIFICATION THAT THE SUBJECT PROPERTY IS ALIENABLE AND
DISPOSABLE IS FATAL TO THEIR APPLICATION FOR LAND REGISTRATION. The Republic submits, however, that the application by both lower courts of Article 457
of the Civil Code was erroneous in the face of the fact that respondents evidence did
IV not establish accretion, but instead the drying up of the Paraaque River.

THE FINDING OF THE COURT OF APPEALS THAT RESPONDENTS HAVE The Republics submission is correct.
CONTINUOUSLY, OPENLY, PUBLICLY AND ADVERSELY OCCUPIED THE
SUBJECT PROPERTY FOR MORE THAN THIRTY (30) YEARS IS NOT SUPPORTED Respondents as the applicants for land registration carried the burden of proof to
BY WELL-NIGH INCONTROVERTIBLE EVIDENCE. establish the merits of their application by a preponderance of evidence, by which is
meant such evidence that is of greater weight, or more convincing than that offered in
To be resolved are whether or not Article 457 of the Civil Code was applicable herein; opposition to it.11 They would be held entitled to claim the property as their own and
and whether or not respondents could claim the property by virtue of acquisitive apply for its registration under the Torrens system only if they established that, indeed,
prescription pursuant to Section 14(1) of Presidential Decree No. 1529 (Property the property was an accretion to their land.
Registration Decree).
Accretion is the process whereby the soil is deposited along the banks of rivers. 12 The
Ruling deposit of soil, to be considered accretion, must be: (a) gradual and imperceptible; (b)
made through the effects of the current of the water; and (c) taking place on land
The appeal is meritorious. adjacent to the banks of rivers.13

I. Accordingly, respondents should establish the concurrence of the elements of accretion


to warrant the grant of their application for land registration.
The CA grossly erred in applying Article 457 of the Civil Code to respondents benefit
However, respondents did not discharge their burden of proof. They did not show that
the gradual and imperceptible deposition of soil through the effects of the current of the
Article 457 of the Civil Code provides that "(t)o the owners of lands adjoining the banks river had formed Lot 4998-B. Instead, their evidence revealed that the property was the
of rivers belong the accretion which they gradually receive from the effects of the dried-up river bed of the Paraaque River, leading both the RTC and the CA to
currents of the waters." themselves hold that Lot 4998-B was "the land which was previously part of the
Paraaque River xxx (and) became an orchard after it dried up."
In ruling for respondents, the RTC pronounced as follows:
Still, respondents argue that considering that Lot 4998-B did not yet exist when the
On the basis of the evidence presented by the applicants, the Court finds that Arcadio original title of Lot 4 was issued in their mothers name in 1920, and that Lot 4998-B
Ivan A. Santos III and Arcadio C. Santos, Jr., are the owners of the land subject of this came about only thereafter as the land formed between Lot 4 and the Paraaque River,
application which was previously a part of the Paraaque River which became an the unavoidable conclusion should then be that soil and sediments had meanwhile been
orchard after it dried up and further considering that Lot 4 which adjoins the same deposited near Lot 4 by the current of the Paraaque River, resulting in the formation of
property is owned by applicant, Arcadio C. Santos, Jr., after it was obtained by him Lot 4998-B.
through inheritance from his mother, Concepcion Cruz, now deceased. Conformably
with Art. 457 of the New Civil Code, it is provided that: The argument is legally and factually groundless. For one, respondents thereby ignore
that the effects of the current of the river are not the only cause of the formation of land
"Article 457. To the owners of the lands adjoining the bank of rivers belong the accretion along a river bank. There are several other causes, including the drying up of the river
which they gradually receive from the effects of the current of the waters." 9 bed. The drying up of the river bed was, in fact, the uniform conclusion of both lower
courts herein. In other words, respondents did not establish at all that the increment of
The CA upheld the RTCs pronouncement, holding: land had formed from the gradual and imperceptible deposit of soil by the effects of the
current. Also, it seems to be highly improbable that the large volume of soil that
ultimately comprised the dry land with an area of 1,045 square meters had been
It could not be denied that "to the owners of the lands adjoining the banks of rivers deposited in a gradual and imperceptible manner by the current of the river in the span
belong the accretion which they gradually receive from the effects of the current of the of about 20 to 30 years the span of time intervening between 1920, when Lot 4 was
waters" (Article 457 New Civil Code) as in this case, Arcadio Ivan Santos III and Arcadio registered in the name of their deceased parent (at which time Lot 4998-B was not yet
Santos, Jr., are the owners of the land which was previously part of the Paraaque in existence) and the early 1950s (which respondents witness Rufino Allanigue alleged
River which became an orchard after it dried up and considering that Lot 4 which to be the time when he knew them to have occupied Lot 4988-B). The only plausible
explanation for the substantial increment was that Lot 4988-B was the dried-up bed of
the Paraaque River. Confirming this explanation was Arcadio, Jr.s own testimony to adversely for more than thirty (30) years because their predecessors-in-interest are the
the effect that the property was previously a part of the Paraaque River that had dried adjoining owners of the subject parcel of land along the river bank. Furthermore, the fact
up and become an orchard. that applicants paid its realty taxes, had it surveyed per subdivision plan Csd-00-000343
(Exh. "L") which was duly approved by the Land Management Services and the fact that
We observe in this connection that even Arcadio, Jr.s own Transfer Certificate of Title Engr. Chito B. Cainglet, OICChief, Surveys Division Land Registration Authority, made
No. 44687 confirmed the uniform conclusion of the RTC and the CA that Lot 4998-B a Report that the subject property is not a portion of the Paraaque River and that it
had been formed by the drying up of the Paraaque River. Transfer Certificate of Title does not fall nor overlap with Lot 5000, thus, the Court opts to grant the application.
No. 44687 recited that Lot 4 of the consolidated subdivision plan Pcs-13-002563, the lot
therein described, was bounded "on the SW along line 5-1 by Dried River Bed."14 Finally, in the light of the evidence adduced by the applicants in this case and in view of
the foregoing reports of the Department of Agrarian Reforms, Land Registration
That boundary line of "SW along line 5-1" corresponded with the location of Lot 4998-B, Authority and the Department of Environment and Natural Resources, the Court finds
which was described as "bounded by Lot 4079 Cad. 299, (Lot 1, Psu-10676), in the and so holds that the applicants have satisfied all the requirements of law which are
name of respondent Arcadio Santos, Jr. (Now Lot 4, Psd-13-002563) in the essential to a government grant and is, therefore, entitled to the issuance of a certificate
Northeast."15 of title in their favor. So also, oppositor failed to prove that the applicants are not entitled
thereto, not having presented any witness.
The RTC and the CA grossly erred in treating the dried-up river bed as an accretion that
became respondents property pursuant to Article 457 of the Civil Code. That land was In fine, the application is GRANTED.
definitely not an accretion. The process of drying up of a river to form dry land involved
the recession of the water level from the river banks, and the dried-up land did not As already mentioned, the CA affirmed the RTC.
equate to accretion, which was the gradual and imperceptible deposition of soil on the
river banks through the effects of the current. In accretion, the water level did not recede Both lower courts erred.
and was more or less maintained. Hence, respondents as the riparian owners had no
legal right to claim ownership of Lot 4998-B. Considering that the clear and categorical
language of Article 457 of the Civil Code has confined the provision only to accretion, The relevant legal provision is Section 14(1) of Presidential Decree No. 1529 (Property
we should apply the provision as its clear and categorical language tells us to. Registration Decree), which pertinently states:
Axiomatic it is, indeed, that where the language of the law is clear and categorical, there
is no room for interpretation; there is only room for application. 16 The first and Section 14. Who may apply. The following persons may file in the proper [Regional
fundamental duty of courts is then to apply the law.17 Trial Court] an application for registration of title to land, whether personally or through
their duly authorized representatives:
The State exclusively owned Lot 4998-B and may not be divested of its right of
ownership. Article 502 of the Civil Code expressly declares that rivers and their natural (1) Those who by themselves or through their predecessors-in-interest have been in
beds are public dominion of the State.18 It follows that the river beds that dry up, like Lot open, continuous, exclusive and notorious possession and occupation of alienable and
4998-B, continue to belong to the disposable lands of the public domain under a bona fide claim of ownership since June
12, 1945, or earlier.
State as its property of public dominion, unless there is an express law that provides
that the dried-up river beds should belong to some other person.19 xxxx

II Under Section 14(1), then, applicants for confirmation of imperfect title must prove the
following, namely: (a) that the land forms part of the disposable and alienable
Acquisitive prescription was agricultural lands of the public domain; and (b) that they have been in open, continuous,
exclusive, and notorious possession and occupation of the land under a bona fide claim
of ownership either since time immemorial or since June 12, 1945. 21
not applicable in favor of respondents
The Republic assails the findings by the lower courts that respondents "took possession
The RTC favored respondents application for land registration covering Lot 4998-B also of the same property continuously, openly, publicly and adversely for more than thirty
because they had taken possession of the property continuously, openly, publicly and (30) years."22
adversely for more than 30 years based on their predecessor-in-interest being the
adjoining owner of the parcel of land along the river bank. It rendered the following
ratiocination, viz:20 Although it is well settled that the findings of fact of the trial court, especially when
affirmed by the CA, are accorded the highest degree of respect, and generally will not
be disturbed on appeal, with such findings being binding and conclusive on the Court,23
In this regard, the Court found that from the time the applicants became the owners the Court has consistently recognized exceptions to this rule, including the following, to
thereof, they took possession of the same property continuously, openly, publicly and wit: (a) when the findings are grounded entirely on speculation, surmises, or
conjectures; (b) when the inference made is manifestly mistaken, absurd, or impossible; admission of respondents themselves that they declared the property for taxation
(c) when there is grave abuse of discretion; (d) when the judgment is based on a purposes only in 1997 and paid realty taxes only from 199928 signified that their alleged
misapprehension of facts; (e) when the findings of fact are conflicting; (f) when in possession would at most be for only nine years as of the filing of their application for
making its findings the CA went beyond the issues of the case, or its findings are land registration on March 7, 1997.
contrary to the admissions of both the appellant and the appellee; (g) when the findings
are contrary to those of the trial court; (h) when the findings are conclusions without Yet, even conceding, for the sake of argument, that respondents possessed Lot 4998-B
citation of specific evidence on which they are based; (i) when the facts set forth in the for more than thirty years in the character they claimed, they did not thereby acquire the
petition as well as in the petitioners main and reply briefs are not disputed by land by prescription or by other means without any competent proof that the land was
respondent; and (j) when the findings of fact are premised on the supposed absence of already declared as alienable and disposable by the Government. Absent that
evidence and contradicted by the evidence on record. 24 declaration, the land still belonged to the State as part of its public dominion.

Here, the findings of the RTC were obviously grounded on speculation, surmises, or Article 419 of the Civil Code distinguishes property as being either of public dominion or
conjectures; and that the inference made by the RTC and the CA was manifestly of private ownership. Article 420 of the Civil Code lists the properties considered as part
mistaken, absurd, or impossible. Hence, the Court should now review the findings. of public dominion, namely: (a) those intended for public use, such as roads, canals,
rivers, torrents, ports and bridges constructed by the State, banks, shores, roadsteads,
In finding that respondents had been in continuous, open, public and adverse and others of similar character; and (b) those which belong to the State, without being
possession of the land for more than 30 years, the RTC declared: for public use, and are intended for some public service or for the development of the
national wealth. As earlier mentioned, Article 502 of the Civil Code declares that rivers
In this regard, the Court found that from the time the applicant became the owners and their natural beds are of public dominion.
thereof, they took possession of the same property continuously, openly, publicly and
adversely for more than thirty years because their predecessor in interest are the Whether the dried-up river bed may be susceptible to acquisitive prescription or not was
adjoining owners of the subject parcel of land along the river banks. Furthermore, the a question that the Court resolved in favor of the State in Celestial v. Cachopero, 29 a
fact that the applicant paid its realty taxes, had it surveyed per subdivision plan Csd-00- case involving the registration of land found to be part of a dried-up portion of the
000343 (Exh. "L") which was duly approved by the Land Management Services and the natural bed of a creek. There the Court held:
fact that Engr. Chito B. Cainglet, OIC Chief, Surveys Division Land Registration
Authority, made a Report that the subject property is not a portion of the Paraaque As for petitioners claim of ownership over the subject land, admittedly a dried-up bed of
River and that it does not fall nor overlap with Lot 5000, thus, the Court opts to grant the the Salunayan Creek, based on (1) her alleged long term adverse possession and that
application. of her predecessor-in-interest, Marcelina Basadre, even prior to October 22, 1966,
when she purchased the adjoining property from the latter, and (2) the right of
The RTC apparently reckoned respondents period of supposed possession to be "more accession under Art. 370 of the Spanish Civil Code of 1889 and/or Article 461 of the
than thirty years" from the fact that "their predecessors in interest are the adjoining Civil Code, the same must fail.
owners of the subject parcel of land." Yet, its decision nowhere indicated what acts
respondents had performed showing their possession of the property "continuously, Since property of public dominion is outside the commerce of man and not susceptible
openly, publicly and adversely" in that length of time. The decision mentioned only that to private appropriation and acquisitive prescription, the adverse possession which may
they had paid realty taxes and had caused the survey of the property to be made. That, be the basis of a grant of title in the confirmation of an imperfect title refers only to
to us, was not enough to justify the foregoing findings, because, firstly, the payment of alienable or disposable portions of the public domain. It is only after the Government
realty taxes did not conclusively prove the payors ownership of the land the taxes were has declared the land to be alienable and disposable agricultural land that the year of
paid for,25 the tax declarations and payments being mere indicia of a claim of entry, cultivation and exclusive and adverse possession can be counted for purposes of
ownership;26 and, secondly, the causing of surveys of the property involved was not an imperfect title.
itself an of continuous, open, public and adverse possession.
A creek, like the Salunayan Creek, is a recess or arm extending from a river and
The principle that the riparian owner whose land receives the gradual deposits of soil participating in the ebb and flow of the sea. As such, under Articles 420(1) and 502(1) of
does not need to make an express act of possession, and that no acts of possession the Civil Code, the Salunayan Creek, including its natural bed, is property of the public
are necessary in that instance because it is the law itself that pronounces the alluvium domain which is not susceptible to private appropriation and acquisitive prescription.
to belong to the riparian owner from the time that the deposit created by the current of And, absent any declaration by the government, that a portion of the creek has dried-up
the water becomes manifest27 has no applicability herein. This is simply because Lot does not, by itself, alter its inalienable character.
4998-B was not formed through accretion. Hence, the ownership of the land adjacent to
the river bank by respondents predecessor-in-interest did not translate to possession of
Lot 4998-B that would ripen to acquisitive prescription in relation to Lot 4998-B. xxxx

On the other hand, the claim of thirty years of continuous, open, public and adverse Had the disputed portion of the Salunayan Creek dried up after the present Civil Code
possession of Lot 4998-B was not even validated or preponderantly established. The took effect, the subject land would clearly not belong to petitioner or her predecessor-in-
interest since under the aforementioned provision of Article 461, "river beds which are
abandoned through the natural change in the course of the waters ipso facto belong to ALL CORNERS NOT OTHERWISE DESCRIBED ARE OLD BL CYL. CONC. MONS 15
the owners of the land occupied by the new course," and the owners of the adjoining X 60CM
lots have the right to acquire them only after paying their value.
All corners marked PS are cyl. conc. mons 15 x 60 cm
And both Article 370 of the Old Code and Article 461 of the present Civil Code are
applicable only when "river beds are abandoned through the natural change in the Surveyed in accordance with Survey Authority NO. 007604-48 of the Regional
course of the waters." It is uncontroverted, however, that, as found by both the Bureau Executive Director issued by the CENR-OFFICER dated Dec. 2, 1996.
of Lands and the DENR Regional Executive Director, the subject land became dry as a
result of the construction an irrigation canal by the National Irrigation Administration.
Thus, in Ronquillo v. Court of Appeals, this Court held: This survey is inside L.C. Map No. 2623, Proj. No. 25 classified as alienable/disposable
by the Bureau of Forest Devt. on Jan. 3, 1968.
The law is clear and unambiguous. It leaves no room for interpretation. Article 370
applies only if there is a natural change in the course of the waters. The rules on Lot 4998-A = Lot 5883} Cad 299
alluvion do not apply to man-made or artificial accretions nor to accretions to lands that
adjoin canals or esteros or artificial drainage systems. Considering our earlier finding Lot 4998-B = Lot 5884} Paranaque Cadastre.
that the dried-up portion of Estero Calubcub was actually caused by the active
intervention of man, it follows that Article 370 does not apply to the case at bar and, Was the notation on the survey plan to the effect that Lot 4998-B was "inside" the map
hence, the Del Rosarios cannot be entitled thereto supposedly as riparian owners. "classified as alienable/disposable by the Bureau of Forest Development on 03 Jan.
1968" sufficient proof of the propertys nature as alienable and disposable public land?
The dried-up portion of Estero Calubcub should thus be considered as forming part of
the land of the public domain which cannot be subject to acquisition by private To prove that the land subject of an application for registration is alienable, an applicant
ownership. xxx (Emphasis supplied) must conclusively establish the existence of a positive act of the Government, such as a
presidential proclamation, executive order, administrative action, investigation reports of
Furthermore, both provisions pertain to situations where there has been a change in the the Bureau of Lands investigator, or a legislative act or statute. Until then, the rules on
course of a river, not where the river simply dries up. In the instant Petition, it is not confirmation of imperfect title do not apply.
even alleged that the Salunayan Creek changed its course. In such a situation,
commentators are of the opinion that the dry river bed remains property of public As to the proofs that are admissible to establish the alienability and disposability of
dominion. (Bold emphases supplied) public land, we said in Secretary of the Department of Environment and Natural
Resources v. Yap34 that:
Indeed, under the Regalian doctrine, all lands not otherwise appearing to be clearly
within private ownership are presumed to belong to the State. 30 No public land can be The burden of proof in overcoming the presumption of State ownership of the lands of
acquired by private persons without any grant, express or implied, from the the public domain is on the person applying for registration (or claiming ownership), who
Government. It is indispensable, therefore, that there is a showing of a title from the must prove that the land subject of the application is alienable or disposable. To
State.31Occupation of public land in the concept of owner, no matter how long, cannot overcome this presumption, incontrovertible evidence must be established that the land
ripen into ownership and be registered as a title.32 subject of the application (or claim) is alienable or disposable.There must still be a
positive act declaring land of the public domain as alienable and disposable. To prove
Subject to the exceptions defined in Article 461 of the Civil Code (which declares river that the land subject of an application for registration is alienable, the applicant must
beds that are abandoned through the natural change in the course of the waters as ipso establish the existence of a positive act of the government such as a presidential
facto belonging to the owners of the land occupied by the new course, and which gives proclamation or an executive order; an administrative action; investigation reports of
to the owners of the adjoining lots the right to acquire only the abandoned river beds not Bureau of Lands investigators; and a legislative act or a statute. The applicant may also
ipso facto belonging to the owners of the land affected by the natural change of course secure a certification from the government that the land claimed to have been
of the waters only after paying their value), all river beds remain property of public possessed for the required number of years is alienable and disposable.
dominion and cannot be acquired by acquisitive prescription unless previously declared
by the Government to be alienable and disposable. Considering that Lot 4998-B was In the case at bar, no such proclamation, executive order, administrative action, report,
not shown to be already declared to be alienable and disposable, respondents could not statute, or certification was presented to the Court. The records are bereft of evidence
be deemed to have acquired the property through prescription. showing that, prior to 2006, the portions of Boracay occupied by private claimants were
subject of a government proclamation that the land is alienable and disposable. Absent
Nonetheless, respondents insist that the property was already classified as alienable such well-nigh incontrovertible evidence, the Court cannot accept the submission that
and disposable by the Government. They cite as proof of the classification as alienable lands occupied by private claimants were already open to disposition before 2006.
and disposable the following notation found on the survey plan, to wit: 33 Matters of land classification or reclassification cannot be assumed. They call for proof."
(Emphasis supplied)
NOTE
In Menguito v. Republic,35 which we reiterated in Republic v. Sarmiento, 36 we have accepted the contents of the certifications as proof of the facts stated therein.
specifically resolved the issue of whether the notation on the survey plan was sufficient Even if the certifications are presumed duly issued and admissible in evidence, they
evidence to establish the alienability and disposability of public land, to wit: have no probative value in establishing that the land is alienable and disposable.

To prove that the land in question formed part of the alienable and disposable lands of xxxx
the public domain, petitioners relied on the printed words which read: "This survey plan
is inside Alienable and Disposable Land Area, Project No. 27-B as per L.C. Map No. The CENRO and Regional Technical Director, FMS-DENR, certifications do not prove
2623, certified by the Bureau of Forestry on January 3, 1968," appearing on Exhibit "E" that Lot 10705-B falls within the alienable and disposable land as proclaimed by the
(Survey Plan No. Swo-13-000227). DENR Secretary. Such government certifications do not, by their mere issuance, prove
the facts stated therein. Such government certifications may fall under the class of
This proof is not sufficient. Section 2, Article XII of the 1987 Constitution, provides: "All documents contemplated in the second sentence of Section 23 of Rule 132. As such,
lands of the public domain, waters, minerals, coal, petroleum, and other mineral oils, all the certifications are prima facie evidence of their due execution and date of issuance
forces of potential energy, fisheries, forests or timber, wildlife, flora and fauna, and other but they do not constitute prima facie evidence of the facts stated therein. (Emphasis
natural resources are owned by the State. x x x." supplied)

For the original registration of title, the applicant (petitioners in this case) must These rulings of the Court indicate that the notation on the survey plan of Lot 4998-B,
overcome the presumption that the land sought to be registered forms part of the public Cad-00-000343 to the effect that the "survey is inside a map classified as
domain. Unless public land is shown to have been reclassified or alienated to a private alienable/disposable by the Bureau of Forest Devt" did not prove that Lot 4998-B was
person by the State, it remains part of the inalienable public domain. Indeed, already classified as alienable and disposable. Accordingly, respondents could not
"occupation thereof in the concept of owner, no matter how long, cannot ripen into validly assert acquisitive prescription of Lot 4988-B.
ownership and be registered as a title." To overcome such presumption, incontrovertible
evidence must be shown by the applicant. Absent such evidence, the land sought to be WHEREFORE, the Court REVERSES and SETS ASIDE the decision of the Court of
registered remains inalienable. Appeals promulgated on May 27, 2003; DISMISSES the application for registration of
Arcadio C. Santos, Jr. and Arcadio Ivan S. Santos III respecting Lot 4998-B with a total
In the present case, petitioners cite a surveyor-geodetic engineers notation in Exhibit area of 1,045 square meters, more or less, situated in Barangay San Dionisio,
"E" indicating that the survey was inside alienable and disposable land. Such notation Paraaque City, Metro Manila; and DECLARES Lot 4998-B as exclusively belonging to
does not constitute a positive government act validly changing the classification of the the State for being part of the dried--up bed of the Parat1aque River.
land in question. Verily, a mere surveyor has no authority to reclassify lands of the
public domain. By relying solely on the said surveyors assertion, petitioners have not Respondents shall pay the costs of suit.
sufficiently proven that the land in question has been declared alienable. (Emphasis
supplied)
SO ORDERED.
In Republic v. T.A.N. Properties, Inc.,37 we dealt with the sufficiency of the certification
by the Provincial Environmental Officer (PENRO) or Community Environmental Officer
(CENRO) to the effect that a piece of public land was alienable and disposable in the
following manner, viz:

x x x it is not enough for the PENRO or CENRO to certify that a land is alienable and
disposable. The applicant for land registration must prove that the DENR Secretary had
approved the land classification and released the land of the public domain as alienable
and disposable, and that the land subject of the application for registration falls within
the approved area per verification through survey by the PENRO or CENRO. In
addition, the applicant for land registration must present a copy of the original
classification approved by the DENR Secretary and certified as a true copy by the legal
custodian of the official records. These facts must be established to prove that the land
is alienable and disposable. Respondent failed to do so because the certifications
presented by respondent do not, by themselves, prove that the land is alienable and
disposable.

Only Torres, respondents Operations Manager, identified the certifications submitted by


respondent.1wphi1 The government officials who issued the certifications were not
presented before the trial court to testify on their contents. The trial court should not

Das könnte Ihnen auch gefallen